[go: up one dir, main page]

85% found this document useful (13 votes)
19K views298 pages

Pocket Medicine High - Yield Board Review 2021

Uploaded by

Omar Hassan
Copyright
© © All Rights Reserved
We take content rights seriously. If you suspect this is your content, claim it here.
Available Formats
Download as PDF, TXT or read online on Scribd
85% found this document useful (13 votes)
19K views298 pages

Pocket Medicine High - Yield Board Review 2021

Uploaded by

Omar Hassan
Copyright
© © All Rights Reserved
We take content rights seriously. If you suspect this is your content, claim it here.
Available Formats
Download as PDF, TXT or read online on Scribd
You are on page 1/ 298

Freemedicalbooks4download

Executive Editor: Sharon Zinner


Development Editor: Thomas Celona
Editorial Coordinators: Ingrid Greenlee and Sean Hanrahan
Editorial Assistant: Victoria Giansante
Marketing Manager: Phyllis Hitner
Production Project Manager: Sadie Buckallew
Design Coordinator: Holly McLaughlin
Art Director, Illustration: Jennifer Clements
Manufacturing Coordinator: Kathy Brown
Prepress Vendor: S4Carlisle Publishing Services

Copyright © 2021 Wolters Kluwer.

All rights reserved. This book is protected by copyright. No part of this book may be reproduced or transmitted in any form or by
any means, including as photocopies or scanned-in or other electronic copies, or utilized by any information storage and retrieval
system without written permission from the copyright owner, except for brief quotations embodied in critical articles and
reviews. Materials appearing in this book prepared by individuals as part of their official duties as U.S. government employees
are not covered by the above-mentioned copyright. To request permission, please contact Wolters Kluwer at Two Commerce
Square, 2001 Market Street, Philadelphia, PA 19103, via email at permissions@lww.com, or via our website at shop.lww.com
(products and services).

987654321

Printed in China

Library of Congress Cataloging-in-Publication Data


ISBN-13: 978-1-975142-43-8 | eISBN: 9781975142469
ISBN-10: 1-975142-43-8

This work is provided “as is,” and the publisher disclaims any and all warranties, express or implied, including any warranties as
to accuracy, comprehensiveness, or currency of the content of this work.

This work is no substitute for individual patient assessment based upon healthcare professionals’ examination of each patient and
consideration of, among other things, age, weight, gender, current or prior medical conditions, medication history, laboratory data
and other factors unique to the patient. The publisher does not provide medical advice or guidance and this work is merely a
reference tool. Healthcare professionals, and not the publisher, are solely responsible for the use of this work including all
medical judgments and for any resulting diagnosis and treatments.

Given continuous, rapid advances in medical science and health information, independent professional verification of medical
diagnoses, indications, appropriate pharmaceutical selections and dosages, and treatment options should be made and healthcare
professionals should consult a variety of sources. When prescribing medication, healthcare professionals are advised to consult
the product information sheet (the manufacturer’s package insert) accompanying each drug to verify, among other things,
conditions of use, warnings and side effects and identify any changes in dosage schedule or contraindications, particularly if the
medication to be administered is new, infrequently used or has a narrow therapeutic range. To the maximum extent permitted
under applicable law, no responsibility is assumed by the publisher for any injury and/or damage to persons or property, as a
matter of products liability, negligence law or otherwise, or from any reference to or use by any person of this work.

Freemedicalbooks4download
shop.lww.com
Contributors

Andrew S. Allegretti, MD, MSc


Director of ICU Nephrology, Attending Physician, Nephrology Division, Massachusetts General
Hospital
Principal Investigator, Kidney Research Center, Massachusetts General Hospital
Instructor of Medicine, Harvard Medical School

Omar Al-Louzi, MD
Neurology Resident, Partners Neurology Residency

Alexander Blair, MD
Internal Medicine Resident, Massachusetts General Hospital

Michael P. Bowley, MD, PhD


Instructor in Neurology, Massachusetts General Hospital
Associate Program Director, Partners Neurology Residency Program

Leeann Brigham Burton, MD


Neurology Resident, Partners Neurology Residency

Alison C. Castle, MD
Internal Medicine Resident, Massachusetts General Hospital

Caitlin Colling, MD
Internal Medicine Resident, Massachusetts General Hospital

Jean M. Connors, MD
Medical Director, Anticoagulation Management Services, Hematology Division, Brigham and
Women’s Hospital & Dana-Farber Cancer Institute
Associate Professor of Medicine, Harvard Medical School

Daniel J. DeAngelo, MD, PhD


Chief, Division of Leukemia, Dana-Farber Cancer Institute
Professor of Medicine, Harvard Medical School

Rachel Frank, MD
Internal Medicine Resident, Massachusetts General Hospital

Freemedicalbooks4download
Robert P. Friday, MD, PhD
Chief, Division of Rheumatology, Newton-Wellesley Hospital
Affiliate Physician, Rheumatology Unit, Massachusetts General Hospital
Instructor in Medicine, Harvard Medical School

Lawrence S. Friedman, MD
The Anton R. Fried, MD, Chair, Department of Medicine, Newton-Wellesley Hospital
Assistant Chief of Medicine, Massachusetts General Hospital
Professor of Medicine, Harvard Medical School
Professor of Medicine, Tufts University School of Medicine

Kristin Galetta, MD
Neurology Resident, Partners Neurology Residency

Kristen Hysell, MD
Infectious Disease Fellow, Massachusetts General Hospital

Tanya E. Keenan, MD, MPH


Hematology-Oncology Fellow, Dana-Farber/Partners CancerCare

Emily Walsh Lopes, MD


Gastroenterology Fellow, Massachusetts General Hospital

Melissa Lumish, MD
Internal Medicine Resident, Massachusetts General Hospital

Jason Maley, MD
Pulmonary Fellow, Massachusetts General Hospital

Michael Mannstadt, MD
Chief, Endocrine Unit, Massachusetts General Hospital
Associate Professor of Medicine, Harvard Medical School

Arielle Medford, MD
Internal Medicine Resident, Massachusetts General Hospital

Nino Mihatov, MD
Cardiology Fellow, Massachusetts General Hospital

Mazen Nasrallah, MD, MSc


Rheumatology Fellow, Massachusetts General Hospital

Walter J. O’Donnell, MD
Staff Physician, Pulmonary/Critical Care Unit, Massachusetts General Hospital
Assistant Professor of Medicine, Harvard Medical School

Michelle L. O’Donoghue, MD, MPH


Senior Investigator, TIMI Study Group
Associate Physician, Cardiovascular Division, Brigham and Women’s Hospital
Affiliate Physician, Cardiology Division, Massachusetts General Hospital
Associate Professor of Medicine, Harvard Medical School

Nilay Patel, MD
Cardiology Fellow, Massachusetts General Hospital

Morgan Prust, MD
Neurology Resident, Massachusetts General Hospital

Stephanie M. Rutledge, MBBCh, BAO, MRCPI


Internal Medicine Resident, Massachusetts General Hospital

David P. Ryan, MD
Clinical Director, Massachusetts General Hospital Cancer Center
Chief of Hematology/Oncology, Massachusetts General Hospital
Professor of Medicine, Harvard Medical School

Marc S. Sabatine, MD, MPH


Chair, TIMI Study Group
Lewis Dexter, MD, Distinguished Chair in Cardiovascular Medicine, Brigham and Women’s
Hospital
Affiliate Physician, Cardiology Division, Massachusetts General Hospital
Professor of Medicine, Harvard Medical School

Harish Seethapathy, MBBS


Nephrology Fellow, BWH/MGH Joint Nephrology Fellowship Program

Shilpa Sharma, MD
Internal Medicine Resident, Massachusetts General Hospital

Harshabad Singh, MBBS


Instructor, Gastrointestinal Cancer Treatment Center, Dana-Farber Cancer Institute

Isaac D. Smith, MD
Internal Medicine Resident, Massachusetts General Hospital

Miranda Theodore, MD
Internal Medicine Resident, Massachusetts General Hospital

Armen Yerevanian, MD
Endocrinology Fellow, Massachusetts General Hospital

Kimon C. Zachary, MD
Assistant Professor of Medicine, Infectious Disease Division, Massachusetts General Hospital

Freemedicalbooks4download
Preface

To my parents, Matthew and Lee Sabatine, to their namesake grandchildren, Matteo and
Natalie, and to my wife, Jennifer.

Many readers of Pocket Medicine have commented that they use that book not only on the
wards, but also for preparation for their various board examinations. To help further address that
need, we have crafted a dedicated Pocket Medicine boards review book.
In Pocket Medicine High-Yield Board Review, we have provided readers with over
500 case-based questions across nine subspecialties. The annotated answers are detailed
and review the key diagnostic and therapeutic principles. Readers will see that the cases
are not only based on the exceptional medical knowledge and clinical acumen of the
teams of authors for each subspecialty for the 7th edition of Pocket Medicine, but also
reflect the recent experience of the more junior members in taking board examinations. A
first edition always presents unique challenges and we welcome any suggestions for
improvement.
I hope that you find Pocket Medicine High-Yield Board Review not only useful for
your examination, but also to help hone your knowledge of internal medicine.
Marc S. Sabatine, MD, MPH
Contents

1. Cardiology
Rachel Frank, Shilpa Sharma, Nino Mihatov, Nilay Patel, Marc S. Sabatine,
Michelle L. O’Donoghue

2. Pulmonary
Miranda Theodore, Jason Maley, Walter J. O’Donnell

3. Gastroenterology
Stephanie M. Rutledge, Emily Walsh Lopes, Lawrence S. Friedman

4. Nephrology
Alexander Blair, Harish Seethapathy, Andrew S. Allegretti

5. Hematology-Oncology
Melissa Lumish, Arielle Medford, Tanya E. Keenan, Harshabad Singh, Jean M.
Connors, Daniel J. DeAngelo, David P. Ryan

6. Infectious Diseases
Alison C. Castle, Kristen Hysell, Kimon C. Zachary

7. Endocrinology
Caitlin Colling, Armen Yerevanian, Michael Mannstadt

8. Rheumatology
Isaac D. Smith, Mazen Nasrallah, Robert P. Friday

9. Neurology
Omar Al-Louzi, Leeann Brigham Burton, Kristin Galetta, Morgan Prust, Michael
P. Bowley

Abbreviations

Freemedicalbooks4download
1
CARDIOLOGY

QUESTIONS

1. A 59-year-old man with a history of hypertension and hyperlipidemia presents with 1


hour of substernal chest pressure rated an 8 on a scale of 1 to 10 with radiation down the
left arm and associated with diaphoresis. Initial vital signs are notable for a blood pressure
(BP) of 92/64 mmHg and a heart rate (HR) of 92 beats/min. His electrocardiogram (ECG)
is shown below:

What is the most likely diagnosis?


A. Anterior ST-elevation myocardial infarction
B. Inferior ST-elevation myocardial infarction
C. Pericarditis
D. ST changes not meeting specific ischemia criteria; additional ECGs should be
obtained.

2. A 72-year-old man with a history of paroxysmal atrial fibrillation (AF) presents with
substernal chest pain following 3 days of rhinorrhea and nasal congestion. His initial vital
signs are notable for the absence of a fever, HR of 58 beats/min, BP of 102/58 mmHg, and
oxygen saturation of 98% on room air. His initial ECG is shown below:

What is the most likely diagnosis?


A. Anterior ST-elevation myocardial infarction
B. Inferior ST-elevation myocardial infarction
C. Pericarditis
D. ST changes not meeting specific ischemia criteria; additional ECGs should be
obtained.

3. A 78-year-old man with a history of heart failure (HF) with reduced ejection fraction
(HFrEF) presents to clinic for posthospitalization follow-up. He reports no current chest
pain, anginal equivalents, or dyspnea on exertion. His ECG is shown below:

Freemedicalbooks4download
What is the correct interpretation?
A. AF with left bundle branch block (LBBB)
B. AF with right bundle branch block (RBBB)
C. AF with RBBB and left anterior fascicular block
D. Complete heart block with RBBB

4. A 72-year-old man with a history of stage 4 chronic kidney disease secondary to


hypertension presents to the hospital with fatigue and malaise for the past 3 days. He
reports no chest pain. His ECG is shown below:

What is the most likely etiology of the ECG changes?


A. Hypercalcemia
B. Hyperkalemia
C. Hypokalemia
D. Ischemia

5. A 39-year-old man with a history of substance use on methadone is admitted with


community-acquired pneumonia and started on levofloxacin. His ECG on admission is
shown below:

Which of the following would increase his risk of life-threatening arrhythmia?


A. Dose-reducing his methadone
B. Ensuring his electrolytes are replete
C. Initiating β-blocker therapy
D. Transitioning from levofloxacin to doxycycline

6. A 45-year-old woman with a history of tobacco use presents with 20 minutes of new-
onset substernal chest pressure. Her vital signs are notable for a HR of 102 beats/min, a
BP of 94/68 mmHg, and oxygen saturation of 90% on room air. Her examination is
notable for jugular venous distention (JVD); tachycardia without murmurs, rubs, or
gallops; and left greater than right lower extremity edema. Her ECG is shown below:

Freemedicalbooks4download
What is the most likely diagnosis?
A. Inferior ST-elevation myocardial infarction
B. Non–ST-elevation myocardial infarction
C. Pericarditis
D. Pulmonary embolism

7. A 68-year-old man presents with severe substernal pressure, nausea, and vomiting for the
past 30 minutes. He reports the chest pressure started over the course of 10 minutes and
has continued to worsen. He is diaphoretic and in moderate distress. Vital signs are
notable for a BP of 118/78 mmHg, a HR of 94 beats/min, and oxygen saturation of 98%
on room air. Physical examination reveals clear lung fields without any murmurs, rubs, or
gallops on cardiac auscultation. ECG is checked and shows an R wave > S wave in V1
with ST-segment depressions in V1 to V3.
What is the next best step?
A. Check posterior leads
B. Rule out pulmonary embolism by computed tomography (CT) of the chest
C. Start colchicine for pericarditis
D. Trial of a proton-pump inhibitor

8. A 69-year-old man with a history of hypertension and hyperlipidemia presents with 90


minutes of substernal chest pain. The pain is described as a pressure sensation with
radiation down the left arm and associated with diaphoresis. Initial vital signs are notable
for a BP of 92/64 mmHg, a HR of 92 beats/min, and oxygen saturation of 94% on room
air. He was given a full-dose aspirin by EMS prior to arrival. His ECG is shown below:
Right-sided leads are obtained that reveal elevation in lead V4R. What is the next
best step?
A. Activate the cardiac catheterization lab for urgent revascularization
B. Administer nitroglycerin
C. Obtain posterior ECG leads
D. Perform transthoracic echocardiogram

9. A 72-year-old man with a history of hypertension presents with new onset of pleuritic and
sharp left-sided chest pain radiating to the shoulders. The pain started subacutely and is
worsened when lying flat and improved by leaning forward. He has not had any preceding
chest pain or dyspnea with exertion. For the past week, he has had rhinorrhea and a dry
cough. Vital signs are notable for tachycardia with a HR of 104 beats/min and a BP of
122/78 mmHg. Oxygen saturation is 98% on room air and respiratory rate is 18
breaths/min. His ECG is shown below:

Freemedicalbooks4download
Which of the following is most consistent with the clinical presentation and ECG?
A. Acute pericarditis
B. Acute pulmonary embolism
C. Aortic dissection
D. ST-elevation myocardial infarction

10. A 76-year-old woman with hypertension and diabetes mellitus presents with chest pain
and is found to have an elevated troponin value. She is brought for cardiac catheterization
for presumed type 1 myocardial infarction (MI). At angiography, she is not found to have
any evidence of epicardial coronary disease. She has an echocardiogram that shows
depressed left ventricular ejection fraction of 35% with global hypokinesis.
What should be considered the next appropriate step in her workup?
A. Cardiac magnetic resonance imaging (MRI)
B. Coronary CT angiography
C. Exercise treadmill test with perfusion imaging
D. Supine bicycle stress echocardiogram

11. A 68-year-old man with a history of hypertension, hyperlipidemia, and diabetes presents
with shortness of breath on exertion, worsening in the past several weeks. His primary
care physician calls you to ask about the optimal stress test to order for him. You review
his chart and see that he has a LBBB at baseline and has had a left knee replacement 10
months ago. He has limited exercise capacity per primary care physician documentation.
Which test should you recommend?
A. Dobutamine stress echocardiography
B. Exercise ECG stress test only
C. Exercise stress test with perfusion imaging
D. Pharmacologic stress test with perfusion imaging

12. A 49-year-old man with hypertension, hyperlipidemia, chronic knee pain requiring a cane,
and family history of early coronary artery disease (CAD) presents to the emergency
department (ED) with several days of chest pain. The pain does not reliably occur with
exertion. When it does occur with exertion, it does not reliably resolve with rest and is not
provoked by more intense exercise. Vital signs are within normal limits. His ECG has no
ischemic changes. His serum troponin is below the upper reference limit and his renal
function is normal.
Which of the following choices would be the most reasonable first test to order?
A. Cardiac MRI without stress testing
B. Coronary angiogram
C. Coronary CT angiography
D. Exercise stress test

13. A 72-year-old man with a remote history of MI, hypertension, hyperlipidemia, and
diabetes (last hemoglobin A1c 11.2%) presents with a HF exacerbation. He is found to
have a reduced left ventricular ejection fraction of 34%. He is subsequently taken to the
cardiac catheterization lab for coronary angiography and was found to have obstructive
disease of the right coronary artery and a chronically occluded left anterior descending
artery. Percutaneous revascularization was deferred due to the presence of diabetes with
left anterior descending disease. An evaluation for coronary artery bypass grafting
(CABG) is undertaken and a cardiac MRI is performed.
What is the purpose of the cardiac MRI?
A. Evaluation of cardiac amyloidosis
B. Evaluation of myocardial viability
C. Evaluation of prior myocarditis
D. Evaluation of valvular disease

14. A 68-year-old man with a history of intermittent substernal exertional chest pain over the
past 3 weeks presents for an ECG exercise stress test. He can complete 4 METs before he
stops due to angina. The maximum HR achieved is 112 beats/min, which is 74% of
maximum predicted HR by age. Initial BP is 136/82 mmHg and at peak exercise is 126/80
mm Hg. The ECG obtained 7 minutes into recovery is shown below:

Freemedicalbooks4download
What is the next best diagnostic test?
A. Cardiac MRI
B. Coronary angiography
C. Coronary CT angiography
D. Repeat pharmacologic stress test with perfusion imaging

15. A 72-year-old man with hypertension presented to primary care clinic with complaints of
exertional chest pain that subsided at rest. His home medications included aspirin and
lisinopril. He was referred for an exercise treadmill test. He achieved 90% of his
maximum predicted HR and experienced chest pain at peak exercise. His ECG exhibited
nonspecific changes. Perfusion imaging demonstrated mild lateral wall ischemia with an
estimated left ventricular ejection fraction of 65%.
Which of the following would be the next most appropriate step?
A. Initiate β-blocker and statin
B. Perform a transthoracic echocardiogram
C. Reassure patient with no change in medical therapy
D. Refer for coronary angiography

16. A 57-year-old man is admitted to the cardiac intensive care unit for closer monitoring
following uncomplicated placement of a drug-eluting stent to the proximal left anterior
descending artery after presenting with an anterior ST-elevation myocardial infarction.
The right femoral artery was accessed for the procedure. Three hours after the procedure,
the patient feels a “pop” in his groin after coughing. Thereafter, a rapidly expanding mass
is observed in the right groin.
Which of the following is the most appropriate first step in his management?
A. Administration of blood products
B. Antiplatelet therapy discontinuation
C. CT of the abdomen
D. Manual compression of the access site

17. A 63-year-old woman with a history of hyperlipidemia and CAD for which she underwent
elective placement of a drug-eluting stent for stable ischemic heart disease 7 months ago
presents to routine primary care clinic follow-up. She remains adherent to her dual
antiplatelet therapy with aspirin and clopidogrel. She strongly wishes to undergo elective
right knee replacement, which would require discontinuation of clopidogrel
periprocedurally.
Which of the following would you recommend?
A. A pharmacologic stress test with perfusion imaging can guide safety of clopidogrel
discontinuation.
B. Clopidogrel can be held to permit a knee replacement.
C. Elective knee replacement must be deferred until 12 months of dual antiplatelet
therapy have been completed.
D. The procedure can be performed with an IV cangrelor bridge once clopidogrel has
been discontinued.

18. A 63-year-old man with no known past medical history presents to the ED with 20
minutes of substernal chest pressure radiating to his left arm and occurring at rest. It
resolved spontaneously shortly following his presentation. His ECG shows no ST-segment
or T-wave changes. Serial troponin values are undetectable.
Which of the following is the next appropriate step in his management?
A. A conservative strategy of aspirin, β-blocker, and nitrates, as needed, followed by
noninvasive risk stratification (stress testing) to help determine if coronary
angiography is appropriate, provided the patient remains asymptomatic
B. Discharge the patient home
C. Plan for coronary angiography within 24-72 hours of presentation
D. Urgent angiography with the intent to revascularize

19. A 45-year-old man with no past medical history presents with acute-onset substernal chest
pain. His physical examination reveals a BP of 98/72 mmHg, a HR of 108 beats/min, and
a respiratory rate of 24 breaths/min. His jugular venous pressure (JVP) is elevated.
Cardiovascular examination is notable for tachycardia with an S3 gallop appreciated. He
has basilar rales on posterior auscultation of his lungs and his lower extremities are cool to
the touch. His presenting 12-lead ECG demonstrates inferior ST-segment elevations
prompting urgent coronary angiography.
At this stage, which of the following medications should be avoided?
A. Aspirin
B. Atorvastatin
C. Metoprolol
D. Ticagrelor

20. A 75-year-old woman presents several hours after symptom onset with an anterior ST-
elevation myocardial infarction. She undergoes emergent coronary angiography with
placement of a drug-eluting stent to her proximal left anterior descending artery via the
right femoral artery. She is admitted to the cardiac intensive care unit for monitoring. On
day 3 in the hospital, she becomes acutely hypotensive, requiring escalating vasopressor

Freemedicalbooks4download
support. Physical examination reveals a harsh holosystolic murmur best heard at the left
lower sternal border with a palpable systolic thrill. Her femoral artery access site is
without evidence of a hematoma.
Which of the following is the next best diagnostic step?
A. Measurement of intracardiac pressures by pulmonary artery (PA) catheter
B. Noncontrast CT of the abdomen/pelvis
C. Urgent repeat coronary angiography to assess stent patency
D. Urgent transthoracic echocardiography

21. A 65-year-old man presents to a non–percutaneous coronary intervention–capable hospital


with crushing substernal chest pain that began while shoveling snow. A 12-lead ECG
reveals 2-mm ST-segment elevations in the anterior leads. He is initiated on appropriate
therapy, but requires vasopressor support for evolving cardiogenic shock. Due to the
inclement weather, the nearest percutaneous coronary intervention–capable hospital is
more than 3 hours away.
Which of the following would be the most appropriate management strategies?
A. Administer alteplase within 30 minutes of hospital arrival and proceed with transfer
to percutaneous coronary intervention–capable facility if evidence of failed
reperfusion or reocclusion
B. Administer alteplase within 30 minutes of hospital arrival and proceed with urgent
transfer to percutaneous coronary intervention–capable facility
C. Administer alteplase within 30 minutes of hospital arrival and transfer to
percutaneous coronary intervention–capable facility within 3 to 24 hours
D. Initiate immediate transfer to a percutaneous coronary intervention–capable facility

22. A 61-year-old man presents with unrelenting substernal chest pain that started 30 minutes
ago and is found to have inferior ST-segment elevation and no other ECG changes. He is
hemodynamically stable. He is brought for urgent coronary angiography where he is
found to have an occluded mid-right coronary artery. He is also found to have an 80%
proximal left anterior descending lesion.
In addition to stenting the right coronary artery, what would be the best course of
action?
A. Make arrangements to stent the left anterior descending in the next 45 days
B. Make no further plans for revascularization
C. Perform fractional flow reserve on the left anterior descending lesion
D. Schedule an outpatient stress test to evaluate the left anterior descending

23. A 78-year-old woman with hypertension presented with non–ST-elevation myocardial


infarction and underwent placement of a drug-eluting stent to her right coronary artery.
Her lipoprotein cholesterol (LDL-C) on a high-intensity statin is 105 mg/dL.
What, if any, changes should be made to her lipid-lowering regimen prior to
discharge?
A. Add a bile sequestrant
B. Add a fibrate
C. Add ezetimibe
D. Continue high-intensity statin alone
24. An 83-year-old woman with a history of hyperlipidemia presented with non–ST-elevation
myocardial infarction and declined coronary arteriography. She was managed
conservatively and initiated on a moderate- to high-intensity statin and a β-blocker.
Her antiplatelet regimen initially should ideally include which of the following
drugs?
A. Aspirin
B. Prasugrel
C. Ticagrelor
D. A and B
E. A and C

25. A 56-year-old man presents with recurrent episodes of substernal chest pain for 15
minutes at rest accompanied by ST-segment depressions on ECG that resolve after
administration of sublingual nitroglycerin. Acute myocardial injury is demonstrated with a
rise and fall of troponin. His serum creatinine is normal.
In addition to appropriate pharmacotherapy, what is the next best strategy for him?
A. CT coronary angiography
B. Inpatient stress test
C. Invasive coronary angiography
D. Outpatient stress test

26. You now consider the timing of angiography for the above patient. What would be the
optimal timing?
A. Immediately
B. Within 24 hours
C. Within 72 hours

27. A 45-year-old woman with a history of migraines presents with acute-onset crushing
substernal chest pain. ECG demonstrates marked anterior ST-segment elevations. After 10
minutes, the patient’s pain abruptly resolves following administration of nitroglycerin and
the ECG returns to normal. Coronary angiography is performed and shows minimal
evidence of obstructive CAD.
Which therapy is most likely to help mitigate the risk of the patient experiencing
recurrent symptoms?
A. Aspirin
B. Clopidogrel
C. Diltiazem
D. Metoprolol

28. A 57-year-old man with hypertension, hyperlipidemia, and diabetes presents with
retrosternal chest pressure radiating to the jaw that began at rest and has now lasted for 1
hour. His presenting troponin is just above the upper limit of normal. The presenting ECG
is shown below:

Freemedicalbooks4download
Which of the following would be the first next best step?
A. CT to assess for pulmonary embolism
B. Posterior leads (leads V7-V9)
C. Right-sided precordial leads (V4R-V6R)
D. Serial troponin measurements

29. A 60-year-old man with diabetes is hospitalized with non–ST-elevation myocardial


infarction and undergoes placement of a drug-eluting stent in his right coronary artery. He
has no other medical problems and is without complaints. His current medication regimen
includes aspirin, clopidogrel, metoprolol, metformin, and atorvastatin. He is at low risk of
bleeding. The patient inquires when he can stop clopidogrel.
What would be your recommendation?
A. Stop clopidogrel at 3 months because it is a drug-eluting stent
B. Stop clopidogrel at 6 months because it is a drug-eluting stent
C. Stop clopidogrel now and continue aspirin monotherapy
D. Take clopidogrel for at least 1 year

30. You are called to the bedside of a 52-year-old man with a history of coronary disease who
is in the intensive care unit (ICU) with hypotension on day 7 of his hospitalization. The
patient is intubated and warm on examination. He underwent placement of a pulmonary
artery (PA) catheter for worsening hemodynamic instability. As you are waiting for labs,
imaging, echo, and ECG, you obtain intracardiac pressures from the PA catheter, which
show central venous pressure 4, PA 28/12, and pulmonary capillary wedge pressure 8 (all
in mmHg). Cardiac index by Fick is 4.2 L/min.
What is the most likely etiology of his shock?
A. Cardiogenic shock
B. Distributive shock
C. Hypovolemic shock
D. Obstructive shock
31. A 53-year-old woman with severe symptomatic mitral regurgitation secondary to mitral
valve prolapse is admitted to the cardiac intensive care unit. A pulmonary artery (PA)
catheter is inserted to understand her intracardiac hemodynamics. Her mean pulmonary
capillary wedge pressure is 20 mmHg.
What would you expect her mean left atrial pressure and left ventricular end-
diastolic pressure (LVEDP) to be in comparison to the wedge pressure?
A. Left atrial—higher; LVEDP—higher
B. Left atrial—higher; LVEDP—same
C. Left atrial—lower; LVEDP—lower
D. Left atrial—same; LVEDP—lower

32. A 61-year-old man with HFrEF (20%) is admitted to the ICU for cardiogenic shock. He is
started on dobutamine at 2 μg/kg/min and started on a lasix drip at 10 mg/h based on his
clinical examination. His HR is 110 beats/min and his BP is 110/70 mmHg. Later that
day, a pulmonary artery (PA) catheter is placed and his relevant filling pressures are:
central venous pressure 12, PA 50/26, pulmonary capillary wedge pressure 24 (all in
mmHg), and systemic vascular resistance 1630 dynes/s/cm−5. Cardiac index by
thermodilution is 1.8 L/min/m2 (cardiac output 3.5 L/min).
Which of the following changes would you make first?
A. Add norepinephrine
B. Add vasopressin
C. Increase dobutamine
D. Increase lasix drip dose

33. An 88-year-old woman is admitted to the ICU for altered mental status, fever, and poor
urinary output. She is found to have a depressed EF with a low cardiac output and is
started on dobutamine. Her BP continues to trend downward and a pulmonary artery (PA)
line is placed. Her BP is 90/45 mmHg (MAP 60 mmHg) on low-dose norepinephrine with
decreasing urine output. Her lactate is 4.5 mg/dL. Her central venous pressure is 11
mmHg and pulmonary capillary wedge pressure 12 mmHg. Her cardiac output on
inotropes is 4 L/min, CI 1.8 L/min/m2, and systemic vascular resistance 1000 dynes/s/cm
−5
.
How would you classify her shock?
A. Mixed—cardiogenic and distributive
B. Purely cardiogenic
C. Purely distributive
D. Tamponade

34. A 79-year-old woman presented late with an anterior MI and underwent primary
percutaneous coronary intervention. She was in cardiogenic shock; a pulmonary artery
(PA) catheter was placed, she was started on medium-dose dobutamine and was admitted
to the cardiac intensive care unit. Her hemodynamics began to slowly improve. On the
third day in the hospital, her BP and urine output decreased and her extremities were
cooler. Conversely, her mixed venous saturation(MVO2) markedly increased from 65% to
85%.
What is the next best step?
A. Add norepinephrine

Freemedicalbooks4download
B. Continue to monitor her MVO2 saturation
C. Decrease the dobutamine
D. Emergent transthoracic echocardiogram

35. A 54-year-old man transfers his care to your primary care practice and you are seeing him
for an initial visit to establish care. He has a history of ischemic cardiomyopathy
following an MI one year ago and has shortness of breath with exertion. A transthoracic
echocardiogram from 1 year ago revealed a left ventricular ejection fraction of 27%. He
underwent implantation of an implantable cardiac defibrillator 10 months ago. You review
his medication regimen with him.
Which of the following medications has NOT been shown to reduce mortality in
patients with symptomatic HF with reduced left ventricular function?
A. Carvedilol
B. Furosemide
C. Lisinopril
D. Spironolactone

36. A 75-year-old woman with HF with preserved EF and stage 3 chronic kidney disease
presents to the ED with 2 weeks of progressive weight gain, increased abdominal girth,
and orthopnea. She has been using escalating doses of diuretics at home without relief of
her symptoms (original home dose furosemide 20 mg daily, now taking 40 mg twice
daily). Vital signs on presentation reveal a HR of 88 beats/min, BP of 146/78 mmHg, and
oxygen saturation of 94% on room air. Physical examination is notable for bibasilar
crackles and JVD, and lower extremities are warm with edema graded 2+ to the thighs.
Laboratory evaluation reveals acute kidney injury with a serum creatinine of 1.7 mg/dL
(baseline 1.2 mg/dL 2 months prior).
What is the next most appropriate step in care?
A. Administer intravenous crystalloid
B. Administer intravenous furosemide
C. Administer oral furosemide
D. Initiate inotropes in the cardiac intensive care unit, given concern for cardiogenic
shock

37. A 60-year-old woman presents to the ED with an acute anterior MI complicated by


cardiogenic shock. She has a history of hypertension, type 2 diabetes, ischemic
cardiomyopathy, AF on warfarin, ascending aortic aneurysm, moderate aortic
regurgitation, and severe mitral regurgitation. Vital signs reveal a HR of 102 beats/min,
BP of 80/50 mmHg, and oxygen saturation of 86% on room air. Her examination is
notable for normal heart sounds, cool distal extremities, and 1+ lower extremity edema.
The cardiac catheterization lab is activated and will be ready in 30 minutes. Your intern
asks if an intra-aortic balloon pump would be beneficial for temporary support. You
review the mechanism of a balloon pump.
Which of the following is NOT a benefit of an intra-aortic balloon pump?
A. Decreased aortic regurgitation
B. Decreased myocardial oxygen demand
C. Increased cardiac output
D. Increased coronary perfusion
38. A 32-year-old man with an idiopathic dilated cardiomyopathy (left ventricular ejection
fraction 18%) is admitted for shortness of breath. On examination, his JVP is 14 cm, he
has bibasilar crackles, and has 2+ pitting edema in his lower extremities. His lower
extremities are warm. His lactate is 1.2 mg/dL, liver function tests are normal, creatinine
is 0.9 mg/dL, and urine output is 100 mL/h over the past 6 hours. BP is 90/60 mmHg.
Which of the following therapies would you start in him?
A. Diuresis + inotropes
B. Diuresis + vasopressors
C. Diuresis alone
D. Inotropes alone

39. A 66-year-old woman with long-standing hypertension and obstructive sleep apnea is
admitted to the hospital with dyspnea on exertion and a 30 lb weight gain. Physical
examination reveals a pulse of 90 beats/min, BP of 175/85 mmHg, JVP of 12 cm,
bibasilar crackles, and 2+ lower extremity edema to her shins. Laboratory testing reveals a
creatinine of 1.2 mg/dL. Transthoracic echocardiography reveals a left ventricular ejection
fraction of 60% without wall motion abnormalities and normal valvular function. She is
admitted to the cardiology service and is diuresed. Her BP is effectively treated with
amlodipine.
What medication can be started in this patient to reduce the risk of
rehospitalization for HF?
A. Aspirin
B. Lisinopril
C. Sildenafil
D. Spironolactone

40. A 56-year-old man with an idiopathic dilated cardiomyopathy (left ventricular ejection
fraction 15%) had stopped all his medications due to financial constraints and is admitted
for acute decompensated HF with pulmonary edema and elevated serum creatinine and
lactate. His BP is 85/50 mmHg. Empiric dobutamine and low-dose norepinephrine are
started. A pulmonary artery (PA) line is placed, his filling pressures are high, and his
cardiac index on medium-to-high-dose dobutamine is 1.5 L/min/m2. He is anuric despite
diuretics, and his lactate remains elevated.
What is the next best step?
A. Increase the inotropes and diuretics
B. Increase the norepinephrine
C. Insert a percutaneous left ventricular assist device (eg, Impella or Tandem Heart)
D. Insert an intra-aortic balloon pump

41. A 21-year-old college student is referred to the ED for generalized fatigue and dyspnea.
His presenting systolic BP is 60 mmHg and his lower extremities are cool. A bedside
ultrasound reveals severely impaired left and right ventricular function. His high-
sensitivity troponin T level is very elevated at 2320 ng/L. A rapid influenza test
subsequently returns positive. A pulmonary artery (PA) catheter is inserted and he is
initiated on dobutamine, norepinephrine, and vasopressin for hemodynamic support. Over
the subsequent 6 hours, the patient’s pharmacologic support is escalated to maximal
ionotropic and vasopressor support with marginal hemodynamics.

Freemedicalbooks4download
Which of the following would be the next most appropriate steps in management?
A. Emergency coronary arteriography
B. Initiation of venoarterial extracorporeal membrane oxygen
C. Insertion of a percutaneous ventricular assist device
D. Insertion of an intra-aortic balloon pump

42. A 33-year-old man presents to the ED with 1 day of fever, myalgias, and dyspnea. He has
a history of a long-standing polysubstance use disorder, including alcohol, cocaine, and
intravenous heroin. Over the last 6 months, he has not been able to climb a flight of stairs
without experiencing shortness of breath. Vital signs are notable for sinus tachycardia
with HRs in the 110 to 120 beats/min range.
An influenza A nasal swab returns positive. A chest radiograph shows interstitial
pulmonary opacities and bilateral pleural effusions. A transthoracic echocardiogram is
obtained and shows a dilated cardiomyopathy with a left ventricular ejection fraction of
18%. He undergoes a thorough investigation for causes of his cardiomyopathy. Coronary
angiography reveals no significant CAD. A cardiac MRI does not suggest acute
myocarditis. Laboratory testing results include a negative human immunodeficiency virus
(HIV) test, a normal serum protein electrophoresis, normal iron studies, an antinuclear
antibody (ANA) titer positive at 1:40, and a normal thyroid-stimulating hormone (TSH).
He is started on oseltamivir and furosemide with significant improvement in symptoms.
He is seen by psychiatry and is committed to abstinence from substance use.
Which of the following ia left ventricular ejection fractionhe next best step
regarding management of his cardiomyopathy?
A. Discharge on oseltamivir and furosemide with plans to repeat transthoracic
echocardiogram in 1 month; if EF is still depressed, initiation of neurohormonal
blockade
B. Initiation of angiotensin-converting enzyme (ACE) inhibitor and β-blocker; repeat
transthoracic echocardiogram in 3 months
C. Myocardial biopsy to determine etiology of cardiomyopathy
D. Placement of an implantable cardioverter-defibrillator prior to discharge

43. A 52-year-old woman with hypertrophic obstructive cardiomyopathy and a left ventricular
outflow gradient of 60 mmHg is admitted to the ICU with septic shock. Two liters of
intravenous crystalloid and broad-spectrum antibiotics are administered. Central and
arterial lines are placed at the bedside. Central venous pressure is now 12 mmHg. The
mean arterial pressure is 55 mmHg and the intensivist recommends initiation of
vasopressors for BP augmentation. She is started on norepinephrine, and despite rapid
dose escalation, her BP continues to decline.
What is the next best step in this patient’s care?
A. Administer additional intravenous crystalloid
B. Discontinue norepinephrine and initiate phenylephrine
C. Start dobutamine in addition to norepinephrine
D. Start trial of diuretics

44. A 72-year-old man presents to the ED with bilateral leg swelling. He has a history of
hypertension, AF on warfarin, multiple myeloma, stage 3 chronic kidney disease, and
Parkinson’s disease. Oxygen saturation on room air is 86%, which improves to 94% with
2 L of supplemental oxygen by nasal cannula. Physical examination is notable for JVP to
14 cm and pitting edema to the knees. Cardiac examination reveals a II/VI mid-systolic
crescendo-decrescendo murmur at the right upper sternal border and an S4 gallop.
Pulmonary examination reveals bilateral crackles at the lung bases.
A chest radiograph reveals interstitial pulmonary opacities and a right pleural
effusion. NT-proBNP is 8000 pg/mL (previously 500 pg/mL). An ECG is notable for low
voltages and diffuse T-wave abnormalities. A transthoracic echocardiogram from 1 year
ago reveals a left ventricular ejection fraction of 66%, biventricular wall thickening, an
enlarged left atrium, and evidence of diastolic dysfunction.
What is the most likely etiology of his diastolic HF?
A. Amyloidosis
B. Hypertrophic cardiomyopathy
C. Long-standing hypertension
D. Sarcoidosis

45. A 32-year-old man with a history of hypertrophic obstructive cardiomyopathy presents to


your clinic. He feels well today and has no new symptoms. His echocardiogram reveals a
left ventricular ejection fraction of 75% with increased left ventricle (LV) wall thickness
of 15 mm (normal 7-11 mm) and systolic anterior motion of his mitral valve. He recently
was reading information about his cardiac disease online and is worried about the risk of
sudden cardiac death.
Which of the following is NOT a high-risk feature for sudden cardiac death in
patients with hypertrophic obstructive cardiomyopathy?
A. Family history of sudden cardiac death
B. LV wall thickness of 15 mm
C. Nonsustained ventricular tachycardia
D. Unexplained syncope

46. A 58-year-old woman with hypertension and hyperlipidemia presents to the ED with chest
pressure and shortness of breath. Her husband died unexpectedly 2 days ago. Her ECG
shows ST elevations in leads V2 to V6 and her high-sensitivity troponin T is slightly
elevated to 28 ng/L. A transthoracic echocardiogram reveals an akinetic apex with a
hyperkinetic base. Left ventricular ejection fraction is 34%.
Which of the following is the next best step?
A. Coronary angiography
B. Observation for 24 hours with serial troponin and then discharge if no arrhythmias or
hemodynamic instability
C. Reassurance, discharge, and repeat transthoracic echocardiogram in 1 to 2 weeks
D. Trial of antacid and anxiolytic

47. A 56-year-old man presents with focal neurologic deficits and was subsequently found to
have a stroke involving the left middle cerebral artery (MCA). As part of his stroke
workup, he undergoes a transthoracic echocardiogram. Echocardiogram reveals severe
biventricular dysfunction with dilated left ventricular ejection fraction of 25%. There are
no regional wall motion abnormalities noted. An ischemia evaluation is unrevealing. On
further history, the patient reports drinking six 4 oz alcoholic drinks per night. During the
hospitalization, he was found to have paroxysmal AF with rapid ventricular response

Freemedicalbooks4download
lasting 5 minutes and terminating spontaneously.
Which of the following is the best advice for the patient regarding his alcohol use?
A. Absolute abstinence from alcohol
B. Limiting alcohol use to less than 7 drinks per week
C. Limiting alcohol use to less than 14 drinks per week
D. No limitations on alcohol use

48. You are asked to evaluate a 68-year-old man who presents to the ED with syncope. He has
a history of HFrEF, CAD, and type 2 diabetes. He was walking to the bathroom when he
felt light-headed and collapsed, awaking seconds afterward. He noted no chest pain or
palpitations preceding the event. His initial vital signs include a HR of 64 beats/min, BP
of 106/62 mmHg, and oxygen saturation of 96% on room air. His ECG shows normal
sinus rhythm and repolarization abnormalities consistent with left ventricular hypertrophy.
A transthoracic echocardiogram is performed, which reveals a left ventricular ejection
fraction of 30%, thickening and calcification of the aortic valve, valve area of 0.8 cm2, and
mean gradient across the aortic valve of 28 mmHg.
Which of the following is the next best step?
A. Proceed to dobutamine stress echo
B. Proceed to surgical aortic valve replacement
C. Proceed to transcatheter aortic valve replacement
D. Repeat transthoracic echocardiogram in 1 year

49. A 52-year-old woman with no significant past medical history is seen by her primary care
physician and found to have a new murmur. She feels well and maintains an active
lifestyle, running 2 to 3 miles every day without symptoms. Her vital signs reveal a HR of
65 beats/min and BP of 100/60 mmHg. Cardiac auscultation reveals a II/VI systolic mid-
peaking crescendo-decrescendo murmur. A transthoracic echocardiogram is obtained,
which shows a bicuspid aortic valve with aortic valve area of 1.2 cm2 and mean gradient
of 32 mmHg.
What is the next best step regarding her aortic valve disease?
A. Consult cardiothoracic surgery for aortic valve replacement during this admission
B. Outpatient referral for aortic valve replacement
C. Repeat transesophageal echocardiogram (TEE) next month
D. Repeat transthoracic echocardiogram every 1 to 2 years

50. A 69-year-old man with hypertension presents to cardiology clinic with dyspnea on
exertion for 1 month. He subsequently undergoes invasive coronary angiography, which
shows nonobstructive CAD, and a transthoracic echocardiogram, which shows severe
aortic stenosis. He has a trileaflet aortic valve. His Society of Thoracic Surgery (STS)
predicted risk of surgical mortality is 2%.
Which of the following is the next best step in management?
A. Heart team approach to determine recommendation for surgical versus transcatheter
aortic valve replacement
B. Referral to cardiothoracic surgery for surgival aortic valve replacement
C. Referral to interventional cardiology for transcatheter aortic valve replacement
D. Trial of medical management prior to aortic valve replacement
51. A 62-year-old obese man presents to the ED with acute onset of dyspnea. He has a history
of smoking, hypertension, and hyperlipidemia. He reports that 3 days prior to admission,
he had substernal chest pain lasting hours that he thought was heartburn, although it was
not alleviated with over-the-counter antacids. Vital signs are notable for HR of 120
beats/min, BP of 70/50 mmHg, and oxygen saturation of 84% on room air. Physical
examination is notable for JVD and normal heart sounds with a I/VI holosystolic murmur
at the apex without thrill. Pulmonary auscultation reveals bilateral crackles halfway up the
lung fields. ECG shows Q waves in II, III, and aVF.
What is the most likely etiology of his presentation?
A. Left ventricular dysfunction
B. Mitral regurgitation
C. Pulmonary embolism
D. Ventricular septal rupture

52. An 81-year-old woman presents to the hospital after a mechanical fall and is found to have
a hip fracture. She has a history of mechanical aortic valve replacement 10 years ago, AF
diagnosed 3 years ago, hypothyroidism, and hypertension. Home medications include
warfarin, amlodipine, and levothyroxine. She has a history of cardiac thromboembolism 6
years ago, after which her international normalized ratio (INR) goal was increased to 2.5
to 3.5. Vital signs reveal HR of 60 beats/min and BP of 120/60 mmHg. Physical
examination is remarkable for left leg external rotation, abduction, and shortening
compared to the right leg. You are consulted for perioperative management of her
anticoagulation. She is on coumadin and has been taking her anticoagulation regularly.
INR on presentation is 2.7.
Which of the following would be recommended?
A. Stop coumadin 2 days prior to surgery and restart 12-24 hours afterward without any
heparin bridge
B. Stop coumadin indefinitely given fall risk
C. Stop coumadin now and bridge with unfractionated heparin
D. Switch from coumadin to dabigatran

53. A 52-year-old woman presents to your clinic for evaluation of a heart murmur. She has a
history of hypertension that is well treated with amlodipine. She reports that she continues
to live an active lifestyle and enjoys running for exercise. She denies any dyspnea or
angina on exertion. Vital signs are notable for HR of 80 beats/min, BP of 110/60 mmHg,
and oxygen saturation of 99% on room air. Physical examination is notable for a regular
rate, normal heart sounds with a II/VI holosystolic murmur at the apex without a thrill.
Echocardiogram reveals a left ventricular ejection fraction of 55%, anterior mitral valve
prolapse with severe mitral regurgitation, and a left ventricular end-systolic dimension
(LVESD) of 41 mm.
What is the next best step in management?
A. Initiate furosemide
B. Order ambulatory ECG monitoring to screen for AF
C. Refer for mitral valve surgery
D. Repeat transthoracic echocardiogram in 6 months

54. A 32-year-old woman in the 22nd week of her first pregnancy presents to your clinic for

Freemedicalbooks4download
evaluation of a heart murmur. She has a history of rheumatic mitral stenosis. Prior to her
pregnancy, she felt well and had no limitations to exercise or daily activity. Over the past
2 weeks, she has had progressive dyspnea with minimal exertion. Vital signs are notable
for HR of 60 beats/min, BP of 120/70 mmHg, and oxygen saturation of 99% on room air.
Physical examination is notable for a regular rate, normal heart sounds with a high-pitched
early diastolic sound followed by a mid-diastolic rumble heard at the apex.
Echocardiogram reveals a left ventricular ejection fraction of 55%, rheumatic mitral
stenosis with mitral valve area of 1.2 cm2, and mean gradient of 11 mmHg.
What is the next best step in management?
A. Initiate metoprolol
B. Initiate warfarin
C. Refer for mitral valve surgery
D. Refer for percutaneous mitral balloon commissurotomy as anatomy allows

55. A 58-year-old woman with a history of breast cancer presents with shortness of breath.
She reports progressive shortness of breath over the past 3 days that has steadily
worsened. Initial vital signs are notable for a HR of 104 beats/min, a BP of 88/64 mmHg,
and a respiratory rate of 28 breaths/min. Physical examination reveals jugular venous
pulsation at 12 cm and muffled heart sounds. You check for a pulsus paradoxus and find
that it is 18 mmHg.
What is the next best step in treatment?
A. Consult cardiothoracic surgery for pericardial window
B. Consult interventional cardiology for pericardiocentesis
C. Give 500 mL fluid bolus
D. Obtain a CT scan to evaluate for pulmonary embolism

56. A 72-year-old woman with a history of hypertension, AF, and breast cancer treated with
resection and radiation presents with abdominal pain, lower extremity edema, weight gain,
and shortness of breath. She reports no chest pain. Her vital signs are notable for a HR of
106 beats/min, a BP of 98/64 mmHg, a respiratory rate of 20 breaths/min, and oxygen
saturation of 92% on room air.
Examination is notable for lungs that are clear to auscultation bilaterally. The
jugular venous pulsation is noted to rise with inspiration. Cardiac auscultation reveals an
irregularly irregular rhythm without murmurs, rubs, or gallops. Abdominal examination
reveals hepatomegaly with shifting dullness concerning for ascites and bilateral lower
extremity edema.
The 12-lead ECG demonstrates low voltages. Transthoracic echocardiography
reveals increased inspiratory flow across the tricuspid valve with decreased inspiratory
flow across the mitral valve. No effusion is present. The patient is sent for simultaneous
left and right heart catheterization and is found to have discordant left ventricular and
right ventricular peaks during the respiratory cycle as well as equalization of the right
ventricular and left ventricular end-diastolic pressure.
What are these findings most consistent with?
A. Acute viral pericarditis
B. Constrictive pericarditis
C. Restrictive cardiomyopathy
D. Severe pulmonary hypertension
57. A 32-year-old man presents with sharp 8/10 substernal chest pain. The pain started a few
hours ago and is worse with inspiration. He reports a prodromal episode of rhinorrhea and
cough about 7 days prior to presentation. Vital signs are notable for a HR of 88 beats/min,
a BP of 120/74 mmHg, and an oxygen saturation of 100% on room air. A troponin test is
drawn and in process. A 12-lead ECG is shown below:

What is the most appropriate next step in management?


A. Emergent coronary angiography
B. Initiation of high-dose nonsteroidal anti-inflammatory drugs (NSAIDs)
C. Initiation of steroids
D. Pharmacologic stress testing with perfusion imaging

58. A 52-year-old man presents to your clinic for his annual physical. He is an active smoker
with a history of chronic obstructive pulmonary disease and gastroesophageal reflux
disease. His vital signs are notable for a HR of 65 beats/min and BP of 135/85 mmHg.
Looking through his history, you note that his BP has ranged from 130/80 to 140/85
mmHg for the last 2 years. The patient asks you whether he should be on medications for
his BP.
In addition to counseling him on diet and smoking cessation, you recommend
which of the following?
A. Calculating his atherosclerotic cardiovascular disease risk score and, if >10%,
starting an antihypertensive agent now
B. Monitoring BP every 6 months and starting antihypertensives when BP >140/90
mmHg
C. Monitoring BP every 12 months and starting antihypertensives when BP >150/90
mmHg
D. Starting an antihypertensive medication now

59. A 50-year-old woman presents to your clinic to reestablish longitudinal care after not

Freemedicalbooks4download
seeing a physician in 10 years. Her vital signs are notable for a BP of 148/92 mmHg. Her
physical examination is otherwise unremarkable. Laboratory evaluation is notable for
hemoglobin A1c of 8.4% and creatinine of 1.2 mg/dL. You have her return for a repeat BP
check and urine sample. Repeat BP is 150/94 mmHg and urine microalbumin/creatinine
ratio is 45 mg/g.
Which of the following antihypertensives would you recommend?
A. Amlodipine
B. Carvedilol
C. Hydrochlorothiazide
D. Lisinopril

60. A 65-year-old man presents to the ED with a complaint of tearing chest pain radiating to
the back. He has a history of poorly controlled hypertension, hyperlipidemia, and CAD
requiring coronary artery bypass graft surgery 5 years ago. Initial vital signs reveal a HR
of 110 beats/min, BP of 190/110 mmHg, and oxygen saturation of 94% on room air. The
remainder of his physical examination is unremarkable. Workup includes a CT
angiography (CTA) that shows a large aortic dissection distal to the takeoff of the left
subclavian artery.
Which of the following agents should be given first to control his BP?
A. Intravenous hydralazine
B. Intravenous labetalol
C. Intravenous nitroprusside
D. Oral labetalol

61. A 32-year-old obese woman presents to your clinic for an annual visit. She has a history
of hypertension that is well controlled on lisinopril. She tells you that she is trying to
conceive.
In addition to prescribing her a prenatal multivitamin, which change would you
make to her antihypertensive regimen?
A. Stop lisinopril and monitor BPs as you would expect her BP to decrease during
pregnancy
B. Substitute hydrochlorothiazide for lisinopril
C. Substitute labetalol for lisinopril
D. Substitute spironolactone for lisinopril

62. A 48-year-old man presents as a new patient to your clinic. He reports always having high
BP. His BP today is 150/90 mmHg. His current medication regimen is amlodipine 10 mg
daily, lisinopril 40 mg daily, and carvedilol 25 mg BID.
Which of the following medications should be added to his regimen?
A. Chlorthalidone
B. Clonidine patch
C. Doxazosin
D. Hydralazine

63. A 29-year-old woman with hypertension presents to your clinic for a follow-up visit. Over
the last few months, her BP medications have been uptitrated, with only mild
improvement in her BP. Her BP today is 168/92 mmHg. On physical examination, you
auscultate an abdominal bruit. She has a normal serum creatinine.
Which of the following is the next best diagnostic test to order?
A. MR angiography (MRA) of the renal arteries
B. Noncontrast CT of the thoracic and abdominal aorta
C. Serum metanephrines
D. Sleep apnea testing

64. A 63-year-old man with hypertension, hyperlipidemia, and a smoking history is


incidentally discovered to have a thoracic aortic aneurysm (TAA) measured at 4 cm by
CT of the chest.
When should the next assessment of aneurysm size be performed?
A. 3 months
B. 6 months
C. 12 months
D. 24 months

65. A 45-year-old man with no past medical history undergoes a transthoracic


echocardiogram for an incidentally heard murmur. A bicuspid aortic valve is identified
with a thoracic aortic aneurysm (TAA) measured at 4.5 cm.
Which of the following recommendations would you offer the patient with regard
to screening of family members?
A. All first-degree male relatives should be screened.
B. All first-degree relatives should be screened.
C. Family members previously informed of a murmur should be screened.
D. No additional screening is required.

66. A 64-year-old woman with hypertension, hyperlipidemia, and coronary disease has an
abdominal aortic aneurysm diameter measured at 4.9 cm that has increased in size by 0.6
cm in the preceding 6 months.
Which feature of her history merits consideration for early surgical repair of her
aortic aneurysm?
A. Aneurysm size of 4.9 cm
B. Female sex
C. Rate of growth of >0.5 cm in 6 months
D. Surgical repair is not indicated

67. A 78-year-old man with a history of hypertension and hyperlipidemia presents with chest
pain. The pain initially started suddenly, radiated to his back, and was rated a 10 on a
scale of 1 to 10. His pain is now an 8/10. He is diaphoretic and appears in distress. Initial
vital signs are notable for a HR of 112 beats/min, BP of 168/72 mmHg, and oxygen
saturation of 92% on room air. He undergoes emergent CTA of the chest, which reveals a
large ascending aortic dissection. The cardiothoracic surgical team is consulted.
What is the first step in medication management?
A. Esmolol drip
B. Metoprolol IV push
C. Nitroglycerin drip
D. Nitroprusside drip

Freemedicalbooks4download
68. A 65-year-old man with a history of tobacco use disorder, hypertension, and
hyperlipidemia presents with sudden-onset ripping chest pain that radiates to his back.
Pain was maximal at onset and has subsequently decreased to an 8/10. He is diaphoretic
and in moderate distress on arrival. Vital signs are notable for sinus tachycardia at a rate
of 110 beats/min, a BP of 172/94 mmHg, and an oxygen saturation of 98% on room air. A
chest x-ray is obtained and is shown below:

The chest x-ray is concerning for which of the following pathologies?


A. Aortic dissection
B. Pericardial effusion
C. Pneumothorax
D. Pulmonary edema

69. A 68-year-old man is admitted to the cardiac intensive care unit following an inferior-
posterior ST-elevation myocardial infarction. He is currently supported by an intra-aortic
balloon pump and norepinephrine. On reassessment 2 hours later, his left radial arterial
line has lost its tracing and the radial and brachial pulse is difficult to palpate. Right radial
and brachial artery pulses are intact and easily palpable. By pulmonary artery (PA)
catheter, cardiac output and cardiac index are unchanged from prior.
What is the most likely diagnosis?
A. Aortic dissection
B. Radial artery spasm
C. Thromboembolism
D. Worsening cardiogenic shock

70. A 38-year-old man presents with acute-onset 10/10 chest pain radiating to his back. Initial
BP is 190/110 mmHg. ECG is without ischemic changes. Examination is notable for a tall,
slender gentleman in moderate distress. Pectus excavatum is noted, and on cardiac
examination he is noted to have a III/IV early diastolic murmur best heard at the right
upper sternal border. Lungs are notable for bibasilar rales, with equal breath sounds
bilaterally. A CTA chest is in process.
Based on the above presentation, what will be the most likely definitive treatment?
A. Coronary angiogram
B. Medical management alone
C. Needle decompression
D. Surgery

71. A 74-year-old man with a history of tobacco use disorder, hypertension, and
hyperlipidemia is admitted to the ICU for hemodynamic monitoring for a type B aortic
dissection extending from T4 to the diaphragmatic hiatus. His current medications include
an esmolol drip and nitroprusside. The patient is noted to have diminishing urine output.
The nitroprusside and esmolol doses have been reduced to maintain systolic BP of 100 to
120 mmHg and HR <60 beats/min. Repeat labs are obtained and shown below:

Creatinine: 1.8 mg/dL (0.9 mg/dL previously)


Lactate: 4.2 mmol/L (1.8 mmol/L previously)
Arterial blood gas: pH: 7.25, PCO2: 28 mmHg, PO2: 98 mmHg, FiO2 30%

What is the most likely etiology of these hemodynamic changes?


A. Cyanide toxicity
B. Extension of the dissection below the diaphragmatic hiatus
C. Extension of the dissection proximally into the ascending aortic arch
D. Sedation effect

72. You evaluate a 22-year-old man who presents with intermittent fevers and
lightheadedness. Approximately 4 weeks earlier, he had been hiking in Massachusetts and
shortly thereafter noted a rash on his left thigh that resolved on its own. Vital signs reveal
temperature of 38.5°C (101.3°F), HR of 25 beats/min, and BP of 80/55 mmHg. Cardiac
examination is only notable for bradycardia. The remainder of the examination is
unremarkable. ECG reveals complete heart block with a slow junctional escape rhythm at
25 beats/min. He receives appropriate IV fluid resuscitation; however, his BP remains
80/55 mmHg.
What is the most appropriate acute management of this arrhythmia?
A. IV ceftriaxone
B. Oral doxycycline
C. Permanent pacemaker implantation
D. Transvenous temporary pacemaker

73. You are asked to evaluate a 68-year-old man for an abnormal ECG. He has a history of
chronic obstructive pulmonary disease with reasonable symptom control on inhaled
corticosteroids and long-acting β-agonists. Vital signs reveal an irregular HR of 101
beats/min and BP of 122/70 mmHg. Cardiopulmonary examination is otherwise
unremarkable. ECG reveals an irregular rhythm with three distinct P-wave morphologies.
What is the most likely mechanism of this arrhythmia?
A. Chaotic atrial activation originating from the pulmonary veins
B. Increased automaticity at multiple sites in the atria

Freemedicalbooks4download
C. Reentry through dual pathways in the atrioventricular node
D. Reentry through the atrioventricular node and an accessory pathway

74. You are asked to evaluate a 44-year-old man with a known history of ventricular
preexcitation (Wolff-Parkinson-White [WPW]) in the ED for palpitations that abruptly
started an hour prior to presentation. Vital signs reveal an irregular HR of 120 beats/min
and BP of 120/65 mmHg. ECG reveals AF with preexcitation.
What is the appropriate initial therapy for management of this arrhythmia?
A. Adenosine
B. Digoxin
C. Emergent direct current cardioversion
D. Procainamide

75. You are called emergently to the bedside after a 54-year-old healthy woman developed
unstable polymorphic ventricular tachycardia. She was admitted to the hospital 3 days
earlier for treatment of a right lower lobe pneumonia and had been improving on broad-
spectrum antibiotics. Prior to this hospitalization, she had been healthy and performed
moderate- to high-intensity aerobic exercise for at least 40 minutes a day. After emergent
defibrillation, she regains spontaneous circulation. An ECG is obtained.
Which of the following is most likely to be present on the ECG?
A. LBBB
B. Long QT interval
C. Pseudo-RBBB with ST-segment elevation in V1 to V3
D. T-wave inversion in V1 to V3

76. A 54-year-old woman is admitted with unstable monomorphic ventricular tachycardia


prompting defibrillation from her implantable cardiac defibrillator. She has a history of an
ischemic cardiomyopathy (left ventricular ejection fraction 20%) and three
hospitalizations for monomorphic ventricular tachycardia prompting implantable cardiac
defibrillator discharge in the past 2 months. Her most recent coronary angiogram last
month reveals nonobstructive disease. Current medications include aspirin, atorvastatin,
metoprolol, amiodarone, mexiletine, and furosemide. Vital signs reveal temperature of
37°C (98.6°F), HR of 75 beats/min, and BP of 110/60 mmHg. Cardiac examination
reveals a laterally displaced apical impulse and normal heart sounds without murmurs.
There is no JVD, lung sounds are clear, and there is no lower extremity edema.
What is the next most appropriate step in care?
A. Flecainide
B. Intra-aortic balloon pump
C. IV magnesium
D. Radiofrequency ablation

77. A 65-year-old man with a history of recurrent urinary tract infections presents to the
hospital with pyelonephritis. Overnight, you are called by his nurse because he is
bradycardic with an HR of 50 beats/min. BP is 95/60 mmHg. The following ECG is
obtained:
What is the next best step?
A. Adenosine
B. Atropine
C. Observation
D. Temporary pacemaker

78. A 21-year-old man with no medical history presents to the ED with palpitations. The
following ECG is obtained:

What is the most likely diagnosis?


A. AF
B. Atrial flutter with 2:1 block
C. Atrioventricular nodal reentrant tachycardia

Freemedicalbooks4download
D. Sinus tachycardia

79. You are asked to evaluate a 66-year-old woman with a history of hypertension in the ED
for palpitations during the past several days. Vital signs reveal a HR of 150 beats/min and
BP of 100/60 mmHg. Cardiac examination is notable for a rapid irregular heart rhythm
without murmurs or gallops. The remainder of the examination is unremarkable. ECG
reveals new onset of AF with rapid ventricular response.
She is admitted to the hospital and her ventricular response rates are controlled to a
range of 110-130 beats/min after atrioventricular nodal blockade, but her low BP limits
further titration. Transthoracic echocardiogram reveals mild left atrial dilation and left
ventricular ejection fraction of 40%. After thorough investigation, the etiology of her
systolic dysfunction is determined to be tachycardia-induced cardiomyopathy.
What is the appropriate management of this arrhythmia?
A. Continue rate control with current regimen
B. Perform cardioversion without transesophageal echocardiogram (TEE)
C. Perform transesophageal echocardiogram (TEE); if no left atrial thrombus, perform
cardioversion
D. Refer for ablation

80. You are asked to evaluate a 69-year-old woman with a history of an ischemic
cardiomyopathy in the cardiac intensive care unit. Vital signs reveal a HR of 125
beats/min, BP of 95/65 mmHg, and oxygen saturation of 92% on 3 L of supplemental
oxygen. There is JVD. Cardiac examination is notable for a rapid, irregular heart rhythm
with an S3 gallop. Pulmonary examination reveals diffuse crackles. ECG reveals new
onset of AF with rapid ventricular response. Her medications include aspirin, atorvastatin,
metoprolol, lisinopril, and torsemide.
She is admitted to the hospital and started on unfractionated heparin and
intravenous furosemide. Her respiratory symptoms improve by day 3 of hospitalization.
Transthoracic echocardiogram reveals left atrial dilation and left ventricular ejection
fraction of 30%. After consultation from the cardiology team, a rhythm control strategy is
recommended for long-term management of her AF. The team would like to initiate drug
therapy prior to consideration of cardioversion to help maintain long-term sinus rhythm.
What is the most appropriate medical therapy?
A. Amiodarone
B. Diltiazem
C. Flecainide
D. Propafenone

81. A 52-year-old woman presents to your clinic for a yearly physical examination. She has a
history of hypothyroidism and rheumatic mitral stenosis. She reports that she has felt well,
but over the recent months she notes intermittent episodes of palpitations. Medications
include levothyroxine. Vital signs reveal a HR of 90 beats/min and BP of 105/55 mmHg.
You perform an ECG in the office, which reveals new onset of AF. Her echocardiogram
demonstrates moderate rheumatic mitral stenosis with a preserved left ventricular ejection
fraction.
Which of the following is the best choice regarding a decision about
anticoagulation?
A. Do not start anticoagulation
B. Start aspirin
C. Start rivaroxaban
D. Start warfarin

82. You are asked to evaluate a 77-year-old woman in the ED after a fall. She has a history of
CAD, hypertension, and AF. She reports that she was vacuuming in her home when she
tripped on the edge of her rug and fell, landing on her outstretched arms and hitting her
head against the coffee table. Medications include aspirin, metoprolol, furosemide,
lisinopril, apixaban, and a multivitamin. Vital signs reveal a regular HR of 90 beats/min
and BP of 105/55 mmHg. Cardiac and neurologic examinations are unremarkable. CT of
the head is performed that reveals a large subdural hemorrhage. She is evaluated by
neurosurgery and is planned to go to the operating room for surgical evacuation.
What is the next most appropriate pharmacologic therapy?
A. Andexanet alfa
B. Fresh frozen plasma
C. Idarucizumab
D. Vitamin K

83. A 43-year-old woman with a history of rheumatic mitral stenosis and AF returns to your
office for routine follow-up. Her current medications include metoprolol and warfarin.
She reports having seen an advertisement on television for apixaban and would like to
learn more.
Which of the following statements about direct-acting oral anticoagulants is true?
A. Direct-acting oral anticoagulants are indicated for long-term antithrombotic therapy
for patients with mechanical heart valves.
B. Direct-acting oral anticoagulants are not indicated for patients with AF with
rheumatic mitral stenosis.
C. Direct-acting oral anticoagulants carry a higher risk of intracranial hemorrhage
compared to warfarin.
D. There are no reversal agents available for direct-acting oral anticoagulants in case of
life-threatening bleeding.

84. A 70-year-old woman has a history of long-standing, permanent AF. She also has amyloid
angiopathy and is now admitted with a large intracerebral hemorrhage.
What option should be considered to reduce her risk of stroke or systemic
embolism?
A. Amiodarone
B. Aspirin alone
C. Half-dose apixaban
D. Percutaneous left atrial appendage occlusion

85. A 63-year-old man with permanent AF on metoprolol, diltiazem, and digoxin presents to
the ED following a syncopal episode. He reports that his syncopal episode was preceded
by a very short period of light-headedness and a sensation of his heart racing. He has
experienced one similar episode in the past approximately 1 week prior to presentation.
He is monitored on 48 hours of telemetry without notable arrhythmic events. A decision is

Freemedicalbooks4download
made to discharge the patient with an ambulatory ECG monitoring device.
Which type of monitor might best capture his arrhythmia?
A. 48-Hour Holter monitor
B. Event recorder
C. Implantable loop recorder
D. Long-term external cardiac event monitoring device

86. A 72-year-old man with known moderate aortic stenosis and hypertension presents with
an episode of syncope while climbing a flight of stairs. He denies a prodromal syndrome.
Which one of the following may offer the highest yield for elucidating an etiology
of his syncopal episode?
A. A transthoracic echocardiogram
B. An ECG
C. Carotid artery Dopplers
D. History and physical

87. A 74-year-old woman with no past medical history presents with syncope that occurred
during micturition. The event was witnessed by her daughter, who notes that there was no
evidence of seizure. Orthostatic vital signs on arrival demonstrate a 5 mmHg drop in
systolic BP from supine to standing with a 20 beats/min increase in HR. Her physical
examination is otherwise unremarkable. Her ECG is unrevealing. The patient presently
has no complaints.
What is the most likely etiology for her syncope?
A. Arrhythmogenic
B. Neurocardiogenic
C. Neurologic
D. Orthostatic hypotension

88. You are asked to evaluate an 83-year-old man who was admitted to the cardiology service
who underwent implantation of a dual-chamber pacemaker earlier in the day for sick sinus
syndrome and sinus pauses resulting in syncope. Your evaluation reveals a HR of 110
beats/min and BP of 70/55 mmHg. Heart sounds are normal and pulmonary examination
reveals clear breath sounds. His neck veins are elevated to 15 cm H2O with a blunted Y
descent. There are no cannon A waves. ECG reveals sinus tachycardia and narrow QRS
complex.
What is the most likely etiology of his presentation?
A. Flash pulmonary edema
B. Lead perforation leading to cardiac tamponade
C. Pacemaker-mediated tachycardia
D. Pacemaker syndrome

89. You evaluate a 60-year-old man with a history of ischemic cardiomyopathy following an
anterior MI in 2009. He reports New York Heart Association class III symptoms,
including dyspnea on mild exertion and rare orthopnea. Vital signs reveal a HR of 60
beats/min and BP of 90/60 mmHg. Examination reveals no JVD, normal heart sounds,
clear breath sounds on pulmonary auscultation, and no lower extremity edema.
Medications include aspirin, atorvastatin, furosemide, metoprolol, lisinopril, and
spironolactone. ECG reveals normal sinus rhythm with a LBBB (QRS = 152 ms). His last
transthoracic echocardiogram reveals a left ventricular ejection fraction of 25%.
Which of the following will reduce his mortality risk?
A. Cardiac resynchronization therapy
B. Dual-chamber pacemaker
C. Implantable defibrillator
D. Both A and C

90. You are asked to evaluate a 65-year-old man with a history of sick sinus syndrome
requiring pacemaker implantation 5 months ago. He has no complaints today; however,
his wife has noted intermittent fevers over the past few days. Vital signs revealed a
temperature of 39.4°C (103°F), HR of 70 beats/min, and BP of 90/60 mmHg.
Examination reveals normal heart sounds with erythema and tenderness over his left
pectoral pacemaker implantation site. The remainder of his examination is unrevealing.
He is admitted and started on intravenous antibiotics. Blood cultures grew methicillin-
sensitive Staphylococcus aureus (MSSA). Transesophageal echocardiogram (TEE)
reveals no vegetations on cardiac valves or implanted hardware. By hospital day 3, he is
afebrile and feels well.
What is the next best step in care?
A. Convert to oral antibiotics to complete a 6-week course and discharge
B. Discharge and continue intravenous antibiotics to complete a 6-week course
C. Plan for pacemaker system removal
D. Refer for tricuspid valve replacement

91. A 55-year-old woman is admitted to the hospital for elective open sigmoid colectomy for
recurrent diverticulitis (last episode 3 months ago). She has a history of diabetes mellitus,
hypertension, hyperlipidemia, and an inferior MI 3 years ago requiring percutaneous
coronary intervention. Vital signs reveal temperature of 37°C (98.6°F), HR of 75
beats/min, and BP of 110/60 mmHg. Physical examination is unremarkable. Medications
include aspirin, metoprolol, atorvastatin, and a multivitamin. She denies any recent angina
or dyspnea on exertion since her MI. She continues to go to the gym weekly and runs 5
miles on the treadmill at least five times per week.
What is the next appropriate step in care?
A. Discontinue aspirin perioperatively
B. Perform stress testing
C. Proceed to surgery
D. Start clopidogrel to decrease risk of perioperative MI

92. You evaluate a 75-year-old woman in clinic for preoperative evaluation one week prior to
an elective cholecystectomy for cholelithiasis. She also has a history of severe
symptomatic aortic stenosis and will be undergoing a valve replacement in one month.
Vital signs reveal temperature of 37°C (98.6°F), HR of 75 beats/min, and BP of 110/60
mmHg. Cardiac examination reveals a harsh III/VI late-peaking systolic ejection murmur.
There is no JVD, lung sounds are clear, and there is no lower extremity edema.
Medications include aspirin and a multivitamin. She reports mild dyspnea on exertion that
has progressed over the past year.
What is the next appropriate step in care?

Freemedicalbooks4download
A. Initiate furosemide
B. Perform stress testing
C. Postpone cholecystectomy until valve intervention is completed
D. Proceed to cholecystectomy as scheduled

93. A 75-year-old man is seen in the emergency room with chest pain and anterior ST-
segment elevations on ECG. HeJVD referred for coronary angiography and found to have
a thrombotically occluded left anterior descending coronary artery and undergoes
placement of a drug-eluting stent. He is started on aspirin, clopidogrel, metoprolol, and
atorvastatin. Near the time of discharge, you learn that he is scheduled for elective knee
surgery in 2 weeks and the surgeons would need to stop his clopidogrel.
What is the next appropriate step in care?
A. Postpone knee replacement for 1 month
B. Postpone knee replacement for 3 months
C. Postpone knee replacement for 6 to 12 months
D. Proceed to knee replacement as scheduled in 2 weeks

94. A 68-year-old man with a history of nonobstructive CAD and stage 4 chronic kidney
disease presents with a nonhealing ulcer on the lateral aspect of his left foot. The ulcer has
been present for 3 months and has no purulence or erythema. On further history, he
describes muscle fatigue and cramping in his legs that has limited his exercise tolerance.
What is the best initial diagnostic test of choice?
A. Ankle-brachial index
B. Arterial lower extremity angiogram in the cardiac catheterization lab
C. CTA of the lower extremities
D. MRA of the lower extremities

95. A 59-year-old man with a history of CAD, stage 4 chronic kidney disease, and who is
nonadherent with medications presents with acute-onset pain in the left leg. On
examination, his leg is cool to the touch. He describes numbness and tingling in his foot.
Neither posterior tibial nor dorsalis pedis pulses are palpable. His foot is white and cold,
with pain on palpation.
While you are obtaining confirmatory imaging, what is the next best step?
A. Initiate low-molecular-weight heparin
B. Obtain arterial Dopplers in the morning
C. Obtain urgent vascular surgery or vascular medicine consultation
D. Place warm blankets on foot

96. A 69-year-old man with a history of advanced chronic kidney disease, hypertension, and
hyperlipidemia presents with painful leg cramps while ambulating. He does not have pain
at rest. You suspect he may have peripheral arterial disease. You order ankle-brachial
index. The right leg ankle-brachial index is 0.85 and the left leg ankle-brachial index is
0.75.
What would be the next best diagnostic test of choice?
A. Angiography in catheterization lab
B. CTA of the lower extremities
C. Segmental ankle-brachial index with pulse volume recording
D. Venous ultrasonography of the lower extremities

ANSWERS

1. The correct answer is: B. Inferior ST-elevation myocardial infarction. The patient has
ST elevations ≥1 mm in two contiguous leads (III, aVF), meeting criteria for ST-elevation
myocardial infarction. The presence of ST depressions in leads I and aVL reflects
reciprocal changes, while the presence of ST depressions in leads V2 and V3 may indicate
posterior myocardial ischemia. In addition, there are T-wave inversions in V4 to V6. The
patient meets ST-elevation myocardial infarction criteria and requires urgent
revascularization. The absence of anterior (V3-V4) elevations makes anterior involvement
unlikely, and the territorial nature of the ECG changes argues against pericarditis.

2. The correct answer is: C. Pericarditis. The patient has diffuse concave upward ST
elevations across multiple vascular territories, making pericarditis the most likely
diagnosis. In addition, lead aVR has an ST depression with elevation of the PR segment.
In sum, this ECG is most suggestive of pericardial inflammation. Given the viral
prodrome, this patient most likely has viral pericarditis.
Anterior ST-elevation myocardial infarctions present with elevations in the anterior
precordial leads (V3 and V4) and represent occlusion of the left anterior descending artery
or its branches. Inferior ST-elevation myocardial infarctions present with ST elevations in
leads II, III, and aVF and represent occlusion of the right coronary artery (or its branches)
in 85% of patients and the left circumflex artery (or its branches) in 15% of patients (in
whom the posterior descending artery arises from the left circumflex artery).

3. The correct answer is: C. AF with right bundle branch block (RBBB) and left
anterior fascicular block. The irregular ventricular rate with lack of visible P waves
makes AF the most likely underlying atrial rhythm. The patient has a RBBB manifesting
with classic morphology in V1 to V3. A RBBB is defined by a QRS ≥120 ms (110-119 =
intraventricular conduction delay or “incomplete”), rSR in R precordial leads (V1, V2), and
wide S wave in I and V6. Sometimes, ST depressions or T-wave inversions are seen in the
right precordial leads. In addition, this patient has a left axis deviation, which does not
normally occur with a RBBB, suggesting a left anterior fascicular block is also present.
The presence of a RBBB with a left anterior fascicular block is called a bifascicular block.
Complete heart block requires evidence of atrial and ventricular dissociation with
an atrial rate that is regular and exceeds the ventricular rate, and a regular ventricular rate.
The width and rate of the ventricular depolarizations are contingent on where in the His-
Purkinje system the ventricular beats are originating from (higher junctional escape beats
have narrower complexes and faster rates than those originating from the Purkinje
system).
The patient does not have a LBBB. LBBBs are defined by a QRS ≥120 ms with a
broad, slurred, monophasic R in I, aVL, and V5-V6. RS may be present in V5-V6 if the
patient has cardiomegaly. In addition, there is an absence of Q waves in I, V5, and V6. The
patient does not meet these criteria and therefore does not have a LBBB.

Freemedicalbooks4download
4. The correct answer is: B. Hyperkalemia. Hyperkalemia is characterized by ECG
changes that may include hyperacute or peaked T waves. This can progress to a shortening
of the QT interval, PR prolongation, and atrioventricular block and at extreme elevations
widened QRS and ST elevation. Hypercalcemia results in shortening of the QT interval
with flattened T waves, P waves, and J point elevation. Hypokalemia results in flattening
of the T waves on ECG, presence of U waves, and prolongation of the QT interval.
Although ischemia can also present with hyperacute T waves, the lack of chest pain or
anginal equivalents makes this diagnosis less likely.

5. The correct answer is: C. Initiating β-blocker therapy. The ECG shows QTc
prolongation. The QT segment is measured from the beginning of the QRS complex to the
end of the T wave. The QTc is corrected for the HR using Bazett’s formula: QTc = QT
√RR interval (RR is measured in seconds). Bazett formula may overcorrect at high HRs
and undercorrect at a low HR. The normal QTc for men is <440 ms and for women <460
ms. Once the QTc is >500 ms, the risk of torsades de pointes increases. Slower HRs,
potentially induced by nodal agents such as β-blockers, increase the risk of torsades de
pointes in the presence of QTc prolongation. Therefore, when initiating any QT-
prolonging medications, ECG monitoring for QTc prolongation is prudent.
The differential for QT prolongation is broad but includes certain antiarrhythmics
(such as class Ic, III), antipsychotics (ie, chlorpromazine, haloperidol, ziprasidone,
quetiapine), antimicrobials (ie, fluoroquinolones, macrolides, azoles), antiemetics (ie,
ondansetron, droperidol), pain medications including methadone, and electrolyte
disturbances (ie, hypocalcemia, hypokalemia). For this patient, the combination of
methadone and new initiation of a fluoroquinolone may have contributed to this QTc
prolongation.
For patients in whom QT prolongation is seen, serial ECG monitoring, correction
of PR lectrolytes, and discontinuation of nonessential QT-prolonging medications are
advised. No guidelines exist for discontinuation of QT-prolonging medications, and
clinical judgment should be used.

6. The correct answer is: D. Pulmonary embolism. The ECG shows sinus tachycardia at a
rate of approximately 100 beats/min. In addition, there is evidence of right heart strain
manifested by the prominent S wave in lead I, T-wave inversion and Q wave in lead III as
well as a RBBB. Additional findings of right heart strain (not seen on this ECG) include
ST elevations in precordial leads. Given the patient’s risk factor (tobacco use), physical
examination findings (including left greater than right lower extremity edema), and ECG
findings, pulmonary embolism is the most likely diagnosis.
Inferior ST-elevation myocardial infarctions present with chest pain or other chest
pain equivalent as well as >1 mm ST elevations in the inferior leads (II, III, aVF).
Pericarditis manifests early with diffuse ST elevations with PR depressions (except aVR,
which has PR elevation and ST depression). Non–ST-elevation myocardial infarctions
may or may not have ECG findings, including ST depressions and T-wave inversions
with positive troponin, but typically would not manifest with new RBBB.

7. The correct answer is: A. Check posterior leads. Posterior leads (V7-V9) should be
checked in patients with a history consistent with an acute coronary syndrome when the
initial standard 12-lead ECG reveals ST depressions in V1 to V3 (corresponding to
posterior ST-segment elevations), R wave > S wave in V1 (corresponding to a posterior Q
wave), or no ST-segment changes. If posterior leads reveal ST-segment elevations, it
would mandate urgent revascularization as this would be consistent with an acute ST-
elevation myocardial infarction. While pulmonary embolism could cause chest pain, it is
more commonly pleuritic. ECG changes in pulmonary embolism include evidence of right
ventricular strain such as S wave in lead I, Q wave in lead III, T-wave inversion in lead
III, RBBB, right axis deviation, and ST-segment elevations in the anterior precordial
leads. While pericarditis can cause chest pain it is typically a sharp pain worse with
inspiration and decreased with sitting forward. On physical examination, a pericardial
friction rub may be present and the ECG may show diffuse ST elevation with PR
depressions. While colchicine is the first-line treatment, the history and examinations are
not consistent with pericarditis. Gastroesophageal reflux disease can cause a burning chest
pain. It often occurs following eating, may be associated with an acidic taste in mouth,
and worsens postprandially and with recumbency. The chest pain history in this vignette is
more concerning for an acute coronary syndrome.

8. The correct answer is: A. Activate the cardiac catheterization lab for urgent
revascularization. The ECG is concerning for an acute inferoposterior ST-elevation
myocardial infarction and the patient should go for urgent coronary angiography. ST-
segment elevation in the right-sided leads (most typically V4R) suggests right ventricle
involvement. In these cases, the right ventricle is preload dependent and administration of
a vasodilator such as nitroglycerin will decrease preload, which could lead to hypotension.
A transthoracic echocardiogram may be utilized to evaluate for wall motion abnormalities
to support a diagnosis of ischemia, but in this instance, the history and ECG are diagnostic
and an echocardiogram may contribute to delays in primary percutaneous coronary
intervention. Posterior leads are not expected to be useful in this case as ST-elevation
myocardial infarction has already been diagnosed and will therefore not change
management.

9. The correct answer is: A. Acute pericarditis. The vignette is most consistent with acute
pericarditis. The description of a sharp, pleuritic pain that is improved by leaning forward
and worsened by lying back is most consistent with pericarditis. This may have been
precipitated by a recent viral illness. Classic ECG findings include PR depressions with a
diffuse pattern of ST elevations that do not correlate to a unique coronary vascular
distribution. An aortic dissection may also present with acute onset of sharp chest pain
although the pain is typically not positional and often radiates to the back. There are often
no ECG findings in aortic dissection, although it may include low voltages if there is a
pericardial effusion or ST elevations if the dissection extends into a coronary artery
(typically the right coronary artery). Acute coronary syndrome may have a variety of
clinical presentations, including chest pain, pressure, shortness of breath, neck or jaw pain,
arm pain, and arm numbness or tingling. Pain may even be epigastric or abdominal and
include nausea or vomiting. Although ST elevations always raise concern for an acute
coronary syndrome, the presence of upsloping ST elevations across multiple vascular
territories is suggestive of a diffuse process such as pericarditis. Acute pulmonary
embolism may also present with sudden-onset pleuritic chest pain. Hemodynamic markers
include hypoxemia, tachycardia, and hypotension, depending on the size of the pulmonary

Freemedicalbooks4download
embolism. ECG findings may range from sinus tachycardia to evidence of right ventricle
strain. ST elevations in multiple vascular territories would be atypical.

10. The correct answer is: A. Cardiac magnetic resonance imaging (MRI). Despite a
clinical presentation suggestive of non–ST-elevation myocardial infarction, women are
less likely than men to have obstructive coronary disease at catheterization. This entity is
known as MI with no obstructive CAD. Individuals with MI with no obstructive CAD
remain at increased risk for major adverse cardiac events, and alternate etiologies for the
patient’s presentation should be investigated. Coronary films should be reviewed for
missed dissection, plaque erosion/disruption, emboli, or spasm. A left ventricle (LV)
functional assessment should be pursued. In this patient with depressed LV function and
elevated troponin, a diagnosis of myocarditis should be considered, which can be
confirmed with a cardiac MRI.

11. The correct answer is: D. Pharmacologic stress test with perfusion imaging. Given
the patient has a LBBB at baseline, he cannot undergo an ECG exercise tolerance test as
the ECG alone will not be diagnostic for ischemia. In addition, his recent knee
replacement and poor exercise tolerance at baseline would make him a poor candidate for
exercise stress testing. Exercise stress tests require attainment of at least 85% of a
maximum predicted HR to maximize diagnostic yield. If a patient is unable to exercise
long enough to reach this end point, the test will not be interpretable. For patients with
underlying LBBB, a vasodilator stress is typically recommended, preferentially before
either an exercise or chronotropic evaluation since there is a higher risk of a false positive
at higher HRs due to ventricular dyssynchrony.

12. The correct answer is: C. Coronary CT angiography. The patient has a somewhat
atypical chest pain history, but does have risk factors including family history of early
CAD, hypertension, and hyperlipidemia, giving him an intermediate probability of CAD.
A coronary CT angiography will give information about the anatomy of the coronary
arteries as well as any stenoses and is therefore a reasonable test to exclude coronary
disease. Although it has a high negative predictive value, positive findings may need to be
confirmed with a coronary angiogram. Cardiac MRI without stress testing can be
employed to examine cardiac function and structure, but is not optimal in the emergency
room setting due to the length of the test. An exercise stress test has the benefit of
evaluating functional capacity and provocation of symptoms with exercise. However, the
patient’s chronic knee pain may limit his ability to reach 85% of the maximum predicted
HR, thus limiting the test’s diagnostic ability.

13. The correct answer is: B. Evaluation of myocardial viability. Cardiac MRI is one of
the imaging studies utilized to evaluate myocardial viability. In a patient with multivessel
disease, there is limited benefit to surgical revascularization if a large portion of the
myocardium acute coronary syndrome scarred irreversibly. A viability assessment can be
performed through different modalities, including cardiac MRI, cardiac positron emission
tomography (PET), thallium scans, and dobutamine stress echocardiography. These
studies can help distinguish myocardium that is irreversibly damaged versus myocardium
that is hibernating and may regain function after reperfusion. While prior myocarditis may
present with late gadolinium enhancement (often in a patchy distribution), the patient’s
cardiomyopathy is likely best explained by ischemia, given his severe multivessel disease.
Cardiac amyloidosis may cause decreased EF late in the disease course and may have a
distinct late gadolinium enhancement on cardiac MRI. While cardiac MRI can evaluate
valvular disease, transthoracic echocardiography is the preferred modality for the
evaluation of valvular disease.

14. The correct answer is: B. Coronary angiography. The patient has a high-risk stress test
for several reasons. He achieved only 4 METs before stopping due to angina.
Additionally, his BP did not augment with exercise, raising concern for left main or
multivessel CAD. The ECG at 7 minutes into recovery has >2-mm ST depressions
diffusely and an ST elevation in aVR. High-risk ECG findings are ST depressions ≥2 mm
or ≥1 mm in stage 1 of an exercise stress test, depressions occurring in ≥5 leads or ≥5
minutes into recovery. In sum, the findings in the vignette are highly concerning for
significant CAD requiring definition of his coronary anatomy and consideration of
revascularization.
Other high-risk features on stress tests include a Duke treadmill score <−11, a drop
in EF during stress (if imaging is obtained), ≥1 large or 2 moderate perfusion defects,
transient left ventricular dilation, or increased lung uptake of radionuclide tracer (if
perfusion imaging is obtained).

15. The correct answer is: A. Initiate β-blocker and statin. The patient presents with a
syndrome consistent with stable angina. Exercise stress testing confirms the diagnosis of
CAD with demonstrable ischemia in a single territory. The next best step in management
would be the institution of optimal medical therapy that includes low-dose aspirin, β-
blocker, and statin therapy (and possibly other lipid-lowering therapy). Coronary
angiography should be considered in those patients with refractory symptoms despite
optimal medical therapy and in those patients with high-risk stress test findings (low EF,
transient ischemic dilation [suggestive of three-vessel disease], hypotensive response to
exercise, ≥2 territories of moderate ischemia or >1 territory of severe ischemia, exercise-
induced arrhythmias). In the absence of left main disease on coronary CT angiography, it
is reasonable to first consider a trial of optimized medical therapy before considering
coronary revascularization in patients with moderate-to-severe ischemia (ISCHEMIA
trial). Transthoracic echocardiogram would help identify structural heart abnormalities,
which would be unlikely to explain the patient’s symptoms.

16. The correct answer is: D. Manual compression of the access site. Access site bleeding
should be suspected in postprocedural patients with hypotension, lower abdominal or back
pain, or rapidly expanding hematoma. The first step in managing active hemorrhage
requires primary bleeding control with manual compression of the common femoral
artery. Thereafter, consideration can be given to whether blood products should be
administered empirically to facilitate hemodynamic stabilization. CT of the abdomen can
be utilized when the diagnosis is uncertain and hemodynamic parameters have stabilized.
While most bleeding stops with manual pressure, surgical or percutaneous intervention
can be considered following failure of manual compression. Aspirin and P2Y12 inhibitor
therapy does not reverse quickly and thus stopping would not help in the acute setting.
Any decisions about altering antiplatelet therapy long term (which would be unlikely)
should be done in consultation with the cardiologist.

Freemedicalbooks4download
17. The correct answer is: B. Clopidogrel can be held to permit a knee replacement. In
this clinical scenario, clopidogrel can be held to permit the patient to undergo a knee
replacement. The initial decision on dual antiplatelet therapy duration depends on the
indication for stent placement. Patients who undergo stent placement for management of
an acute coronary syndrome are considered to be at higher risk for stent thrombosis and
recurrent MI; therefore, dual antiplatelet therapy is ideally recommended for a minimum
of 12 months, regardless of stent type. In contrast, stents placed for stable ischemic heart
disease mandate dual antiplatelet therapy for a minimum of 6 months if a drug-eluting
stent is placed. The intravenous P2Y12 inhibitor cangrelor does not have a role for
elective procedures but can be considered as an off-label IV bridge for semi-urgent
procedures for patients in whom the risk of stent thrombosis off of dual antiplatelet
therapy is considered high. There is no role for stress testing to guide the duration of dual
antiplatelet therapy.

18. The correct answer is: A. A conservative strategy of aspirin, β-blocker, and nitrates
as needed, followed by noninvasive risk stratification (stress testing) to help
determine if coronary angiography is appropriate, provided the patient remains
asymptomatic. All patients presenting with unstable angina or non–ST-elevation
myocardial infarction should undergo risk assessment, which can be performed with two
commonly used scores: Thrombolysis in Myocardial Infarction (TIMI) or Global Registry
of Acute Coronary Events risk scores. Urgent/immediate diagnostic angiography with the
intent to revascularize should be pursued in patients with non–ST-elevation myocardial
infarction who have refractory angina or hemodynamic/electrical instability. Early (within
24 hours) diagnostic angiography with the intent to revascularize is indicated in stabilized
patients with non–ST-elevation myocardial infarction who are at high risk of clinical
events. In patients, such as this example, at low risk (TIMI score 0-2 or Global Registry of
Acute Coronary Events score ≤108), a conservative strategy of optimal medical therapy
with coronary angiography pursued in those patients with persistent or recurrent
symptoms or high-risk stress test findings can be considered.

19. The correct answer is: C. Metoprolol. In this patient presenting with ST-elevation
myocardial infarction with plans for primary percutaneous coronary intervention, the
clinical examination suggests features of evolving cardiac pump failure (elevated JVP,
basilar rales, and an S3) with suggestion of hypoperfusion (cool lower extremities) and
marginal hemodynamics (tachycardia, hypotension with narrow pulse pressure). β-blocker
therapy is recommended within the first 24 hours in patients with ST-elevation myocardial
infarction but should be deferred in patients with HF, a low-output state, or at risk of
developing cardiogenic shock as it may block the compensatory sympathetic response.

20. The correct answer is: D. Urgent transthoracic echocardiography. Rapid reperfusion
therapy has decreased the incidence of mechanical complications of MI. Nonetheless, a
high index of suspicion is required. Patients with late presentations of larger territory MI,
especially of the anterior wall, are at highest risk of developing a ventricular septal defect.
The harsh holosystolic murmur at the lower sternal border, especially in the presence of a
palpable systolic thrill, is pathognomonic for a ventricular septal defect. The diagnosis is
typically made by transthoracic echocardiogram, although an increase of venous oxygen
saturation between the right atrium and the pulmonary artery (PA) by right heart
catheterization can be suggestive. Definitive management necessitates surgical repair.
Afterload reducing agents (ie, nitroprusside) and interventions (ie, intra-aortic balloon
pump) can help decrease left to right shunting through the ventricular septal defect as a
bridge to surgery.

21. The correct answer is: B. Administer alteplase within 30 minutes of hospital arrival
and proceed with urgent transfer to percutaneous coronary intervention–capable
facility. In patients presenting to a non–percutaneous coronary intervention–capable
hospital with ST-elevation myocardial infarction who are candidates for reperfusion,
transfer to a percutaneous coronary intervention–capable hospital should be pursued if the
anticipated time from first medical contact to device therapy is less than 120 minutes.
When the anticipated transfer time is in excess of 120 minutes, fibrinolytic therapy should
be administered within 30 minutes of hospital arrival. Patients with cardiogenic shock or
severe HF should be transferred to a percutaneous coronary intervention–capable hospital
as soon as possible, irrespective of the time delay since MI. In the absence of cardiogenic
shock or HF, transfer can be pursued urgently if there is evidence of failed pharmacologic
reperfusion (ongoing symptoms) or within 24 hours for those patients in whom an
invasive strategy is being pursued.

22. The correct answer is: A. Make arrangements to stent the left anterior descending in
the next 45 days. Randomized trials, including the COMPLETE trial, showed a lower
risk of recurrent major adverse cardiovascular events when nonculprit lesions are stented
in addition to the culprit lesion in patients with ST-elevation myocardial infarction. This
can be pursued either during the index hospitalization or within the first 45 days.
Fractional flow reserve would be reasonable for a moderate stenosis (50%-69% stenosis),
but is not required for more severe stenoses. The stenting of nonculprit lesions should not
be pursued for patients with signs or symptoms of cardiogenic shock.

23. The correct answer is: C. Add ezetimibe. All acute coronary syndrome patients should
be prescribed a high-intensity statin, regardless of the baseline LDL-C. Data from the
IMPROVE-IT trial suggest additive benefit for ezetimibe on a background of moderate-
intensity statin and is recommended for patients in whom LDL-C is still not at goal.

24. The correct answer is: E. A and C. Even in the absence of stenting, patients with acute
coronary syndrome should be considered for dual antiplatelet therapy for approximately
12 months. Some studies support longer durations of therapy. Prasugrel has not been
shown to benefit patients with non–ST elevation acute coronary syndrome who are
conservatively managed. A dose reduction may also need to be considered with prasugrel
for patients over the age of 75. In contrast, ticagrelor has been shown to reduce the risk of
major adverse cardiovascular events on a background of aspirin, regardless of whether
patients undergo coronary stenting. More recent data may suggest that aspirin can be
safely stopped with continued P2Y12 inhibitor monotherapy 1 to 3 months post–
percutaneous coronary intervention (TWILIGHT). Nonetheless, the initial treatment
strategy should include dual antiplatelet therapy.

25. The correct answer is: C. Invasive coronary angiography. The patient is experiencing
non–ST-elevation myocardial infarction, and per American College of Cardiology and

Freemedicalbooks4download
American Heart Association (ACC/AHA) guidelines, an invasive strategy is
recommended in these higher-risk patients to reduce the risk of recurrent spontaneous MI.

26. The correct answer is: B. Within 24 hours. For high-risk patients with elevated
troponin, ST-segment changes, or high Global Registry of Acute Coronary Events score
(>140), early angiography within 24 hours is recommended. Lower-risk patients (eg, those
without the above features, but with diabetes, chronic kidney disease, percutaneous
coronary intervention in the past 6 months, prior CABG, or left ventricular ejection
fraction <40%) can undergo angiography within 72 hours. Patients with refractory or
recurrent angina or hemodynamic or electrical instability should undergo angiography
immediately.

27. The correct answer is: C. Diltiazem. Vasospastic or Prinzmetal angina is a clinical
syndrome of rest angina with transient ST-segment elevation, nitrate-responsive angina, or
angiographically apparent coronary spasm. Obstructive coronary disease may be the cause
of vasospasm and thus should be excluded by invasive or noninvasive means. Calcium
channel blockers are first-line therapy for vasospastic angina as they prevent
vasoconstriction and promote vasodilation. Long-acting nitrates can also be effective, but
nitrate tolerance makes them less desirable. β-blockers, particularly nonselective β-
blockers, should be avoided as they can precipitate vasospasm. Aspirin should be used
with caution as it is a prostacyclin inhibitor at high doses and may also precipitate
vasospasm.

28. The correct answer is: B. Posterior leads (leads V7-V9). In a patient with cardiovascular
risk factors and a clinical history consistent with acute coronary syndrome, a high index of
suspicion should be maintained. If the presenting ECG is nondiagnostic, posterior leads
should be checked to assess the distal left circumflex and right coronary artery territory
that may supply the posterior wall of the left ventricle (LV). Posterior ST elevations can
help diagnose a posterior ST-elevation myocardial infarction that may not be electrically
apparent on a standard 12-lead ECG. Right-sided precordial leads are useful in patients
presenting with an inferior MI to help detect right ventricle involvement and would not be
useful in this case. Serial troponin measurements would be appropriate following
completion of the posterior lead ECG. In this patient with rest symptoms highly
suggestive of acute coronary syndrome, pulmonary embolus CT would not be the next
best step (but could be pursued after acute coronary syndrome was definitely ruled out).

29. The correct answer is: D. Take clopidogrel for at least 1 year. Current ACC/AHA
guidelines support continued use of dual antiplatelet therapy for at least 12 months for
patients after acute coronary syndrome. At 12 months, consideration can be given to
continuing the P2Y12 inhibitor for a longer duration if the patient is believed to be at low
bleeding risk. For patients who undergo elective percutaneous coronary intervention for
stable CAD, shorter courses of dual antiplatelet therapy may be possible. Emerging
evidence suggests that monotherapy with a P2Y12 inhibitor can be considered with
discontinuation of aspirin 1 to 3 months post–percutaneous coronary intervention, but this
approach has not yet been adopted by the guideline recommendations.

30. The correct answer is: B. Distributive shock. This patient’s normal cardiac output
points toward distributive shock because with all the other etiologies one would expect a
diminished cardiac output. For patients who are hospitalized, distributive shock should
always remain on the differential given the high frequency of sepsis from a nosocomial
infection. Patients with distributive shock typically have high cardiac output, low systemic
vascular resistance, and low/normal filling pressures. In cardiogenic shock, we expect low
cardiac output and high filling pressures. In hypovolemic shock, we expect both low
cardiac output and low filling pressures. The cardiac output is low in obstructive shock
and filling pressures vary based on etiology of the obstructive shock.

31. The correct answer is: D. Left atrial—same; left ventricular end-diastolic pressure
(LVEDP)—lower. The pulmonary artery (PA) wedge pressure is an estimate of the
LVEDP under ideal no-flow conditions. This occurs at the end of diastole when the left
atrial and left ventricle (LV) pressures are equal (assuming there is no transmitral
gradient). The mean PA wedge pressure is often used to estimate LVEDP. Mitral
regurgitation is one of several conditions in which mean PA pressure can significantly
overestimate LVEDP because of chronic regurgitant flow returning to the left atrium
during each cardiac systole. In mitral regurgitation, mean left atrial pressure is higher than
the LVEDP. The mean left atrial pressure transmits to the wedge pressure. Therefore, in
this scenario, wedge pressure = left atrial pressure > LVEDP.

32. The correct answer is: C. Increase dobutamine. This patient has a low cardiac index,
high systemic vascular resistance, and high filling pressures consistent with ongoing
cardiogenic shock. Increasing the dobutamine will increase contractility, allowing for
increased forward flow and diuresis. Increasing the diuretic dose without increasing
contractility will have minimal benefit. Adding norepinephrine and vasopressin will
increase the afterload on the heart and therefore will not be as beneficial as an inodilator
like dobutamine.

33. The correct answer is: A. Mixed—cardiogenic and distributive. Her hypotension is
suggestive of a mixed picture of both cardiogenic (with reduced cardiac output despite
inotropic support due to diminished left ventricle [LV] function) and distributive (low end
of normal systemic vascular resistance but on a vasopressor) shock. In pure cardiogenic
shock and in tamponade, we would expect high filling pressures. In pure distributive
shock in a patient with sepsis, we would expect a normal/high cardiac output.

34. The correct answer is: D. Emergent transthoracic echocardiogram. The sudden
change in hemodynamics in a post-MI patient is worrisome for a mechanical complication
of MI. The rise in MVO2 does not fit with her clinical picture of worsening cardiogenic
shock and likely reflects development of a ventricular septal defect. Urgent
echocardiography is needed to confirm the diagnosis, the treatment for which is typically
urgent surgery. On physical examination, one might expect to hear a new, harsh
holosystolic murmur with a thrill and see a step-up (>7%) in oxygen saturation from
central venous (pre–left-to-right shunt) to mixed venous (post–left-to-right shunt).

35. The correct answer is: B. Furosemide. Diuretics have been shown in multiple trials to
reduce symptoms related to congestion; however, they have never been proven to reduce
mortality. The agents that have been shown to reduce mortality in patients with

Freemedicalbooks4download
symptomatic HF with reduced EF include ACE inhibitors, angiotensin receptor blockers
(ARBs; if ACE inhibitors are not tolerated), ARNIs (ARB+ neprilysin inhibitors), β-
blockers (specifically carvedilol, metoprolol, and bisoprolol), and mineralocorticoid
receptor antagonists (spironolactone and eplerenone). Of note, ARNIs have been
demonstrated to be superior to ACE inhibitors in patients with symptomatic HF and
reduced left ventricular function, but the two should never be used in combination.
Emerging evidence may support the use of SGLT2 inhibitors in these patients as well.

36. The correct answer is: B. Administer intravenous furosemide. In this patient with
suggestive evidence of a HF exacerbation, her acute kidney injury is likely explained by
cardiorenal syndrome that is believed to be driven by a variety of factors, including
venous congestion. The initial treatment of choice is diuresis, which should improve her
symptoms of congestion and likely will improve her creatinine. She is mentating well with
normal perfusion (warm extremities) and normal pulse pressure. Intravenous crystalloid
likely would worsen her HF exacerbation. Given her limited response to oral diuretics, she
may be experiencing poor gut absorption from bowel edema. Therefore, the most
appropriate step is to administer intravenous furosemide or other intravenous diuretic.

37. The correct answer is: A. Decreased aortic regurgitation. The benefits of an intra-
aortic balloon pump are 2-fold: (a) inflation during diastole results in augmentation of
coronary perfusion and (b) deflation during systole decreases afterload and improves
cardiac output, which may further decrease myocardial oxygen demand. One of the major
risks of the intra-aortic balloon pump is that inflation during diastole will increase
preexisting aortic regurgitation. Aortic aneurysms and aortic regurgitation are relative
contraindications to balloon pump placement.

38. The correct answer is: C. Diuresis alone. When patients present with acute
decompensated HF, it is important to assess their degree of congestion (wet vs. dry) and
adequacy of perfusion (warm vs. cold). This patient’s elevated JVP, crackles on lung
examination, and lower extremity edema are suggestive of volume overload (wet). His
normal lactate, liver function tests, creatinine, warm extremities, and reasonable urine
output suggest adequate end-organ perfusion or forward flow (warm). In a patient who is
“warm and wet,” the initial step in therapy is diuresis alone. If inadequate perfusion is
demonstrated, then the patient may also benefit from the addition of inotropes.

39. The correct answer is: D. Spironolactone. Many clinical trials have failed to
demonstrate the benefit of pharmacotherapy to reduce mortality among patients with heart
failure with preserved ejection fraction. In a clinical trial, spironolactone did not reduce
mortality, but was shown to decrease the risk of rehospitalization for HF (Pitt B, Pfeffer
MA, Assmann SF, et al. NEJM. 2014;370(15):1383). The ACC/AHA guidelines currently
offer a class IIa recommendation (should be considered) for the use of spironolactone in
patients with heart failure with preserved ejection fraction with either elevated natriuretic
peptides or hospitalization with HF in the past year. ACE inhibitors, phosphodiesterase
type 5 inhibitors, and aspirin have not been shown in a clinical trial to reduce the risk of
HF hospitalization among patients with heart failure with preserved ejection fraction.
Trials of SGLT2i in heart failure with preserved ejection fraction patients are ongoing.
40. The correct answer is: C. Insert a percutaneous left ventricular assist device (eg,
Impella or Tandem Heart). This patient is in cardiogenic shock due to profound acute
decompensated HF. Increasing the pressor dose would not address his cardiogenic shock.
Increasing the dobutamine is unlikely to be sufficient and could risk further end-organ
damage. An intra-aortic balloon pump is also unlikely to offer adequate support. A
percutaneous left ventricular assist device can provide up to 5 L/min cardiac output and
would be the next best step. It should be noted, however, that randomized trials supporting
the use of assist devices are currently lacking.

41. The correct answer is: B. Initiation of venoarterial extracorporeal membrane


oxygen. The patient has evidence of biventricular HF from a presumed influenza
myocarditis. With supportive care, myocardial recovery is expected, and thus with
escalation of pharmacologic support, mechanical circulatory support should also be
considered. Given the patient’s biventricular failure, a pure left ventricular support device
would be insufficient. Thus, the patient should be considered for venoarterial
extracorporeal membrane oxygen. It should be noted that large randomized trials
supporting this approach have not yet been conducted.

42. The correct answer is: B. Initiation of angiotensin-converting enzyme (ACE)


inhibitor and β-blocker; repeat transthoracic echocardiogram in 3 months. In this
case, the patient presents with a dilated cardiomyopathy and is found to have influenza
infection. While this may be the etiology of his systolic HF, the cardiac MRI did not
support a diagnosis of acute myocarditis. He also has a history of significant toxin
exposure including alcohol and cocaine, which may lead to systolic dysfunction. He
should pursue abstinence and immediately start appropriate guideline-directed medical
therapy to promote positive left ventricle (LV) remodeling and possibly improve his EF.
Transthoracic echocardiogram can be repeated in 3 months to assess for recovery of
function; 1 month may be too soon to expect any significant improvement. If he has
persistent significant LV dysfunction, an implantable cardiac defibrillator can then be
considered at this time. A primary prevention implantable cardiac defibrillator should not
be placed if there is a reasonable expectation for recovery in left ventricular function.
Myocardial biopsy is not indicated in this case, as it is typically pursued in cases of
hemodynamic or electrical instability when pathology is expected to change management.

43. The correct answer is: B. Discontinue norepinephrine and initiate phenylephrine.
The patient has a history of hypertrophic obstructive cardiomyopathy, which indicates
septal hypertrophy with a narrow subaortic outflow tract given. β-agonists, such as
norepinephrine, will increase chronotropy and inotropy, which can worsen outflow
obstruction and can lead to worsening hypotension. In this case, the vasopressor of choice
would be phenylephrine, which is a pure α-agonist, and therefore will increase systemic
vascular resistance without increasing chronotropy or inotropy. Slower HRs may be
advantageous by increasing left ventricular filling time and therefore decreasing the
outflow gradient. The addition of other inotropes, such as dobutamine or epinephrine, will
likely worsen her shock. Volume depletion can also worsen the obstruction, but at this
time, she appears to be euvolemic with a central venous pressure of 12 mmHg on her most
recent tracing.

Freemedicalbooks4download
44. The correct answer is: A. Amyloidosis. The discordance of low voltages on ECG,
despite thick ventricular walls on transthoracic echocardiogram, is a classic ECG finding
for cardiac amyloidosis for someone who is suspected to be at risk. Additional diagnostic
tests include serum and urine protein electrophoresis, quantification of serum free light
chain ratio, and possible fat pad biopsy and cardiac MRI. Sarcoidosis is another type of
infiltrative cardiomyopathy; however, his history (including multiple myeloma and renal
failure) and ECG findings are more characteristic of amyloidosis. Patients with
sarcoidosis do not classically have increased left ventricle (LV) wall thickness as
involvement may be quite patchy. With long-standing hypertension or hypertrophic
cardiomyopathy, a transthoracic echocardiogram will commonly show left ventricular
wall thickening due to concentric hypertrophy; however, this is typically reflected by
normal or high voltages on ECG.

45. The correct answer is: B. Left ventricle (LV) wall thickness of 15 mm. LV wall
thickness ≥15 mm is one of the diagnostic criteria for hypertrophic cardiomyopathy. LV
wall thickness of ≥30 mm, as well as answer choices A, C, and D, are all high-risk
features for sudden cardiac death. An implantable cardiac defibrillator is recommended
for all patients with hypertrophic cardiomyopathy and a history of cardiac arrest or
sustained ventricular tachycardia. Patients with hypertrophic cardiomyopathy should be
evaluated for high-risk features and, if any exist, considered for implantable cardiac
defibrillator implantation.

46. The correct answer is: A. Coronary angiography. The next best step is to obtain an
urgent coronary angiogram to exclude obstructive CAD. While the history and
echocardiogram are suggestive of Takotsubo cardiomyopathy due to the recent stressor
and evidence of apical ballooning, it is necessary to definitively rule out acute plaque
rupture prior to diagnosing a stress-induced cardiomyopathy.

47. The correct answer is: A. Absolute abstinence from alcohol. While the cause of his
new cardiomyopathy could be from alternate etiologies, it may be due to alcohol use or
alcohol may be contributing. The treatment in addition to guideline-directed medical
therapy for HF with reduced EF and rate or rhythm control for AF is complete abstinence
from alcohol.

48. The correct answer is: A. Proceed to dobutamine stress echo. This patient should be
evaluated with a dobutamine stress echo prior to consideration of aortic valve replacement
in order to better understand whether he exhibits low-flow, low-gradient, severe aortic
stenosis. In patients with reduced EF and aortic valve area <1 cm2, it may be challenging
to distinguish if the estimated aortic valve area is low because of the reduced EF or due to
true severe aortic stenosis. If the calculated aortic valve area increases substantially with
administration of dobutamine, then this would suggest “pseudo-aortic stenosis” as the
initial aortic valve area estimate was likely explained by poor forward flow. Similarly, it is
important to know whether the mean gradient across the valve significantly increases in
response to dobutamine. If the mean gradient increases substantially and the aortic valve
area does not change, then the patient benefits from aortic valve replacement.

49. The correct answer is: D. Repeat transesophageal echocardiogram every 1 to 2 years.
This patient was incidentally found to have a bicuspid aortic valve and echocardiographic
evidence of moderate aortic stenosis while undergoing workup for her fevers. Aortic valve
intervention is not indicated for moderate aortic stenosis; therefore, she needs routine-
periodic monitoring with repeat transthoracic echocardiogram yearly to every other year.
Patients with bicuspid aortic valve are also at risk of developing thoracic aortic aneurysms
(TAAs). In these cases, aortic valve replacement may be indicated in patients with aortic
stenosis or regurgitation at the time of surgical intervention on the ascending aorta.

50. The correct answer is: A. Heart team approach to determine recommendation for
surgical versus transcatheter aortic valve replacement. This patient has symptomatic
severe aortic stenosis; therefore, aortic valve replacement is indicated. The STS score is
used to determine the risk of surgical mortality for many surgical procedures, including
aortic valve replacement. Transcatheter aortic valve replacement has been shown to be
noninferior to surgical aortic valve replacement for high STS score (>8%) patients
(PARTNER A trial), intermediate STS score (4%-8%) patients (PARTNER 2 trial), and
most recently low STS score (<4%) patients (PARTNER 3 trial, published in 2019). A
transcatheter aortic valve replacement approach typically offers faster hospital recovery,
but has a higher incidence of permanent pacemaker placement. For this patient with a low
STS score, either surgical or transcatheter aortic valve replacement may be appropriate,
and the decision of which to recommend to the patient should be made by a
multidisciplinary heart team.

51. The correct answer is: B. Mitral regurgitation. This patient most likely had a missed
inferior MI that occurred 3 days ago as suggested by his symptoms and the appearance of
inferior Q waves on his ECG. Patients are at risk of mechanical complications, including
papillary muscle rupture, ventricular septal rupture, and pericardial wall rupture
approximately 2 to 7 days following MI. This is particularly true in larger infarcts that are
not revascularized. The posteromedial papillary muscle is typically supplied by the
posterior descending artery off of the right coronary artery. Due to its sole blood supply, it
is more prone to rupture than the anterolateral papillary muscle. Papillary muscle rupture
is typically manifested with acute onset of dyspnea, hypoxia, pulmonary edema,
holosystolic murmur (low pitched and may be hard to appreciate), and hypotension. This
requires immediate surgical repair and can be evaluated by echocardiogram. A ventricular
septal defect is another complication after MI but typically exhibits a louder systolic
murmur and is often accompanied by a thrill. Although left ventricular dysfunction and a
pulmonary embolus are possible, a mechanical complication should be first considered
during this time frame after MI.

52. The correct answer is: C. Stop coumadin now and bridge with unfractionated
heparin. Patients with a mechanical valve in the aortic position and additional stroke risk
factors such as AF, prior thromboembolism, hypercoagulability, and EF <30% to 35%
should be managed with a coumadin goal of 2.5 to 3.5. The novel anticoagulants,
including the direct thrombin inhibitor dabigatran and factor Xa inhibitors, are
contraindicated in patients with a mechanical valve. Patients with a mechanical mitral
valve or a mechanical aortic valve with risk factors for stroke should be bridged with
heparin when coumadin is discontinued. For patients with a mechanical aortic valve
replacement without high-risk features, the decision to bridge with anticoagulation may

Freemedicalbooks4download
depend on patient factors, the procedure being performed, and other medical
comorbidities.

53. The correct answer is: C. Refer for mitral valve surgery. Patients with chronic severe
primary mitral regurgitation due to mitral valve prolapse should be referred for mitral
valve replacement if they have an EF between 30% and 60% or a left ventricular end-
systolic dimension (LVESD) >40 mm, regardless of symptom status. She has no evidence
of congestion, so furosemide is not indicated. Since she has an indication for mitral valve
intervention, additional testing with ambulatory ECG monitoring or repeat
echocardiography is not indicated.

54. The correct answer is: D. Refer for percutaneous mitral balloon commissurotomy as
anatomy allows. Patients with rheumatic mitral stenosis may have prolonged
asymptomatic phases; however, the additional volume load of pregnancy often results in
overt symptoms of HF. She currently describes symptoms consistent with New York
Heart Association class III HF. In the presence of moderate-to-severe symptoms and
pregnancy, consideration needs to be given to possible mitral valve intervention if the
pregnancy is to be continued. For patients in whom the anatomy is amenable, a
percutaneous procedure is preferred. Women of child-bearing age with known mitral
valve stenosis should be counseled about the possible risks of pregnancy.

55. The correct answer is: C. Give 500 mL fluid bolus. The patient is presenting with
Beck’s triad (distant heart sounds, elevated jugular vein pulsation, and hypotension)
concerning for tamponade. Upfront treatment of suspected tamponade relies on volume
expansion while a diagnostic and therapeutic plan is formulated. An emergent
transthoracic echocardiogram must be obtained to measure the size and location of the
effusion. Effusions amenable to pericardiocentesis are anterior in location and at least 10
mm in size (measured during diastole). A pericardial window may be needed in posterior
effusions not amenable to pericardiocentesis. The history and physical are most suggestive
of a pericardial effusion, so a pulmonary embolism CT scan would not be the first step.

56. The correct answer is: B. Constrictive pericarditis. Constrictive pericarditis may occur
in 1% to 2% of cases following pericarditis. In patients with tuberculosis, bacterial
infections, neoplasm, or, as in this case, exposure to radiation therapy, the risk of
developing constrictive pericarditis is higher. Constrictive pericarditis occurs when there
is adhesion between the visceral and parietal pericardium, resulting in a rigid pericardium
limiting diastolic filling and increasing venous pressures. The limitation of venous return
occurs only after the rapid filling stage following opening of the tricuspid valve. These
patients often have Kussmaul sign present, which manifests as a jugular venous pulsation
that does not decrease with inspiration. Sometimes, a pericardial knock may be present.
ECG may show low voltages, but constrictive pericarditis does not cause conduction
disease, which is more commonly seen with restrictive cardiomyopathy. Clinically,
patients often have signs and symptoms of right-sided HF on examination with clear
lungs. Diagnostically, transthoracic echocardiogram reveals respirophasic variation, where
during inspiration there is increased flow seen across the tricuspid valve and decreased
flow across the mitral valve. Other findings include expiratory hepatic vein flow reversal.
On simultaneous left and right heart catheterization, there is equalization of the ventricular
end-diastolic pressures between the right and left ventricles and discordance of the right
ventricular and left ventricular pressure peaks during the respiratory cycle.

57. The correct answer is: B. Initiation of high-dose nonsteroidal anti-inflammatory


drugs (NSAIDs). Pericarditis may manifest with ECG changes, including ST elevations
diffusely (often crossing multiple vascular territories such as leads I and II). The ST-
elevation morphology is classically concave upward. PR depression may be seen as well.
The exception to these changes is in lead aVR where there may be ST depression and PR
elevation as seen here. The ECG may ultimately reveal T-wave inversions.
High-dose NSAIDs remain the backbone of acute pericarditis treatment. In patients
with a recent MI, aspirin can be used. Colchicine should be added for a period of 3
months to decrease the risk of refractory or recurrent pericarditis. Steroids should only be
utilized in those patients with contraindications to NSAIDs or refractory pericarditis as
steroids have been shown to increase the risk of recurrence.

58. The correct answer is: A. Calculating his atherosclerotic cardiovascular disease risk
score and, if >10%, starting an antihypertensive agent now. Per the most recent 2017
ACC/AHA BP guidelines, a confirmed BP between 130-139/80-89 mmHg is considered
to be stage 1 hypertension. For this patient population, the recommendation is to initiate
an antihypertensive if clinical cardiovascular disease is present (ie, ischemic heart disease,
HF, stroke) or their calculated atherosclerotic cardiovascular disease risk is >10%.

59. The correct answer is: D. Lisinopril. First-line antihypertensives include ACE
inhibitors, calcium channel blockers, and diuretics. The choice of which antihypertensive
to begin with is guided by the presence of comorbidities. Patients with diabetes mellitus
with microalbuminuria should be treated first line with an ACE inhibitor, in the absence of
contraindications.

60. The correct answer is: B. Intravenous labetalol. In this patient with hypertensive
emergency complicated by aortic dissection, intravenous agents are needed initially to
achieve rapid HR and BP control. The use of a pure vasodilator can lead to reflex
tachycardia, thereby increasing left ventricular contractile force and potentially
propagating the aortic dissection. As such, the first choice of intravenous agent should
limit HR response such as a β-blocker. A vasodilator can then be added to achieve BP
control.

61. The correct answer is: C. Substitute labetalol for lisinopril. ACE inhibitors should be
discontinued in women planning to conceive, given the risk of fetus malformation during
the first trimester and renal failure in the second/third trimesters. The preferred agents for
hypertension during pregnancy are methyldopa, labetalol, and nifedipine. Hypertension
can worsen during pregnancy and cause significant harm to the fetus. Therefore, it is not
recommended to stop antihypertensive medications during pregnancy in women with an
indication for their continued use.

62. The correct answer is: A. Chlorthalidone. Occult volume overload often underlies
difficult-to-control hypertension. Therefore, all patients with concern for resistant
hypertension should be on a diuretic. The definition of resistant hypertension includes ≥3

Freemedicalbooks4download
drugs with at least one agent being a diuretic.

63. The correct answer is: A. MR angiography (MRA) of the renal arteries. In this young
patient with severe hypertension and an abdominal bruit, renal artery stenosis secondary to
fibromuscular dysplasia should be strongly suspected. An MRA of the renal arteries
would be a reasonable first step in her diagnostic evaluation. Although a duplex
ultrasound can be considered, it cannot definitively exclude fibromuscular dysplasia. A
noncontrast CT scan will not visualize the vasculature adequately. In some instances,
direct visualization with renal angiography may be required if clinical suspicion remains
high, but a diagnosis cannot be made noninvasively. For patients with confirmed
fibromuscular dysplasia in whom BP cannot be controlled, percutaneous transluminal
renal angioplasty should be considered.

64. The correct answer is: B. 6 months. All patients who have a newly discovered aortic
aneurysm should undergo repeat assessment at 6 months to ensure aneurysm stability and
rate of growth. Thereafter, thoracic aortic aneurysms (TAAs) can be monitored yearly if
stable.

65. The correct answer is: B. All first-degree relatives should be screened. Screening for
thoracic aortic aneurysms (TAAs) is recommended in all patients with a bicuspid aortic
valve. First-degree relatives of patients with a TAA, bicuspid aortic valve, or connective
tissue disorder should undergo screening for TAAs.

66. The correct answer is: C. Rate of growth of 0.5 cm in 6 months. Surgical repair of
abdominal aortic aneurysms should be performed in patients with aneurysms ≥5.5 cm in
size or those associated with symptoms. Surgical repair should be considered in females
with aneurysms ≥5 cm and in those patients in whom the rate of growth is >0.5 cm/y.

67. The correct answer is: A. Esmolol drip. The patient has a type A aortic dissection and
should have prompt surgical evaluation. In the interval, the key to upfront medical
management is to decrease the impulse (dP/dt) with each heartbeat, targeting a HR <60
beats/min and systolic BP <120 mmHg. BP should be titrated off the highest BP reading
as dissection involving the subclavian artery can result in falsely low readings. The
optimal first agent would be an esmolol drip—a β-blocker that provides predictable
pharmacokinetics with easy titratability. Starting with either nitroprusside or nitroglycerin
would cause reflex tachycardia and can propagate the dissection. Either of these may be
started following β-blocker initiation. Nitroprusside should be avoided with renal
dysfunction as it can precipitate cyanide toxicity. A continuous IV β-blocker drip would
likely offer better control and opportunities for titration than a single metoprolol IV push.

68. The correct answer is: A. Aortic dissection. The patient’s mediastinal silhouette is
enlarged, raising concern for aortic pathology such as dissection. An abnormal chest x-ray
is seen in 60% to 90% of patients with aortic dissection. A normal chest x-ray, however,
does not rule out aortic dissection or pathology. Given the clinical history of a tearing
back pain that was maximal at onset and remains severe, with hypertension,
hyperlipidemia, and tobacco use history, the patient in the clinical vignette is most likely
experiencing an acute aortic dissection.
There are lung markings extending to the periphery of the lung fields, making
pneumothorax incorrect. While the trachea appears to be displaced to the right, it may be
the result of the tortuous aorta and does not represent a tension pneumothorax, as both
lungs appear fully inflated in the chest x-ray. While there may be subtly increased
vascular markings in the lung fields, pulmonary edema is not the predominant finding. A
pericardial effusion that results in an increased cardiac silhouette is more likely to be
associated with tamponade (hypotension, tachycardia, JVD) than hypertension.

69. The correct answer is: A. Aortic dissection. Intra-aortic balloon pumps are used to
increase coronary perfusion pressure and augment cardiac output. One of the
complications of intra-aortic balloon pumps includes aortic dissection. Propagation of the
dissection into a subclavian, femoral, or carotid artery may cause pulse deficits. In the
clinical vignette, the balloon caused dissection, which propagated to involve the left
subclavian artery leading to loss of radial arterial line tracing. While worsening cardiac
output can cause distal pulses to be more difficult to palpate (in the setting of increased
systemic vascular resistance), the cardiac output and cardiac index are unchanged in the
vignette. In addition, the asymmetry of the examination makes a global process, such as
worsening cardiogenic shock unlikely. Radial artery spasm can occur, particularly if the
artery is catheterized, but it should not affect the brachial artery. Thromboembolism can
occur with large anterior MIs in the setting of left ventricular akinesis. This patient,
however, experienced an inferior-posterior ST-elevation myocardial infarction that is less
likely to lead to left ventricular thrombus compared to anterior MIs.

70. The correct answer is: D. Surgery. The patient has a body habitus that may be consistent
with Marfan syndrome, and his diastolic murmur may be suggestive of a proximal
dissection. Acute aortic regurgitation is present in approximately 44% of patients with a
proximal aortic dissection. The patient also has new signs of HF (bilateral pulmonary
rales), which may be a sign of proximal dissection leading to aortic regurgitation.
The treatment for a proximal aortic dissection (type A) is typically urgent surgery.
Distal dissections (type B) may be managed medically by decreasing dP/dt and targeting
HR <60 beats/min and central systolic BP <120 mmHg. While patients with Marfan
syndrome are at increased risk for pneumothorax, this patient has a diastolic murmur,
hypertension, and back pain, which is therefore concerning for a proximal dissection.

71. The correct answer is: B. Extension of the dissection below the diaphragmatic hiatus.
The acute renal failure, decreased urine output, and lactic acidosis are concerning for
impaired blood flow into the renal arteries and visceral blood supply. This could be due to
direct extension of the dissection into additional arteries or due to decreased blood flow
due to obstruction by a false lumen in the dissection flap. While proximal dissection can
cause hypotension, the mechanism is typically through acute aortic regurgitation,
tamponade, or cardiogenic shock. This patient has signs of malperfusion below the
diaphragm, making this the more likely etiology. Cyanide toxicity is a complication of
patients being treated with nitroprusside. Risk factors for cyanide toxicity include
hypoalbuminemia and high doses of nitroprusside. However, the patient has no evidence
of decreased oxygenation on his arterial blood gas (ABG), making this diagnosis less
likely. Alternative medications for afterload reduction should be considered given the
acute renal failure. Sedation effect should not cause renal failure or lactic acidosis.

Freemedicalbooks4download
72. The correct answer is: D. Transvenous temporary pacemaker. This patient has a
clinical syndrome concerning for early disseminated Lyme disease, including intermittent
fevers, recent history of a target-shaped rash, and travel to an endemic area. In this setting,
his ECG findings of complete heart block with a slow escape rhythm are likely secondary
to acute Lyme carditis. His complete heart block may resolve following a course of
antibiotics; therefore, a permanent pacemaker is not immediately indicated. However, a
temporary pacemaker should be placed in this scenario as he remains hemodynamically
unstable.

73. The correct answer is: B. Increased automaticity at multiple sites in the atria. The
ECG reveals an irregular rhythm with three distinct P-wave morphologies, which is
consistent with a multifocal atrial tachycardia (MAT). The mechanism of this arrhythmia
is believed to be through increased automaticity at multiple sites in the atria (each site
responsible for a distinct P wave). MAT is not common but is sometimes seen in patients
with chronic lung disease. Chaotic atrial activation originating from the pulmonary veins
is often the presumptive mechanism of AF (no P waves seen on ECG). Reentry through
dual pathways in the atrioventricular node is the mechanism of atrioventricular nodal
reentrant tachycardia. Reentry through the atrioventricular node and an accessory pathway
is the mechanism of atrioventricular reentrant tachycardia. Atrioventricular nodal
reentrant rhythms may exhibit a retrograde P wave on ECG.

74. The correct answer is: D. Procainamide. For patients with WPW, a rapid
supraventricular tachycardia may conduct through both the atrioventricular node and
accessory pathway. If a drug is administered that purely blocks the atrioventricular node,
then there is a risk that the rhythm might travel exclusively down the accessory pathway
and degenerate to ventricular fibrillation. For this reason, procainamide is the drug of
choice as it will stabilize the atrial rhythm. Digoxin and adenosine will primarily target the
atrioventricular node and are therefore contraindicated. Direct current cardioversion may
eventually be required but is not emergent at this time if the patient is stable.

75. The correct answer is: B. Long QT interval. This clinical vignette presents an otherwise
healthy 54-year-old woman who is admitted with pneumonia and is undergoing treatment
with antibiotics. Her recent exposure to broad-spectrum antibiotics and a polymorphic
ventricular tachycardia arrest raise concern for an acquired long QT syndrome. A pseudo-
RBBB with ST-segment elevation in V1 to V3 is a classic ECG pattern seen in Brugada
syndrome (a rare, congenital type of sodium channel mutation associated with sudden
cardiac death). T-wave inversion in V1 to V3 can be seen in arrhythmogenic right
ventricular change as well as other right ventricle cardiomyopathies; however, these are
typically associated with monomorphic ventricular tachycardia and are rare. LBBB can
suggest ischemia, which is an important cause of polymorphic ventricular tachycardia;
however, she is a relatively young woman with no cardiovascular symptoms or risk
factors and excellent cardiovascular fitness. In this case, the most likely cause of her
polymorphic ventricular tachycardia is an acquired long QT interval. Any QT-prolonging
therapies should be immediately discontinued.

76. The correct answer is: D. Radiofrequency ablation. The patient presents with a fourth
episode of monomorphic ventricular tachycardia over the past 2 months. This has been
occurring despite adequate medical therapy (β-blockers, mexiletine, amiodarone). Her
most recent coronary angiogram would suggest that her ventricular tachycardia is not
driven by ischemia. For recurrent ventricular tachycardia or ventricular tachycardia storm,
radiofrequency ablation can be an effective therapy and is the next most appropriate step
in care.

77. The correct answer is: C. Observation. The ECG tracing reveals second-degree
atrioventricular block type 1 (also called Mobitz I or Wenckebach). This arrhythmia is
often asymptomatic and almost typically can be monitored without intervention.
Symptomatic patients with second-degree atrioventricular block type 2 (also called
Mobitz II) or third-degree heart block may require a pacemaker. Atropine is indicated for
symptomatic bradycardia.

78. The correct answer is: C. Atrioventricular nodal reentrant tachycardia. The patient
presents with a narrow complex tachycardia at 150 beats/min. ECG reveals a regular rate.
A retrograde P wave can be seen immediately following the QRS complex in leads V1 and
V2, suggesting this rhythm is atrioventricular nodal reentrant tachycardia. ECGs of
patients with atrial flutter would reveal flutter waves at a typical circuit of 300 beats/min
that are best visualized in leads II, III, and aVF. AF is an irregular rhythm.
Atrioventricular nodal reentrant tachycardia can typically be terminated with IV adenosine
as it disrupts the reentrant path involving the atrioventricular node.

79. The correct answer is: C. Perform transesophageal echocardiogram (TEE); if no left
atrial thrombus, perform cardioversion. The patient presents with new AF complicated
by a tachycardia-induced cardiomyopathy. In patients with AF, a strategy of rate control
has been shown to have comparable outcomes to a strategy of rhythm control. However,
in this instance, she has inadequate rate control on her current regimen and her BP has
limited further titration. Therefore, restoration of sinus rhythm is likely to provide her the
best opportunity to recover left ventricle (LV) function. As the duration of her AF is
unknown, she should undergo TEE prior to any attempts at cardioversion. All patients
after electrical cardioversion should receive several weeks of anticoagulation. Decisions
about long-term anticoagulation should be made based on calculation of her risk of
embolic stroke. Typically, an ablation for AF is only pursued after several recurrences.

80. The correct answer is: A. Amiodarone. The patient has new-onset AF and a history of
an ischemic cardiomyopathy. Both flecainide and propafenone are class Ic antiarrhythmic
drugs and can be useful for the management of AF. However, class Ic drugs are
contraindicated in patients with ischemia or structural heart disease. Although amiodarone
has potential side effects, it can be used to maintain sinus rhythm in patients with
structural heart disease. A non-dihydropyridine calcium channel blocker such as diltiazem
may be helpful for rate control but is not typically considered to be an antiarrhythmic
therapy and is contraindicated in HF.

81. The correct answer is: D. Start warfarin. This patient presents with valvular AF
secondary to rheumatic mitral stenosis. The CHA2DS2-VASc score should be used to help
assess a patient’s risk of embolic stroke or systemic embolic events in patients with
nonvalvular AF. However, anticoagulation is indicated for patients with moderate or

Freemedicalbooks4download
severe rheumatic mitral stenosis in the presence of AF. Patients with moderate or severe
rheumatic mitral stenosis were excluded from trials of the novel or direct-acting oral
anticoagulants, including rivaroxaban, and therefore are not indicated for use. Clinical
practice guidelines recommend use of vitamin K antagonists, such as warfarin, to reduce
the risk of stroke in these patients.

82. The correct answer is: A. Andexanet alfa. This patient presents with a life-threatening
traumatic intracranial hemorrhage following a mechanical fall. She currently takes both
aspirin and apixaban, which increases her risk of bleeding. In the acute setting, reversal of
the effect of both agents would be indicated with both platelet transfusion and andexanet
alfa. Andexanet alfa is a recombinant factor Xa protein that reverses the effect of factor
Xa inhibitors, including apixaban. Idarucizumab is a monoclonal antibody that reverses
the effect of the direct thrombin inhibitor dabigatran. Fresh frozen plasma contains all
factors in the soluble coagulation system and can be utilized to restore factor deficiencies
in patients who are bleeding or are planned to undergo procedures. Vitamin K will reverse
the effects of warfarin but not a factor Xa inhibitor.

83. The correct answer is: B. Direct-acting oral anticoagulants are not indicated for
patients with AF with rheumatic mitral stenosis. Direct-acting oral anticoagulants are
not indicated for patients with AF with rheumatic mitral stenosis as these patients are at
very high risk of stroke and were excluded from trials assessing efficacy of direct-acting
oral anticoagulants in prevention of stroke in AF. Warfarin is the only oral anticoagulant
that should be used for patients with a mechanical heart valve due to a trial that indicated
that dabigatran is inferior to warfarin in that setting. Reversal agents are now available for
all the approved direct-acting oral anticoagulants. In head-to-head clinical trials, the
direct-acting oral anticoagulants have been shown to have a lower risk of intracranial
hemorrhage when compared to warfarin in patients with AF.

84. The correct answer is: D. Percutaneous left atrial appendage occlusion. Percutaneous
left atrial appendage occlusion offers an alternative to anticoagulation for reduction of
stroke and systemic embolism. The efficacy of half-dose apixaban to prevent stroke has
not been established and may still increase her risk of further intracranial hemorrhage.
Aspirin alone is not believed to significantly attenuate risk of stroke or systemic embolic
event in a patient with AF and will still increase the risk of bleeding. Antiarrhythmic
therapy such as amiodarone does not sufficiently lower the risk of stroke in a patient with
AF to preclude anticoagulation.

85. The correct answer is: D. Long-term external cardiac event monitoring device. The
salient features of this case include a syncope presentation with the frequency of
symptoms occurring on a weekly basis. Thus, a 48-hour Holter monitor may not offer a
sufficiently durable time interval to capture the event. Event recorders rely on activation
by the patient, which may be impractical in patients experiencing syncope with limited
prodrome. Implantable loop recorders can record up to 3 years and are generally
considered in patients with a suspicion of arrhythmogenic syncope too infrequent to be
captured by alternative means. A long-term external cardiac event monitoring device
provides continuous monitoring for up to 14 days, a sufficient period of time to identify a
potential cause in this case and does not rely on patient activation for recording.
86. The correct answer is: D. History and physical. Evaluation of syncopal presentations is
challenging and an etiology may not be determined in upward of 40% of cases. A history
and physical with orthostatic vital signs have been shown to have the highest yield and
cost-effectiveness in identifying a cause for syncope. ECG can demonstrate abnormalities
in 50% of syncope cases but is only helpful in elucidating a cause for syncope in 10% of
cases. A transthoracic echocardiogram can be considered to evaluate for structural heart
disease, but even in the presence of structural heart disease, more than one potential cause
may be contributing for which a history and physical remain most useful. Further
evaluation for evidence of severe carotid stenosis should be guided by clinical
examination.

87. The correct answer is: B. Neurocardiogenic. About 25% of syncopal episodes are
deemed to be from a neurocardiogenic or vasovagal cause, often precipitated by increased
vagal tone related to cough, defecation, carotid hypersensitivity, or micturition. This leads
to a reactive decrease in HR and BP. Evaluation of orthostatic causes is important.
Orthostatic vital signs are considered positive if 15 seconds after moving from a supine to
standing position there is a >20 mmHg drop in systolic BP, >10 mmHg drop in diastolic
BP, or a >30 beats/min increase in HR. Arrhythmic cardiovascular causes are important to
consider. In patients with no known cardiovascular disease at baseline, <5% of patients
will have a cardiovascular explanation for their syncope. Neurologic causes explain about
10% of syncopal presentations. Given the presence of collateral information and the
benign clinical examination with complete resolution of symptoms, a neurocardiogenic
cause is more likely.

88. The correct answer is: B. Lead perforation leading to cardiac tamponade. This
patient’s hypotension and elevated neck veins in the context of a pacemaker implantation
earlier in the day are suggestive of cardiac tamponade secondary to lead perforation at the
time of the procedure, resulting in a pericardial effusion. A blunted Y descent is typically
observed in the jugular veins because there is impaired right ventricle filling during
diastole. Pacemaker syndrome is typically described in patients who have a pacemaker
that does not allow for synchrony between the atrium and ventricle, thereby leading to a
sensation of pulsations and fullness in the neck due to the atrium contracting against a
closed tricuspid valve. Pacemaker-mediated tachycardia is a wide, complex tachycardia
that occurs when retrograde P waves (due to the loss of atrioventricular synchrony) are
sensed and tracked in an atrial tracking mode by the pacemaker. Flash pulmonary edema
is not consistent with the history and the physical examination, which was notable for
clear breath sounds.

89. The correct answer is: D. Both A and C. Patients with an irreversible ischemic
cardiomyopathy and a left ventricular ejection fraction <35% have been shown to have a
mortality benefit from placement of a primary prevention implantable cardiac
defibrillator. Cardiac resynchronization therapy has also been shown to have a mortality
benefit in patients with symptomatic HF and a reduced EF if they also have a wide QRS
on ECG (>120 ms, ideally LBBB >150 ms). Patients should first be medically optimized
before device placement is considered. Cardiac resynchronization is achieved by
placement of a left ventricular lead that allows the left ventricle (LV) to contract with
greater synchrony. In addition to a mortality benefit, cardiac resynchronization may

Freemedicalbooks4download
increase the EF and reduce HF symptoms. A dual-chamber pacemaker is typically
indicated for atrioventricular block, sinus node dysfunction, and specific forms of
syncope; however, right ventricle pacing may worsen HF by creating more dyssynchrony.

90. The correct answer is: C. Plan for pacemaker system removal. This patient presents
with evidence of pacemaker infection as evidenced by fever, positive blood cultures
growing S. aureus, and erythema around his device site. Despite a negative
transesophageal echocardiogram (TEE), he has staphylococcal bacteremia and merits
definitive therapy with system removal. Continuing oral or intravenous antibiotics without
system removal would be inadequate therapy. He has no evidence of endocarditis and
therefore does not merit valve surgery.

91. The correct answer is: C. Proceed to surgery. This patient presents for elective open
colectomy. She denies active cardiac conditions such as angina or HF and maintains >4
METs in physical activity. For this reason, she should proceed with planned surgical
intervention without additional testing. Patients with surgical emergencies can also be
brought to the OR without further testing. She should continue her aspirin and β-blocker
perioperatively as she has an indication to continue taking both.

92. The correct answer is: C. Postpone cholecystectomy until valve intervention is
completed. This patient presents for elective cholecystectomy; however, she also has
severe symptomatic aortic stenosis requiring surgery. Given the risk for procedural
complications with untreated severe valvular disease, her elective abdominal surgery
should be postponed until after her valve intervention. There is no clear indication for
stress testing or initiation of furosemide in this patient.

93. The correct answer is: C. Postpone knee replacement for 6 to 12 months. The patient
just had an MI and underwent placement of a drug-eluting stent. Ideally, he should be
continued on dual antiplatelet therapy for at least 12 months after his acute coronary
syndrome with stent. In patients with emergent indications for surgery, surgery can
proceed and ideally the patient would be continued on dual antiplatelet therapy.
Consideration can be given to stopping dual antiplatelet therapy at approximately 6
months in certain lower risk patients who require elective procedures.

94. The correct answer is: A. Ankle-brachial index. The ankle-brachial index is a
measurement comparing the BP of a patient’s arm with the BP in the ankle (posterior
tibial and dorsalis pedis), utilizing a BP cuff and ultrasonography. A normal ankle-
brachial index is between 1.0 and 1.4, borderline values are between 0.91 and 0.99. An
abnormal ankle-brachial index is ≤0.9. Conversely, if the value is ≥1.40, the test is
nondiagnostic as the vessel may be incompressible due to calcifications. If the ankle-
brachial index is ≥1.40, the next best step would be to check pulse volume recording,
which may help localize disease in calcified vessels. The other diagnostic tests listed are
utilized to diagnose, quantify, and plan intervention on peripheral artery disease (PAD).
However, with stage 4 chronic kidney disease, contrast-sparing studies (such as ankle-
brachial index) are the optimal first test.

95. The correct answer is: C. Obtain urgent vascular surgery or vascular medicine
consultation. The patient is presenting with signs of acute limb ischemia. This is an
emergent condition and requires immediate imaging and urgent consultation with either
vascular surgery or vascular medicine. Anticoagulation may be utilized if there is concern
for acute arterial thrombosis. With severe renal dysfunction, however, low-molecular-
weight heparin is contraindicated and unfractionated heparin should be used
preferentially. Arterial Dopplers are a diagnostic test for acute limb ischemia. However,
delaying until the morning would not be appropriate given the threat to the limb. Placing
warm blankets on the foot will not reverse the acute limb ischemia and could be
dangerous in a patient with decreased sensation.

96. The correct answer is: C. Segmental ankle-brachial index with pulse volume
recording. On a pulse volume recording, the arterial waveform will dampen when blood
flow is impaired in an artery. This will allow the areas of peripheral artery disease (PAD)
to be localized. A venous ultrasound is useful for evaluation of deep venous thrombosis
but does not have a role in the evaluation of peripheral arterial disease. While angiography
in the cardiac catheterization lab and CTA do offer delineation of vascular anatomy,
modalities that do not require contrast should be favored in this patient with chronic
kidney disease.

Freemedicalbooks4download
2
PULMONARY

QUESTIONS

1. A 75-year-old man with a history of hypertension and peptic ulcer disease presents with
dyspnea on exertion to the emergency department (ED). He notes black-colored stools for
the past 2 weeks. His initial examination is notable for conjunctival pallor, clear lung
fields without rales, wheezing, or rhonchi, as well as tachycardia with a regular rhythm,
normal s1 and s2, and without murmurs, rubs, or gallops. Initial laboratory workup reveals
a hemoglobin of 5.5 g/dL, reduced from a baseline of 12 g/dL on routine outpatient
laboratory testing from 3 months prior. His basic metabolic panel, arterial blood gas
(ABG), and lactate are all within normal limits. A chest radiograph (CXR) is normal.
What is the mechanism of this man’s dyspnea on exertion?
A. Decreased cardiac output
B. Decreased oxygen delivery
C. Decreased systemic vascular resistance
D. High-output heart failure
E. Pulmonary edema

2. A 75-year-old woman with a 50-pack-year history of cigarette smoking presents to her


primary care physician (PCP) with 10 years of progressive dyspnea on exertion. She notes
episodic wheezing and experiences a “chest cold” about two times per year. Her
examination reveals distant breath sounds to auscultation and hyper-resonant chest to
percussion, with otherwise unremarkable examination. A CXR demonstrates
hyperinflation with flattening of the bilateral hemidiaphragms. Pulmonary function tests
(PFTs) reveal a forced expiratory volume (FEV1) of 50% predicted, forced vital capacity
(FVC) of 70% predicted, and FEV1/FVC of 0.50. There is no response to bronchodilator.
What pattern best describes her PFT results?
A. Cannot determine without more information
B. Normal
C. Obstructive
D. Restrictive
3. A 65-year-old man with a 20-pack-year history of cigarette smoking is referred to
pulmonary clinic with 6 months of progressive dyspnea on exertion. He has been
prescribed an albuterol inhaler by his PCP without improvement. He reports an occasional
dry cough, but otherwise denies chest pain, fevers, chills, weight loss, or other associated
symptoms. His examination reveals fine inspiratory crackles at the bases of both lung
fields, with a normal cardiovascular examination, absence of any lower extremity edema,
absence of fingernail clubbing, and otherwise unremarkable examination. A CXR
demonstrates scattered reticular opacities at the bilateral lung bases. PFTs reveal an FEV1
of 50% predicted, FVC of 45% predicted, and FEV1/FVC of 0.95. There is no response to
bronchodilator.
What pattern best describes his PFT results?
A. Cannot determine without more information
B. Normal
C. Obstructive
D. Restrictive

4. A 40-year-old woman without past medical history presents for evaluation of 3 months of
progressive dyspnea on exertion. She denies other associated symptoms such as cough,
chest pain, or hemoptysis. She undergoes PFTs, which reveal a reduced diffusion capacity
of 45% predicted. Her spirometry and total lung capacity are within normal limits.
Which of the following diseases could the PFT findings NOT support?
A. Early interstitial lung disease
B. Neuromuscular disease
C. Pulmonary embolism
D. Pulmonary hypertension

5. A 30-year-old woman with a history of asthma presents to her PCP complaining of


occasional shortness of breath and wheezing despite her current asthma treatment. She
currently takes a low-dose inhaled corticosteroid and is requiring her albuterol rescue
inhaler daily.
Which of the following management options would NOT be appropriate at this
time?
A. Addition of a leukotriene receptor antagonist to her current regimen
B. Addition of a long-acting β-agonist to her current regimen
C. Addition of prednisone 5 mg oral daily to her current regimen
D. Increasing the inhaled corticosteroid to medium dose

6. A 25-year-old man with a history of asthma, prescribed fluticasone/salmeterol twice daily


and albuterol, as needed, presents to the ED with wheezing and severe shortness of breath
for the past day. Additionally, he has had 2 days of rhinorrhea and dry cough. He states
that his symptoms began after exposure to a sibling who is suffering from an upper
respiratory infection (URI). His vital signs are as follows: temperature (T) 37°C, heart rate
(HR) 90 beats/min, blood pressure (BP) 130/80 mmHg, respiratory rate (RR) 24
breaths/min, and SaO2 97% on 2 L/min via nasal cannula. His examination is notable for
decreased air movement throughout all lung fields with scattered wheezing. A CXR is
unremarkable.

Freemedicalbooks4download
What is the next best step in his management?
A. Administer albuterol nebulizers and IV methylprednisolone
B. Administer epinephrine 1:1000 dilution, 0.3 mL IM
C. Administer magnesium 2 g IV
D. Apply noninvasive positive-pressure ventilation
E. Perform computed tomography (CT) scan of his chest

7. A 50-year-old woman with a history of asthma is referred to a pulmonary clinic for recent
worsening of her symptoms with frequent intermittent wheezing. She has a 20-year
history of asthma, which had been well-controlled with albuterol and a low-dose inhaled
corticosteroid. She denies known exposure to cigarette smoke, vaping, or new allergens.
She has lived in the same home for the past 5 years and has recently been renovating her
basement. Basic laboratory evaluation is notable for a normal white blood cell (WBC)
count with an absolute eosinophil count of 2000 cells/μL. PFTs reveal a worsening
obstructive pattern.
Which of the following is the next most appropriate test?
A. Antinuclear antibody (ANA)
B. Flexible bronchoscopy
C. Methacholine challenge PFT
D. Total IgE and Aspergillus-specific IgE

8. A 40-year-old man has long-standing asthma that has been difficult to control with
standard therapies. He requires rescue albuterol treatment multiple times daily despite
treatment with a high-dose inhaled corticosteroid, a long-acting β-agonist, a leukotriene
receptor antagonist, and oral corticosteroids. He has removed all allergens from his home,
and evaluation with an allergist did not reveal any modifiable allergen exposures.
What is the next best step in his management?
A. Adding daily albuterol nebulizer treatments
B. Checking serum eosinophil count and IgE to consider immunologic therapy
C. Increasing the dose of oral corticosteroids
D. No change to management
E. Repeating allergy testing

9. A 45-year-old man with non-Hodgkin lymphoma experiences wheezing, dyspnea, and


hypotension several minutes after the initiation of his second infusion of rituximab
treatment. He was asymptomatic at the time of arrival to the infusion appointment. His
vital signs are as follows: T 37.4°C, HR 115 beats/min, BP 100/60 mmHg, RR 28
breaths/min, and SaO2 98% on room air. On examination, he is noted to be in acute
distress with diffuse wheezing and urticaria on his abdomen and chest.
What is the immediate first-line management?
A. Administer 50 mg diphenhydramine IV
B. Administer 0.3 mL of 1:1000 dilution epinephrine IM
C. Administer 0.3 mL of 1:10 000 dilution epinephrine IV
D. Perform emergent intubation

10. A 23-year-old man with a history of allergic reaction to peanuts presents to the ED with
urticaria after an accidental exposure to peanuts at a restaurant. His vital signs are stable,
and his examination is only notable for urticaria on the chest and upper back. He reports a
history of similar reaction and denies any associated symptoms such as dyspnea, facial
swelling, or throat tightness at this time.
Which of the following statements about management of allergic reactions is
FALSE?
A. A biphasic reaction with recurrence of anaphylaxis may occur anywhere from 8 to 72
hours after an initial episode of anaphylaxis.
B. All patients should be admitted to an inpatient ward for management.
C. Observation for 6 to 12 hours is appropriate for reactions limited to urticaria.
D. Patients who are observed with resolution of symptoms should be prescribed
epinephrine autoinjector pens at the time of discharge.

11. A 52-year-old man with a history of chronic obstructive pulmonary disease (COPD)
presents to his PCP’s office for a routine visit. He is on albuterol four times a day as
needed. He reports that overall, he has had an increase in symptoms in the last 6 months.
He reports that he was treated for a COPD flare with steroids about 5 months ago. He has
had no further flares requiring him to go to the hospital, but he has had worsening
shortness of breath and increased sputum production recently. His PFTs are notable for an
FEV1 of 71%.
What medication should be added to his medication regimen?
A. Increase frequency of albuterol to six times a day
B. Start daily azithromycin
C. Start tiotropium
D. Start triple therapy (an inhaled corticosteroid–a long-acting muscarinic antagonist – a
long-acting β-agonist)

12. Two months later, the patient in Question 11 presents to the ED with shortness of breath
and wheezing in the setting of a recent viral URI. He is found to be hypoxemic to 86% on
room air. He is started on nasal cannula oxygen and his SpO2 improves to 99%. He is later
found to be somnolent by his nurse.
What is the next best step in his management?
A. Decrease oxygen for goal SpO2 88% to 92%
B. Give stacked ipratropium nebs
C. Initiate noninvasive positive-pressure ventilation
D. Start methylprednisolone 125 mg IV Q6H

13. An 80-year-old woman with a history of severe COPD presents to her PCP’s office for
increased dyspnea over the past 9 months. PFTs demonstrate a progressive worsening in
her obstructive deficit over the past 5 years—currently, her FEV1 is 18% of predicted. She
takes a combination of a long-acting β-agonist, a long-acting muscarinic antagonist, and
an inhaled corticosteroid daily. She uses albuterol two times per day. Her oxygen
saturation is noted to be 87% at rest, which has declined from 90% at the last visit.
Which of the following statements about oxygen therapy in COPD is true?
A. Eliminates the need for an inhaled corticosteroid
B. Prolongs survival in patients with oxygen saturation of 88% or less at rest
C. Reduces frequency of exacerbations

Freemedicalbooks4download
D. Reduces hospitalizations for COPD

14. A 75-year-old woman with a history of hypertension and frequent pulmonary infections
for the past 5 years presents to her PCP’s office with chronic cough, shortness of breath,
and copious sputum production. Examination is notable for inspiratory squeaks but is
otherwise unremarkable. CXR demonstrates tram-tracking in the lower lung fields.
Which of the following tests is most likely to determine the etiology of this
patient’s symptoms?
A. Antinuclear antibody (ANA)
B. Chest CT and sputum cultures (including mycobacterial and fungal cultures)
C. IgA, IgG, IgM, and IgG subclasses
D. Sweat testing

15. Two weeks later, the patient in Question 14 presents to the ED with severe hemoptysis.
She has expectorated approximately 600 mL of blood in the past 24 hours and she
continues to have episodes of hemoptysis. Her HR is 89 beats/min, BP is 129/89 mmHg,
and O2 saturation is 93% on 2 L by nasal cannula. CXR shows a patchy opacification in
the right mid-lung field.
What is the next best step in her management?
A. Give cough suppressant
B. Obtain stat chest CT
C. Place the patient R-side down
D. Transfuse 1 U packed red blood cells

16. A 22-year-old man with cystic fibrosis is discharged from the hospital following an acute
exacerbation. He completes 4 weeks of antibiotic treatment targeting Pseudomonas
aeruginosa from his sputum culture. He continues to take lumacaftor as prescribed.
Which additional steps are essential to maintaining lung function and avoiding
future exacerbation?
A. 9% NaCl nebulizer at least twice daily
B. Airway clearance at least twice daily with a high-frequency oscillation vest
C. Monthly bronchoscopy
D. Weekly monitoring of pulmonary function

17. A 70-year-old man with a history of hypertension, coronary artery disease, and without
past tobacco use presents to an outpatient pulmonary clinic for evaluation of a solitary
pulmonary nodule. The nodule was found incidentally on a chest CT performed 3 weeks
earlier in an ED for chest pain. He denies cough, dyspnea, hemoptysis, weight loss,
anorexia, or other associated symptoms. Review of his chest CT reveals a 5-mm solid
nodule with smooth borders in the right upper lobe, without other associated abnormalities
noted.
Which of the following statements is INCORRECT regarding solitary pulmonary
nodules in general?
A. A 9-mm high-risk pulmonary nodule should be followed with CT chest every 12
months.
B. A history of smoking and cancer are risk factors for malignant nodules.
C. Positron emission tomography (PET)-CT is a sensitive study for ruling out
malignancy of nodules >8 mm.
D. The majority of solitary pulmonary nodules are benign.

18. What is the next best step in management of the patient in Question 17?
A. Bronchoscopy with biopsy
B. No further workup necessary
C. Serial CT imaging to follow nodule at 12-month intervals
D. Surgical lung biopsy

19. A 35-year-old woman presents to her PCP’s office with cough and shortness of breath.
She denies joint pain or swelling, rashes, fevers, or night sweats. She does recall a tender
red rash on her R shin about 2 months prior, which self-resolved. The patient works as a
secretary and does not have any pets. She has never smoked. Examination is notable for
fine crackles throughout the lung fields, but no skin or joint findings. CT scan is
performed, which shows enlarged hilar lymph nodes and fibrotic changes. PFTs are
notable for a mixed obstructive and restrictive pattern.
What is the most likely diagnosis?
A. Hypersensitivity pneumonitis
B. Idiopathic interstitial pneumonia
C. Sarcoidosis
D. Scleroderma

20. For the patient in Question 19, angiotensin-converting enzyme (ACE) level, antinuclear
antibody (ANA), rheumatoid factor, antineutrophilic cytoplasmic antibody (ANCA),
cyclic citrullinated peptide (CCP), SSA/SSB, and Scl-70 are all within normal limits.
What is the most appropriate diagnostic step?
A. Bronchoscopy with bronchoalveolar lavage (BAL)
B. Endobronchial ultrasound (EBUS)-guided lymph node biopsy
C. Myositis antibody panel
D. Video-assisted thoracoscopic surgery (VATS) lung biopsy

21. A 65-year-old man with a 25-pack-year smoking history presents to the pulmonary clinic
for continued management of idiopathic pulmonary fibrosis. After a recent exacerbation
of idiopathic pulmonary fibrosis, he underwent a CT scan of his chest to reassess the
extent of his disease.
Which of the following is NOT a CT feature of idiopathic pulmonary fibrosis?
A. Apical cystic changes
B. Bibasilar honeycombing
C. Subpleural reticular opacities
D. Traction bronchiectasis

22. A 55-year-old man undergoes chest CT scan after presenting to the ED with blood-tinged
cough and pleuritic chest pain. The CT scan shows bilateral nodular opacities and masses
in the peripheral lung fields with some areas of cavitation. Pulmonary emboli are not
present. In the past 6 months, he has also noted new hearing loss and intermittent
epistaxis. Additional testing is notable for a urinalysis with many red blood cells present
on microscopy.

Freemedicalbooks4download
Which of the following tests would support this man’s diagnosis?
A. Antineutrophilic cytoplasmic antibody (ANCA)
B. CT scan with interstitial lung disease protocol
C. D-dimer
D. Transthoracic echocardiogram

23. A 46-year-old woman with a history of diabetes, ischemic cardiomyopathy (ejection


fraction [EF] 45%), and current smoking presents to the ED with shortness of breath and
productive cough. Vital signs are as follows: T 37.6°C, HR 96 beats/min, BP 112/68
mmHg, RR 28 breaths/min, and SpO2 91% on room air. CXR is performed that shows a
right-sided pleural effusion. Thoracentesis is performed; 500 mL of fluid is removed.
Fluid studies are notable for lactate dehydrogenase (LDH) of 202 units/L, protein 4.1
g/dL, cholesterol 52 mg/dL,glucose 49 mg/dL, and pH 7.1. Serum protein is 6.8 g/dL and
serum LDH is 243 units/L. There is a predominance of polymorphonuclear neutrophils
(PMNs) on cell count.
This effusion is most likely due to which of the following underlying conditions?
A. Congestive heart failure exacerbation
B. Infection
C. Malignancy
D. Rheumatoid arthritis

24. What is the next best therapeutic step for the patient in Question 23?
A. Indwelling pleural catheter
B. No intervention
C. Serial thoracentesis
D. Tube thoracostomy

25. A 72-year-old man with a history of heart failure (EF of 38%), type 2 diabetes, and a
remote history of stage 1 lung cancer status post resection presents to the ED with
worsening dyspnea on exertion and orthopnea. Vital signs are as follows: T 35.9°C, HR
67 beats/min, BP 120/78 mmHg, RR 18 breaths/min, and SpO2 91% breathing ambient
air. Examination is notable for diminished breath sounds at the bases bilaterally and
diffuse coarse crackles, elevated jugular venous pressure, and 2+ pitting edema in the
lower extremities bilaterally. Chest x-ray is notable for bilateral pleural effusions and
interstitial edema. The patient is admitted to the hospital for IV diuresis. On hospital day
4, the patient has lost 4 kg of weight and his lower extremity edema has improved
significantly; however, he continues to complain of shortness of breath. Repeat chest x-
ray demonstrates persistence of the bilateral effusions but improved interstitial edema.
The medical team proceeds with right-sided thoracentesis, and 700 mL of straw-colored
fluid is removed. Pleural fluid studies are notable for LDH 77 units/L, protein 4.1 g/dL,
cholesterol 35 mg/dL, glucose 100 mg/dL, and pH 7.4. There are no WBCs or red blood
cells present. Cytology examination is unremarkable. Serum studies are notable for LDH
187 units/L, protein 7.2 g/dL, and glucose 110 mg/dL.
What is the most likely cause of this effusion?
A. Congestive heart failure
B. Pneumonia
C. Pulmonary embolism
D. Recurrence of lung cancer

26. A 77-year-old man with a history of diabetes, hypertension, and colon cancer is admitted
for surgical resection.
Which of the following should be used for thromboprophylaxis?
A. Apixaban
B. Enoxaparin
C. Fondaparinux
D. Mechanical prophylaxis

27. The patient in Question 26 has a complicated postoperative course and a prolonged
hospitalization. He is eventually discharged to a rehab facility. On his fifth day at rehab,
he develops shortness of breath and tachycardia. His SpO2 is 89% on room air. CXR is
unremarkable and labs are pending.
What is the best diagnostic test to use in this patient?
A. CT pulmonary angiogram
B. D-dimer
C. Echocardiogram
D. V/Q scan

28. The patient in Questions 26 and 27 is found to have a pulmonary embolus in the right
pulmonary artery. He is started on a heparin drip.
What agent should be used in this patient to treat his pulmonary embolus on
discharge?
A. Apixaban
B. Argatroban
C. Low-molecular-weight heparin (LMWH)
D. Warfarin

29. A 45-year-old woman with a history of type 2 diabetes presents to her PCP with pain and
swelling in her left lower extremity. D-dimer was 1200 ng/mL. A lower extremity
ultrasound demonstrates a deep vein thrombosis (DVT) in the left popliteal vein. She has
no known risk factors or recent provoking factors that would explain this DVT. She
undergoes thorough evaluation, including age-appropriate cancer screening, which is
unremarkable.
Which of the following anticoagulation approaches is most appropriate?
A. Apixaban for 3 months, then reassessment with patient counseling regarding ongoing
anticoagulation
B. Apixaban for 4 weeks
C. Lifelong apixaban
D. No anticoagulation; compression stockings only

30. The patient in Question 29 is worried about developing a pulmonary embolus and asks if
there is anything she can do to prevent this.
What is the next best step in her management?
A. Continue apixaban
B. Continue apixaban and place inferior vena cava (IVC) filter

Freemedicalbooks4download
C. Discontinue apixaban and place IVC filter
D. Start half-dose apixaban and place IVC filter

31. A 52-year-old woman who has not seen a doctor in over 20 years comes to the ED for
evaluation of dyspnea on exertion. She has an oxygen saturation of 92%. She is admitted
to the internal medicine service for further workup. CXR is within normal limits and CT
scan is without interstitial process. PFTs show mild restriction and no obstruction.
Transthoracic echocardiogram shows an EF of 54%, normal valves, and right ventricular
systolic pressure of 55 mmHg. She then undergoes right heart catheterization. Pulmonary
capillary wedge pressure is 13 mmHg and mean pulmonary artery pressure is 35 mmHg.
What is the most likely cause of her dyspnea?
A. COPD
B. Heart failure
C. Idiopathic pulmonary arterial hypertension (PAH)
D. Interstitial lung disease

32. The patient in Question 31 follows up with a pulmonary hypertension specialist who
decides to start her on a medication.
What is the first-line treatment for this patient?
A. Epoprostenol
B. Nifedipine
C. Riociguat
D. Sildenafil

33. A few months later, the patient in Questions 31 and 32 presents to the ED with an episode
of presyncope when walking her dog. She is taken to the ED. Vital signs on arrival are
notable for T 37°C, HR 118 beats/min, BP 78/41 mmHg, RR 22 breaths/min, and SpO2
82% on room air (that improves to 88% on 6 L via nasal cannula). Examination is notable
for elevated jugular venous pressure and cool extremities. Lactic acid is elevated to 4.0
mmol/L. Bedside ultrasound shows a dilated and diffusely hypokinetic right ventricle.
What is the next best step in her management?
A. Give 1 L bolus of 0.9% NaCl solution
B. Increase sildenafil dose
C. Initiate inotropic and vasopressor support
D. Intubate

34. A 100-kg, 77-year-old man is admitted to the medical intensive care unit (ICU) for
multifocal pneumonia and intubated for hypoxemia. Following intubation, his SpO2 is
87% on volume control with the following settings: VT 500 mL, RR 15 breaths/min,
positive end-expiratory pressure 5 cm H2O, and FiO2 70%.
How should the ventilator be adjusted?
A. Increase FiO2
B. Increase positive end-expiratory pressure
C. Increase RR
D. Increase VT
35. On hospital day 2, the patient in Question 34 has an acute desaturation event and the
ventilator alarms for high pressures (peak inspiratory pressure 46 cm H2O, previously 27
cm H2O). Plateau pressure is unchanged at 22 cm H2O.
What is the most likely etiology of this change?
A. Auto-positive end-expiratory pressure
B. Excessive secretions
C. Pneumothorax
D. Vent asynchrony

36. The patient in Questions 34 and 35 shows steady improvement, and the team plans to
work toward extubation and decreases the pressure support from 10/5 cm H2O with FiO2
30% to 5/5 cm H2O with FiO2 30%.
Which of the following would suggest that the patient is not ready for extubation?
A. Decrease in mean arterial pressure (MAP) from 75 to 65 mmHg
B. Decrease in RR from 20 to 12 breaths/min; decrease in tidal volumes from 380 to 340
mL
C. Fall in SpO2 from 96% to 90%
D. Increase in RR from 20 to 30 breaths/min; decrease in tidal volumes from 380 to 280
mL

37. A 26-year-old man with a history of asthma develops fevers to 38.8°C, chills, sore throat,
and myalgias. He presents to his PCP’s office where he is found to be influenza A-
positive. He is started on oseltamivir. Two days later he develops severe shortness of
breath. His roommate becomes concerned and brings him to the ED.
Vital signs on arrival to the ED are as follows: T 40°C, HR 119 beats/min, BP
96/54 mmHg, RR 28 breaths/min, and SpO2 83% on room air. He is placed on high-flow
nasal cannula with improvement in his SpO2 to 93%. During the ED course, he is given 2
L lactated Ringer (LR) for ongoing hypotension as well as broad-spectrum antibiotics. On
arrival to the ICU, he is noted to be hypoxemic to 79% despite high-flow nasal cannula at
maximal settings. He is subsequently intubated. Postintubation CXR is notable for diffuse
bilateral infiltrates. Postintubation ABG is 7.3/30/80 on 100% FiO2.
Which of the following should NOT be part of the ventilator strategy?
A. Keeping FiO2 <0.6
B. Low tidal volume ventilation with VT <6 mL/kg
C. Maintaining pH >7.3
D. Titrating positive end-expiratory pressure to prevent tidal alveolar collapse

38. Despite optimizing positive end-expiratory pressure and initiating paralytics, the patient in
Question 37 continues to have a P/F ratio <100.
Which of the following interventions has been found to have a mortality benefit in
acute respiratory distress syndrome?
A. Early extracorporeal membrane oxygenation (ECMO)
B. Proning
C. Pulmonary vasodilators (iNO, prostacyclins)
D. Steroids

Freemedicalbooks4download
39. A 66-kg, 75-year-old man with a history of hypertension, atrial fibrillation, COPD,
diabetes, and chronic kidney disease on hemodialysis presents to the ED from his assisted
living facility after his wife found him confused and unable to get off of the couch the
evening after a dialysis session. On EMS arrival, the patient was found to be hypotensive
to 78/33 mmHg, so he was taken to the ED for further management. He received 1 L by
nasal cannula en route to the hospital.
Vital signs on arrival to the ED are as follows: T 35.6°C, HR 82 beats/min, BP
92/52 mmHg, RR 28 breaths/min, and SpO2 88% on room air. On examination he is
somnolent. Lungs are clear and abdomen is soft. Labs are notable for WBC count of 11 ×
109/L , creatinine 4.9 mg/dL, and lactate 3.7 mmol/L. Blood cultures are pending.
What is the next best step in his management?
A. Give an additional liter of normal saline for a total of 2 L
B. Start broad-spectrum antibiotics
C. Start hydrocortisone and fludrocortisone
D. Start norepinephrine

40. What antibiotic regimen is most appropriate in the patient from Question 39?
A. Ceftriaxone/azithromycin
B. Levofloxacin
C. Vancomycin/cefepime
D. Vancomycin/cefepime/metronidazole/micafungin

41. The BP of the patient in Questions 39 and 40 falls to 75/40 mmHg despite 2 L lactated
Ringer (LR) fluid resuscitation.
What is the initial pressor of choice in this patient?
A. Dopamine
B. Norepinephrine
C. Phenylephrine
D. Vasopressin

42. A 44-year-old woman with a history of depression, hypertension, and asthma is taken to
the ED by EMS after being found down at home with a bottle of empty pills nearby. Vital
signs on arrival to the ED are as follows: T 36.2°C, HR 88 beats/min, BP 80/40 mmHg,
RR 20 breaths/min, and SpO2 92% on room air. Examination is notable for a somnolent
woman who is slow to respond to questions. Tongue lacerations are present. Neurologic
examination is notable for confusion. Reflexes are normal. Electrocardiogram (ECG) is
notable for a QRS of 140 ms and a QT interval of 560 ms.
Overdose of which medication is most likely responsible for her presenting
symptoms?
A. Amitriptyline
B. Carvedilol
C. Diltiazem
D. Lithium

43. What is the next best step in the management of the patient from Question 42?
A. Flumazenil
B. Glucagon
C. IV sodium bicarbonate
D. Urgent dialysis

44. A 45-year-old man with cystic fibrosis has had progressive decline over the past 2 years.
His FEV1 is now 28% and he requires 6 L nasal cannula at all times. He is referred to a
lung transplant center, and 3 months later, he undergoes bilateral lung transplantation.
He initially does well; however, 2 months later he develops fever, cough, and
shortness of breath, and when he checks his home pulse oximeter, he notes that he is
hypoxemic to 88% on room air. He is seen by his pulmonologist who performs a CXR
that is unremarkable.
What etiology of his new symptoms must be ruled out?
A. Acute rejection
B. Chronic rejection
C. Pneumonia
D. Primary graft dysfunction

45. What is the next best diagnostic step for the patient in Question 44?
A. Antibody testing
B. Chest CT
C. Sputum culture
D. Transbronchial biopsy

46. How should the condition of the patient in Questions 44 and 45 be treated?
A. Azithromycin, montelukast, and change in immunosuppressant therapy
B. Broad-spectrum antibiotics
C. Increasing immunosuppression
D. No intervention

ANSWERS

1. The correct answer is: B. Decreased oxygen delivery. This patient is likely suffering
from an upper gastrointestinal (GI) bleed, as evidenced by melena, new anemia, and
history of peptic ulcer disease. His presenting symptom of dyspnea on exertion is
secondary to decreased oxygen delivery to tissues. Delivery of oxygen to tissues is
dictated by cardiac output and the oxygen content of blood. The major determinant of
oxygen-carrying capacity in blood is the hemoglobin concentration. His examination and
testing do not support a diagnosis of heart failure or pulmonary edema. High-output heart
failure may develop in the setting of chronic anemia; however, his known hemoglobin 3
months prior was normal.

2. The correct answer is: C. Obstructive. The patient’s presentation is suggestive of


COPD. Her PFTs demonstrate a reduced FEV1/FVC, supportive of an obstructive
ventilatory deficit.

3. The correct answer is: D. Restrictive. The patient’s presentation is suggestive of

Freemedicalbooks4download
interstitial lung disease. Her PFTs demonstrate a reduced FEV1 and FVC with a normal
FEV1/FVC ratio, supportive of a restrictive ventilatory deficit.

4. The correct answer is: B. Neuromuscular disease. A reduced diffusion capacity


indicates a reduced surface area of lung participating in gas exchange and/or a disruption
to diffusion at the interface of alveoli, interstitium, and capillaries. In the absence of other
abnormalities on PFT, this may indicate pulmonary vascular disease or early interstitial
lung disease. Reduced diffusion capacity is routinely corrected for hemoglobin
concentration to exclude anemia as a cause. Neuromuscular disease would not affect
reduced diffusion capacity, but rather would result in progressive restriction and reduced
forces generated during inspiration and expiration.

5. The correct answer is: C. Addition of prednisone 5 mg oral daily to her current
regimen. This patient presents with continued uncontrolled asthma symptoms despite her
current regimen of low-dose inhaled corticosteroid. It would not be appropriate to add
daily oral corticosteroids given that the patient has many other options for escalation in
her asthma management. Appropriate changes at this time would include increasing the
dose of her inhaled corticosteroid, or addition of another medication such as a long-acting
β-agonist or a leukotriene receptor antagonist.

6. The correct answer is: A. Administer albuterol nebulizers and IV


methylprednisolone. This patient with a history of asthma presents with evidence on
history and physical examination of acutely worsened airflow obstruction in the setting of
a likely viral exacerbation of asthma, without evidence of pneumonia. The mainstays of
treatment in the acute setting include short-acting β-agonist therapy at frequent intervals,
corticosteroids, and supportive care. There is no clear benefit to epinephrine over usual
care in asthma, and this patient does not have other signs to suggest anaphylaxis.
Magnesium may be considered in patients who fail to respond to first-line therapies but is
not the best initial step. Noninvasive positive-pressure ventilation is controversial in
asthma as it cannot reverse the inflammatory lung process and may contribute to
hyperinflation and complications such as pneumothorax and hemodynamic instability.

7. The correct answer is: D. Total IgE and Aspergillus-specific IgE. This patient presents
with previously controlled asthma that has suddenly worsened. This warrants a search for
exacerbating factors or asthma mimics. Given her recent exposure to a basement
renovation and elevated serum eosinophil count, one consideration would be allergic
bronchopulmonary aspergillosis (ABPA). This appears in patients with asthma or cystic
fibrosis who have been exposed to Aspergillus—this may occur in areas such as damp
basements, water-damaged homes, or wet organic matter such as from leaves, compost,
and barns. Central bronchiectasis on chest imaging would support this diagnosis. The tests
necessary to fulfill diagnostic criteria include elevated total IgE and Aspergillus-specific
IgE. Additionally, Aspergillus-specific IgG and absolute eosinophilia support this
diagnosis.

8. The correct answer is: B. Check serum eosinophil count and IgE to consider
immunologic therapy. This patient has poorly controlled asthma despite escalation of
inhaled therapy and adjunctive treatments. Given a desire to avoid long-term oral
corticosteroid treatment, continuation of the current regimen or an increased dose of
corticosteroids is not a preferred option. Similarly, repeat allergy testing or albuterol
nebulizers are not preferred as the patient has failed to benefit from allergy testing and
albuterol only serves as a rescue therapy. Obtaining serum, eosinophil, and IgE levels to
evaluate candidacy for anti-IgE and anti-interleukin-5 therapies would be the next best
step in management.

9. The correct answer is: B. Administer 0.3 mL of 1:1000 dilution epinephrine IM. The
patient is experiencing anaphylaxis secondary to a rituximab infusion. The first most
appropriate and important step is to administer epinephrine intramuscularly. A dilution of
1:1000 refers to 1 mg/mL concentration. Therefore, to administer 0.3 mg, one administers
0.3 mL IM. Intravenous epinephrine is reserved for refractory shock despite initial
treatment and aggressive fluid administration, or cardiac arrest. Histamine antagonists,
glucocorticoids, and inhaled β-2 agonists are adjunctive therapies for anaphylaxis but do
not serve as first-line management.

10. The correct answer is: B. All patients should be admitted to an inpatient ward for
management. Patients with urticaria or mild bronchospasm may be observed in an ED or
observation unit for at least 6 hours, obviating the need for universal admission of allergic
reactions.

11. The correct answer is: C. Start tiotropium. The patient has had increased symptoms
over the past 6 months, but his FEV1 is Global Initiative for Chronic Obstructive Lung
Disease (GOLD) stage 2 and he has fewer than two exacerbations per year, so starting a
long-acting muscarinic antagonist (tiotropium) alone at this time is reasonable. Increasing
the albuterol frequency without adding another controller medication is not appropriate.
Daily azithromycin was found to decrease exacerbations, but it is not routine to start daily
azithromycin, particularly in this patient who does not have frequent exacerbations. Triple
therapy is not necessary at this time, although it could be considered in the future if his
COPD continues to worsen.

12. The correct answer is: C. Initiate noninvasive positive-pressure ventilation. The
patient likely is altered in the setting of hypercarbia and noninvasive positive-pressure
ventilation should be initiated. Targeting a lower O2 goal may help prevent CO2 retention
by avoiding V/Q mismatch, maintaining respiratory drive, and preventing the Haldane
effect (more oxygenated blood has less CO2-carrying capacity); however, now that the
patient is altered, he likely needs noninvasive positive-pressure ventilation to improve
ventilation and breathe off the CO2. Ipratropium nebulizers and steroids are also important
in managing acute COPD exacerbations, but are unlikely to improve his CO2 and his
mental status now that he has already begun to retain.

13. The correct answer is: B. Prolongs survival in patients with oxygen saturation of
88% or less at rest. This patient has severe COPD with a decline in lung function and
worsening symptoms over time. She also has severe resting hypoxemia, with an oxygen
saturation of 87% at rest. In patients with COPD and a resting oxygen saturation of 88%
or less, long-term oxygen treatment has been shown to improve survival. However,

Freemedicalbooks4download
routine treatment with inhaled therapies and adjunctive treatments remain necessary to
reduce the frequency of exacerbations and hospitalizations.

14. The correct answer is: B. Chest CT and sputum cultures (including mycobacterial
and fungal cultures). This patient is presenting with symptoms consistent with
bronchiectasis, most likely secondary to recurrent pulmonary infection. Thus, chest CT
and sputum cultures are the next most appropriate diagnostic steps. Cystic fibrosis
generally does not present in late age, so sweat testing would not be a first-line diagnostic
test in this patient. Immunodeficiency (diagnosed with IgA, IgG, IgM, and IgG
subclasses) is also a cause of bronchiectasis, but is less likely in this patient given she
lacks a history of frequent infections throughout her life. Autoimmune diseases such as
systemic lupus erythematosus and rheumatoid arthritis are also possible, but she lacks
examination findings to suggest either of these as probable etiologies.

15. The correct answer is: C. Place the patient R-side down. The patient is experiencing
massive hemoptysis, defined as >500 mL of expectorated blood in 24 hours, likely in the
setting of her bronchiectasis. The first step in managing this patient is stabilizing her and
preventing asphyxiation. The patient should be placed with affected side down, and if she
becomes unstable from a respiratory perspective, she should be intubated (selectively
intubate the nonaffected lung) while she awaits definitive treatment. Cough suppressants
may actually increase the risk of asphyxiation. Blood transfusion is not necessary at this
time given that the patient is hemodynamically stable, and the patient is much more likely
to die from asphyxiation than from exsanguination. Chest CT may be useful to better
determine the location/source of the bleeding, but the first step is to stabilize the patient.

16. The correct answer is: B. Airway clearance at least twice daily with a high-frequency
oscillation vest. Airway clearance is an essential component of the daily care of patients
with cystic fibrosis. Techniques such as manual chest physical therapy or wearing a vest
serve to mobilize thickened secretions, which would otherwise predispose patients to
infections. These techniques are accompanied by the use of nebulized hypertonic saline
and dornase alfa, which serve to thin secretions. When performed routinely, airway
clearance techniques can help maintain lung function and decrease exacerbation of cystic
fibrosis due to infection. A 0.9% or normal saline nebulizer would not be sufficient to thin
the airway mucous of patients with cystic fibrosis. While PFTs are performed
intermittently, they are not done as frequently as weekly intervals. Finally, routinely
scheduled bronchoscopy does not play a role in the treatment of cystic fibrosis.

17. The correct answer is: A. A 9-mm high-risk pulmonary nodule should be followed
with CT chest every 12 months. For patients with solid solitary pulmonary nodule,
PET/CT or biopsy should be considered, and if deferred, then a CT scan should be
performed at a 3-month interval. Following at 12 months would not be appropriate for a
large nodule in a high-risk patient.

18. The correct answer is: C. Serial CT imaging to follow nodule at 12-month intervals.
Serial imaging is appropriate for this patient without risk factors for lung cancer and with
a solitary, solid 5-mm nodule without high-risk appearance.
19. The correct answer is: C. Sarcoidosis. Sarcoidosis is the most likely diagnosis. The
patient’s demographics are consistent (female in fourth decade of life). Other salient
historical features are the patient’s history of painful red rash on her shin (erythema
nodosum), findings of hilar lymphadenopathy on CT scan, and PFTs with a mixed
obstructive and restrictive pattern. She does not have any exposures to suggest
hypersensitivity pneumonitis, and while she is the right demographic for collagen vascular
disease, she does not have any skin, joint, or muscle involvement that would suggest
scleroderma. Idiopathic interstitial pneumonia is considered when the cause of a patient’s
interstitial lung disease is unknown.

20. The correct answer is: B. Endobronchial ultrasound (EBUS)-guided lymph node
biopsy. ACE has only 60% sensitivity for sarcoidosis without symptoms (closer to 90%
with symptoms). EBUS-guided lymph node biopsy is the best diagnostic test for
evaluation of sarcoidosis. Video-assisted thoracoscopic surgery is often used for diagnosis
of the idiopathic interstitial pneumonia and might be used if lymph node biopsy does not
yield a diagnosis. Bronchoalveolar lavage can be used to diagnose infection, hemorrhage,
and eosinophilic syndromes. Because the concern is highest for sarcoidosis, myositis
panel would be a low-yield diagnostic test.

21. The correct answer is: A. Apical cystic changes. The characteristic features of
idiopathic pulmonary fibrosis include a subpleural, bibasilar distribution of reticular
opacities, honeycombing, and traction bronchiectasis. Ground-glass opacities are
classically absent, although their presence does not exclude idiopathic pulmonary fibrosis.
Apical cystic changes are not seen as part of idiopathic pulmonary fibrosis.

22. The correct answer is: A. Antineutrophilic cytoplasmic antibody (ANCA). This man
presents with cavitating peripheral lung nodules and masses. The differential diagnosis for
this includes cancer (eg, primary lung or metastasis), vasculitis (eg, ANCA associated),
pulmonary embolism (when just nodules with cavitation are present), autoimmune disease
(eg, rheumatoid arthritis), infection (eg, fungal pneumonia, tuberculosis), and congenital
abnormalities. The presence of hearing loss, bloody nasal discharge, and hematuria raises
concern for a systemic vasculitis, specifically the ANCA-associated vasculitis
granulomatosis with polyangiitis. The pulmonary imaging manifestations of
granulomatosis with polyangiitis include solid and ground-glass nodules with or without
cavitation, waxing-and-waning pulmonary opacities, and lymphadenopathy.

23. The correct answer is: B. Infection. This is an exudative effusion with PMN
predominance, most consistent with a parapneumonic effusion. Congestive heart failure
effusions are generally transudative. Both malignancy and rheumatoid arthritis can cause
exudative effusions but do not have a PMN predominance on cell count.

24. The correct answer is: D. Tube thoracostomy. Tube thoracostomy is indicated for
complicated parapneumonic effusions given the concern for development of organization
and potential need for decortication later on. Serial thoracentesis is not appropriate in this
case as there is a need for continual source control. Indwelling pleural catheters are used
in some patients with malignancy; one is not indicated in this case as the effusion is not
expected to be recurrent. Because this is a complicated parapneumonic effusion, it must be

Freemedicalbooks4download
tapped as explained above; thus, B (no intervention) is not appropriate.

25. The correct answer is: A. Congestive heart failure. Despite meeting Light’s criteria for
an exudate by effusion protein/serum protein ratio >0.5, this effusion is most likely a
“pseudo-exudate” (in this case, an effusion that is due to congestive heart failure but has
elevated total protein following diuresis). He does not meet more specific criteria for
exudate (in this case, with a low-effusion cholesterol). The other effusions listed here will
have pleural fluid studies consistent with an exudate (including elevated cholesterol) and
should demonstrate other findings such as elevated WBCs or abnormal cytology.

26. The correct answer is: B. Enoxaparin. LMWH (enoxaparin) is the most appropriate
prophylaxis in this patient who is having surgery. Mechanical prophylaxis is more
appropriate for ambulatory patients having minor surgery. Direct-acting oral
anticoagulants (DOACs) are increasingly being studied for prophylaxis but are not
currently a standard practice. Fondaparinux would be appropriate if this patient had a
contraindication to heparin products (such as history of heparin-induced
thrombocytopenia).

27. The correct answer is: A. CT pulmonary angiogram. Because there is a high suspicion
for pulmonary embolus, and the patient does not have any obvious contraindications to
CT scan, CT pulmonary angiogram is the most appropriate test. V/Q would be appropriate
if the patient had contraindication to CT scan. D-dimer will almost certainly be positive in
this patient and thus will not help narrow the differential diagnosis. Echocardiogram may
be useful for risk stratification; however, it is not sensitive for pulmonary embolus.

28. The correct answer is: C. Low-molecular-weight heparin (LMWH). Because this
patient has cancer, LMWH is the preferred agent. Direct-acting oral anticoagulants
(DOACs; notably edoxaban) have been studied in cancer and are felt to be as effective,
but are associated with higher bleeding risk, especially in GI malignancy. Warfarin is
inferior to LMWH in cancer patients. Argatroban is an IV direct thrombin inhibitor that is
sometimes used in the inpatient setting in patients with a history of heparin-induced
thrombocytopenia.

29. The correct answer is: A. Apixaban for 3 months, then reassessment with patient
counseling regarding ongoing anticoagulation. For a first, unprovoked proximal deep
vein thrombosis (DVT), anticoagulation should be continued for at least 3 months. There
is likely benefit for prolonged anticoagulation, but the decision should be made
considering the nature of the clot, the patient’s bleeding risk, and patient preference.
Compression stockings alone or 4 weeks of anticoagulation is an option for superficial
venous thrombosis (depending on risk factors for development of DVT), but not for DVT.

30. The correct answer is: A. Continue apixaban. The next most important step is to
continue full-dose apixaban. There is no proven benefit to adding an IVC filter in patients
who are able to take anticoagulation (this patient has no contraindications). Historically,
IVC filters have been used in patients who have a contraindication to anticoagulation, but
they are falling out of favor due to numerous potential complications and a recent study
raising the question of increased mortality in patients with contraindications to
anticoagulation who had IVC filters placed.

31. The correct answer is: C. Idiopathic pulmonary arterial hypertension (PAH). The
patient has a mean pulmonary artery pressure >25 mmHg, which is elevated, the
pulmonary capillary wedge pressure is normal, and the transpulmonary gradient (mean
pulmonary artery pressure-pulmonary capillary wedge pressure) is 22 mmHg (normal
<12-15 mmHg), suggesting precapillary pulmonary hypertension (ruling out answer B).
Given that the patient has no evidence of COPD on PFTs or interstitial lung disease on
CT, this is less likely a group 3 pulmonary hypertension, and overall consistent with
idiopathic PAH.

32. The correct answer is: D. Sildenafil. The first-line agent in this patient should be
sildenafil given the minimal side effect profile. All of the other agents are also approved
in pulmonary arterial hypertension (PAH).

33. The correct answer is: C. Initiate inotropic and vasopressor support. This patient is
presenting with decompensated right heart failure in the setting of her known pulmonary
arterial hypertension (PAH). The first step is to improve her systemic BP with the
initiation of an agent such as norepinephrine or vasopressin. Once her MAP has improved,
the addition of milrinone will be appropriate to increase contractility (and decrease
vascular resistance). Additional inhaled pulmonary vasodilators may be appropriate once
the patient has stabilized. Intubation in patients with decompensated right heart failure can
cause hemodynamic collapse and should be avoided if not absolutely necessary.
Intravenous fluid administration is also contraindicated in this patient as it would likely
increase her right ventricular filling pressures that are already high as evidenced by the
patient’s elevated jugular venous pulse and result in worsening right heart failure.

34. The correct answer is: B. Increase positive end-expiratory pressure. The patient’s
SpO2 is below goal (goal 88%-92%). Because the patient is already on a high FiO2 (>0.6),
increasing the positive end-expiratory pressure is the most appropriate next step.
Increasing the RR and VT improves ventilation and should be employed if there are
problems with hypercapnia.

35. The correct answer is: B. Excessive secretions. Auto-positive end-expiratory pressure,
pneumothorax, and vent asynchrony all can lead to increased peak inspiratory pressures;
however, they are also associated with increased plateau pressures. In this case, the peak
inspiratory pressure is elevated, but the plateau is unchanged, indicating increased
resistances. Bronchospasm, secretions, aspiration, and problems with ventilator tubing can
all lead to increased resistance (elevated peak inspiratory pressure-Pplat).

36. The correct answer is: D. Increase in RR from 20 to 30 breaths/min; decrease in tidal
volumes from 380 to 280 mL. A number of parameters must be considered when
liberating patients from ventilator support. An increase in RR and decrease in tidal
volume, and thus an elevated rapid shallow breathing index (RR/tidal volume [L]) >105
predicts failure and may indicate that the patient is not ready for extubation. Fall in MAP
merits close monitoring but does not necessarily mean that the patient is not ready for
extubation.

Freemedicalbooks4download
37. The correct answer is: C. Maintaining pH >7.3. The major goal in ventilating patients
with acute respiratory distress syndrome is to avoid ventilator-induced lung injury. This is
done in a number of ways, including low tidal volume ventilation (<6 mL/kg), titrating
positive end-expiratory pressure, and avoiding high FiO2 to prevent worsened V/Q
mismatch (as well as theoretical risk of O2-induced lung injury). To allow for low tidal
volume ventilation, permissive hypercapnia is common practice (but goal should be to
maintain pH >7.15).

38. The correct answer is: B. Proning. Many interventions have been studied in acute
respiratory distress syndrome. Both proning and lung-protective low tidal volume
ventilation have been shown to decrease mortality. There has been no proven mortality
benefit to pulmonary vasodilators. In one trial, while steroids were associated with
increased number of vent-free days and shock-free days during the first 28 days, they were
associated with increased mortality at both 60 and 180 days when started ≥14 days after
the onset of acute respiratory distress syndrome; they are not recommended in acute
respiratory distress syndrome. There is no proven mortality benefit for early
extracorporeal membrane oxygenation in acute respiratory distress syndrome.

39. The correct answer is: B. Start broad-spectrum antibiotics. It is important to start
empiric IV antibiotics as soon as sepsis/septic shock is identified (patient meets 3/3 quick
Sequential Organ Failure Assessment criteria: RR >22 breaths/min, change in mental
status, and systolic BP <100 mmHg). Every hour of delay in starting antibiotics is
associated with increasing mortality. Resuscitation with fluid (~30 mL/kg) and pressors
may also play a role to maintain MAP >65 mmHg; however, at this time the patient is
maintaining his MAP and priority should be given to starting antibiotics. There are some
data to support the use of hydrocortisone and fludrocortisone in refractory shock;
however, this is not an initial intervention.

40. The correct answer is: C. Vancomycin/cefepime. Broad-spectrum antibiotics should be


initiated for septic shock. Generally, if a patient is without risk factors for fungal infection,
antifungals are not necessary. Treatment for community-acquired pneumonia with either
ceftriaxone/azithromycin or levofloxacin is insufficient in this patient with septic shock
(particularly given his additional risk factors for bacteremia because he is on dialysis).

41. The correct answer is: B. Norepinephrine. Norepinephrine is generally the first-line
pressor of choice in septic shock. It was associated with fewer arrhythmias when
compared with dopamine (De Backer D, Biston P, Devriendt J, et al. NEJM.
2010;362:779). The action of norepinephrine on α-receptors counteracts the vasodilatory
state of sepsis, and β-agonist provides increased cardiac output. There may be benefit to
adding vasopressin to norepinephrine in less severe septic shock (patients on 5-14
mcg/min of norepinephrine) (Russell JA, Walley KR, Singer J, et al. NEJM.
2008;358:877).

42. The correct answer is: A. Amitriptyline. The patient is presenting with seizures,
hypotension, and prolonged QRS and QTc, consistent with tricyclic antidepressant
overdose. Calcium channel blockers and β-blockers are associated with bradycardia,
atrioventricular block, and hypotension. Lithium is not only associated with seizure and
long QT, but also nausea/vomiting/diarrhea, hyperreflexia, clonus, atrioventricular block,
and bradycardia.

43. The correct answer is: C. IV sodium bicarbonate. IV NS and IV sodium bicarbonate
are used in tricyclic antidepressant overdose. Glucagon may be used in β-blocker toxicity.
Flumazenil can be used for benzodiazepine overdose; however, it should be avoided as it
can precipitate withdrawal/seizures. Dialysis may be needed in the case of certain
ingestions such as lithium, digoxin, methanol, or ethylene glycol.

44. The correct answer is: A. Acute rejection. Acute rejection must be ruled out in any post
transplant patient presenting with fever, cough, shortness of breath, or any decline in
pulmonary function. Pneumonia is less likely given unremarkable CXR. Primary graft
dysfunction generally occurs immediately in the posttransplant period. Chronic rejection
generally manifests itself as worsening obstruction on PFTs and is less associated with
acute symptoms.

45. The correct answer is: D. Transbronchial biopsy. Whenever there is concern for
rejection, transbronchial biopsy must be obtained.

46. The correct answer is: C. Increasing immunosuppression. Increased


immunosuppression is the treatment for acute rejection. Azithromycin, montelukast, and
change in immunosuppressant therapy may be used in chronic rejection. The patient does
not appear to have an infection, so broad-spectrum antibiotics are not warranted.

Freemedicalbooks4download
3
GASTROENTEROLOGY

QUESTIONS

1. A 75-year-old woman with a history of diabetes mellitus, hypertension, hyperlipidemia,


and peripheral artery disease presents with a chief complaint of cough while eating. On
further history, she describes trouble initiating her swallow as well as recent dysarthria.
Which of the following is the most likely cause of her symptoms?
A. Achalasia
B. Esophageal web
C. Stroke
D. Zenker diverticulum

2. A 55-year-old man with a history of diabetes mellitus, hypertension, obesity, tobacco


smoking, and gastroesophageal reflux disease (GERD) presents to his primary care
physician (PCP) with the new onset of trouble swallowing. He describes a sensation of
solid food becoming stuck in his mid-chest. He has been taking omeprazole 20 mg daily
for GERD for the past 3 years. He only occasionally misses a dose of this medication.
Which of the following is the next best step in the management of this patient?
A. Do a barium swallow
B. Do an esophagogastroduodenoscopy (EGD)
C. Encourage the patient not to miss any doses of omeprazole
D. Increase the dose of omeprazole to 40 mg daily

3. A 35-year-old man presents with epigastric pain. The pain is gnawing and burning in
nature and worsens after eating. Since a recent shoulder injury, he has been taking
ibuprofen “around the clock” for pain control. He denies recent weight loss, nausea and
vomiting, dysphagia, melena, or hematochezia. He has no family history of
gastrointestinal (GI) cancers. On examination, vital signs are normal, and the abdomen is
benign.
Which of the following is the next most reasonable step in his management?
A. Begin a proton pump inhibitor
B. Do an esophagogastroduodenoscopy
C. Prescribe quadruple therapy for Helicobacter pylori
D. Provide reassurance

4. A 65-year-old man with a history of hypertension, diabetes mellitus, coronary artery


disease (CAD), and gout is brought to the emergency department (ED) by his wife for
hematochezia at home. Five days ago, he began taking prednisone and ibuprofen for a
gout flare. His other medications include aspirin, clopidogrel, metformin, and
hydrochlorothiazide. Yesterday, he reported gnawing epigastric pain that worsened with
meals. This morning, he reported feeling dizzy and began passing large amounts of bright
red blood per rectum. EMS was called, and on arrival to the ED, his vital signs were as
follows: pulse 120/min, blood pressure (BP) 89/50 mmHg, temperature 37°C, and O2
saturation 100% on room air. Two large-bore IV lines were placed in the field.
Which of the following is the most appropriate immediate next step?
A. Gastroenterology consultation for esophagogastroduodenoscopy
B. Pantoprazole 40 mg IV
C. Type and cross
D. Volume resuscitation with crystalloid fluids

5. A 56-year-old woman with a history of cirrhosis secondary to alcohol use and ongoing
alcohol use disorder presents with two episodes of hematemesis at home. She has no
history of bleeding. She underwent esophagogastroduodenoscopy screening 3 years ago
and had no varices at that time. She currently takes furosemide and spironolactone. Her
initial vital signs are as follows: pulse 110/min, BP 90/56 mmHg, temperature 37°C, and
O2 saturation 98% on room air. Her physical examination is notable for dried blood
around the lips, spider telangiectasias on the chest, palmar erythema, shifting dullness and
splenomegaly, and 1+ lower extremity edema. Two large-bore IVs are placed, and she is
given 2 L of normal saline, with normalization of her BP. Since arrival to the ED, she has
had no further hematemesis. She is started on IV pantoprazole, IV ceftriaxone, and an
octreotide bolus followed by a continuous octreotide infusion.
Which of the following is the next most appropriate step in her management?
A. Gastroenterology consultation for esophagogastroduodenoscopy (EGD)
B. Interventional radiology consultation for arteriography and coiling of likely gastric
varices
C. Interventional radiology consultation for placement of a transjugular intrahepatic
portosystemic shunt (TIPS)
D. Placement of a Sengstaken-Blakemore tube

6. A 75-year-old man with a history of obesity and hypertension presents to the ED with
hematochezia. He was at home in his usual state of health when he began having bright
red blood per rectum several hours ago. He estimates blood loss of approximately 700 mL
at home. By the time he arrived at the ED, his bleeding had stopped. His vital signs are
normal. He denies dizziness, light-headedness, or abdominal pain.
The presentation is most consistent with which of the following?
A. Bleeding from peptic ulcer
B. Diverticular bleeding
C. Ischemic colitis
D. Variceal bleeding

Freemedicalbooks4download
7. A 75-year-old woman with a history of diabetes mellitus and CAD is seen by her PCP for
progressive dyspnea on exertion. She was recently seen by her cardiologist with similar
complaints. Her examination and workup at that time were not suggestive of a cardiac
cause of dyspnea, and a stress test was negative for cardiac ischemia. Lab work, however,
did demonstrate a new anemia with a hemoglobin of 8.5 g/dL, a mean corpuscular volume
(MCV) of 75 fL per cell (normal 80-100), and a red cell distribution width of 19%
(normal 11.5-14.5). At her PCP’s office, she reports that she has had regular, normal
bowel movements without melena, hematochezia, or hematemesis. Physical examination
reveals a nontoxic-appearing woman with a nonfocal abdominal examination. Digital
rectal examination reveals no masses, fissures, or hemorrhoids but is notable for brown
stool that is positive for occult blood. A colonoscopy followed by an
esophagogastroduodenoscopy are both unrevealing for a source of blood loss.
Which of the following is the next best step in her management?
A. Computed tomography (CT) arteriography
B. Reassurance
C. Tagged white blood cell (WBC) scan
D. Video capsule endoscopy

8. An 18-year-old woman presents for evaluation of diarrhea. She has a several-year history
of intermittent cramping, abdominal pain, and bloating, and for the past 4 to 6 months she
has had worsening diarrhea. She reports that her stools are foul-smelling and float in the
toilet water. The diarrhea is relieved by fasting. Workup reveals an elevated fecal fat, as
well as iron deficiency anemia.
Which of the following best describes the nature of the diarrhea?
A. Acute inflammatory diarrhea
B. Acute noninflammatory diarrhea
C. Chronic malabsorptive diarrhea
D. Chronic secretory diarrhea

9. A 65-year old woman with osteoporosis is hospitalized for repair of a hip fracture.
Preoperatively, she is given cephalexin to reduce the risk of postoperative wound
infection. Postoperatively, she is placed on MiraLAX to prevent constipation while she
remains on narcotics for pain control. On postoperative day 1, she reports four to six loose
stools. Her vital signs are normal. Laboratory testing shows a WBC count of 10 × 103/μL.
A stool Clostridioides difficile polymerase chain reaction (PCR) is positive, and stool C.
difficile enzyme-linked immunoassay (EIA; toxin A/B test) returns negative.
Which of the following is the next most appropriate step in her management?
A. Discontinue the MiraLAX
B. Order glutamate dehydrogenase
C. Prescribe PO (oral intake) vancomycin
D. Prescribe PO vancomycin and metronidazole

10. A 35-year-old woman develops fever, diarrhea, and abdominal cramping after consuming
raw eggs. Stool culture reveals nontyphoidal Salmonella species. She has normal PO
intake and appears nontoxic, with a reassuring abdominal examination, and conservative
management is employed.
For which of the following groups should antibiotics be considered in the treatment
of Salmonella infection?
A. Antibiotics should never be employed for nontyphoidal Salmonella infection
B. Immunocompromised patients
C. Patients <20 years old
D. Patients with a history of prior Salmonella infection

11. A 22-year-old man presents with acute-onset nonbloody, nonbilious vomiting occurring
10 times a day. He has had occasional episodes like this in the past, lasting days at a time,
and notes that hot showers have improved his symptoms during prior episodes. Episodes
occur two or three times a year and last up to a week. He has no abdominal pain, change
in stool pattern, weight loss, or evidence of GI bleeding. In between episodes, he feels
normal without GI symptoms. He takes no regular medications and has no other past
medical history. The social history is notable for regular marijuana use (multiple times
daily) and intermittent cigarette use. He is given IV fluids for hydration as well as
antiemetics for his symptoms.
Which of the following would you also recommend?
A. Cessation of marijuana
B. Esophagogastroduodenoscopy
C. Lorazepam long-term
D. Ondansetron long-term

12. A 45-year-old woman with a history of alcohol use disorder is hospitalized in the
intensive care unit (ICU) for severe pancreatitis requiring vasopressors. She is given IV
fluids and IV morphine for pain control. On hospital day 3, she is noted to have absent
flatus and bowel movements, and physical examination reveals a distended abdomen with
decreased bowel sounds in all four quadrants. Abdominal CT shows interstitial edematous
pancreatitis without necrosis and new, diffuse colonic dilatation (cecal diameter of 11 cm)
without evidence of obstruction. She is made NPO, a nasogastric tube is placed for
decompression, and she is given methylnaltrexone, without effect.
Which of the following would be the next most appropriate step?
A. Administer neostigmine
B. Minimize narcotic administration
C. Place a rectal tube
D. All of the above

13. In the patient in Question 12, a rectal tube is placed for decompression, and narcotics are
weaned to minimize bowel-slowing effect. On hospital day 5, she is noted to have
spontaneous passage of both stool and flatus. She is weaned off vasopressors, and the
rectal tube is able to be discontinued. The nurse notes that the patient has been NPO for
several days and is receiving IV D5NS; she inquires about the next steps in her nutrition
plan.
Which of the following is most appropriate?
A. Begin enteral nutrition
B. Begin peripheral parenteral nutrition
C. Begin total parenteral nutrition
D. Continue IV D5NS

Freemedicalbooks4download
14. A 70-year-old man with a history of obesity and hypertension presents with 2 days of
fever and left lower quadrant pain. He has noted loose stools in the past 48 hours but
denies nausea, vomiting, diarrhea, or constipation. He reports a normal appetite and PO
intake. In the ED, he is febrile to 38.2°C, and the remainder of his vital signs are normal.
Abdominal examination demonstrates focal left lower quadrant tenderness to deep
palpation without rebound tenderness or guarding. CT of the abdomen and pelvis with IV
and oral contrast is performed and demonstrates focal diverticulitis of the sigmoid colon
without evidence of abscess or fistula. He is given acetaminophen with adequate pain
control.
Which of the following is the next most appropriate step in his management?
A. Hospitalization with IV piperacillin-tazobactam for 7 days
B. Interventional radiology consultation for abdominal drain placement
C. Outpatient therapy with PO metronidazole and ciprofloxacin for 7 days
D. Surgical consultation for sigmoid resection

15. The patient in Question 14 is treated successfully with outpatient antibiotic therapy and
returns to his PCP for follow-up. He is worried about a recurrence of diverticulitis.
Which of the following is TRUE about recurrence and prevention of diverticulitis?
A. Because his first episode was uncomplicated, he can be almost certain that a
subsequent episode will also be uncomplicated.
B. Surgical resection should only be considered after a fourth recurrence of
diverticulitis.
C. The combination of amoxicillin/clavulanic acid has proven to be efficacious for the
prevention of recurrent diverticulitis.
D. The risk of recurrence within 10 years is 10% to 30%.

16. A 50-year-old woman with a history of hypertension, diabetes mellitus, CAD with a prior
myocardial infarction (MI), and heart failure with reduced ejection fraction (40%)
presents for her first screening colonoscopy. She is currently feeling at her baseline. She is
a nonsmoker and does not drink alcohol. Her medications include aspirin, lisinopril,
furosemide, and metoprolol. Her family history is notable for a mother with colorectal
cancer diagnosed at age 59 years, a maternal cousin with ulcerative colitis, and early heart
attack in her father.
Which of the following elements of her history places her at increased risk of
colorectal cancer compared with the baseline population?
A. Aspirin use
B. Family history of colorectal cancer in a first-degree relative
C. Family history of ulcerative colitis in a cousin
D. Personal history of hypertension

17. A 19-year-old man presents with worsening bloody diarrhea and abdominal pain. His
symptoms began 6 days ago with low abdominal pain that was quickly followed by
overtly bloody diarrhea accompanied by intense rectal urgency and tenesmus. Over the
past several days, his stools have become more frequent, now occurring 8 to 10 times per
day. He has had no recent travel, antibiotic use, or hospitalization. He denies fever, oral
ulcers, ocular symptoms, or joint or dermatologic symptoms. Family history is positive for
ulcerative colitis in his mother and a maternal cousin. You suspect that the patient may
have ulcerative colitis.
If so, which of the following would you expect a colonoscopy to show?
A. Deep ulcerations in a cobblestone pattern with skip lesions throughout the colon
B. Grossly normal-appearing mucosa with biopsies showing active chronic colitis
C. Inflammation of the colonic mucosa in a contiguous pattern starting at the rectum and
progressing proximally
D. Pseudomembranes throughout the colon

18. The patient in Question 17 undergoes colonoscopy, which demonstrates contiguous,


circumferential overt ulceration beginning in the rectum and extending proximally to the
splenic flexure. Stool culture and C. difficile testing return negative. Biopsies from his
colonoscopy demonstrate severe, active, chronic colitis with crypt distortion and
abscesses, most consistent with a diagnosis of ulcerative colitis. He asks what he can
expect with this disease.
Which of the following would you tell him?
A. Cigarette smoking places him at increased risk of ulcerative colitis.
B. He has an increased risk of colorectal cancer compared to the baseline population
rate.
C. His overall life expectancy is decreased compared with that of the general
population.
D. Unlike Crohn disease, patients with ulcerative colitis are not at risk for stricturing
disease.

19. A 21-year-old woman with a known history of stricturing Crohn disease involving the
ileum has undergone several ileal resections for bowel obstruction secondary to strictures.
With her history of ileal disease and resection, she is at increased risk for which of
the following?
A. Gallstones
B. Kidney stones
C. Osteoporosis
D. All of the above

20. A previously healthy 27-year-old woman presents with 4 weeks of new-onset bloody
diarrhea, accompanied by lower abdominal pain, rectal urgency, and tenesmus. On
evaluation, she has normal vital signs and is nontoxic-appearing. Examination reveals
tenderness to deep palpation in the left lower quadrant without accompanying guarding or
rebound. Digital rectal examination reveals red blood on the gloved finger with no
fissures, fistulas, or masses. She undergoes a colonoscopy, which reveals circumferential
ulceration, friability, and contact bleeding that extends contiguously from the rectum to
the descending colon. Biopsies show severely active chronic colitis with few crypt
abscesses, confirming a diagnosis of ulcerative colitis with severe disease activity. She is
started on IV steroids with improvement.
Which of the following would be the most appropriate medication for long-term
maintenance therapy?
A. Cyclosporine
B. Infliximab
C. Methotrexate

Freemedicalbooks4download
D. Oral prednisone

21. A 79-year-old nursing home resident with atrial fibrillation (AF; not on anticoagulation
because of frequent falls), hypertension, prior cerebrovascular accident (CVA), diabetes
mellitus, and dyslipidemia presents with the sudden onset of severe abdominal pain,
which he describes as “the worst of his life.” He was reported to be hypotensive (70/40
mmHg) at the nursing home but now has a BP of 102/55 mmHg. On examination, the
abdomen is mildly tender to palpation, and there is a small amount of bright red blood in
the rectal vault. Labs are notable for a serum lactate level of 4.0 mmol/L, WBC of 18
000/mm3, normal lipase, and normal bilirubin. The serum creatinine level is 0.7 mg/dL.
Which of the following would be the next most appropriate step?
A. CT angiography of the abdomen
B. Duplex ultrasonography of mesenteric vessels
C. Flexible sigmoidoscopy
D. Magnetic resonance imaging (MRI) abdomen with gadolinium

22. An 83-year-old woman with CAD, prior coronary artery bypass grafting (CABG),
hypertension, chronic kidney disease (baseline creatinine 2.0 mg/dL), tobacco use, and
diabetes mellitus is admitted to the hospital after presenting with 6 months of worsening
abdominal pain associated with eating. She has lost 10 lb due to a reluctance to eat (eating
triggers the pain). On examination, vital signs are within normal limits, and she is well-
appearing. Laboratory results, including chemistries, liver biochemical tests, complete
blood count (CBC), and lactate, are normal. The abdomen is nontender to palpation, and
she currently has no pain.
Which of the following is the most likely pathophysiologic cause of her
symptoms?
A. Decreased blood flow to the gut due to mesenteric atherosclerosis
B. Decreased perfusion in the superior mesenteric artery territory due to arterial
embolism
C. Increased venous congestion due to mesenteric vein thrombus
D. Ischemic colitis due to decreased blood flow to the colon

23. The patient described in Question 22 undergoes workup for her symptoms.
Which of the following is the next most appropriate step in her diagnosis?
A. CT angiography with venous phase contrast
B. Invasive angiography
C. Magnetic resonance angiography
D. Ultrasonography with Dopplers of the mesenteric vessels

24. A 45-year-old woman with long-standing alcohol use disorder, multiple prior episodes of
pancreatitis, and depression presents to the ED with abdominal pain radiating to the back.
She had 10 alcoholic drinks on the day of presentation. On examination, she has a pulse of
110/min, dry mucous membranes, epigastric tenderness, and is vomiting green liquid. The
hemoglobin is 10 g/dL (baseline), creatinine is 1.4 mg/dL (baseline 0.5 mg/dL), lactate is
2.0 mmol/dL (normal <1), and lipase is 32 U/L (normal <40). Abdominal ultrasonography
shows no cholelithiasis, and the pancreas is obscured by bowel gas.
Which of the following is the next most appropriate step in her management?
A. IV lactated Ringer’s solution and pain management
B. IV lorazepam (for alcohol withdrawal)
C. IV pantoprazole
D. Oral fluids, with antiemetics, if needed

25. A 42-year-old woman with a history of alcohol use disorder (at least four beers daily) is
admitted with epigastric pain, nausea, and vomiting and found to have a serum lipase level
of 3200 U/L. Ultrasonography of the abdomen reveals no cholelithiasis or bile ductal
dilatation, and the patient is diagnosed with a first episode of alcohol-related acute
pancreatitis. She is treated with IV lactated Ringer’s solution and analgesia. Within 4 days
of hospital admission, she develops worsening abdominal pain and hypotension. She has a
fever of 38.8°C, pulse 120/min, and BP 76/40 mmHg. On examination, she has marked
abdominal tenderness, particularly in the epigastrium. Urgent CT abdomen with IV
contrast reveals nonenhancing low-attenuating regions within a mildly enlarged pancreas.
She is started on carbapenem and levofloxacin and is no better after 12 hours.
Which of the following would you recommend?
A. Endoscopic placement of a pancreatic duct stent
B. IV fluids and octreotide
C. Open surgical debridement (necrosectomy)
D. Percutaneous drainage

26. A 35-year-old man with no history of alcohol, drug, or tobacco use presents to the GI
clinic with 6 months of nonspecific epigastric pain, nausea after eating, and greasy foul-
smelling stools. Initial testing reveals a decreased fecal elastase level and the presence of
fecal fat. The HbA1c level is 5.0%. Blood testing reveals elevated serum alkaline
phosphatase of 350 U/L and total bilirubin of 3.2 mg/dL, and normal aspartate
aminotransferase (AST) and alanine aminotransferase (ALT) levels.
While awaiting imaging, which of the following would be the next most
appropriate lab tests to send?
A. Antinuclear antibody and IgG subtypes
B. Blood and stool cultures
C. H. pylori serology and human immunodeficiency virus (HIV) testing
D. Serum alcohol level

27. A 74-year-old woman with diabetes mellitus, hypertension, dyslipidemia, chronic


obstructive pulmonary disease (COPD; on azithromycin three times a week),
osteoarthritis, recurrent urinary tract infections (UTIs; on suppressive nitrofurantoin), and
depression is seen by her PCP for an annual physical. Liver enzymes are drawn as part of
routine blood work and reveal an elevated total bilirubin of 2.5 mg/dL (direct bilirubin 1.5
mg/dL), an elevated alkaline phosphatase of 650 U/L, ALT 15 U/L, AST 20 U/L,
international normalized ratio (INR) 1.2, and albumin 3.5 g/dL.
Which of the following medications would be most likely to cause this pattern of
abnormalities?
A. Acetaminophen
B. Azithromycin
C. Hydralazine
D. Nitrofurantoin

Freemedicalbooks4download
28. A 29-year-old man undergoes an annual physical examination for an immigration visa.
His workup reveals abnormal liver enzymes. The total bilirubin level is elevated to 3.0
mg/dL with direct bilirubin of 0.5 mg/dL. Serum AST and ALT levels are 20 and 18 U/L,
respectively. The alkaline phosphatase is 113 mg/dL and the INR, CBC, blood smear,
reticulocyte count, and albumin are within normal limits. He has no symptoms and denies
alcohol use. He takes no regular medications.
Which of the following would be the next best step in his diagnosis?
A. Abdominal ultrasonography
B. Checking a γ-glutamyl transpeptidase
C. Reviewing prior lab results and asking about a current illness or stressor
D. Urine ethyl glucuronide for surreptitious alcohol use disorder

29. A 59-year-old woman with a history of osteomyelitis and depression presents to her PCP
with itching for 2 months. She has no new exposures to potential allergens. She denies a
rash or family history of autoimmune liver disease. She takes no medications. On
examination, there is no conjunctival icterus and she is well-appearing. The abdominal
examination is unremarkable. Fasting laboratory tests are notable for a normal serum
creatinine, blood urea nitrogen (BUN), and CBC. The total bilirubin is 0.9 mg/dL, AST 19
U/L, ALT 23 U/L, and alkaline phosphatase is elevated to 198 U/L.
Which of the following would you recommend?
A. An oral antioxidant
B. Antimitochondrial antibodies
C. Gamma-glutamyl transpeptidase
D. Ultrasound elastography of the liver

30. A 26-year-old male who has sex with men (one partner, uses barrier protection) presents
to the ED with new-onset right upper quadrant (RUQ) abdominal pain, conjunctival
icterus, nausea, and vomiting. He reports no recent travel but may have been exposed to
sick contacts (young cousins with nausea, vomiting, and diarrhea) over the past few
weeks. On examination, he is jaundiced with tenderness to palpation over the RUQ.
Mental status is normal. He denies IV drug use and has not taken any medications. He
drinks two alcoholic drinks daily and has had no tattoos or blood transfusions. Labs are
notable for a serum ALT level of 5430 U/L, AST 3598 U/L, alkaline phosphatase 234
U/L, bilirubin 7.8 mg/dL, and INR 1.2.
Which of the following is the test most likely to confirm the diagnosis?
A. Anti-HBc
B. Hepatitis C virus (HCV) RNA
C. IgM anti-hepatitis A virus (HAV)
D. Serum ethanol

31. A 37-year-old man from China presents to a hepatology clinic to establish care after
moving to the United States. He is asymptomatic but has been told that he has had
hepatitis B virus (HBV) since childhood. Serologic testing reveals that he is HBsAg
positive, anti-HBs negative, IgG anti-HBc positive, HBeAg positive, and anti-HBe
negative. HBV DNA (viral load) is 120 000 IU/mL. The serum ALT level is 256 U/L, but
the alkaline phosphatase and bilirubin levels are normal.
Which of the following is the next most appropriate step in his management?
A. Begin sofosbuvir/ledipasvir
B. Begin tenofovir
C. Repeat testing in 3 to 6 months
D. Vaccinate against HBV

32. A 37-year-old man with a history of alcohol use disorder, IV drug use, and multiple prior
admissions for alcohol-associated hepatitis presents with abdominal pain, fever, ascites,
and conjunctival icterus. He was recently discharged from the hospital after treatment for
spontaneous bacterial peritonitis (SBP). On examination, he is tachycardic with dry
mucous membranes and is tender in the RUQ. The serum bilirubin level is 11.3 mg/dL
and INR is 2. Maddrey discriminant function is 57. A repeat paracentesis shows 650
nucleated cells (80% polymorphonuclear leukocytes [PMNs]) and gram-negative rods on
Gram stain.
Which of the following would be the most appropriate management?
A. IV ceftriaxone
B. Oral prednisolone
C. Oral prednisolone and pentoxifylline
D. Referral for urgent liver transplant

33. A 57-year-old woman with type 2 diabetes mellitus complicated by nephropathy (chronic
kidney disease stage IV with a baseline estimated glomerular filtration rate [eGFR] of 25
mL/min) and retinopathy, hypertension, prior intravenous drug use, and HCV infection
presents to residents’ clinic to establish care. She was diagnosed with HCV infection
approximately 5 years ago, after antibody screening test returned positive and was
confirmed on repeat testing. Her current medications are insulin, cinacalcet, lisinopril,
cholecalciferol, furosemide, and doxazosin. Examination reveals no stigmata of liver
disease.
Labs are notable for platelets of 205 × 109/L, total bilirubin 0.6 mg/dL, albumin 4.1
g/dL, INR 1.1, AST 21 U/L, and ALT 18 U/L. The HCV genotype is 1b, and the HCV
RNA level is 2 305 000 IU/mL. HBV serologies are consistent with immunity to HBV as
a result of prior vaccination. HIV testing is also negative. Ultrasound of the abdomen
shows a normal-sized liver with slightly increased echogenicity and no splenomegaly.
Vibration-controlled transient elastography reveals a liver stiffness of 5.5 kPa (normal).
She inquires about treatment for her HCV.
What would you recommend that she do?
A. Defer treatment, given that there is no evidence of liver fibrosis or cirrhosis
B. Initiate treatment with pegylated interferon and ribavirin
C. Perform a liver biopsy before offering treatment
D. Suggest treatment with elbasvir-grazoprevir (a direct-acting antiviral regimen)

34. A 21-year-old woman with a history of depression, anxiety, and prior suicide attempts is
brought in by ambulance after taking an intentional overdose of 40 g of acetaminophen 20
hours prior to presentation. On arrival to the ED, she is rousable but lethargic with
asterixis. The serum bilirubin level is 5.6 mg/dL and INR is 1.6. The acetaminophen level
is pending.
In addition to supportive care in an ICU, checking arterial ammonia and lactate,
and transfer to a liver transplant center, which of the following treatments should be

Freemedicalbooks4download
initiated first?
A. Activated charcoal
B. N-acetylcysteine
C. Pentoxifylline
D. Urgent hemodialysis

35. The patient described in Question 34 is transferred to a liver transplant center ICU. She
becomes more obtunded and unable to protect her airway. She is then intubated, and
intracranial pressure monitoring is initiated. The arterial ammonia is 200 μmol/L, and the
intracranial pressure is elevated. The serum sodium is 135 mEq/L, and bilirubin continues
to rise (now 21.4 mg/dL). INR is increased to 2.3. She has no evidence of bleeding.
What is the best therapeutic measure to perform next for the elevated intracranial
pressure?
A. Hypothermia
B. IV barbiturate
C. IV mannitol
D. Lactulose

36. A 34-year-old pregnant woman with acute liver failure due to hepatitis E virus (HEV) is
intubated in the ICU and listed for a liver transplantation. She is being covered with
broad-spectrum antibiotics for fever. She has hypoglycemia and requires a continuous
dextrose infusion. Her labs are notable for an INR of 1.5, platelet count of 21 000/mm3,
fibrinogen of 123 mg/dL, and BUN of 45 mg/dL. She needs to have central access placed
in the ICU.
Which of the following is the most appropriate blood product to administer
immediately prior to line insertion?
A. Cryoprecipitate
B. Fresh frozen plasma
C. Platelets
D. Vitamin K

37. A 57-year-old man with biopsy-proven cirrhosis due to HCV infection, MELD (Model for
End-Stage Disease) score of 20, and prior sustained virologic response with
sofosbuvir/ledipasvir treatment presents with 3 days of abdominal pain and fever. He is
hemodynamically stable without encephalopathy. He has no history of spontaneous
bacterial peritonitis (SBP) but meets the criteria for SBP prophylaxis and is taking
ciprofloxacin daily. A diagnostic paracentesis demonstrates 500 nucleated cells (75%
neutrophils); fluid cultures are sent.
Which of the following is the best empiric antibiotic to start in this case?
A. IV ceftriaxone
B. IV ciprofloxacin
C. IV vancomycin and meropenem
D. PO trimethoprim-sulfamethoxazole

38. A 60-year-old woman with cirrhosis caused by nonalcoholic fatty liver disease, a prior
variceal bleed, refractory ascites (requiring a paracentesis every 2 weeks, on furosemide
and spironolactone), and a baseline serum creatinine level of 0.6 mg/dL presents with
confusion and coffee-ground emesis. An esophagogastroduodenoscopy reveals a gastric
ulcer with a visible vessel, which is cauterized. She has not been adequately fluid
resuscitated, and the serum creatinine level is noted on admission to be 1.5 mg/dL; the
urine sediment is bland, and mean arterial pressure (MAP) is around 60 mmHg (which is
close to her baseline BP).
Which of the following is the first best step in her management?
A. Draw blood cultures and administer IV meropenem
B. Hold diuretics and give IV albumin 1 g/kg
C. Reduce the dose of β-blocker
D. Transfer her to the ICU for IV norepinephrine and octreotide

39. A 45-year-old woman with a history of alcohol-related Child-Pugh class C cirrhosis and
ongoing alcohol use presents with hematemesis. She has no history of esophageal varices.
She is initially hypotensive with a BP of 75/45 mmHg and tachycardic with a pulse of
120/min. The hemoglobin is 6.6 g/dL (with a baseline of 10 g/dL). Medications prior to
admission were furosemide, spironolactone, lactulose, rifaximin, and omeprazole. She is
resuscitated with 2 units of packed red blood cells and admitted to the ICU for urgent
esophagogastroduodenoscopy, which reveals grade 3 esophageal varices with positive red
wale signs.
Which of the following would be the most appropriate management for this
patient?
A. Endoscopic variceal ligation and nonselective β-blocker
B. Nonselective β-blocker
C. Sengstaken-Blakemore tube placement
D. Transjugular intrahepatic portosystemic shunt (TIPS)

40. A 50-year-old male veteran with HCV-related cirrhosis (no prior


esophagogastroduodenoscopy [EGD] or hepatocellular carcinoma screening),
hypertension, and diabetes mellitus presents with abdominal pain for 5 days. He is
hemodynamically stable with no change in the serum bilirubin, INR, and creatinine from
baseline. CT with IV contrast is performed, which reveals a heterogeneous liver, moderate
amount of ascites, no focal liver lesions, and acute thrombosis of the portal vein. He is
feeling well and would like to be discharged home.
Which of the following is the most appropriate management plan for this patient?
A. Admission to the hospital for IV heparin
B. Admission to the hospital for monitoring of clot progression and pain
C. Discharge home with repeat CT of the abdomen in 1 month
D. Esophagogastroduodenoscopy screening for varices and initiation of anticoagulation

41. A 75-year-old man with end-stage renal disease (on hemodialysis), type 2 diabetes
mellitus, hypertension, Waldenström macroglobulinemia, dyslipidemia, and obesity
presents to the ED with acute onset of RUQ pain and new onset of ascites. His wife notes
that his eyes have been yellow, and he has been confused for the past day. He has no
history of liver disease. On examination, he is nontoxic-appearing with conjunctival
icterus, large-volume ascites, and RUQ tenderness. Labs are notable for a serum bilirubin
of 6.5 mg/dL, INR of 2.0, AST 350 U/L, ALT 235 U/L, and alkaline phosphatase 323
U/L.

Freemedicalbooks4download
Which of the following would be the next most appropriate diagnostic test?
A. Abdominal ultrasonography with Dopplers
B. Liver elastography
C. MRI with MR venography (MRV) of the liver
D. Noncontrast CT of the abdomen and pelvis

42. A 20-year-old man, 12 days after hematopoietic stem cell transplantation (HSCT) for
acute myelogenous leukemia, develops hepatomegaly, RUQ pain, ascites, weight gain of
12 lb over 2 days, and lower extremity edema. He has no rash or diarrhea. Two days later,
his serum bilirubin level rises to 6.5 mg/dL with mildly elevated AST and ALT levels.
Ultrasonography demonstrates reversal of flow in the portal vein.
Which of the following is the most likely diagnosis?
A. Acute cytomegalovirus infection
B. Acute graft-versus-host disease
C. Drug toxicity from pre-HSCT conditioning chemotherapy
D. Sinusoidal obstruction syndrome

43. A 46-year-old woman from Puerto Rico presents to the ED with new ascites, malaise,
weight loss, and conjunctival icterus. A diagnostic paracentesis reveals an ascitic protein
concentration of 0.8 g/dL, ascitic albumin of 0.5 g/dL (serum albumin of 3.0 g/dL), and
145 nucleated cells (65% neutrophils).
Based on the ascitic fluid studies, which of the following is the most likely
diagnosis?
A. Nephrotic syndrome
B. Peritoneal carcinomatosis
C. Portal hypertension
D. Tuberculous ascites

44. A 68-year-old man with a history of AF, type 2 diabetes mellitus, hypertension,
dyslipidemia, CAD, and prior percutaneous coronary intervention presents to the ED with
abdominal pain, fever, and chills 9 days after open laparotomy for bowel obstruction. He
is tender to palpation in the left upper quadrant. Broad-spectrum antibiotics are started,
and a diagnostic paracentesis reveals polymicrobial growth within 24 hours, fluid lactate
dehydrogenase (LDH) of 697 mg/dL, and glucose of 16 mg/dL. The following day he
becomes acutely hypotensive with increasing abdominal pain.
Which of the following is the next most appropriate diagnostic test?
A. CT of the abdomen and pelvis
B. Esophagogastroduodenoscopy with mucosal biopsies of the stomach and duodenum
C. Repeat paracentesis
D. Ultrasonography of the abdomen with Dopplers

45. A 67-year-old woman with a history of type 1 diabetes mellitus, end-stage renal disease
(on peritoneal dialysis), hypertension, hypercholesterolemia, and COPD presents with
abdominal pain, fever, and nausea. She reports that her dialysate fluid looks cloudy.
Diagnostic peritoneal fluid studies reveal 320 WBCs (96% polymorphonuclear
neutrophils). Her pulse is 110/min, BP 95/40 mmHg, and temperature 39°C.
Initial empiric antibiotic therapy should be with which of the following?
A. IV cefepime
B. IV ciprofloxacin and metronidazole
C. IV meropenem
D. IV vancomycin and gentamicin

46. An 80-year-old white woman with a history of morbid obesity, hypertension, gout, and
type 2 diabetes mellitus presents with 2 days of RUQ pain, confusion, fever of 40°C,
chills, and lethargy in the nursing home where she resides. On arrival to the ED, she is
toxic-appearing, tachycardic to 110/min, and hypotensive to 86/46 mmHg, and is noted to
have conjunctival icterus in addition to severe RUQ tenderness. She is volume-
resuscitated in the ED, empiric antibiotics are given, and blood cultures are drawn.
Abdominal ultrasonography shows dilatation of the bile duct to 1.5 cm with dilated
intrahepatic ducts and cholelithiasis without cholecystitis. The serum bilirubin level is 6.8
mg/dL, ALT 154 U/L, AST 125 U/L, alkaline phosphatase 609 U/L, and WBC 30 x 103 /
μL. In the ED, the patient remains hypotensive despite adequate fluid resuscitation and is
started on vasopressors with escalating requirements.
Which of the following is the next most appropriate step in her management?
A. Broad-spectrum antibiotics
B. Cholecystostomy tube
C. Endoscopic retrograde cholangiopancreatography (ERCP)
D. Surgical consultation for cholecystectomy

47. A 41-year-old morbidly obese woman with a history of hypertension and type 2 diabetes
mellitus and who is currently 12 weeks pregnant presents with new onset of belching,
nausea, and RUQ pain after eating, particularly after eating fatty foods. The pain lasts
approximately 1 to 2 hours and radiates toward her right scapula. She has a family history
of peptic ulcer disease. On examination, she is clinically well-appearing and afebrile with
a normal heart rate and BP. She also has mild RUQ tenderness. Liver enzymes and CBC
are within the normal range. She is concerned about the pain and asks what she should do
next.
Which of the following is most appropriate?
A. Esophagogastroduodenoscopy
B. Esophageal probe monitoring
C. Referral for cholecystectomy
D. Transabdominal ultrasonography

48. A 56-year-old man with idiopathic pulmonary fibrosis who is 2 weeks post-bilateral lung
transplantation remains in the ICU with postoperative pneumonia and multiple failed
extubations. He is on total parenteral nutrition due to an inability to tolerate tube feeds. He
is persistently febrile with a pressor requirement despite broad-spectrum antibiotics. CT of
the abdomen and pelvis reveals a thick gallbladder wall with intramural gas,
pericholecystic fluid, and sludge in the gallbladder without gallstones.
Which of the following is the next most appropriate step in his management?
A. Cholecystectomy
B. Cholecystostomy drain
C. Discontinue total parenteral nutrition
D. Endoscopic retrograde cholangiopancreatography (ERCP)

Freemedicalbooks4download
ANSWERS

1. The correct answer is: C. Stroke. The patient describes trouble initiating a swallow,
which suggests that the origin is oropharyngeal rather than esophageal. The combination
of dysphagia and dysarthria raises concern for a neuromuscular rather than strictly
structural cause. In this patient with known vascular disease, this constellation of
symptoms may be consistent with a stroke.

2. The correct answer is: B. Do an esophagogastroduodenoscopy (EGD). The patient has


long-standing GERD and risk factors for Barrett esophagus and esophageal
adenocarcinoma. The development of dysphagia is an alarm symptom for potential
malignancy. Alarm symptoms include dysphagia, excessive vomiting, weight loss, and
anemia. Risk factors for Barrett esophagus include age more than 50 years, Caucasian
race, hiatal hernia, central adiposity, smoking, and a family history of Barrett esophagus
or esophageal adenocarcinoma. The next best step in the management of this patient is to
refer him for EGD to exclude malignancy. While other complications from GERD, such
as esophagitis and strictures, may cause dysphagia, malignancy should be excluded first.
EGD is more sensitive and specific for Barrett esophagus and malignancy than a barium
swallow.

3. The correct answer is: A. Begin a proton pump inhibitor. This patient presents with
symptoms that are typical for peptic ulcer disease, specifically a gastric ulcer, because his
pain worsens with meals (the pain of a duodenal ulcer is typically relieved with food). His
use of nonsteroidal anti-inflammatory drugs (NSAIDs) is the most likely etiology of the
presumed peptic ulcer disease. With a history of NSAID use, an absence of alarm
symptoms, and a reassuring physical examination, it is reasonable to treat empirically with
acid-suppressive therapy while he remains on NSAIDs and monitor him closely for
symptom resolution. An esophagogastroduodenoscopy is not needed at this point but
should be considered if his symptoms prove refractory to proton pump inhibitor therapy.
Empiric treatment for H. pylori without testing is not recommended. Reassurance alone
would be inappropriate.

4. The correct answer is: D. Volume resuscitation with crystalloid fluids. The patient is
presenting with several risk factors for peptic ulcer disease (ibuprofen and aspirin use in
combination with prednisone use) as well as symptoms and signs of a rapid upper GI
bleed (hypotension, tachycardia in combination with hematochezia). Although red blood
cell transfusion, IV proton pump inhibitor, and esophagogastroduodenoscopy are all
appropriate steps in the management of brisk upper GI bleeding, immediate volume
resuscitation with crystalloid fluids is of utmost priority given his hypotension and the
rapid availability of IV fluids in the emergency room.

5. The correct answer is: A. Gastroenterology consultation for


esophagogastroduodenoscopy (EGD). The patient has cirrhosis and presents with an
upper GI bleed and signs of portal hypertension (ascites and splenomegaly), raising
concern for a possible variceal source. She has been hemodynamically resuscitated, and
appropriate medical management has been initiated. The next best step is to perform an
EGD with endoscopic band ligation if varices are found. A Sengstaken-Blakemore tube is
not necessary, since the bleeding is not uncontrollable. A Sengstaken-Blakemore tube is
typically used in patients with unstable, large-volume hematemesis to allow subsequent
EGD or for refractory bleeding in spite of band ligation to allow subsequent transjugular
intrahepatic portosystemic shunt (TIPS). A TIPS is not an appropriate first management
step without first attempting EGD. Although gastric varices are possible, they must first
be confirmed by EGD.

6. The correct answer is: B. Diverticular bleeding. The patient has had overt
hematochezia but has remained hemodynamically stable. Therefore, the bleeding is most
likely localized to a lower GI bleed (as opposed to a brisk upper GI bleed). The
presentation with painless bleeding, along with a sudden onset and abrupt cessation, is
typical of a diverticular bleed. His age is a clear risk factor for diverticulosis; obesity is
also thought to be a risk factor. Ischemic colitis is less likely and is typically accompanied
by abdominal pain and bloody stool. An upper GI source is unlikely in a patient who is
hemodynamically stable.

7. The correct answer is: D. Video capsule endoscopy. This patient has a new,
symptomatic anemia, with labs suggestive of iron deficiency (low MCV, elevated red cell
distribution width). Although she has no overt symptoms of bleeding, her positive fecal
occult blood test is suggestive of occult GI blood loss. Thus, a full evaluation should be
performed to look for sources of GI bleeding, and reassurance is inappropriate. The next
best step for this particular case would be a video capsule endoscopy, which may be able
to identify small bowel lesions that could account for her blood loss. CT arteriography is
unlikely to be helpful in this case, as this imaging modality depends on a bleeding rate of
roughly 0.5 mL/min or more to detect bleeding from the culprit vessel. Since this patient
has had no overt evidence of bleeding, it is unlikely that she is losing blood from a GI
source at a rate greater than 0.5 mL/min. A tagged WBC scan is a modality used to detect
inflammation and does not help localize occult GI bleeding.

8. The correct answer is: C. Chronic malabsorptive diarrhea. The first step in
determining the cause of a patient’s diarrhea is to characterize it. Diarrhea is considered
acute if the patient has had less than 2 weeks of symptoms and is often secondary to
infection. Diarrhea is considered chronic if symptoms have lasted more than 4 weeks.
(Symptoms lasting 2-4 weeks may be either acute or chronic.) This patient’s symptoms
have lasted several months and presentation is most consistent with chronic diarrhea. A
history of foul-smelling, floating stools with an elevated fecal fat would be most
consistent with either a malabsorptive or maldigestive diarrhea. The patient’s presentation
is most consistent with (but not limited to) an etiology such as celiac disease or small
intestinal bacterial overgrowth. Fecal fat would not be expected to be elevated in chronic
secretory diarrhea, and typically this type of diarrhea does not improve with fasting.

9. The correct answer is: A. Discontinue the MiraLAX. The next most appropriate step is
to decrease or discontinue MiraLAX and monitor the patient clinically. The stool testing
results likely represent C. difficile colonization rather than C. difficile–associated diarrhea
(CDAD). This conclusion is supported by the absence of fever, absence of leukocytosis,

Freemedicalbooks4download
the presence of an alternative explanation for loose stools, and negative C. difficile toxin
result. Up to 20% of hospitalized patients who receive antibiotics may have asymptomatic
C. difficile colonization (as determined by positive C. difficile PCR testing) without
clinical CDAD. Therefore, the likely culprit for her loose stools, laxative use, should be
addressed first, and she should be monitored clinically. Treatment of C. difficile infection
should not be implemented at this time. Glutamate dehydrogenase testing would not yield
additional clarity to the question of colonization versus CDAD.

10. The correct answer is: B. Immunocompromised patients. In patients with a nontoxic
examination and normal PO intake, conservative measures (adequate hydration, PRN
antidiarrheal medications) can be employed for nontyphoidal Salmonella infection.
Immunocompromised patients represent a group of patients that should be considered for
antibiotic treatment (typically with a fluoroquinolone antibiotic). Prior exposure to
Salmonella does not impact treatment decisions for a subsequent episode.

11. The correct answer is: A. Cessation of marijuana. The patient’s history of recurrent,
stereotypic episodes of acute vomiting in the setting of heavy marijuana use raises concern
for cannabinoid hyperemesis syndrome. Alleviation of symptoms with hot showers is a
typical feature of this syndrome. IV fluids, antiemetics, and benzodiazepines may be used
to control acute symptoms. However, agents such as ondansetron and lorazepam are not
recommended for long-term use due to their risk and side effect profiles. The mainstay of
treatment of this disorder is cessation of marijuana use. For refractory symptoms, tricyclic
antidepressants can be considered. Given the self-limited nature of the patient’s episodes
and history consistent with cannabinoid hyperemesis syndrome, an
esophagogastroduodenoscopy is not needed at this time.

12. The correct answer is: D. All of the above. This critically ill patient has developed
paralytic ileus of the colon, sometimes called colonic pseudo-obstruction or Ogilvie
syndrome (although the latter designation was originally applied to paraneoplastic colonic
pseudo-obstruction). Risk factors for colonic pseudo-obstruction in this patient include
critical illness, pancreatitis, hospitalization, and opiate administration. Management of
colonic pseudo-obstruction is centered around bowel rest, bowel decompression, and
restoring normal intestinal peristalsis. Conservative measures such as making the patient
NPO and discontinuing offending medications should be employed first. Bowel
decompression can be attempted with either nasogastric tube or rectal tube placement (or
both). For patients receiving narcotics, methylnaltrexone, a peripheral μ-opioid antagonist
that acts locally in the gut without causing systemic withdrawal symptoms, can be
attempted to reverse the bowel-slowing effects of opiates. Finally, treatment with
neostigmine, a cholinesterase inhibitor that results in parasympathomimetic activity, can
also be attempted.

13. The correct answer is: A. Begin enteral nutrition. At this point, more aggressive
nutritional support should be attempted because the patient is not meeting her caloric
needs on peripheral IV D5NS alone. Enteral nutrition is the preferred route of nutritional
support in the absence of a contraindication. The colonic pseudo-obstruction has resolved,
as evidenced by the spontaneous passage of stool and flatus and ability for the rectal tube
to be discontinued. Therefore, enteral nutrition should be attempted first, and the patient
should be monitored closely. Should she be unable to tolerate enteral nutrition due to
nausea, vomiting, or recurrence of colonic pseudo-obstruction, alternative forms of
nutritional support such as peripheral parenteral nutrition and total parenteral nutrition
should then be considered.

14. The correct answer is: C. Outpatient therapy with PO metronidazole and
ciprofloxacin for 7 days. The patient has risk factors for diverticulosis (age, obesity) and
presents with symptoms of diverticulitis. Cross-sectional imaging confirms this diagnosis
and demonstrates lack of complications (abscess, perforation, or fistula). The episode can
be characterized as mild in severity, because he has few comorbidities, is able to tolerate
PO intake, has pain that is well-controlled with acetaminophen, and has uncomplicated
disease on CT. Therefore, it is reasonable to pursue outpatient therapy with PO antibiotics
for a 7- to 10-day course. Antibiotic regimens may include metronidazole along with an
fluoroquinolone antibiotic or amoxicillin/clavulanic acid. Hospitalization should be
considered for patients who appear toxic, cannot tolerate PO intake, require narcotics for
pain, have significant medical comorbidity, or have complicated disease. Surgery would
not be indicated because this is the patient’s first presentation of diverticulitis and his
disease is uncomplicated. Surgical resection can be considered for multiple episodes of
recurrent diverticulitis. Finally, interventional radiology consultation for drain placement
is not needed at this time because he demonstrates no evidence of abscess formation.

15. The correct answer is: D. The risk of recurrence within 10 years is 10% to 30%.
After a first episode of diverticulitis, the 10-year risk of recurrent diverticulitis is 10% to
30%. Choice A is incorrect because the risk of complicated diverticulitis increases with a
second episode. Choice C is incorrect because there is no evidence to support the use of
amoxicillin/clavulanic acid in the prevention of recurrent disease; there is weak evidence
for mesalazine and rifaximin (Carter F, Alsayb M, Marshall JK, et al. Cochrane Database
Syst Rev. 2017;10:CD009839; Khan MA, Ali B, Lee WM, et al. Am J Gastroenterol.
2016;111:579; Elisei W, Tursi A. Ann Gastroenterol. 2016;29:24). Choice B is incorrect
because there is no specific number of episodes above which surgical resection should be
considered; surgical resection should be considered on a case-by-case basis.

16. The correct answer is: B. Family history of colorectal cancer in a first-degree
relative. Her family history of colorectal cancer in a first-degree relative places her at
increased risk of colon cancer. Guidelines recommend that persons with a family history
of colorectal cancer in a first-degree relative begin colorectal cancer screening at 40 years
of age, or 10 years before the earliest age of colorectal cancer diagnosis in the family
member, whichever comes first. (This patient should have begun screening at 40 years of
age.) There are no data to suggest that a personal history of hypertension or ulcerative
colitis in a cousin increases a patient’s risk of colorectal cancer. Finally, aspirin use is not
associated with increased risk of colorectal cancer; in fact, some data suggest that aspirin
use may be a protective factor against colorectal cancer.

17. The correct answer is: C. Inflammation of the colonic mucosa in a contiguous
pattern starting at the rectum and progressing proximally. In a patient with ulcerative
colitis, colonoscopy would be expected to show contiguous and circumferential
inflammation beginning at the rectum and progressing proximally. Deep cobble-stoning

Freemedicalbooks4download
ulceration with skip lesions is a feature seen in Crohn colitis. With this patient’s degree of
symptoms, including grossly bloody diarrhea, one would not generally expect a grossly
normal-appearing colonoscopy if this patient’s diagnosis is truly ulcerative colitis. Finally,
pseudomembranes are a feature typical of C. difficile infection.

18. The correct answer is: B. He has an increased risk of colorectal cancer compared to
the baseline population rate. The patient should be informed that he is at increased risk
of colorectal cancer with his diagnosis of left-sided ulcerative colitis, and he should
undergo a more aggressive colorectal cancer screening strategy compared with persons at
average risk of colorectal cancer. A typical screening strategy would be to perform a
surveillance colonoscopy 8 to 10 years after the initial diagnosis of ulcerative colitis and
then every 1 to 3 years thereafter. Active smoking is actually associated with a lower risk
of ulcerative colitis (it is, however, associated with a higher risk of Crohn disease).
Persons with ulcerative colitis have a normal life expectancy when compared with persons
without ulcerative colitis. Patients with recurrent inflammation due to ulcerative colitis
can develop strictures, typically in the rectosigmoid colon, although less frequently than
those with Crohn disease.

19. The correct answer is: D. All of the above. This patient is at risk for all of the
complications listed. Patients with extensive ileal disease or ileal resection experience
both fat malabsorption and impaired bile acid reabsorption. Fat malabsorption leads to
excess fat in the intestinal lumen, which binds calcium. Binding of calcium to fat leaves
an excess of available oxalate for colonic absorption, which can then lead to urinary
calcium oxalate stones. Intestinal fat malabsorption also leads to impaired reabsorption of
fat-soluble vitamins, including vitamin D, which can lead to osteopenia or osteoporosis.
Finally, an increased risk of gallstone formation in those with Crohn disease is not
completely understood, although it is thought to be related to impaired bile reabsorption
and circulation leading to hypersaturation of cholesterol into gallstones; however, about
half of the gallstones are bilirubin stones, so other mechanisms are involved.

20. The correct answer is: B. Infliximab. This patient has clinical, endoscopic, and
histologic findings consistent with severely acute ulcerative colitis. She has responded to
IV steroids, and once remission is achieved, she will need to be transitioned to an agent
for long-term maintenance of remission. For severe disease, many clinicians will choose a
biologic agent. This is in contrast to the “step-up” approach, which begins with the least
toxic regimen (typically a nonbiologic agent). From the list provided, infliximab is the
best treatment option for maintenance of remission in those presenting with severe acute
ulcerative colitis. Oral prednisone is incorrect, as the goal should be to transition patients
to a steroid-sparing agent for maintenance of remission, given the adverse effects of long-
term steroid use. Methotrexate is incorrect, as this therapy is only approved for Crohn
disease and not for ulcerative colitis. Cyclosporine is incorrect as this is a therapy used for
the management of acute severe ulcerative colitis but is not used as a long-term
maintenance therapy.

21. The correct answer is: A. CT angiography of the abdomen. This elderly patient who is
not anticoagulated for AF is at risk for arterial embolism. The “pain out of proportion to
physical examination findings” is classic for acute mesenteric ischemia. The elevated
lactate and WBC count are likely due to bowel infarction. Of the possibilities listed, CT
angiography (arterial phase) is the noninvasive test of choice (the venous phase is used for
the diagnosis of mesenteric venous thrombosis). Invasive angiography is the gold
standard. This can be potentially therapeutic (eg, embolectomy) and would be the next
step if CT angiography is suggestive of vascular occlusion.

22. The correct answer is: A. Decreased blood flow to the gut due to mesenteric
atherosclerosis. The presentation is classic for “intestinal angina,” which is characterized
by postprandial abdominal pain, early satiety, and weight loss due to fear of eating. The
pathophysiologic basis is decreased blood flow to the gut due to mesenteric
atherosclerosis, and the syndrome of chronic mesenteric ischemia can often be seen in
patients with vascular disease elsewhere (such as CAD, carotid disease, peripheral artery
disease). The pain is intermittent (postprandial), but if the pain becomes constant, acute
mesenteric thrombosis should be considered. The other choices are more often associated
with constant pain, rectal bleeding, abdominal tenderness, and elevated lactate or WBC
count depending on whether there is bowel infarction.

23. The correct answer is: D. Ultrasonography with Dopplers of the mesenteric vessels.
Ultrasonography with Dopplers or CT angiography (with arterial, not venous, phase
contrast) are reasonable first steps in the diagnosis of chronic mesenteric ischemia.
Although invasive angiography is the gold standard, it is reasonable to obtain imaging
with ultrasonography first, especially because the patient has a history of chronic kidney
disease and should avoid contrast. A possible role for tonometry, spectroscopic oximetry,
and MR angiography in the diagnosis of chronic mesenteric ischemia has been suggested,
but the clinical usefulness of these studies, which are still under investigation, has not
been established.

24. The correct answer is: A. IV lactated Ringer’s solution and pain management. The
patient has alcohol-related chronic pancreatitis. She has a history of multiple prior
episodes of acute pancreatitis, which explains why the lipase level is normal. Her story of
epigastric acute pain radiating to the back associated with nausea and vomiting is classic.
She has signs of volume depletion on examination. It is likely too early for alcohol
withdrawal, given that she had multiple drinks on the day of presentation. The mainstay of
treatment for pancreatitis is aggressive fluid resuscitation in the first 24 hours. A 20
mL/kg IV bolus should be followed by 3 mL/kg/h. Lactated Ringer’s solution may be
superior to normal saline.

25. The correct answer is: D. Percutaneous drainage. The patient has infected pancreatic
necrosis (5% of all cases, 30% of severe pancreatitis), which is associated with high
mortality. This should be treated with a carbapenem or metronidazole plus a
fluoroquinolone. She is clinically unstable and should undergo percutaneous drainage
(often followed by minimally invasive surgical debridement) or endoscopic necrosectomy;
these management options have been shown to be superior to open necrosectomy. IV
fluids and antibiotics may temporize but are not definitive management strategies.

26. The correct answer is: A. Antinuclear antibody and IgG subtypes. This patient
presents with steatorrhea and abdominal pain, which are likely a result of chronic

Freemedicalbooks4download
pancreatitis. Edema of the pancreas may cause compression of the bile duct, leading to
cholestatic liver biochemical levels. The patient is not a diabetic, thus endocrine function
of the pancreas has not been affected. The next best step in laboratory workup in this
young, otherwise healthy patient would be antinuclear antibody and IgG subtypes to
assess for autoimmune pancreatitis (IgG4-related autoimmune pancreatitis). There is no
evidence of infection, so cultures are not warranted. HIV, H. pylori testing, and a serum
alcohol level are unlikely to help elucidate the etiology of chronic pancreatitis in this case.
Glucocorticoids would be first-line treatment for autoimmune pancreatitis, and
immunomodulators (azathioprine, mycophenolate mofetil, cyclophosphamide, rituximab)
may be used if the patient relapses.

27. The correct answer is: B. Azithromycin. Of the options given, azithromycin is the most
likely drug to cause a pure cholestatic pattern of abnormal liver chemistries, as may other
macrolide antibiotics, azathioprine, chlorpromazine, estrogens, and 6-mercaptopurine
Acetaminophen classically causes hepatocellular injury with “towering” aminotransferase
elevation after an overdose. Hydralazine may also cause hepatocellular injury.
Nitrofurantoin, amoxicillin-clavulanic acid, azathioprine, carbamazepine, mirtazapine, and
penicillin may cause a “mixed” pattern with elevated AST, ALT, and alkaline phosphatase
levels. The next best step would be to discontinue the culprit medication (azithromycin)
and consider liver imaging if the abnormalities do not resolve.

28. The correct answer is: C. Review prior lab results and ask about a current illness or
stressor. This patient likely has Gilbert syndrome, as demonstrated by an elevated
unconjugated bilirubin with normal liver enzymes, normal liver synthetic function, and no
symptoms, medications, or alcohol use. In this patient, review of prior lab work would
reveal previous elevated indirect bilirubin levels, which improved after a period of illness
or stress. Gilbert syndrome is the most common inherited disorder of bilirubin
glucuronidation and is a benign condition (present in 4%-16% of the population). It is
characterized by recurrent episodes of jaundice and may be triggered by, among other
things, dehydration, fasting, intercurrent disease, menstruation, and overexertion. Other
than jaundice, patients are typically asymptomatic. The diagnosis is made by excluding
other causes of unconjugated hyperbilirubinemia, although genetic testing is available. A
presumptive diagnosis can be made in patients with the following features:

Unconjugated hyperbilirubinemia on repeated testing


A normal CBC, blood smear, and reticulocyte count
Normal serum aminotransferase and alkaline phosphatase levels

29. The correct answer is: C. Gamma-glutamyl transpeptidase. This patient has an
isolated elevated alkaline phosphatase, which may or may not relate to her itching. The
first step in working up an isolated elevated alkaline phosphatase is to repeat the test
fasting (a level 1.5-2 × ULN can be seen postprandially). Pregnancy and medications are
other common causes of an elevated alkaline phosphatase, but when these causes have
been ruled out, the next best step is to establish whether the alkaline phosphatase is of
bone or liver origin. Checking a γ-glutamyl transpeptidase would be the test of choice. If
the γ-glutamyl transpeptidase is elevated, ultrasonography should be done to look for bile
ductal dilatation, and testing antimitochondrial antibodies should be done for primary
biliary cholangitis. This patient is the right demographic for primary biliary cholangitis,
but the first step is to establish whether this is a bone or liver source of alkaline
phosphatase. Antioxidants are not indicated.

30. The correct answer is: C. IgM anti-hepatitis A virus (HAV). This patient has a story
typical for an acute hepatitis as demonstrated by the extremely high aminotransferase
levels (ALT > AST) and acute presentation with nausea, vomiting, jaundice, and RUQ
pain. There is an increased rate of HAV among males who have sex with men, and he has
also had sick contacts who may have been in day care centers (over the incubation period
of 2-6 weeks). HBV and HCV may be less likely, given the lack of IV drug use or other
bloodborne exposures. Alcohol hepatitis is even less likely, especially given the pattern
and degree of aminotransferase elevations (typically AST > ALT and ALT <500 U/L in
alcohol-associated hepatitis).

31. The correct answer is: B. Begin tenofovir. This patient has HBeAg-positive chronic
HBV (immunoreactive phase). This is demonstrated by the fact that he is HBeAg-positive
(anti-HBe negative) and has an elevated ALT level. On a liver biopsy specimen, he would
likely have moderate-to-severe inflammation and possible fibrosis. The HBV DNA level
is over 20 000 IU/mL. Chronic HBV infection should be treated in the immunoreactive
and immune reactivation phases and in cirrhotic patients with an elevated HBV DNA
level or decompensation. First-line treatment is with a nucleo(s/t)ide analog, entecavir or
tenofovir, which are well-tolerated and associated with a low rate of resistance at 5 years.
HBeAg seroconversion occurs in 30% to 40% of treated patients, with loss of HBsAg in
5% to 10%. Tenofovir is preferred if there is a history of lamivudine resistance.

32. The correct answer is: A. IV ceftriaxone. The patient has alcohol-associated hepatitis
and a high Maddrey discriminant function (>32); therefore, his mortality rate is high (1-
month mortality of 35%-45%). It is worth considering oral glucocorticoids, but he has
ongoing spontaneous bacterial peritonitis (SBP) and an active infection is a
contraindication to prednisolone (contraindications include active bacterial or fungal
infection, chronic HCV infection, or HBV infection). Although pentoxifylline would be a
therapeutic possibility in a patient with contraindications to glucocorticoids, it would not
be recommended in conjunction with glucocorticoids. He has had multiple admissions for
alcohol-associated hepatitis and would not be accepted for liver transplantation at this
time. Instead, he should be managed by treating his SBP, optimizing hydration, nutrition,
stress ulcer prophylaxis, and abstinence from alcohol.

33. The correct answer is: D. Suggest treatment with elbasvir-grazoprevir (a direct-
acting antiviral regimen). Treatment with direct-acting antiviral therapy should be made
available to all patients with chronic HCV infection, unless life expectancy is short, in
order to prevent complications such as cirrhosis and hepatocellular carcinoma. The direct-
acting antivirals especially given the pattern and degree of aminotransferase elevations
(typically AST > ALT and ALT <500 U/L in alcohol-associated hepatitis) can achieve
sustained virologic responses or cure in a high (>95%) proportion of patients and have
few adverse events. HCV treatment with direct-acting antivirals has been shown to
improve mortality, even in the absence of cirrhosis (Carrat F, Fontaine H, Dorival C.
Lancet. 2019;393:1453), and the World Health Organization (WHO) has set an

Freemedicalbooks4download
elimination target for HCV by 2030.
In patients with an eGFR <30 mL/min or on dialysis, the most robust data are for
the direct-acting antivirals glecaprevir-pibrentasvir (Mavyret) and elbasvir-grazoprevir
(Zepatier). Elbasvir-grazoprevir is a pangenotypic regimen that has been shown to have
excellent sustained viral response rates with 12 weeks of treatment in patients with
chronic kidney disease or end-stage renal disease (including those on dialysis).
Treatment with pegylated interferon and ribavirin is rarely used, as it has a worse
side effect profile and lower sustained viral response rate than direct-acting antivirals.
There is no need to perform a liver biopsy to confirm absence or presence of cirrhosis,
given that there is no evidence of fibrosis or cirrhosis on imaging and vibration-controlled
transient elastography. Moreover, compensated cirrhosis would not change treatment
recommendation in this case.

34. The correct answer is: B. N-acetylcysteine. Adult patients who present soon (within 4
hours) after a potentially toxic ingestion of acetaminophen (single dose ≥7.5 g) are likely
to benefit from GI decontamination with activated charcoal. Charcoal should be withheld
in patients who are sedated and may not be able to protect their airway. This patient’s
presentation is too delayed and she is too lethargic for administration of charcoal.
Acetaminophen is not removed by hemodialysis, and pentoxifylline is not useful in the
treatment of acute liver failure due to acetaminophen. N-acetylcysteine is the accepted
antidote for acetaminophen poisoning and is given to all patients at significant risk for
hepatotoxicity. Although the level is not yet back, she has evidence of liver damage and a
history of acetaminophen ingestion.
Indications for N-acetylcysteine therapy include:

Serum acetaminophen concentration drawn at 4 hours or more following acute ingestion of


an immediate-release preparation is above the “treatment” line of the treatment nomogram
for acetaminophen poisoning
A suspected single ingestion of greater than 150 mg/kg (7.5 g total dose regardless of
weight) in a patient for whom the serum acetaminophen concentration will not be available
until more than 8 hours from the time of ingestion
An unknown time of ingestion and a serum acetaminophen concentration >10 μg/mL
A history of acetaminophen ingestion and any evidence of liver injury
A delayed presentation (>24 hours after ingestion) of laboratory evidence of liver injury
(ranging from mildly elevated aminotransferase levels to acute liver failure) and a history of
excessive acetaminophen ingestion

35. The correct answer is: C. IV mannitol. The patient has developed grade 3
encephalopathy (somnolence to stupor, confusion, gross disorientation) requiring
intubation for airway protection. Acute hyperammonemia (as in acute liver failure)
induces accumulation of glutamine inside astrocytes and causes cerebral edema, a
common cause of death in this patient population. Although the evidence is mixed for
routine intracranial pressure monitoring in acute liver failure, it should be considered in
patients with grade 3 or 4 encephalopathy. If intracranial pressure is found to be elevated,
mannitol, 0.5 to 1.0 mg/kg IV, should be given. In patients with grade 3 or 4
encephalopathy, acute kidney injury, or on a vasopressor, consider prophylactic 3%
hypertonic saline with a goal of a serum sodium concentration 145 to 155 mEq/L. In
patients with refractory elevated intracranial pressure despite these measures, consider
initiation of a barbiturate and therapeutic hypothermia. Lactulose should generally be
avoided in patients with acute liver failure and elevated ammonia because of the risk of
volume depletion from a high stool output.

36. The correct answer is: C. Platelets. Patients with acute liver failure are at risk for
bleeding due to multiple hematologic abnormalities (thrombocytopenia, elevated
prothrombin time [PT] and partial thromboplastin time, decreased fibrinogen, decreased
synthesis of coagulation factors, although this is somewhat balanced by decreased protein
C and S levels, and disseminated intravascular coagulation). The INR is not helpful for
determining the bleeding risk because of the decrease in protein C and S production,
which are not directly measured. Neither fresh frozen plasma nor cryoprecipitate are
administered to “correct” the PT/INR prior to a procedure, because several large reviews
of available evidence have shown no clinical benefit. In this patient, an INR of 1.5 would
not be significantly changed by fresh frozen plasma administration (studies have shown
that transfusions of fresh frozen plasma will not reliably decrease the INR below about
1.7). Cryoprecipitate should be given to patients who are bleeding and who have a
fibrinogen level of ≤100 to 120 mg/dL. Vitamin K can be given if there is concern for
vitamin K deficiency to assess whether the INR corrects, but IV vitamin K takes 6 to 8
hours to be effective and would not be useful right before a procedure. It is reasonable to
aim for a platelet count of 50 000/mm3 in the setting of bleeding or prior to a procedure
(as in this case), but in the absence of bleeding or an imminent procedure, platelet
administration should be avoided unless the platelet count is <10 000 to 15 000.

37. The correct answer is: A. IV ceftriaxone. Empiric antibiotic therapy for spontaneous
bacterial peritonitis (SBP) pending culture results, include third-generation cephalosporin
such as ceftriaxone or amoxicillin-clavulanic acid for 5 days. In cases that occur in areas
with high fluoroquinolone resistance or when the patient is already taking a
fluoroquinolone for SBP prophylaxis, it is better to avoid this class of antibiotic. There are
increasing rates of extended-spectrum β-lactamase (ESBL) infections and infections
caused by other resistant organisms. Therefore, if the patient has a history of resistant
infections, has had multiple recent hospitalizations, or is systemically unwell, it is
reasonable to consider broader empiric coverage such as a fourth-generation
cephalosporin or a carbapenem.

38. The correct answer is: B. Hold diuretics and give IV albumin 1 g/kg. The patient may
have hepatorenal syndrome. Common precipitants include GI bleeding, overdiuresis,
infection, serial large-volume paracenteses, and drugs such as NSAIDs. In suspected
hepatorenal syndrome, the first step is to rule out prerenal acute kidney injury with
volume expansion by holding diuretics and giving IV albumin 1 g/kg/d for 2 days. In this
case, it would also be appropriate to discontinue the β-blocker (rather than simply reduce
the dose). If the renal function and MAP do not improve with an albumin challenge, a trial
of midodrine and octreotide or a vasopressor to raise the patient’s blood pressures and
improve renal perfusion would be appropriate. There is no evidence of infection, so a
workup and treatment for sepsis are not currently indicated.

39. The correct answer is: A. Endoscopic variceal ligation and nonselective β-blocker.

Freemedicalbooks4download
Primary prevention for medium-to-large esophageal varices that have not yet bled
includes a β-blocker or endoscopic variceal ligation (usually not both). Secondary
prevention for all patients after a first variceal bleed includes β-blocker and endoscopic
variceal ligation (rather than either alone) because of the ~50% risk of rebleeding and
~30% mortality rate. Transjugular intrahepatic portosystemic shunt (TIPS) can also be
considered in patients with Child class B or class C cirrhosis within 72 hours of admission
for a variceal bleed (less rebleeding, more encephalopathy, no change in mortality)
(Holster IL, Tjwa ET, Moelker A. Hepatology. 2016;63:581), or for refractory bleeding
not controlled with endoscopic therapy. In this case, choice D is not the next most
appropriate management step, as the patient is not currently bleeding, and therapy with
endoscopic variceal ligation and nonselective β-blocker should first be attempted. A
Sengstaken-Blakemore tube may be placed in a patient with uncontrollable bleeding, often
followed by TIPS placement.

40. The correct answer is: D. Esophagogastroduodenoscopy screening for varices and
initiation of anticoagulation. Treatment of acute portal vein thrombus in a noncirrhotic
patient includes low-molecular-weight heparin, followed by warfarin for 6 months, or
indefinitely if the cause is irreversible. In a patient with cirrhosis, anticoagulation
increases the rates of recanalization of the portal vein without increasing the risk of
bleeding (Loffredo L, Pastori D, FarcomenA, et al. Gastroenterology. 2017;153:480). The
patient should be screened for high-risk varices prior to initiation of anticoagulation (Qi
X, Han G, Fan D. Nat Rev Gastroenterol Hepatol. 2014;11:435) and, if identified, variceal
bleeding prophylaxis is initiated. For chronic portal vein thromboses, anticoagulation
should be initiated if the patient is noncirrhotic or has a hypercoagulable state. If the
patient is cirrhotic, anticoagulation may be considered if the patient is symptomatic or is
demonstrated to have progression of the thrombus on follow-up imaging.

41. The correct answer is: A. Abdominal ultrasonography with Dopplers. The presenting
features are suggestive of Budd-Chiari syndrome (BCS, a rare disorder, occurring in one
per million people). BCS should be suspected in patients with a history of oral
contraceptive pill (OCP) use, prior venous thrombosis, myeloproliferative disorder such as
polycythemia rubra vera, or malignancy. In 25% of cases, BCS is idiopathic. The
presentation often consists of RUQ abdominal pain, ascites, and hepatomegaly. Serum
AST, ALT, and alkaline phosphatase levels are often elevated. Acute liver failure may be
present. The diagnosis is generally made using abdominal ultrasonography with Dopplers,
which shows an occlusion in the hepatic veins or inferior vena cava (IVC). Alternative
approaches include MRI and MRV or CT of the abdomen and pelvis with IV contrast. In
this patient’s case, MRI with gadolinium should be avoided due to the risk of nephrogenic
systemic fibrosis associated with end-stage renal disease.

42. The correct answer is: D. Sinusoidal obstruction syndrome. Sinusoidal obstruction
syndrome (formerly known as hepatic veno-occlusive disease) results from the occlusion
of hepatic venules and sinusoids due to a toxic insult to hepatic vein endothelium. The
syndrome is associated with a 20% mortality rate. It is characterized by hepatomegaly,
RUQ pain, jaundice, and ascites, most often occurring in patients undergoing HSCT and
less commonly following the use of certain chemotherapeutic agents in nontransplant
settings, ingestion of alkaloid toxins, high-dose radiation therapy, or liver transplantation.
Treatment is supportive and consists largely of fluid management with diuretics.

43. The correct answer is: C. Portal hypertension. The serum-ascites albumin gradient
equals the serum albumin concentration minus the ascites albumin concentration (in
g/dL). A serum-ascites albumin gradient ≥1.1 signifies portal hypertension with ~97%
accuracy. If portal hypertension plus another cause of ascites (seen in ~5% of cases)
coexist, the serum-ascites albumin gradient is still ≥1.1. In this patient’s case, the serum-
ascites albumin gradient is 2.5, suggestive of portal hypertension (most likely due to
cirrhosis). The other causes would typically present with a serum-ascites albumin gradient
of <1.1.

44. The correct answer is: A. CT of the abdomen and pelvis. The patient has secondary
bacterial peritonitis, which may be due to an intra-abdominal abscess or perforation and
which should be suspected in a patient who has recently undergone laparotomy. Runyon
criteria include the following: ascitic fluid total protein >1 g/dL, glucose <50 mg/dL, and
LDH >ULN for serum. Cultures may reveal polymicrobial growth. Treatment is with a
third-generation cephalosporin and metronidazole. Urgent abdominal imaging with CT is
most appropriate in this case, and the patient may require an exploratory laparotomy.

45. The correct answer is: D. IV vancomycin and gentamicin. Peritonitis in the setting of
peritoneal dialysis is usually caused by intraluminal contamination (suboptimal sterile
technique when handling the catheter, or “touch contamination”). Approximately 45% to
65% of cases are caused by gram-positive organisms and 15% to 35% by gram-negative
organisms. In up to 40% of cases, a causative organism is never found, and empiric
antibiotics should cover both gram-negative and gram-positive organisms. Coagulase-
negative staphylococcus will cause the majority of infections, and some will be
methicillin-resistant. Of the antibiotic choices, vancomycin and gentamicin cover
methicillin-resistant gram-positive organisms as well as gram-negative organisms
effectively.

46. The correct answer is: C. Endoscopic retrograde cholangiopancreatography


(ERCP). This patient has acute (“ascending”) cholangitis, likely due to a bile duct stone
(although this is not seen on the ultrasonography, the bile duct is dilated, which is
suggestive of choledocholithiasis in the right clinical setting). She has “Reynolds pentad,”
which is a syndrome of RUQ pain, jaundice, and fever and chills (Charcot triad) plus
shock and confusion (present in ~15% of patients). Antibiotics should be initiated as soon
as possible and should be broad-spectrum to cover common bile pathogens (eg, ampicillin
+ gentamicin [or levofloxacin] with or without metronidazole [if severe]; carbapenems;
piperacillin-tazobactam). If there is a history of resistant organisms, carbapenem can be
used initially. Approximately 80% of patients respond to conservative treatment with
antibiotics, in which case biliary drainage can be performed on an elective basis.
Approximately 20% require urgent biliary decompression via endoscopic retrograde
cholangiopancreatography (papillotomy, stone extraction, and/or stent insertion), as in this
case.

47. The correct answer is: D. Transabdominal ultrasonography. This patient has many of
the risk factors for cholelithiasis (female, obese, pregnant, age >40 years). She is

Freemedicalbooks4download
presenting with symptoms typical of biliary pain (“colic”): episodic RUQ or epigastric
abdominal pain that begins abruptly, is continuous, resolves slowly and lasts for 30
minutes to 3 hours, with or without radiation to scapula, and is associated with nausea and
belching. Although it is possible that she has GERD, gastritis, and/or peptic ulcer disease,
the symptoms are typical of biliary pain related to gallstones, which should be explored
first with abdominal ultrasonography. Management of biliary pain is usually supportive in
pregnant women, but if bouts of biliary pain are frequent, primary surgical management
during pregnancy is reasonable because recurrence is common with conservative therapy
and surgical therapy appears to be safe for mother and fetus. The preferred time for
surgery is the second trimester.

48. The correct answer is: A. Cholecystectomy. Acalculous cholecystitis is caused by


gallbladder stasis and ischemia (in the absence of cholelithiasis), resulting in a
necroinflammatory response, and occurs mainly in critically ill or hospitalized patients.
Acalculous cholecystitis often occurs after major surgery and may be associated with total
parenteral nutrition, sepsis, trauma, burns, opiates, immunosuppression, and infections
such as cytomegalovirus, Candida, Cryptosporidiosis, Campylobacter, and typhoid fever.
Although gallbladder abnormalities such as biliary sludge are present in many critically ill
patients, gas in the gallbladder wall or lumen, lack of gallbladder wall enhancement, and
edema around the gallbladder have the highest specificity for acalculous cholecystitis
(99%, 95%, and 92%, respectively). In this patient with a persistent fever and shock,
acalculous cholecystitis may be the source of sepsis. Indications for an emergency
cholecystectomy include gallbladder necrosis, emphysematous cholecystitis (as in this
case) due to a gas-forming organism, and gallbladder perforation. In acalculous
cholecystitis without these features, a trial of a cholecystostomy tube would be the
appropriate initial treatment.
4
NEPHROLOGY

QUESTIONS

1. A 25-year-old man with type 1 diabetes mellitus is admitted with lethargy and altered
mental status. His roommate reports that the patient recently lost his job, started drinking
again, and has been rationing his insulin. He takes topiramate for seizure disorder. On
examination, the patient is barely arousable to stimuli. His blood pressure (BP) is 90/50
mmHg and his pulse is 120/min. Laboratory studies show: Na 128 mEq/L, K 5.3 mEq/L,
Cl 94 mEq/L, HCO3 14 mEq/L, glucose 560 mg/dL, serum osmolality 303 mmol/kg, and
serum lactate 3.5 mmol/L. Arterial blood gas (ABG) shows a pH of 7.20, PaCO2 of 23
mmHg, and PaO2 of 88 mmHg. Urinalysis is positive for ketones and glucose and a urine
toxicology screen is positive for ethanol and cannabinoids.
What is the cause of metabolic acidosis in this patient?
A. Alcohol intoxication
B. Diabetic ketoacidosis
C. Lactic acidosis
D. Renal tubular acidosis

2. A 35-year-old man with a history of alcohol use disorder complicated by seizures is


brought into the emergency department (ED) as he was found stumbling in the park by
pedestrians. He is afebrile, with heart rate (HR) in the 90s and BP 120/70s. His labs are
notable for pH 7.20, pCO2 32 mmHg (ABG) with Na 125 mEq/L, Cl 90 mEq/L, HCO3 15
mEq/L, and albumin of 3.0 g/dL with a lactate elevated at 6 mmol/L. The ED orders 2 L
lactated Ringer’s, with repeat labs showing pH 7.18 with Cl 89 mEq/L, HCO3 20 mEq/L,
and repeat lactate 7 mmol/L.
What is the most likely cause of this patient’s acid-base disturbance?
A. Alcoholic ketosis
B. Hypovolemia
C. Sepsis
D. Thiamine deficiency
E. Vomiting

Freemedicalbooks4download
3. A 38-year-old man with unknown past medical history is brought into the ED after he
was found lying on the sidewalk. He is disoriented but reports intermittent blurry vision.
He is afebrile with a pulse rate of 105/min, BP of 128/80 mmHg, and respiratory rate (RR)
of 28/min. Examination is notable for pupillary dilation, poor dentition, and mild diffuse
abdominal pain. Labs reveal Na 132 mEq/L, Cl 92 mEq/L, HCO3 22 mEq/L, blood urea
nitrogen (BUN) 30 mg/dL, Cr 1.2 mg/dL, Ca 7.4 mg/dL, glucose 160 mg/dL, and albumin
3 g/dL. Liver enzymes are normal. The serum alcohol level is 160 mg/dL and the
measured serum osmolality is 350 mOsm/kg.
What is the most likely ingestion causing this patient’s presentation?
A. Aspirin toxicity
B. Ethylene glycol
C. Isopropyl alcohol
D. Methanol
E. Propylene glycol

4. A 38-year-old man comes to the hospital after a fall. He is found to have an ankle
fracture, which is managed conservatively. He admits extensive alcohol use and reports he
would like to quit. He develops symptoms of alcohol withdrawal 48 hours into the
hospitalization and is treated with intravenous (IV) diazepam per the Clinical Institute
Withdrawal Assessment for Alcohol Scale (CIWA) scale; 36 hours later, he develops
worsening confusion and respiratory failure. Medications at this time include high-dose
IV diazepam drip, IV thiamine, multivitamin, and acetaminophen. BP is 160/80 mmHg
and his pulse is 108/min. On examination, he is somnolent and responds only to pain.
Labs show an Na of 145 mEq/L, K of 3.8 mEq/L, Cl of 101 mEq/L, and HCO3 of 20
mEq/L. Serum osmolality is 390 mOsm/kg, glucose 130 mg/dL, creatinine 1.6 mg/dL, and
lactate 5 mmol/L. ABG shows a pH of 7.30, PaCO2 of 42 mmHg, and PaO2 of 88 mmHg.
Urinalysis is mildly positive for ketones. Ethanol level is 2 mg/dL.
What is the most likely cause of this patient’s metabolic abnormalities?
A. Alcoholic ketoacidosis
B. Diazepam
C. Ethylene glycol
D. Infection
E. Isopropyl alcohol

5. A 55-year-old woman presents with weakness after 3 days of abdominal pain, diarrhea,
and poor PO intake. She ate at a Chinese restaurant and 8 hours later developed crampy
abdominal pain followed by multiple episodes of watery nonbloody diarrhea. She presents
with weakness and inability to stand. In the ED, she is afebrile with HR 108 bpm, BP
100/60 mmHg, and RR 22 breaths/min. She has dry mucous membranes and no lower
extremity edema. Labs are notable for pH of 7.18 with a pCO2 of 28 mmHg on an ABG
with Na 130 mEq/L, Cl 108 mEq/L, HCO3 12 mEq/L, K 3.0 mEq/L, and albumin 4.0
g/dL.
What is the most likely cause of this patient’s acid-base disorder?
A. Diarrhea
B. Hyperventilation
C. Lactic acidosis
D. Starvation ketosis
E. Vomiting

6. A 48-year-old woman with recently diagnosed rheumatoid arthritis is referred to a


nephrologist for a low bicarbonate level. On evaluation, she has no complaints. Labs
reveal Na 138 mEq/L, K 3.2 mEq/L, Cl 118 mEq/L, HCO3 8 mEq/L, and albumin 4 g/dL
with a serum pH of 7.28 and a pCO2 of 24 mmHg. Urine studies are notable for Na 40
mEq/L, K 20 mEq/L, and Cl 30 mEq/L with urinalysis notable for trace blood, no protein,
and a pH of 6.0.
What is the most likely complication if this condition is left untreated?
A. Hyperkalemia
B. Kidney stones
C. Osteomalacia
D. Sepsis
E. Venous thromboembolism

7. A 69-year-old man presents to the hospital with difficulty breathing. He reports that he
has had trouble “catching his breath,” especially when he walks up the stairs. He was
previously very active and regularly played tennis with friends, which he is no longer able
to do. He has a recent history of bladder cancer and underwent a cystoprostatectomy and
ileal neobladder creation 2 months ago. He reports that, recently, he has had to change his
urine bag less frequently but does not report any abdominal pain or blood in the urine. He
takes calcium, vitamin D, and docusate. On examination, his BP is 110/75 mmHg, pulse
rate is 110/min, and RR is 24/min. His lungs are clear to auscultation. Chest radiograph
(CXR) and electrocardiogram (ECG) are normal. Laboratory studies show an Na of 135
mEq/L, K of 3.7 mEq/L, Cl of 110 mEq/L, and HCO3 of 14 mEq/L. ABG shows a pH of
7.28, pCO2 of 30 mmHg, and pO2 of 78 mmHg. Urinalysis shows 20 to 50 red blood cells
(RBCs) and 20 to 50 white blood cells (WBCs) per high-power field (hpf).
What is the cause of this patient’s acid-base abnormalities?
A. Neobladder dysfunction
B. Pulmonary embolism
C. Renal tubular acidosis
D. Type B lactic acidosis
E. Urinary tract infection

8. A 32-year-old man presents to the ED with nausea and vomiting after eating sushi the
night prior. Vitals include a pulse rate of 108/min, BP of 110/74 mmHg, and RR of
20/min. Examination is notable for dry mucous membranes and epigastric tenderness.
Labs reveal Na 126 mEq/L, K 3.2 mEq/L, Cl 87 mEq/L, and HCO3 20 mEq/L with
albumin 2 g/dL. An ABG shows a pH of 7.38 and a pCO2 of 36 mmHg.
What is the cause of the patient’s acid-base disturbance?
A. High anion gap metabolic acidosis
B. High anion gap metabolic acidosis, metabolic alkalosis with compensatory
respiratory alkalosis
C. Metabolic alkalosis and non–gap metabolic acidosis with compensatory respiratory
acidosis

Freemedicalbooks4download
D. No acid-base abnormality
E. Non–gap metabolic acidosis

9. A 28-year-old woman presents to the ED with 3 days of emesis with inability to tolerate
PO intake. She ate at a seafood restaurant 8 hours prior to symptom onset. Vitals are
notable for HR 115/min, BP 92/52 mmHg, and RR 12 breaths/min. She is found to have
dry mucous membranes with no lower extremity edema. Labs are notable for pH 7.50,
PaCO2 48 mmHg with Na 130 mEq/L, Cl 88 mEq/L, HCO3 30 mEq/L, and K 2.8 mEq/L.
What is the best treatment for this patient?
A. Acetazolamide
B. Hemodialysis
C. Intubation for mechanical ventilation
D. IV normal saline
E. Observation

10. An ABG shows pH 7.15, pCO2 20 mmHg, pO2 80 mmHg, and HCO3 12 mEq/L and a
chemistry panel shows an anion gap of 16 mEq/dL (normal 10-12 mEq/L).
What is the acid-base disturbance here?
A. Anion gap metabolic acidosis
B. Anion gap metabolic acidosis + non–gap metabolic acidosis
C. Anion gap metabolic acidosis + non–gap metabolic acidosis + respiratory alkalosis
D. Anion gap metabolic acidosis + respiratory alkalosis
E. Respiratory acidosis + metabolic alkalosis

11. A 78-year-old woman is hospitalized after a fall and is found to have a subarachnoid
hemorrhage. She is managed conservatively with nimodipine. BP is 100/75 mmHg and
HR is 90/min. Her physical examination is unremarkable except for mild visual field
deficits as well as absence of axillary sweat and dry mucous membranes. Admission labs
are Na 139 mEq/L, K 4.5 mEq/L, and Cr 0.9 mg/dL. Five days later, her labs show Na
126 mEq/L, K 3.6 mEq/L, and Cr 0.9 mg/dL. Serum osmolality is 262 mOsm/kg, urine
sodium is 49 mEq/L, and urine osmolality is 360 mOsm/kg. AM serum cortisol is 8 μg/dL
(normal 5-25 μg/dL) and thyroid-stimulating hormone (TSH) is 8 μU/mL (normal 0.5-5
μU/mL). Free triiodothyronine (T3) and thyroxine (T4) levels are normal. After
administration of 1 L of IV saline, serum Na is 127 mEq/L while urine Na is 60 mEq/L
and urine osmolality is 260 mOsm/kg.
What is the etiology of hyponatremia in this patient?
A. Cerebral salt wasting
B. Hypoadrenalism
C. Hypothyroidism
D. Hypovolemia
E. Syndrome of inappropriate antidiuretic hormone secretion (SIADH)

12. A 34-year-old man presents to the ED after he was found to be confused by his friend at
home. He is alert but disoriented and unable to answer questions. His friend relays that he
was recently fired from his job and has been at home for the last week drinking 3 to 4 six-
packs of beer per day. Vitals: he is afebrile, pulse 68/min, and BP 140/86 mmHg.
Examination is notable for moist mucous membranes with normal capillary refill and skin
turgor. Serum sodium is found to be 126 mEq/L with a glucose level of 120 mg/dL. Urine
studies are sent. Urine osmolality is 75 mOsm/kg.
What is the next best step in management?
A. Administer loop diuretic
B. Fluid restrict
C. IV normal saline
D. Tolvaptan

13. A 76-year-old woman with a history of lung cancer presents to the ED with weakness 2
days following outpatient chemotherapy treatment. Vitals are notable for pulse 80/min, BP
110/70 mmHg, and RR 20 breaths/minute. Labs are notable for Na 120 mEq/L. She
receives 2 L of normal saline over 1 hour. She suffers a tonic-clonic seizure 30 minutes
later. Repeat labs show Na 116 mEq/L.
What is the most likely etiology of this patient’s hyponatremia?
A. Acute kidney injury
B. Chemotherapy
C. Hypovolemia
D. Inappropriate antidiuretic hormone (ADH) release
E. Vomiting

14. A 72-year-old woman with hypothyroidism presents to the ED after her family found her
confused in her apartment. She weighs 65 kg, is afebrile, with a pulse rate of 108/min, BP
122/74 mmHg, and RR 18/min. She does not require supplementary oxygen. She is
complaining of nausea and headache but is arousable. In the ED, she has an episode of
vomiting. Examination is notable for moist mucous membranes. Labs are notable for Na
118 mEq/L, BUN 18 mg/dL, and glucose 87 mg/dL with a serum osmolality of 250
mOsm/kg. UNa is 35 mEq/L and urine osmolality 450 mOsm/kg.
What is the next best step in management?
A. Bolus 2 L normal saline
B. Check TSH and free T4 levels
C. Give 3% hypertonic saline as a 100 mL bolus in 10 minutes, then recheck labs
D. Restrict free water and check labs in 4 hours
E. Start IV normal saline at 250 mL/h and recheck labs in 4 hours

15. A 76-year-old woman with a history of Alzheimer disease and amyloidosis presents to the
ED accompanied by her family, who noticed that she has been more confused and
lethargic in the last few weeks. She is incontinent at baseline and the family does not think
she has had any change in her urine output. Vitals are notable for HR 98/min, BP 118/76
mmHg, and RR 18 breaths/min. On examination, she is somnolent but easily arousable,
with dry mucous membranes. Serum sodium is 156 mEq/L with glucose 140 mg/dL.
Urine osmolality is found to be 225 mOsm/kg with a serum osmolality of 335 mOsm/kg.
A Foley catheter is placed and IV normal saline is started. Over the next 12 hours, she
voids 1.5 L and repeat serum Na is 154 mEq/L and urine osmolality is 255 mOsm/kg.
Desmopressin (DDAVP) is administered and urine osmolality increases from 285 to 310
mOsm/kg.
What is the most likely diagnosis?
A. Central diabetes insipidus

Freemedicalbooks4download
B. Gastrointestinal (GI) free water loss
C. Lack of access to free water
D. Nephrogenic diabetes insipidus
E. Osmotic diuresis

16. A 26-year-old pregnant woman is admitted at 32 weeks of gestation with suspected


preeclampsia. She reports increased thirst and urinary frequency in the last few weeks.
She has a history of bipolar disorder, which was managed with lithium in the past. Her BP
is 155/80 mmHg and her physical examination is unremarkable. Her blood counts and
liver function tests are normal. She is treated with bed rest, IV magnesium, and labetalol.
When her polyuria persists despite fluid restriction, a desmopressin challenge is
administered.
Baseline 6 hours of fluid restriction 2 hours after desmopressin challenge
Sodium 138 mEq/L 143 mEq/L 141 mEq/L
Serum osmolality 287 mOsm/kg 295 mOsm/kg 290 mOsm/kg
Urine osmolality 90 mOsm/kg 95 mOsm/kg 310 mOsm/kg
Urine volume 8 L/24 hours 3 L/6 h 300 mL/2 h
What is the cause of her polyuria?
A. Central diabetes insipidus
B. Gestational diabetes insipidus
C. Nephrogenic diabetes insipidus
D. Preeclampsia
E. Primary polydipsia

17. A 38-year-old man presents to the ED with palpitations and diarrhea. The evening prior,
he participated in an eating contest and ate a large quantity of black licorice, after which
he has felt ill and has developed diarrhea. Vitals are notable for a pulse rate of 90 to
95/min (frequent premature ventricular contractions), BP 172/102 mmHg, and RR 22/min.
Examination is notable for an anxious man without distress; mucous membranes are
moist; and abdomen is mildly tender. Labs are notable for Na 142 mEq/L, Cl 102 mEq/L,
HCO3 18 mEq/L, K 2.8 mEq/L, WBC 9 × 109/L, urine Na 10 mg/dL, K 40 mg/dL, Cl 60
mg/dL, and Cr 2 mg/dL.
What is the most likely etiology for this patient’s hypokalemia?
A. Diarrhea
B. Licorice ingestion
C. Renal tubular acidosis
D. Thiazide
E. Vomiting

18. A 70-year-old woman is admitted to the intensive care unit (ICU) after an out-of-hospital
cardiac arrest. She undergoes coronary revascularization and is then started on a
therapeutic hypothermia protocol. Her temperature is 32.5°C after 12 hours. She is
intubated, sedated, and receiving vasopressin and norepinephrine infusions. The resident
on call notices that the patient’s potassium has fallen from 3.9 mEq/L at admission to 2.8
mEq/L. The urine output recorded over the last 24 hours is about 3.5 L and the serum
creatinine is normal. Stool output is 200 mL.
What is the cause of this patient’s hypokalemia?
A. Cardiac revascularization
B. Diarrhea
C. Hyperaldosteronism renal tubular acidosis
D. Hypothermia
E. Mechanical ventilation

19. A 72-year-old man with a history of chronic lymphocytic leukemia presents to his
oncologist for follow-up. He has been doing well and has no complaints. Labs show Na
132 mEq/L, K 8.2 mEq/L, HCO3 26 mEq/L, BUN 28 mg/dL, Cr 1.2 mg/dL, glucose 220
mg/dL, and WBC 372/μL. ECG shows normal sinus rhythm and no peaked T waves. Ca
gluconate, insulin, and dextrose are administered. Repeat potassium is 8.5 mEq/L and
repeat ECG shows no changes.
What is the most likely etiology of this patient’s hyperkalemia?
A. Hyperglycemia
B. Hyporeninemic hypoaldosteronism
C. Pseudohyperkalemia
D. Renal failure
E. Tumor lysis syndrome

20. A 52-year-old man with a history of diabetes, hypertension, chronic kidney disease
(baseline Cr 1.5), and chronic obstructive pulmonary disease (COPD) is admitted to the
hospital with a COPD exacerbation. He is on day 3 of his hospitalization and is being
treated for COPD with short-acting bronchodilators and oral prednisone. Today, his labs
are notable for pH 7.32 with Na 130 mEq/L, K 5.5 mEq/L, HCO3 28 mEq/L, BUN 30
mg/dL, Cr 1.7 mg/dL, and glucose 450 mg/dL.
What is the most likely cause of this patient’s hyperkalemia?
A. Acidemia
B. Acute kidney injury
C. Excessive dietary intake
D. Hyperglycemia
E. Medications

21. A 68-year-old man with hypertension, diabetes, and end-stage renal disease (ESRD) on
hemodialysis on the surgical service is post-op day 2 from a small bowel resection after
presenting to the ED with a small bowel obstruction. The patient is anuric at baseline.
Vitals are notable for a HR of 86/min and BP 150/80 mmHg. On examination, mucous
membranes are moist, jugular venous pressure (JVP) is 8 cm, and abdomen is mildly
tender with a well-healing vertical midline incision. He has a right brachiocephalic fistula
with a palpable thrill. Labs are notable for Na 134 mEq/L, K 5.8 mEq/L, and BUN 50
mg/dL. ECG is reviewed, which shows a normal sinus rhythm with stable T-wave
inversions in V4-V6. The patient is scheduled to have dialysis in 2 days, so you decide to
give 10 U regular insulin IV with two amps of D50W and would like to give sodium
polystyrene sulfonate (SPS) (Kayexalate).
What is a contraindication to SPS administration?
A. COPD
B. Hyperglycemia

Freemedicalbooks4download
C. Hyponatremia
D. Recent bowel surgery
E. Sepsis

22. A 60-year-old man with a history of coronary artery disease (CAD) s/p coronary artery
bypass grafting (CABG) is found down at home by his family members. EMS is called;
when they arrive, he is found pulseless and cardiopulmonary resuscitation (CPR) is
initiated. He is found to be in ventricular fibrillation; shock is administered and return of
spontaneous circulation is achieved. He is intubated and brought to the ED. CXR is
notable for diffuse opacities. He is requiring 70% FiO2 and 12 of positive end-expiratory
pressure to achieve a PaO2 70 mmHg. Mean arterial pressures (MAPs) are in the 70s on
30 norepinephrine. Labs are notable for potassium 6.5 mEq/L with peaked T waves. A
Foley is placed and there is no urine output. He receives Ca gluconate, insulin, dextrose,
and IV furosemide. After 1 hour, urine output is 5 mL. Repeat labs show a potassium of
6.6 mEq/L.
What is the next best step in management?
A. Consult nephrology for renal replacement therapy
B. Give IV albuterol
C. Give normal saline
D. Give potassium binding resin
E. Repeat a basic metabolic panel

23. A 67-year-old man with a history of CAD, hypertension, and heart failure with reduced
ejection fraction presents to the hospital with 7 days of worsening dyspnea on exertion,
orthopnea, and weight gain. In the ED, he is tachypneic and requires 4 L of oxygen
supplementation to maintain O2 saturation >94%. His examination is notable for elevated
JVP, diffuse crackles, and lower extremity edema extending up to his knees. A CT
pulmonary embolism is obtained, which shows diffuse pulmonary edema. Labs are
notable for Na of 122 mEq/L and Cr 2.2 mg/dL with normal baseline of 0.9 mg/dL.
Urinalysis is without protein or blood.
What is the most likely etiology of this patient’s acute kidney injury?
A. Antineutrophilic cytoplasmic antibody (ANCA) vasculitis
B. Contrast
C. Heart failure
D. Nonsteroidal anti-inflammatory drug (NSAID) use
E. Poor oral intake

24. A 62-year-old man with metastatic melanoma is admitted with a creatinine of 3.2 mg/dL
(baseline 1.0 mg/dL) after cycle 3 of pembrolizumab. He reports being thrilled with the
response of his cancer to therapy but is concerned about his kidney failure. He reports
recent diarrhea and poor oral intake. He denied dizziness, shortness of breath, or urinary
symptoms. His medications include omeprazole and oxycodone. He had a BP of 96/50
mmHg and his HR was 90/min. His examination showed dry mucous membranes and was
otherwise normal. Labs showed Na 134 mEq/L, K 3.4 mEq/L, HCO3 19 mEq/L, and Cl 90
mEq/L. Liver function tests were normal. Urinalysis 5 to 10 WBCs/hpf and urine
sediment was bland. He is given 2 L of IV saline and a day later, his Cr was 2.9 mg/dL.
His urine output has been 1.3 L over the last 24 hours. His renal ultrasound shows normal
sized kidneys with no hydronephrosis.
What is the next best step in management?
A. Continue IV hydration
B. Insert a Foley catheter
C. Kidney biopsy
D. No further intervention at present
E. Prednisone

25. A previously healthy 26-year-old man comes to the ED after experiencing myalgias and
noticing dark-colored urine. He reports he has been pushing himself at the gym and has
been lifting heavy weights. His vital signs are within normal limits and his examination is
normal. His creatine kinase (CK) comes back at 62 000 IU/L and his creatinine, serum
potassium, and serum calcium are 1.7 mg/dL, 6.0 mEq/L, and 6.7 mg/dL, respectively.
What is the next best step in management?
A. Hemodialysis
B. IV furosemide
C. IV saline at 1 L/h
D. IV saline at 200 mL/h
E. IV sodium bicarbonate at 200 mL/h

26. A 45-year-old male with a family history of premature CAD, hypertension, and diabetes
mellitus presents with atypical chest pain. After an abnormal nuclear stress test, he
undergoes a coronary angiogram. Postangiogram, he experiences contrast-induced acute
kidney injury. His creatinine rises to 2.0 mg/dL from a baseline of 0.9 mg/dL. With
conservative management, his creatinine trends down to 1.1 mg/dL in the next 2 days.
What is the association between acute kidney injury and the risk of the following
adverse outcomes?
A. Acute kidney injury is associated with an increased risk of chronic kidney disease
and end-stage renal disease (ESRD) but not with an increased risk of mortality
B. Acute kidney injury is associated with an increased risk of chronic kidney disease but
not ESRD
C. Acute kidney injury is associated with an increased risk of chronic kidney disease,
ESRD, and mortality
D. Acute kidney injury is not associated with an increased risk of chronic kidney disease
or ESRD

27. A 55-year-old woman with a history of hypertension, diabetes mellitus, and chronic
kidney disease stage 4 is admitted with chest pain. Initial ECG did not show any ST
abnormalities but subsequent ECGs a few hours later showed ischemic ST segment
changes. A myocardial infarction is suspected and a cardiac catheterization is planned.
Medications include lisinopril, torsemide, aspirin, carvedilol, and insulin. BP is 110/66
mmHg and pulse is 60/min. There is no peripheral edema and the lungs are clear to
auscultation.
What is the best intervention to reduce the risk of contrast-associated acute kidney
injury in this patient?
A. Hemodialysis immediately after the procedure

Freemedicalbooks4download
B. IV N-acetylcysteine
C. IV normal saline
D. Oral fluids

28. A 63-year-old man with hypertension, diabetes mellitus, and end-stage kidney disease on
hemodialysis three times a week is admitted from the dialysis unit with shortness of breath
and chest discomfort. He presented to the dialysis unit with these symptoms and hence
could not be started on dialysis. He denies fever, chills, cough, or dizziness. BP is 160/70
mmHg, pulse is 110/min, and weight is 73 kg. He is 1 kg above his dry weight. Physical
examination shows clear lungs and a normal JVP. On admission, his high-sensitivity
troponin T level is 135 ng/L and his second troponin T level, a few hours later, was 405
ng/L (reference <9 ng/L). CXR shows clear lung fields. ECG shows signs of left
ventricular hypertrophy (LVH) and new, nonspecific ST segment abnormalities.
What is the most appropriate next step in management?
A. Aspirin, nitrates, and cardiology consult for cardiac catheterization
B. CT pulmonary embolism
C. Observation
D. Repeat troponin levels a few hours later
E. Urgent hemodialysis and ultrafiltration

29. A 65-year-old man is admitted with decompensated congestive heart failure after failing
outpatient diuresis. He reports a gain of 25 lb over the last 3 to 4 weeks and dyspnea at
rest. He has been on torsemide 60 mg twice a day during that period. A furosemide bolus
of 100 mg IV is administered and he is started on an infusion at 10 mg/h. He responds
well and loses 10 lb in the first 5 days but continues to require supplemental oxygen (2
L/min nasal cannula), has dyspnea on walking, and has bilateral crackles. On day 6, his
HCO3 on the chemistry panel is noticed to be 45 mEq/L and an ABG drawn shows pH of
7.51 and PaCO2 of 52 mmHg. In a team meeting, it is decided that IV diuresis needs to be
continued.
What is the most appropriate treatment?
A. Add acetazolamide
B. Add amiloride
C. Add spironolactone
D. Infuse normal saline
E. Insist that the furosemide be stopped

30. A 60-year-old woman with hypertension presents to the clinic with persistently high home
BP readings. She is currently on lisinopril 40 mg daily, amlodipine 10 mg daily, and
chlorthalidone 12.5 mg daily. She follows a strict salt-restricted diet, exercises every day,
and does not drink alcohol or smoke tobacco. Her home BP measurements are in the 150-
165/85-95 mmHg range. Her comorbidities include CAD and well-controlled diabetes
mellitus. Her lab work shows an estimated glomerular filtration rate (eGFR) of 40 mL/min
and a potassium of 4.9 mEq/L.
What is the next best step in management?
A. Add carvedilol
B. Add spironolactone
C. Change chlorthalidone to furosemide 40 mg daily
D. Increase chlorthalidone to 25 mg daily
E. Increase lisinopril to 60 mg daily

31. A 33-year-old gravida 2 para 1 pregnant woman with a history of preeclampsia in her
previous pregnancy is admitted with a BP of 170/95 mmHg. She has no symptoms.
What antihypertensive should NOT be used in pregnancy?
A. Hydralazine
B. Labetalol
C. Lisinopril
D. Methyldopa
E. Nifedipine

32. A 60-year-old woman is just admitted to the medical ICU with septic shock secondary to a
foot infection that was left untreated. She is on 6 μg/kg/min norepinephrine and 0.04
U/min vasopressin and her lactate is 8 mmol/L. She is intubated and initiated on
mechanical ventilation. An ABG reveals a pH of 7.25 and a pO2 of 140 mmHg on 30%
FiO2 and a positive end-expiratory pressure of 5 mmHg. A chest x-ray reveals mild
pulmonary edema. Her BUN and Cr are 63 mg/dL and 4.2 mg/dL, respectively. Her urine
output is 25 mL/h.
What is the next best step in management?
A. Give IV sodium bicarbonate
B. Monitor closely with IV furosemide as needed
C. Start continuous renal replacement therapy
D. Start intermittent hemodialysis

33. A 59-year-old woman with type 2 diabetes mellitus comes to the hospital with complaints
of cough, congestion, and fever for the last 2 weeks. Her BP is 85/50 mmHg and a CXR
showed right lower lobe pneumonia. Her admission creatinine was 11 mg/dL (baseline 2.6
mg/dL) and her potassium was 6.5 mEq/L. Nonspecific ST segment abnormalities are
seen on her ECG. A Foley catheter is inserted with no return of urine. She is started on IV
antibiotics and IV fluids. One hour later, she continues to have no urine output and her
potassium rises to 7 mEq/L. She is started on emergent hemodialysis. Over the next week,
she received three dialysis sessions and her infection has resolved. Her urine output has
progressively increased and she now makes around 2.1 L of urine a day. Her BUN is 65
mg/dL, creatinine is 4.1 mg/dL, and potassium is 4.3 mEq/L. Her last dialysis session was
48 hours ago. Her postdialysis BUN was 55 mg/dL and her creatinine was 3.3 mg/dL.
What is the next best step in management?
A. Dialysis today
B. Dialysis until serum creatinine is <2 mg/dL
C. IV furosemide to augment urine output
D. Observation only

34. A 45-year-old woman with end-stage renal disease (ESRD) on three times weekly
hemodialysis is admitted to the hospital with cellulitis. While she is admitted, the team
notices that she has a left brachiocephalic fistula with a good thrill and bruit but has been
receiving dialysis through a central venous catheter. The patient reports that the fistula
was placed 3 months ago. The fistula is examined, is deemed to have matured well, and is

Freemedicalbooks4download
used for hemodialysis that day with full blood flow rates (400 mL/min). Immediately on
return to the inpatient unit, the patient complains of numbness and severe pain in her left
hand. The radial pulse is nonpalpable and the fingers are cold to the touch. Her vital signs
are stable.
What is the next best step in management?
A. Elevate the arm
B. Observe
C. Perform surgical evaluation
D. Prescribe high-dose gabapentin
E. Reduce blood flow rates during next dialysis session

35. A 72-year-old Hispanic man who recently immigrated to the United States from Colombia
is seen at the outpatient primary care clinic 4 weeks after a recent hospital admission for
pneumonia. At admission, his creatinine was found to be elevated (6 mg/dL). With fluids
and supportive care, his creatinine trended down quickly to 3.1 mEq/L within 48 hours of
admission and stayed at that level at hospital discharge. A repeat renal panel shows a
creatinine of 3.2 mEq/L, which equates to an eGFR of 18 mL/min/1.73 m2. His blood
group type is A and his body mass index (BMI) is 31 kg/m2. He reveals that he has had
difficulty controlling his diabetes for 35 years and is on insulin. He was told he had
diabetic kidney disease years ago and reports hypertension, for which he is on amlodipine.
He then shows you a stack of papers from his home country, which indicate that he had
surgery for a <1 cm basal cell carcinoma of the skin (forearm) 2 years ago. He has a
supportive family and plays golf on the weekends. He wants to avoid dialysis as much as
possible and asks you about the possibility of being listed for a deceased donor kidney
transplant.
What should you tell him?
A. He does not qualify for a kidney transplant because of his age
B. He does not qualify for a kidney transplant due to his history of cancer
C. He should explore the option of a living donor because the wait time will be shorter
D. He should be evaluated and listed for a kidney transplant now
E. He will qualify for a kidney transplant once he is on dialysis

36. A 50-year-old woman with alcoholic cirrhosis is admitted with worsening abdominal pain
due to progressive ascites and altered mental status. She just achieved 6-month sobriety
and was listed for a liver transplant 12 days ago. Examination shows a tense and distended
abdomen and her BP is 95/56 mmHg. Her mental status improves with lactulose. Her
serum creatinine on admission is 4.2 mg/dL from a baseline of 1.5 mg/dL and her serum
albumin is 2.6 m/dL. Her MELD (Model for End-Stage Disease) score is 28. Her urine
sediment is bland, and her fractional excretion of sodium is <1%. Despite resuscitation
with IV fluids and albumin and treatment with midodrine and octreotide, her creatinine
worsens over the next 72 hours to 6.2 mg/dL and her mental status worsens again. A
discussion regarding the initiation of dialysis is held with her and her family.
What is her most favorable prognostic factor if she were to be initiated on dialysis?
A. Age
B. Etiology of acute kidney injury
C. Listed for liver transplant
D. MELD score
37. Which of the following statements is false?
A. Home hemodialysis is associated with improved survival compared with
conventional in-center hemodialysis
B. Peritoneal dialysis is associated with better preservation of cognitive functions
compared with hemodialysis
C. Quality of life in transplant recipients is the same as that in patients on dialysis
D. Survival on peritoneal dialysis and survival on hemodialysis are comparable

38. A 35-year-old African American woman diagnosed with systemic lupus erythematosus at
age 30 is seen by her primary care physician during a routine visit. She has no complaints,
except for a malar rash. She has no other signs of systemic lupus and her BP is 145/92
mmHg. She is on oral contraceptive pills. A spot urine sample from a week ago showed
800 mg/g of proteinuria. There is no hematuria on the urinalysis and her renal panel is
normal with a creatinine of 0.7 mg/dL. Her complement levels are mildly low.
What is the next best step in management?
A. Initiate immunosuppression with steroids and mycophenolate mofetil
B. Obtain a 24-hour protein/creatinine ratio
C. Refer her for kidney biopsy
D. Repeat urine/protein creatinine ratio in a month
E. Start her on lisinopril

39. A 27-year-old Asian man presents to the outpatient clinic after experiencing recurrent
episodes of gross hematuria. He reports one such recent episode this past winter after he
had “flu-like” symptoms. He is not on any medications. His BP is 135/90 mmHg. A
urinalysis shows hematuria and he is found to have a urine microalbumin/creatinine ratio
of 0.6 g/g. His serum creatinine is 0.9 mg/dL. Renal ultrasound is normal. A renal biopsy
reveals findings consistent with immunoglobulin A (IgA) nephropathy.
With which of the following should he be treated?
A. Cyclophosphamide
B. Fish oil
C. Lisinopril
D. Prednisone

40. A 44-year-old Caucasian male with relapsed chronic myeloid leukemia is in clinical
remission after an allogenic hematopoietic stem cell transplantation 15 months ago. His
cyclosporine was tapered and discontinued 3 months before admission. He now presents
with a rash on his face and forearms for 2 weeks along with lower extremity swelling. He
is found to have nephrotic syndrome with 4.2 g of proteinuria and a serum albumin of 2.3
mg/dL. His creatinine was elevated at 1.4 mg/dL from 0.9 mg/dL a month ago. Hepatitis
and human immunodeficiency virus (HIV) serologies were negative. Serum and urine
electrophoresis, C3, C4, antinuclear antibody (ANA), and antineutrophilic cytoplasmic
antibody (ANCA) levels were normal. His hemoglobin is 13 g/dL, WBC count is 8000/
μL, and platelet count is 176 500/μL.
What is the most likely diagnosis?
A. ANCA vasculitis
B. Focal segmental glomerulosclerosis
C. Membranous nephropathy

Freemedicalbooks4download
D. Minimal change disease
E. Thrombotic microangiopathy

41. A 53-year-old woman presented with hematuria, nephrotic range proteinuria (3.7 g/d), and
elevated creatinine (1.9 g/dL). A kidney biopsy revealed monoclonal deposition disease
with proliferative glomerulonephritis.
In patients with monoclonal gammopathy of renal significance, what is the best
way to detect the monoclonal protein in the circulation?
A. Serum (SPEP) and urine protein electrophoresis
B. SPEP, serum immunofixation, and serum free light chain assay
C. Urine protein electrophoresis, urine immunofixation, and urine free light chain assay
D. Urine free light chain assay

42. A 38-year-old woman is admitted with abdominal pain and fevers for the past week. Her
chemistry panel shows a serum creatinine of 4.1 mg/dL (1.0 mg/dL 1 month ago) and her
urine sediment shows dysmorphic red cells. Serologies are sent and are pending. A kidney
biopsy is done and crescentic glomerulonephritis is seen on light microscopy.
What is the least likely diagnosis in this patient?
A. Antineutrophilic cytoplasmic antibody (ANCA) vasculitis
B. Anti–glomerular basement membrane disease
C. Hemolytic uremic syndrome
D. IgA nephropathy
E. Lupus nephritis

43. A 45-year-old woman with hepatitis C presents to the ED with malaise, joint pain, and
change in the color of her urine. She has been lost to GI follow-up and has not been
treated for hepatitis C. Over the past week, she has developed worsening joint pain and
swelling around her ankles. Vitals are notable for a BP of 164/98 mmHg. Examination is
notable for moist mucous membranes, bibasilar crackles, no ascites, 1+ ankle edema, and
purpura along her extremities. Labs are notable for a Cr 2.4 (baseline 1.2), decreased C3
and C4. A Foley catheter is placed, yielding 300 mL of tea-colored urine. A urinalysis
shows 2+ blood and 1+ protein. A representative image from her urine sediment is
displayed.
Image courtesy of Harish Seethapathy, MBBS.

What is the most likely diagnosis?


A. Cryoglobulinemic glomerulonephritis
B. Henoch-Schönlein purpura
C. Hepatorenal syndrome
D. Postrenal acute kidney injury
E. Prerenal acute kidney injury

44. A 57-year-old man presents to an urgent care visit because of a change in the color of his
urine for a week. He describes his urine as looking “tea colored.” He denies any recent
trauma, kidney stones, or Foley catheter insertion. The patient has a history of diabetes
and chronic back pain using ibuprofen as needed. Vitals are normal and physical
examination is unremarkable. A serum chemistry panel shows normal creatinine.
What is the most appropriate next step in management?
A. CT urography
B. Observation
C. Renal ultrasound
D. Urinalysis + sediment
E. Urology referral

45. A normal urinalysis in acute kidney injury helps exclude which of the following
diagnoses?
A. Amyloidosis
B. Cardiorenal syndrome
C. Multiple myeloma cast nephropathy
D. Thrombotic microangiopathy
E. Urinary tract obstruction

46. Which one of the following statements is not true?


A. Calcium oxalate stones are the most common type of kidney stones.
B. In a patient with calcium oxalate stones who drinks less than 1.5 L/d, dietary
modification is unlikely to prevent another kidney stone unless the fluid intake is
increased simultaneously.
C. Most kidney stones of <0.5 cm in diameter will pass spontaneously.
D. Most patients who pass a kidney stone will never experience another one in their
lifetime.
E. Potassium citrate can prevent new uric acid stone formation but may increase the risk
of calcium phosphate stone formation.

47. A 47-year-old man with Crohn’s disease is admitted to the hospital with severe diarrhea
for the last 3 days. He reports that he has a history of chronic diarrhea and has intermittent
bouts of severe diarrhea. He also has a history of diet-controlled diabetes mellitus, with
his last HbA1c being 6.6%, and a history of hypertension, for which he takes 5 mg of
amlodipine. He reports passing a kidney stone 3 years ago. His creatinine on admission
was 4.5 mg/dL and decreased to 3.8 mg/dL after fluid resuscitation. You notice that his
creatinine was 0.5 mg/dL 5 years ago and 1.3 mg/dL 8 months ago when he was seen by

Freemedicalbooks4download
his primary care physician. His urine protein creatinine ratio is 0.2 g/dL. His renal
ultrasound is normal.
What is the most likely etiology of his chronic kidney disease?
A. Diabetic nephropathy
B. Hypertensive nephropathy
C. Oxalate nephropathy
D. Recurrent acute kidney injury

48. A 45-year-old woman is admitted with flank pain and hematuria. On imaging, she was
found to have three kidney stones, one of which was obstructing the ureter. One day after
admission, she passed the stone and her pain and hematuria resolved. Her serum creatinine
has been between 1.3 and 1.6 mg/dL over the last few months. Her serum calcium is 9
mg/dL (normal 8.5-10 mg/dL) and her vitamin D level is 53 ng/mL (normal 30-60
ng/mL). She reports childhood asthma but no other medical conditions. She works a desk
job and keeps herself well hydrated. She extensively researches health issues online and
takes medications that she feels improve her health, including cholecalciferol 500 units
daily; multivitamins, including vitamin B complex and vitamin C (2 g daily); aspirin 81
mg daily; and fish oil 1000 mg daily.
Which of the following is most likely contributing to kidney disease and formation
of kidney stones in this patient?
A. Fish oil
B. Vitamin B
C. Vitamin C
D. Vitamin D

ANSWERS

1. The correct answer is: B. Diabetic ketoacidosis. This patient most likely has diabetic
ketoacidosis with an anion gap metabolic acidosis, along with hyperglycemia, glucosuria,
and ketonuria. Initially in the acute phase of diabetic ketoacidosis, while ketoacids build
up, an anion gap metabolic acidosis becomes apparent; however, in patients with normal
renal function, diabetic ketoacidosis can also cause normal gap metabolic acidosis later on
when the ketoacids (which can potentially be converted to bicarbonate in the liver) are
excreted in the urine, thereby causing loss of bicarbonate. Renal tubular acidosis due to
topiramate usually causes a mild proximal renal tubular acidosis along with hypokalemia.
Ethanol intoxication can lead to ketoacidosis due to starvation but hypoglycemia is
usually present along with it. Mild lactate elevation, as seen in this patient, does not cause
this degree of acidosis. Moreover, the overall picture fits diabetic ketoacidosis much
better.

2. The correct answer is: D. Thiamine deficiency. This patient presents with a pH of 7.20
with a high anion gap metabolic acidosis (HAGMA), most likely due to type B lactic
acidosis in the setting of thiamine deficiency. This patient is acidemic given a pH <7.40
with an anion gap of 22 (125 − 90 − 15) + 2 for the albumin of 3 g/dL. The change in
anion gap over change in bicarbonate (ie, delta/delta) is 1:1, so there is no metabolic
alkalosis (vomiting unlikely etiology). As this patient’s lactic acidosis did not improve
with 2 L of hypotonic IV fluid resuscitation and he is afebrile with a normal BP, type A
lactic acidosis due to ischemia is unlikely (eg, hypovolemia, septic shock, hypoxemia). As
such, type B (nonhypoxic) lactic acidosis is more likely. Etiologies of type B lactic
acidosis include thiamine deficiency, seizures, metformin, propofol, niacin, and glycol
intoxications. Thiamine is an essential cofactor for pyruvate dehydrogenase, which
converts pyruvate to acetyl-CoA, which enters the Kreb’s cycle (aerobic metabolism). In
thiamine deficiency, pyruvate is instead converted to lactate. Patients with alcohol use
disorder are at risk for thiamine deficiency, which can be treated with IV thiamine.

3. The correct answer is: D. Methanol. This patient presents with altered mental status and
blurry vision with an elevated anion and osmolar gap, which is mostly likely due to
methanol (also known as wood alcohol) ingestion. He has vision changes with an anion
gap of 20.5 (18 uncorrected for albumin) and an osmolar gap of 23. An elevated anion gap
and osmolar gap is seen with methanol, ethylene glycol, propylene glycol, and diethylene
glycol ingestion; however, blurry vision, pupillary dilation, and papilledema are seen
specifically with methanol ingestion at doses as low as 10 mL. In terms of the other
choices, isopropyl alcohol usually presents with normal anion gap and increased osmolar
gap and typically causes pancreatitis. Aspirin toxicity is associated with fever, tinnitus,
respiratory alkalosis (early), and metabolic acidosis (late) and does not increase the
osmolar gap. Ethylene glycol causes heart failure with hypocalcemia, acute kidney injury,
and calcium oxalate crystal formation. This patient had a normal BP, normal calcium (8.2
g/dL when corrected for albumin), and normal Cr. Propylene glycol can cause acute
kidney injury and lactic acidosis, which are not noted here.

4. The correct answer is: B. Diazepam. This patient most likely has propylene glycol
toxicity due to high doses of diazepam administration. He has an increased anion gap
metabolic acidosis and an increased osmolar gap. Propylene glycol is the solvent used to
administer IV diazepam, lorazepam, as well as phenobarbital. It can be toxic in high
doses, leading to increased lactate levels and acute kidney injury. Treatment involves
discontinuing the offending agent and dialysis in severe cases. Ethylene glycol can cause a
similar picture with anion gap acidosis and an increased osmolar gap. It is usually found
in antifreeze solutions and causes profound acidosis and acute tubular necrosis with
calcium oxalate crystal formation in the urine. These manifestations are usually found at
admission and not during hospitalization. Isopropyl alcohol can lead to an increased
osmolar gap but not an anion gap acidosis.

5. The correct answer is: A. Diarrhea. This patient presents with a pH 7.18 with a normal
anion gap of 10, consistent with a non–gap metabolic acidosis in the setting of severe
diarrhea. GI fluid above the ligament of Treitz (eg, stomach) is rich in acid, whereas GI
fluid below the ligament of Treitz (small and large bowel) is rich in bicarbonate. As such,
large volume emesis causes a chloride depletion alkalosis due to loss of volume, chloride,
and potassium, whereas diarrhea often causes a nongap acidosis due to loss of
bicarbonate-rich fluid. While severe hypovolemia can cause lactic acidosis and severe
anorexia can cause starvation ketosis, this patient has a normal anion gap, so these choices
are incorrect.

Freemedicalbooks4download
6. The correct answer is: B. Kidney stones. This patient’s labs are suggestive of a distal
(type I) renal tubular acidosis, which is likely acquired from her rheumatoid arthritis. She
has a non–gap metabolic acidosis with appropriate respiratory compensation. The positive
urine anion gap (Na + K − Cl) suggests ineffective secretion of NH4+ cations because one
would expect a negative urine anion gap as NH4+ is secreted with Cl in a state of acidosis.
Distal renal tubular acidoses are due to ineffective secretion of NH4+, resulting in an
inappropriately higher urine pH of typically >5.3 that distinguishes them from proximal
renal tubular acidoses. Distal renal tubular acidoses are more prone to kidney stones
because high urine pH favors precipitation of Ca phosphate and proximal reabsorption of
citrate, which typically prevents Ca stone formation. In terms of the other options,
osteomalacia is more commonly seen in proximal renal tubular acidoses due to concurrent
proximal phosphate wasting. Venous thromboembolism is not associated with any renal
tubular acidoses. Hyperkalemia is associated with type IV renal tubular acidoses in the
setting of hypoaldosteronism. Proximal and distal renal tubular acidoses are both prone to
hypokalemia due to distal K+ secretion in lieu of NH4+. Patients with renal tubular
acidoses are not at increased risk for infection.

7. The correct answer is: A. Neobladder dysfunction. This patient has a normal anion gap
metabolic acidosis with appropriate respiratory compensation. He most likely has a
neobladder dysfunction. An ileal conduit has an abundance of Na+-H+ exchangers. Urea
traveling through the neobladder is metabolized to NH4+ and HCO3−. The NH4+ competes
with Na+ at the Na+-H+ exchanger and is reabsorbed in exchange for H+. Cl− in the urine is
also absorbed in exchange for HCO3− at the Cl−/HCO3− exchanger. The absorbed NH4+ is
metabolized in the liver to urea and H+, resulting in the gain of a proton. Under well-
functioning conditions, this is not an issue; however, if the transit time increases, as in the
case of a neobladder dysfunction, the urine stays in contact with the intestinal luminal wall
for a longer period and a normal anion gap acidosis develops. These patients require
chronic bicarbonate replacement and sometimes revision of the conduit. Other causes,
such as sepsis secondary to a urinary tract infection (UTI) or type B lactic acidosis that
occurs in short gut syndrome, produce an anion gap acidosis. This patient does not have
any major reasons to suspect renal tubular acidosis. Most renal tubular acidoses are mild
and do not produce sudden severe acidosis.

8. The correct answer is: B. High anion gap metabolic acidosis, metabolic alkalosis with
compensatory respiratory alkalosis. This patient presents with a normal pH, but with
multiple acid-base disturbances. His anion gap is 24 when corrected for low albumin with
only a 4 mEq/L decrease in HCO3 as opposed to a decrease in HCO3 of 12 mEq/L that one
would expect; hence, he has an additional metabolic alkalosis (delta-delta 3). His pCO2 is
slightly low, suggestive of a respiratory alkalosis. Using Winter’s formula (1.5 × HCO3− +
8 ± 2), the expected pCO2 is 36 to 40 mmHg, so his low pCO2 is the expected respiratory
compensation for his metabolic acidosis. He likely has an elevated anion gap from lactic
acidosis and ketosis in the setting of poor oral intake and metabolic alkalosis from
vomiting. Treatment should involve resuscitation with normal saline and aggressive
potassium repletion as the alkalemia will persist in a state of hypochloremia and
hypokalemia. In terms of the other choices, while the pH is normal, there is an
abnormality as this is a triple acid-base disturbance that is masked by concurrent
metabolic acidosis and alkalosis. The patient has a High anion gap metabolic acidosis, but
one would expect a pH ~7.25 with HCO3 12 mEq/L and pCO2 28 mmHg if he has a pure
High anion gap metabolic acidosis. This patient has an anion gap of 24 when corrected for
albumin, so a non–gap metabolic acidosis is incorrect. Anion gap can be calculated as Na
− Cl − HCO3, which is 126 − 87 − 20 = 19 in this patient; however, one needs to correct
for albumin, by adding 2.5 to the anion gap for each 1 g/dL decrease in albumin below 4
g/dL. Hence, the corrected anion gap is 19 + 2.5 × 2 = 24.

9. The correct answer is: D. IV normal saline. This patient presents with emesis, resulting
in loss of Cl-rich fluids and therefore a hyperchloremic, hypokalemic metabolic alkalosis.
The treatment of choice for saline-responsive metabolic alkalosis is IV saline. Normally,
the kidneys can easily handle bicarbonate excess; however, after loss of HCl, volume, and
K (the generation phase of the metabolic alkalosis), the kidney will preferentially
prioritize volume retention at the expense of persistent alkalemia (the maintenance phase).
The macula densa senses low Cl and stimulates renin-angiotensin-aldosterone system and
aldosterone secretion, which leads to not only Na retention but also H+ and K secretion in
principal cells. In addition, low potassium stimulates H+ secretion in α-intercalated cells.
As such, correction of metabolic alkalosis requires volume resuscitation with chloride-rich
isotonic fluid (eg, normal saline) with aggressive potassium repletion (K > 4.0 mEq/L). As
the alkalemia is corrected, potassium will shift extracellularly as well.

10. The correct answer is: C. Anion gap metabolic acidosis + non–gap metabolic acidosis
+ respiratory alkalosis.

Step 1: Evaluate the pH: This patient clearly has an acidosis with a pH of 7.15.
Step 2: Identify whether it is respiratory or metabolic: pCO2 here is 40 mmHg and the HCO3
is 12 mmol/L; hence, it is clearly metabolic.
Step 3: Look at the anion gap: The anion gap is elevated in this case; hence, the patient has
an anion gap metabolic acidosis.
Step 4: Check whether the respiratory response is adequate: The expected pCO2 by Winter’s
formula should be (1.5 × (serum HCO3) + 8 ± 2); hence, the expected pCO2 is 26 ± 2
mmHg. But the patient’s pCO2 is 20 mmHg; hence, there is a coexistent respiratory
alkalosis.
Step 5: Calculate ∆anion gap/∆HCO3 (delta/delta): Does the rise in anion gap match the fall
in HCO3? In this case the delta/delta is 4/12, which is <1; hence, there is a concomitant
non–gap metabolic acidosis.

11. The correct answer is: A. Cerebral salt wasting. Cerebral salt wasting and syndrome of
inappropriate antidiuretic hormone secretion (SIADH) share many common features. Both
are hypo-osmolar hyponatremias, which can occur after an intracranial pathology, and
both have hyper-osmolar urine, hypouricemia (due to high excretion), and a high urine
sodium. The differentiating feature is volume status—SIADH patients are euvolemic or
mildly volume overloaded while cerebral salt wasting leads to hypovolemia due to
excessive urinary sodium losses. Cerebral salt wasting, while much less common than
SIADH, usually occurs after a subarachnoid hemorrhage. The liberal use of IV fluids to

Freemedicalbooks4download
prevent vasospasm in these patients usually helps maintain the serum sodium in the 125 to
130 mEq/L range. Patients with cerebral salt wasting respond to IV saline by diluting their
urine and lowering their urine osmolality since the hypovolemic stimulus for antidiuretic
hormone (ADH) is removed, while the urine osmolality for patients with SIADH typically
does not change and will likely worsen the serum sodium since more water is retained.
The response to IV saline may become muddled in patients with both cerebral salt wasting
and SIADH features.

12. The correct answer is: C. IV normal saline. This patient presents with euvolemic
hyponatremia with a history concerning for low-solute diet (alcohol intake without food)
due to beer potomania. Administration of IV normal saline would provide solute, enabling
free water excretion. Free water clearance (cH2O) = solute excretion/Uosm. A normal US
dietary solute load is 750 mOsm/d and the kidney’s maximum diluting capacity is 50
mOsm/L, so under normal circumstances, we can drink about 15 L of free water prior to
holding on to free water and developing hyponatremia. In beer potomania, daily solute
intake ranges from 200 to 300 mOsm, so patients are only able to drink 4 to 6 L of free
water (or beer) prior to developing hyponatremia. A six-pack is 72 ounces or ~2100 mL,
so drinking 2 six-packs would overwhelm this patient’s diluting capacity, after which he
would hold on to free water. As he is maximally diluting his urine, his urine osmolality is
<100 mOsm/kg. Administration of solute should increase this patient’s serum sodium, so
C is the correct answer. In terms of other options, A and B do not increase solute intake,
and thus are not good choices for this patient. Antidiuretic hormone (ADH) activity would
be low as serum Osm <300 mOsm/kg. Tolvaptan is a vasopressin receptor (and therefore
ADH) antagonist used as second line in syndrome of inappropriate antidiuretic hormone
secretion (SIADH) and hypervolemic hyponatremia.

13. The correct answer is: D. Inappropriate antidiuretic hormone (ADH) release. This
patient’s hyponatremia worsened from 120 to 116 mEq/L, resulting in a seizure after
normal saline resuscitation suggestive of syndrome of inappropriate antidiuretic hormone
secretion (SIADH). Inappropriate ADH release is seen in many disorders, including
stroke, malignancy, carbamazepine, selective serotonin reuptake inhibitors (SSRIs), and
pneumonia. This patient has a history of lung cancer, which is associated with
inappropriate ADH release. Typically, urine Na is >20 mEq/L, suggestive of euvolemia,
and urine osmolality is >100 mOsm/kg, suggestive of ADH activity and free water
reabsorption. With normal saline administration, sodium levels typically decrease. In this
patient, aggressive IV normal saline administration decreased her Na to a dangerously low
level, resulting in seizure. At this point, hypertonic saline should be administered in
consultation with nephrology.

14. The correct answer is: C. Give 3% hypertonic saline as a 100 mL bolus in 10
minutes, then recheck labs. This patient presents with severe hyponatremia (defined as
serum Na <120 mEq/L) of unknown chronicity and is at risk of life-threatening symptoms
(seizures, respiratory failure, coma) due to cerebral edema. Both European and American
expert panel guidelines recommend aggressive therapy with 3% saline in the setting of
severe symptoms, defined as vomiting, deep somnolence, seizures, or coma. Hypertonic
saline should be administered in consultation with nephrology but is typically given as a
150 mL bolus over 20 minutes (EU guidelines) or 100 mL over 10 minutes (US
guidelines). An increase of serum Na by ~3 mEq/L is sufficient to reverse severe
symptoms. In terms of other options, while this patient’s hyponatremia may be due to
underlying hypothyroidism, the risk of life-threatening symptoms warrants initial rapid
correction. Isotonic saline is the treatment of choice for hypovolemic hyponatremia;
however, the severity of symptoms warrants hypertonic saline. In addition, this patient
appears euvolemic with UNa >20 mEq/L, so syndrome of inappropriate antidiuretic
hormone secretion (SIADH) is the more likely diagnosis; 2 L of isotonic saline without
rechecking serum Na could further worsen serum Na. Free water restriction would be
warranted in SIADH and is the next step, but this patient warrants more rapid correction
given the severity of symptoms.

15. The correct answer is: D. Nephrogenic diabetes insipidus. This patient presents with
hypernatremia of unknown chronicity and is found to have symptomatic hypernatremia.
The patient’s serum osmolality is very high (335 mOsm/kg) and her urine osmolality is
<300 mOsm/kg, suggesting insufficient antidiuretic hormone (ADH). After resuscitation
with normal saline, she remains polyuric with ~675 mOsm/d, suggesting water diuresis as
opposed to osmotic diuresis, so diabetes insipidus is most likely. Administration of
DDAVP will differentiate central from nephrogenic diabetes insipidus. As she has less
than a 50% increase in urine osmolality with DDAVP, she likely has nephrogenic diabetes
insipidus.

A. Incorrect as DDAVP administration increased urine osmolality by <50%, so production of


DDAVP is not the main etiology behind her diabetes insipidus.
B. Incorrect as GI free water loss would be expected from significant watery diarrhea.
C. Incorrect—while lack of free water would cause hypernatremia, this patient has adequate
access to free water and remains polyuric due to inability to concentrate her urine.
D. Incorrect as this patient is below the threshold for glycosuria and polyuria with ~675
mOsm/d consistent with a water diuresis, whereas one would expect a urine osmolality
>1000 mOsm/d with very high serum glucose levels in osmotic diuresis.

16. The correct answer is: B. Gestational diabetes insipidus. This patient most likely has
gestational diabetes insipidus caused by the production of placental vasopressinase, which
degrades normally produced vasopressin. It usually presents in the third trimester and
resolves over weeks in the postpartum period. Diagnosis and treatment are with
desmopressin, which is resistant to endogenous vasopressinase and is effective in raising
the urine osmolality. Central diabetes insipidus is another possibility in this patient, who
responds to desmopressin; however, the more obvious cause in this patient is gestational
diabetes insipidus. Lithium can cause nephrogenic diabetes insipidus, sometimes after
treatment cessation but nephrogenic diabetes insipidus is unlikely since the polyuria
responded to desmopressin. Primary polydipsia resolves with fluid restriction, which was
not the case here. Preeclampsia is a risk factor for gestational diabetes insipidus since the
liver, which is the site of vasopressinase degradation, may be affected, but preeclampsia
by itself does not lead to diabetes insipidus.

17. The correct answer is: B. Licorice ingestion. This patient presents with hypokalemia
with urine K/Cr ratio >13 and high BP in the setting of large ingestion on black licorice.
Glycyrrhizic acid, an ingredient in licorice, inhibits 11-β-hydroxysteroid dehydrogenase,

Freemedicalbooks4download
the enzyme responsive for inactivating cortisol in the adrenal glands. Black licorice,
ingested in large quantities, can result in hypercortisolism. In states of cortisol excess,
cortisol activates mineralocorticoid receptors in the principal cell, causing sodium
retention and potassium excretion and resulting in severe hypokalemia. Both salt retention
and hypercortisolism contribute to elevated BP. In terms of other options, while this
patient has diarrhea, which can cause extrarenal losses of potassium, the UK/Cr is >13. If
this patient’s hypokalemia were due to diarrhea, the UK/Cr would be <13. Thiazides cause
renal losses of potassium; however, this patient does not take a thiazide and his BP is
high. Hypokalemia is frequently associated with proximal (type II) and distal (type I)
renal tubular acidoses; however, this patient’s urine anion gap is −10. This patient does
not have a history of vomiting and one would expect metabolic alkalosis with
hypochloremia and elevated bicarbonate, whereas this patient has normal chloride and low
bicarbonate.

18. The correct answer is: D. Hypothermia. Therapeutic hypothermia can cause
intracellular shift of potassium by increased activity of the Na+-K+-ATPase and can also
cause some urinary loss of potassium due to cold-induced diuresis; hence, this patient’s
hypokalemia is consistent with the initiation of therapeutic hypothermia. Serum
electrolytes should be closely monitored during rewarming since rebound hyperkalemia
can occur. The other causes are not consistent with this presentation. Mechanical
ventilation and cardiac revascularization do not cause hypokalemia. The patient’s stool
output is not high enough for diarrhea to be a cause of hypokalemia and
hyperaldosteronism would have been apparent on presentation and is unlikely to present
acutely.

19. The correct answer is: C. Pseudohyperkalemia. This patient, who presents with
persistent severe hyperkalemia on a serum chemistry panel without ECG changes, most
likely has pseudohyperkalemia in the setting of leukocytosis. A clue to
pseudohyperkalemia is the unchanged serum potassium despite hyperkalemia treatment
and a normal ECG. Various leukemias (eg, acute myeloid leukemia and chronic
lymphocytic leukemia) have been associated with pseudohyperkalemia. In leukemias,
fragile cell membranes are prone to cell lysis during transportation and processing,
resulting in falsely high potassium levels. Chronic lymphocytic leukemia with WBCs in
the 400s has been shown to falsely increase the measured serum potassium by
approximately 4 mEq/L. A strategy to determine the true plasma potassium is to check the
potassium level on a venous blood gas where handling and processing are minimized and
cells are less prone to lysis. In terms of other options, while this patient is hyperglycemic,
it is mild and unlikely to cause significant hyperkalemia. Tumor lysis syndrome is most
common with solid tumors during induction chemotherapy. In addition, true serum
potassium levels in the range of 8.2 to 8.5 mEq/L should cause ECG changes.
Hyporeninemic hypoaldosteronism can cause hyperkalemia and can be caused by diabetes
mellitus, NSAID use, HIV, and multiple myeloma. His lack of response to hyperkalemia
treatment and ECG changes make this unlikely and this patient has an elevated
bicarbonate and so is unlikely to have hypoaldosteronism. This patient has a normal
creatinine and is unlikely to have hyperkalemia due to renal failure.

20. The correct answer is: D. Hyperglycemia. This patient has developed mild
hyperkalemia from transcellular shifts in the setting of hyperglycemia. With his
underlying diabetes, he likely has type IV renal tubular acidosis, in which he is prone to
hyperkalemia with an inability to secrete K+ due to aldosterone resistance. With treatment
for his COPD exacerbation, he has become significantly hyperglycemic. Hyperglycemia is
associated with extracellular increased osmolality, which promotes passive movement of
potassium from the intracellular to the extracellular fluid. As such, his glucose of 450
mg/dL and high serum osmolality have resulted in mild hyperkalemia. Correction of his
hyperglycemia with insulin should correct his hyperkalemia.

21. The correct answer is: D. Recent bowel surgery. SPS is a GI cation exchanger that is
used to remove potassium in hyperkalemia. In a retrospective study involving 501 patients
who received SPS for hyperkalemia, serum potassium decreased by a mean of 0.93 mEq/L
within 24 hours (Hagan AE, Farrington CA, Wall GC, et al. Clin Nephrol. 2016;85:38);
however, there were two cases (0.04%) of bowel necrosis related to administration. As
such, experts recommend avoiding SPS in postoperative patients who have undergone
bowel surgery.

22. The correct answer is: A. Consult nephrology for renal replacement therapy. This
patient is anuric despite IV diuresis, so the next best step is to initiate renal replacement
therapy for hyperkalemia with ECG changes and hypoxemic respiratory failure due to
pulmonary edema. He most likely is anuric from acute tubular necrosis, as he suffered a
cardiac arrest with prolonged renal ischemia. Further shifting of potassium with albuterol
will not help with removal. Normal saline would not be beneficial in a patient who is
hypoxemic from pulmonary edema and unlikely to have prerenal acute kidney injury.
Potassium binding resins would help with GI removal of potassium, but not acutely.

23. The correct answer is: C. Heart failure. This patient presents with a congestive heart
failure exacerbation with acute kidney injury, most likely due to hypervolemia causing a
cardiorenal syndrome. Cardiorenal syndrome is a type of prerenal acute kidney injury that
results from venous congestion and decreased renal perfusion. As there is no intrinsic
injury, a urinalysis should have no blood or protein present, as would be seen with
contrast or vasculitis. Treatment includes decongestion with IV diuretics.

24. The correct answer is: C. Kidney biopsy. This patient most likely has acute kidney
injury with symptoms (diarrhea, poor oral intake) and examination (hypotension)
suggestive of prerenal kidney injury. His kidneys were normal at baseline; however, his
creatinine has not significantly improved with IV hydration. This is highly suggestive of
acute interstitial nephritis as a cause of his acute kidney injury. Checkpoint inhibitors
(pembrolizumab, nivolumab, ipilimumab) are a class of drugs that ramp up the native
immune response against tumor antigens. This can have adverse consequences on normal
tissue such as the kidneys, thyroid, adrenals, and intestines. Checkpoint nephritis can
present variably and WBCs/WBC casts in the urine may or may not be present. A kidney
biopsy should be considered in patients at low risk for bleeding and in whom the diagnosis
is not obvious. Empiric prednisone can be considered in patients at high risk of bleeding
from a kidney biopsy, but steroids have adverse effects and a false diagnosis of acute
interstitial nephritis can have important implications for therapy going forward. A Foley
catheter is not indicated in patients with no signs of obstruction and reasonable urine

Freemedicalbooks4download
output. In addition, medications, such as proton pump inhibitors, have been associated
with checkpoint nephritis and should be stopped or switched to H2 blockers.

25. The correct answer is: D. IV saline at 200 mL/h. Individuals with rhabdomyolysis are
volume depleted due to sequestration of fluid within the muscles. Aggressive fluid
resuscitation is key in helping prevent the occurrence of or worsening of acute kidney
injury; hence, isotonic saline administration at 1 to 2 L/h is recommended in severe cases
of rhabdomyolysis. In patients with acute kidney injury who become volume overloaded,
furosemide may be used to augment urine flow and decrease overload, but it should not be
used in isolation as initial therapy. Isotonic sodium bicarbonate may be used in severe
rhabdomyolysis after volume repletion with isotonic saline but there is scant evidence that
alkaline diuresis improves renal outcomes and it may be harmful in patients with
hypocalcemia, such as in this patient. Frequent monitoring of arterial pH and ionized
calcium is recommended. Dialysis may be necessary if patients do not respond to initial
fluid resuscitation and develop severe acute kidney injury but should not be instituted as
initial therapy.

26. The correct answer is: C. Acute kidney injury is associated with an increased risk of
chronic kidney disease, end-stage renal disease (ESRD), and mortality. A meta-
analysis comprising 2 million patients found individuals with acute kidney injury at an
increased risk of new or progressive chronic kidney disease (hazard ratio 2.67, 95%
confidence interval [CI] 1.99-3.58), ESRD (hazard ratio 4.81, 95% CI 3.04-7.62), and
death (hazard ratio 1.80, 95% CI 1.61-2.02) (EJ, Jayasinghe K, Glassford N, et al. Kidney
Int. 2019;95:160). The risk of chronic kidney disease, ESRD, and death increased with the
severity of acute kidney injury, with patients sustaining acute kidney injury stage 3 having
the highest risk for all outcomes. The risk of death in patients who developed acute kidney
injury compared with those who did not was notably higher for patients undergoing limb
or coronary angiography or transcatheter aortic valve replacement (hazard ratio, 3.07;
95% CI, 2.12-4.46) than in patients undergoing cardiovascular surgery (hazard ratio, 1.75;
95% CI, 1.55-1.98), who were in intensive care (hazard ratio, 1.47; 95% CI, 1.32-1.65), or
who were in a general hospital setting (hazard ratio, 1.41; 95% CI, 1.26-1.56).

27. The correct answer is: C. IV normal saline. Contrast-associated acute kidney injury is
typically mild and occurs 48 to 72 hours after the procedure. In most patients, the kidney
function recovers to baseline and dialysis is almost never required. While there is some
controversy regarding the cause and effect relationship between contrast agents and acute
kidney injury, there are sufficient data to suggest that severe kidney injury leading to
serious adverse events, such as requiring dialysis, is extremely rare. Preexisting kidney
disease is the strongest risk factor for the development of acute kidney injury. Isotonic
saline has been shown in multiple studies to reduce the incidence of acute kidney injury
while N-acetylcysteine has not shown to be beneficial. Hemodialysis does not help
prevent contrast-associated acute kidney injury. In low-risk patients who are not expected
to have adverse effects from minimal volume expansion, IV saline is recommended to
reduce the risk of contrast-associated acute kidney injury.

28. The correct answer is: A. Aspirin, nitrates, and cardiology consult for cardiac
catheterization. Symptoms of acute coronary syndrome in patients with end-stage renal
disease (ESRD) are frequently misjudged to be secondary to inadequate dialysis or
volume removal. The presentation is often atypical in these patients and a high degree of
suspicion is required. Typical ST changes on ECG are often absent. The pathophysiology
behind increased troponin levels in ESRD is not well understood. It may be secondary to
chronic structural heart disease with troponin release rather than reduced renal clearance.
This patient has gained just 1 kg after his last dialysis session and does not have major
signs of volume overload. His troponin levels have tripled since admission; hence, dialysis
should be deferred until an evaluation for acute coronary syndrome has been performed.
Acute coronary syndrome is more likely than a pulmonary embolism in this setting and a
CT scan is not necessary at this stage.

29. The correct answer is: A. Add acetazolamide. This patient has diuretic-induced sodium
and chloride loss. The increased sodium delivery to the distal tubule leads to increased
absorption of the sodium through the ENaC channel and increased H+ secretion into the
urine from the intercalated cells in compensation. The chloride depletion also leads to
bicarbonate retention in the distal tubule, thereby causing metabolic alkalosis.
Acetazolamide inhibits carbonic anhydrase in the proximal tubule and leads to loss of
bicarbonate; it also promotes further diuresis, although it is a weak diuretic by itself. This
leads to lowering of the serum bicarbonate concentration and betterment of the diuretic-
induced metabolic alkalosis. Typical doses include 250 to 500 mg twice or thrice a day
until the pH improves to normal levels. Infusion of normal saline may help replete the
sodium and chloride and reverse the alkalosis but would be counterproductive in this
patient with significant volume overload. Stopping the loop diuretic would do the same.
Spironolactone and amiloride are not used for this purpose.

30. The correct answer is: D. Increase chlorthalidone to 25 mg daily. Resistant


hypertension indicates uncontrolled hypertension on optimal doses of three drugs, one of
which should be a diuretic. This patient has uncontrolled hypertension but the dose of one
of the drugs is not optimized. So the best option would be to increase chlorthalidone to 25
mg daily. Adding a fourth drug, such as carvedilol (a good choice with her history of
CAD) or spironolactone, may be considered if her BP remains uncontrolled after
uptitration of the chlorthalidone dose. Lisinopril is already at the maximum dose and an
increase would lead to adverse effects, such as hyperkalemia. Changing to a loop diuretic
may be required when her eGFR drops below 20 to 25 mL/min, but it is not required at
this time.

31. The correct answer is: C. Lisinopril. Methyldopa, labetalol, nifedipine, and hydralazine
are the most studied antiteratogenic drugs that can be safely used during pregnancy.
Labetalol and hydralazine are commonly used in hypertensive crises. These drugs are also
compatible with lactation. Angiotensin-converting enzyme (ACE) inhibitors and
angiotensin receptor blockers (ARBs) are teratogenic, especially during the second and
third trimester of pregnancy, and can lead to fetal pulmonary hypoplasia, malformations,
fetal renal failure, growth retardation, and miscarriages; hence, these drugs should be
switched during pregnancy planning or at conception.

32. The correct answer is: B. Monitor closely with IV furosemide as needed. The IDEAL-
ICU study showed no mortality benefit for early (within 12 hours of acute kidney injury)

Freemedicalbooks4download
versus late (after 48 hours of acute kidney injury) renal replacement therapy in patients
with septic shock. This is consistent with the previously published results of the AKIKI
trial. Notably, 38% of the patients in the delayed group did not receive renal replacement
therapy and only 17% of the patients in the delayed group required emergency renal
replacement therapy. While there was prevailing thought that early dialysis initiation may
improve acid-base, electrolyte, and fluid balance and potentially avoid adverse outcomes,
that has been proven not to be the case. Initiating renal replacement therapy early may
unnecessarily expose patients to the risks associated with dialysis, such as access
complications and prolonged ICU stay; hence, close monitoring and use of IV diuretics to
maintain fluid balance is recommended in the early period of acute kidney injury, unless
there is an emergent indication for renal replacement therapy, such as severe volume
overload, metabolic acidosis (pH < 7.15), and hyperkalemia (>6 mEq/L) refractory to
medical therapy. If dialysis were to be initiated in this patient, continuous renal
replacement therapy would be the preferred choice due to hemodynamic instability. While
indications for bicarbonate therapy are controversial, it is not recommended for patients
with an arterial pH >7.2. There are some data that suggest it may prevent the need for
renal replacement therapy and potentially improve survival in patients with severe acute
kidney injury.

33. The correct answer is: D. Observation only. It can be particularly challenging to decide
when to stop or interrupt dialysis in patients with an underlying chronic kidney disease
and an acute insult. Studies have shown that having a robust urine output is a strong
predictor of dialysis discontinuation. There is no specific laboratory threshold where
dialysis is not indicated. Patients with acute kidney injury should be evaluated daily for
renal replacement therapy needs, including metabolic derangements and volume overload.
IV furosemide is not required unless volume removal takes precedence. In patients outside
the critical care setting, volume removal is not an overwhelming issue; hence, furosemide
can be avoided with a urine output >2 L. In this patient, therefore, it is prudent to wait and
watch to see if she recovers enough kidney function to stay off dialysis. The increase in
BUN and creatinine from postdialysis levels is expected with equilibration postdialysis,
since these concentrations are lowered rapidly by dialysis and then slowly increase during
equilibration when dialysis is discontinued.

34. The correct answer is: C. Perform surgical evaluation. This patient has steal syndrome,
which diverts blood away from the arteries of the hand and can lead to ischemia. It
requires immediate surgical evaluation and, potentially, ligation of the fistula if severe.
Mild steal syndrome can be managed with lowering dialysis blood flow rates if the
symptoms are manageable; however, in this patient with severe pain and a nonpalpable
pulse, a surgical evaluation is paramount. Some patients experience mild numbness for
months to years before steal syndrome becomes apparent, while in others it becomes
apparent with the first use of a fistula. Risk factors for steal syndrome in dialysis patients
include smoking, diabetes, hypertension, and peripheral vascular disease.

35. The correct answer is: D. He should be evaluated and listed for a kidney transplant
now. As of February 2019, there were close to 95 000 patients on the kidney transplant
waiting list, with 20% being Hispanics. There is no age limit for a candidate to be listed.
For most malignancies, a period of 2 to 5 years before transplantation is recommended.
No waiting period is required for cancers such as nonmelanomatous skin cancers, in situ
bladder and cervical cancers, or small (<7 cm) and incidentally discovered renal cancers
that are removed. A wait time of more than 2 years is required for melanomas, breast,
colorectal, and uterine cancers to account for recurrence risk. The median wait times
according to blood group type are as follows: 2 years for AB, 3 years for A, and 5 years
for B and O. This patient is 72 years but leads an active life and seems to have adequate
family support. With a blood group type that has a lower than average wait time, he
should undergo a recipient evaluation and get listed for a transplant. While he is
undergoing workup, he should attempt to lose weight and lower his BMI as obesity is
associated with delayed graft function. The current eGFR cutoff to be listed is ≤20
mL/min and he does not have to wait until he is on dialysis. In fact, listing him as soon as
possible is paramount given he can accrue time on the transplant list now and, potentially,
get a transplant before he is on dialysis. He should be encouraged to look at potential
living donors since they are associated with better patient and graft survival; however, that
should not delay him being listed on the deceased donor transplant list.

36. The correct answer is: C. Listed for liver transplant. A study showed that cirrhotic
patients requiring renal replacement therapy had an extremely poor prognosis with a
median survival of around 2 to 3 weeks in the absence of liver transplant (Allegretti AS,
Parada XV, Eneanya ND, et al. Clin J Am Soc Nephrol. 2018;13:16). There was no
difference in mortality if the acute kidney injury was secondary to acute tubular necrosis
or due to hepatorenal syndrome. Patients of older age with higher MELD scores and
indicators of critical illness, such as vasopressor initiation, mechanical ventilation, or
continuous renal replacement therapy, had a worse outcome. Being listed for a liver
transplant played a big role in determining mortality at 6 months. Almost half the subjects
(48%) listed for a liver transplant obtained a transplant and mortality was also much lower
in the group listed for transplant (45%) compared with those who were not (84%). Our
patient is not young and has a high MELD score. While her overall prognosis is poor, her
transplant listing status gives her a more favorable prognosis compared with not being
listed.

37. The correct answer is: C. Quality of life in transplant recipients is the same as that of
patients on dialysis. In fact, transplant recipients report a superior quality of life
compared with patients on any modality of dialysis. Overall, peritoneal dialysis has not
shown to have a mortality benefit over hemodialysis, although it has been shown to be
associated with better cognitive function. While there are no randomized trials,
observational data suggest a survival benefit with home hemodialysis compared with in-
center hemodialysis.

38. The correct answer is: C. Refer her for kidney biopsy. A kidney biopsy should be
considered for all patients with suspected lupus nephritis. This includes patients with the
following:

a. urine protein creatinine excretion of >0.5 g/d


b. abnormal kidney function
c. findings on urine sediment, such as persistent hematuria or RBC casts, indicating active
kidney involvement

Freemedicalbooks4download
Patients with no signs of systemic lupus disease and normal creatinine may have
active renal involvement. Immune complex–mediated glomerulonephritis is the most
common cause of lupus nephritis and most forms of immune complex–mediated disease
respond to mycophenolate and steroids; however, it is important to define the extent and
chronicity of injury, both of which may modify the intensity and duration of treatment.
This patient with 800 mg/g of proteinuria active complements and hypertension could
have mesangioproliferative (class II), focal proliferative (class III), or diffuse proliferative
(class IV) lupus nephritis. In addition to immunosuppression, intensive BP control is
important. Since she is of childbearing age, she should also undergo contraceptive and
preconception counseling. While spot urine testing is not as reliable as using a 24-hour
urine collection for testing proteinuria, obtaining the results should not delay the referral
for a kidney biopsy. Patients in whom diagnosis and treatment of active renal lupus are
delayed are at a much higher risk of progressing to end-stage renal disease (ESRD).

39. The correct answer is: C. Lisinopril. IgA nephropathy, the most common glomerular
disease in the developed world, most often presents with gross or microscopic hematuria.
Gradual progression to end-stage renal disease (ESRD) over 15 to 20 years occurs in 50%
of patients. Elevated creatinine, persistent urine protein creatinine ratio >1 g/d, and high
BP (>140/90) are markers of progressive loss of kidney function. The mainstays of IgA
nephropathy treatment are BP control (<130/80 mmHg) and proteinuria reduction (<500
mg/g) with ACE inhibitors or ARBs. Fish oil may be used as an add-on therapy in patients
with persistent proteinuria despite ACE inhibitors or ARBs. Immunosuppression, with
either prednisone or cyclophosphamide, is suggested for patients with one or more of the
following:

1. persistent and progressive proteinuria >1 g/d after maximal medical therapy
2. necrotizing glomerulonephritis on kidney biopsy
3. acutely rising serum creatinine

Immunosuppression is not recommended for patients without active inflammatory


changes on a kidney biopsy or those with a stable chronic kidney disease. Recurrent
hematuria, by itself, is a nonspecific marker and is not an indication to treat unless there is
a concurrent significant rise in creatinine and/or proteinuria.

40. The correct answer is: C. Membranous nephropathy. This patient, with recent
discontinuation of immunosuppression and skin findings, most likely has chronic graft-
versus-host disease. Chronic graft-versus-host disease can affect multiple organs, with
skin, liver, and mucosal surfaces most often involved. Since 2000, there has been
increasing recognition that nephrotic syndrome could be the renal manifestation of chronic
graft-versus-host disease. While this has not been proven conclusively, the temporal
relationship between discontinuation of immunosuppression and presentation of nephrotic
syndrome, as well as the coexistence of graft-versus-host disease involving other organs,
suggests that is the case. The loss of immune tolerance as a result of decreased
immunosuppression may result in reactivity of donor lymphocytes to the native kidney.
Membranous glomerulonephritis is the most common lesion seen (~60%), while minimal
change disease (20%-25%) and focal segmental glomerulosclerosis (<10%) are less
common. Immunosuppression with a combination of steroids, cyclophosphamide, and
Rituxan is most commonly used in managing these patients. Thrombotic microangiopathy
is unlikely in this patient with no anemia or thrombocytopenia. While posttransplant
antineutrophilic cytoplasmic antibody (ANCA) vasculitis has been reported in the
literature as a manifestation of chronic graft-versus-host disease, it usually presents with a
nephritogenic pattern and, added to that, our patient’s ANCA levels are normal.

41. The correct answer is: B. SPEP, serum immunofixation, and serum free light chain
assay. Monoclonal gammopathy of renal significance is the name assigned to a group of
disorders in which patients have renal manifestations due to immunoglobulins produced
by a B-cell or plasma cell clone but do not have sufficient circulating immunoglobulins to
meet the criteria for MGUS (monoclonal gammopathy of unclear significance).
Monoclonal gammopathy of renal significance comprises a spectrum of diseases,
including monoclonal Ig deposition disease, proliferative glomerulonephritis with
monoclonal Ig deposits, C3 glomerulonephritis (C3 with masked Ig deposits),
amyloidosis, fibrillary glomerulonephritis, cryoglobulinemia, and immunotactoid
glomerulopathy. These diseases have various rates of detection of monoclonal protein in
the peripheral blood or urine, with light chain deposit disease having detection rates of
65% to 100%, while proliferative glomerulonephritis with monoclonal Ig deposit has the
lowest detection rates (20%-30%). Evaluation of patients with monoclonal gammopathy
of renal significance involves various tests to detect the monoclonal protein, including
SPEP, Urine protein electrophoresis, serum immunofixation, and serum free light chain
assay. Among these tests, the combination of SPEP (monoclonal spike), serum
immunofixation (to confirm monoclonality and type of protein), and serum free light
chain assay (abnormal kappa/lambda ratio inferring monoclonality) has the highest
sensitivity to detect a monoclonal protein. The addition of urine tests fails to improve
diagnostic sensitivity. A bone marrow biopsy should be considered in those in whom a
monoclonal protein is not detected using the aforementioned methods.

42. The correct answer is: C. Hemolytic uremic syndrome. This patient has a rapidly
progressive glomerulonephritis, with crescent formation seen on a kidney biopsy. Anti–
glomerular basement membrane disease and antineutrophilic cytoplasmic antibody
(ANCA)-associated glomerulonephritis are the most common causes of crescentic
glomerulonephritis, with 85% of anti–glomerular basement membrane disease and 50% of
ANCA disease presenting in this manner (>50% crescents on a kidney biopsy). Lupus
nephritis and IgA nephropathy can also rarely present with crescentic glomerulonephritis.
Kidney biopsy in hemolytic uremic syndrome typically demonstrates endothelial injury
and thrombi/fibrin deposition in the glomerular capillaries and arterioles.

43. The correct answer is: A. Cryoglobulinemic glomerulonephritis. This patient who
presented with gross hematuria, hypertension, and acute kidney injury was found to have
protein and blood on urinalysis, with RBC casts most concerning for acute
glomerulonephritis. Cryoglobulinemia results from deposition of cold-sensitive antibodies
and complement proteins within blood vessels, causing decreased perfusion and ischemia.
Cryoglobulinemia is associated with untreated hepatitis C and can cause
membranoproliferative glomerulonephritis with a reduction in C3 and C4. Her urinalysis
is active with protein and blood with an RBC cast shown on her urine sediment.
Treatment involves plasmapheresis and immunosuppression (eg, steroids) with avoidance

Freemedicalbooks4download
of cold temperatures.

A. Incorrect. While Henoch-Schönlein purpura presents with palpable purpura and can cause
glomerulonephritis, one would expect a normal C3.
B. Incorrect. This patient does not have a history of cirrhosis and hepatorenal syndrome is a
diagnosis of exclusion with no signs of intrinsic kidney injury.
C. Incorrect. This patient does not have significant urinary retention and one would expect a
bland urinalysis and sediment with postrenal acute kidney injury.
D. Incorrect. This patient has moist mucous membranes and an active sediment. One would
expect bland urinalysis and sediment with prerenal acute kidney injury.

44. The correct answer is: D. Urinalysis + sediment. This patient presents with gross
hematuria without a recent history of trauma or suspected kidney stone. The next best step
in management to differentiate intrarenal from extrarenal etiologies is a urinalysis with
sediment. Urinalysis will reveal ≥3 RBCs on dipstick. If glomerular in etiology, one
would expect dysmorphic RBCs or RBC casts.

A. Incorrect. As there is no history of flank pain, recent kidney stone, or trauma, CT urography
would not be indicated.
B. Incorrect. Gross hematuria should be initially worked up with CT urography in case of
suspected stone or history of trauma; otherwise, urinalysis is appropriate.
C. Incorrect. Renal ultrasound is indicated in the evaluation of acute kidney injury/chronic
kidney disease, but would not help significantly in workup for gross hematuria.
D. Incorrect. Urology referral at this time is not appropriate as the etiology should first be
evaluated by urinalysis.

45. The correct answer is: A. Amyloidosis. Many conditions can produce a urinalysis that
appears relatively normal with little or no cells, casts, or protein. Vascular injuries, such as
thrombotic microangiopathies and malignant hypertension, can rarely produce red cells in
the urine, but the urine is most often clear. Patients with conditions that obstruct the
tubules (like cast nephropathy), externally obstruct the flow of urine (such as benign
prostatic hypertrophy), or compress the vasculature (such as abdominal compartment
syndrome) have a normal urinalysis. A decrease in effective circulating volume, such as in
hepatorenal syndrome or cardiorenal syndrome, also produces benign-appearing urine.
Other conditions that produce acute kidney injury and a normal urinalysis include
hypercalcemic nephropathy, phosphate nephropathy, and tumor lysis syndrome.
Amyloidosis, a primarily glomerular process, usually presents with glomerular proteinuria
(albuminuria), which would be present on urinalysis.

46. The correct answer is: D. Most patients who pass a kidney stone will never
experience another one in their lifetime. One-third of patients experience a recurrence
of a kidney stone in 5 years and the rate of recurrence increases to almost 50% in 10 years.
It is important to perform a stone compositional analysis or a 24-hour urine metabolic
profile to evaluate the type of stone a patient is at risk for and instill management
appropriately. With regard to the type of stones, calcium stones account for almost 80%,
with calcium oxalate being the most common. Uric acid, cystine, struvite, and mixed-
composition stones are less common. In a patient with calcium oxalate stones and low
fluid intake, increasing the fluid intake in addition to dietary modification (low sodium,
high potassium, low oxalate, low nondairy animal protein, increased fruit and vegetables,
supplemental vitamin C) decreases the chances of new calcium stone formation by half. A
low-calcium diet is not recommended. Alkalinizing the urine (pH > 6) with citrate helps
prevent new uric acid stone formation but can conversely increase the risk of calcium
phosphate stones. Most stones (>80%) <0.5 cm in size pass spontaneously. The chances of
spontaneously passing a stone decreases with increase in size after that, with only 25% of
stones ≥9 mm passing without intervention. Stone passage is also affected by location,
with proximal stones having lower chances of spontaneously passing compared with distal
stones at the ureterovesical junction.

47. The correct answer is: C. Oxalate nephropathy. Enteric hyperoxaluria is the most
common cause of secondary hyperoxaluria and occurs in malabsorptive states, such as
inflammatory bowel disease, pancreatic insufficiency, and bowel resections, or after
bariatric surgery, such as Roux-en-Y gastric bypass. Chronic malabsorption leads to
increased fatty acids in the intestinal lumen, which then bind the calcium that would
otherwise bind with dietary oxalate. The high quantities of soluble oxalate then get
absorbed and can lead to nephrolithiasis and oxalate nephropathy. Renal biopsy shows
extensive tubular and interstitial damage, reflecting the direct toxicity of oxalate to the
tubular epithelial cells, while the glomerulus is not typically affected. A 24-hour urine
collection reveals high urinary oxalate levels, while urine calcium and citrate are low.
Management includes dietary modification to include a diet low in fat and oxalate and
relatively high in calcium. Liberal fluid intake is also recommended.

48. The correct answer is: C. Vitamin C. This patient most likely has calcium oxalate
nephrolithiasis and oxalate nephropathy. Oxalate nephropathy can be an inborn error of
metabolism that manifests at birth or can be secondary to excessive oxalate consumption
or absorption. Ingestion of high-dose vitamin C (1-2 g/d), an oxalate precursor, is
associated with formation of calcium oxalate stones and oxalate deposition in the kidney.
A large study showed that individuals taking vitamin C had twice the risk of kidney stones
as individuals not taking vitamin C. Vitamin D intake with normal calcium levels does not
lead to a higher stone risk. B6 supplementation lowers urinary oxalate and may reduce the
risk of kidney stones.

Freemedicalbooks4download
5
HEMATOLOGY-ONCOLOGY

QUESTIONS

1. A 92-year-old woman with depression presents with fatigue and dyspnea. She lives alone,
and her diet consists primarily of tea and toast. Examination is notable for body mass
index (BMI) of 17, normal deep tendon reflexes, and a steady gait. Labs reveal
hemoglobin (Hgb) 9 g/dL, mean corpuscular volume (MCV) 108 fL/red cell, indirect
bilirubin 1.5 mg/dL (normal <1 mg/dL), lactate dehydrogenase (LDH) 315 U/L (normal
140-280 U/L), homocysteine 75 μmol/L (normal <13 μmol/L), and methylmalonic acid
0.3 μmol/L (normal <0.4 μmol/L). Peripheral smear shows neutrophil hypersegmentation
and macro-ovalocytes.
What is the most likely etiology of her anemia?
A. Anemia of chronic disease
B. Folate deficiency
C. Iron deficiency
D. Vitamin B12 deficiency

2. A 39-year-old man presents with new bilateral hand and foot numbness. He is a vegan
who regularly practices yoga but has noticed worsening imbalance during yoga recently.
Examination reveals absent deep tendon reflexes in the upper and lower extremities. Labs
are notable for Hgb 10 g/dL, MCV 111 fL/red cell, indirect bilirubin 1.8 mg/dL (normal
<1 mg/dL), LDH 332 U/L (normal 140-280 U/L), homocysteine 101 μmol/L (normal <13
μmol/L), and methylmalonic acid 75 μmol/L (normal <0.4 μmol/L). Peripheral smear
shows neutrophil hypersegmentation and macro-ovalocytes.
After confirmatory diagnostic tests, what is the most appropriate treatment?
A. Folate 5 mg oral daily for 3 months
B. Levothyroxine 50 μg daily
C. Oral ferrous gluconate three times daily
D. Vitamin B12 1 mg IM weekly and then monthly

3. A 66-year-old man presents with fatigue and dyspnea. He does not like to see doctors and
does not have a primary care physician (PCP). His wife notes that, over the past few
weeks, he has started eating ice frequently. Examination is notable for angular cheilosis,
atrophic glossitis, and guaiac positive stool. Labs reveal Hgb 9.2 g/dL and MCV 75 fL/red
cell.
What will his iron studies likely show (normal iron 60-170 μg/dL, normal total
iron binding capacity [TIBC] 240-450 μg/dL, normal ferritin 12-250 ng/mL)?
A. Iron 132 μg/dL, TIBC 299 μg/dL, ferritin 204 ng/mL
B. Iron 210 μg/dL, TIBC 350 μg/dL, ferritin 375 ng/mL
C. Iron/TIBC <18%, ferritin 5 ng/mL
D. Iron/TIBC >18%, ferritin 314 ng/mL

4. A 21-year-old man presents with fatigue and gum bleeding. He has no past medical
history (PMH), and he is not taking any medications, including supplements and over-the-
counter (OTC) medications. Labs are notable for white blood cell (WBC) 1100/μL, Hgb
7.3 g/dL, platelets 15 000/μL, and reticulocytes 1%. Viral studies, including human
immunodeficiency virus (HIV), Epstein-Barr virus, parvovirus B19, and human
herpesvirus 6 (HHV-6), are negative. A bone marrow biopsy shows hypocellularity.
What is the most appropriate management?
A. Allogeneic stem cell transplant
B. Immunosuppression
C. Supportive care
D. Thrombopoietin mimetics

5. A 46-year-old woman with lupus on hydroxychloroquine presents with increasing


fatigue, painful arthralgias, and oral ulcers. She takes no other medications, denies alcohol
use, and lives in a home constructed in the last 5 years. Labs show Hgb 8.9 g/dL from
12.2 mg/dL last year, transferrin saturation 16%, ferritin 350 ng/mL (normal 12-25
ng/mL), MCV 82 fL/red cell, and normal creatinine.
What is the most likely diagnosis?
A. Anemia of chronic inflammation
B. Anemia of chronic kidney disease
C. Iron deficiency anemia
D. Sideroblastic anemia

6. A 74-year-old woman recently diagnosed with pneumonia treated with levofloxacin


presents with dyspnea and jaundice. Examination shows tachycardia to 110s,
splenomegaly, and cervical lymphadenopathy. Labs are notable for WBC 15 800/μL,
absolute lymphocyte count 7500/μL (normal <5000/μL), Hgb 6.9 g/dL, platelets 160 000/
μL, direct bilirubin 0.2 mg/dL, indirect bilirubin 2.6 mg/dL (normal <1 mg/dL),
reticulocytes 11% (normal 0.5%-2.5%), LDH 511 U/L (normal 140-280 U/L),
haptoglobin undetectable (normal 30-200 mg/dL), and a positive Coombs test. Peripheral
smear shows spherocytic red blood cells (RBCs) and no schistocytes.
What is the most likely diagnosis?
A. Autoimmune hemolytic anemia
B. Drug-induced hemolytic anemia
C. Hereditary spherocytosis
D. Microangiopathic hemolytic anemia (MAHA)

7. A 35-year-old woman presents for evaluation of acute fatigue over the past week. She

Freemedicalbooks4download
states that 2 weeks ago, her two children, ages 7 and 9, developed a febrile illness
associated with prominent redness of the cheeks. They recovered well and have returned
to school. A few days later, the patient developed a low-grade fever, which resolved
spontaneously, and she is not sure whether she also had the redness of the cheeks. Her
laboratory workup is shown below:
Laboratory studies
WBC count 8500/μL
Hgb 8.4 g/dL (previously 12.1 g/dL)
MCV 89 fL/red cell
Platelets 255 000/μL
Total bilirubin 0.8 mg/dL
Which of the following is the most likely diagnosis?
A. Anemia of chronic disease
B. Aplastic anemia
C. Folate deficiency
D. α-Thalassemia
E. Pure red cell aplasia

8. A 55-year-old woman presents for a routine annual physical and to establish care with her
new provider. Her PCP sends basic laboratory studies, which reveal evidence of anemia
with Hgb 10.1 g/dL, MCV 70 fL/red cell, and transferrin saturation 35%. At the next
follow-up visit, the patient brings her medical records from the time of her first pregnancy
at age 29, which are shown below.
Laboratory studies
WBC count 7200/μL
Hgb 9.8 g/dL
MCV 69 fL/red cell
Platelets 255 000/μL
Total bilirubin 0.8 mg/dL
Transferrin saturation 29%
Hgb electrophoresis
HbA2 <2.0%
HbA 95%
HbF <1.0%
HbH 2.5%
Which of the following is the correct diagnosis?
A. Iron deficiency anemia
B. α-Thalassemia major
C. α-Thalassemia minor
D. β-Thalassemia major
E. β-Thalassemia minor

9. A 42-year-old woman with sickle cell disease presents with a vaso-occlusive pain episode
requiring admission for intravenous pain medications. Social history is notable for current
smoking. On hospital day 2, she develops cough, dyspnea, fever, and a new 4L nasal
cannula oxygen requirement. Physical examination reveals tachypnea, intercostal
retractions, diffuse wheezing, and bibasilar rales. Chest radiograph (CXR) shows new
bilateral lower lobe opacities.
Labs show hematocrit 22%, WBC 12 000/μL with 80% neutrophils, and
reticulocytes 8%. Blood and sputum cultures are sent. Electrocardiogram (ECG) shows
sinus tachycardia with heart rate (HR) 110 beats/min and no ST or T-wave changes. She
is started on broad-spectrum antibiotics for pneumonia. However, her oxygen saturation
drops to 85% on 4L nasal cannula, and she transiently requires a 100% nonrebreather
before stabilization at oxygen saturations of 90% on 6L nasal cannula. She is transferred
to the intensive care unit (ICU) for closer monitoring of her tenuous respiratory status.
What is the most important next treatment step for this patient?
A. Albuterol nebulizer
B. Dexamethasone
C. Exchange transfusion
D. RBC transfusion

10. A 25-year-old man presents with fatigue, fevers, dyspnea on exertion, and a rash on his
lower extremities for the past 4 days. Aside from a fever, his vital signs are normal. Initial
examination reveals a normal neurologic examination and mental status. Labs show
hematocrit 18%, platelets 13 000/μL, and creatinine 1.1 mg/dL (baseline 0.7), LDH 1122
U/L, total bilirubin 1.4 mg/dL, and direct bilirubin 0.2 mg/dL. Initial peripheral smear
shows rare schistocytes.
A few hours later, he develops altered mental status consisting of word salad
speech and an inability to follow commands. Peripheral smear shows increased
schistocytes.
How should this be managed?
A. Fresh frozen plasma
B. Plasma exchange
C. Platelet transfusion
D. RBC transfusion

11. A 32-year-old African American woman presents with fatigue and dyspnea on exertion.
She was given trimethoprim-sulfamethoxazole for a urinary tract infection (UTI) 5 days
ago. She looks pale and has a HR of 105 beats/min. Complete blood count (CBC) reveals
Hgb 6.0 g/dL and hematocrit 19%.
What diagnostic test should be performed next?
A. Flow cytometry
B. Glucose-6-phosphate dehydrogenase (G6PD) testing
C. Hgb electrophoresis
D. Osmotic fragility

12. An 88-year-old woman with a history of coronary artery disease (CAD) for which she had
undergone coronary artery bypass graft surgery 10 years ago, atrial fibrillation on chronic
warfarin therapy, hypertension, heart failure with preserved ejection fraction, and chronic
kidney disease is brought to the emergency room (ER) by her family after a mechanical
fall at home with a complaint of altered mental status.
On examination, she is somnolent, but arousable to voice. A noncontrast head
computed tomography (CT) is performed and demonstrates a 2-cm subdural hematoma
with associated midline shift.
Laboratory studies
Creatinine 1.9 mg/dL

Freemedicalbooks4download
Hgb 12.5 g/dL
Platelets 205 000/μL
PT-INR 3.9
PTT 30 sec
In addition to discontinuing warfarin and any antiplatelet therapy that the patient is
receiving, which of the following is the most appropriate next step in treatment?
A. Administer 4-factor prothrombin complex concentrate (4F-PCC)
B. Administer 10 mg oral vitamin K
C. Administer fresh frozen plasma
D. Administer platelet transfusion
E. Transfuse 1 unit packed RBCs

13. A 65-year-old man with hypertension, type 2 diabetes mellitus (T2DM) on insulin, CAD,
and chronic kidney disease on dialysis presents to his hemodialysis clinic for his
scheduled dialysis treatment after missing his preceding two dialysis runs. On sitting
down in the dialysis chair, he develops spontaneous epistaxis. The bleeding continues
even after 40 minutes despite holding pressure.
On examination, the patient is twitching intermittently and is slow to respond but is
oriented to person, place, and time.
Laboratory studies
BUN 85 mg/d
Creatinine 6 mg/dL
Hgb 12.5 g/dL
Platelets 205 000/μL
PT-INR 1.0
PTT 30 sec
Which of the following best describes the etiology of the patient’s abnormal
hemostasis and the appropriate intervention?
A. Acquired platelet disorder; DDAVP (desmopressin acetate)
B. Acquired platelet disorder; discontinue insulin
C. Acquired platelet disorder; transfuse platelets
D. Inherited platelet disorder; DDAVP
E. Inherited platelet disorder; transfuse platelets

14. A 27-year-old man presents to the emergency department (ED) with a complaint of
abdominal pain and lower extremity rash for 2 to 3 days. Although he has been drinking
adequate fluids, he has been urinating less frequently.
On examination, there is a raised, purpuric rash over the bilateral lower
extremities. Temperature is 37°C (98.6°F), blood pressure is 122/84 mmHg, pulse rate is
90/min and regular, and respiration rate is 18/min. Pulmonary examination reveals
decreased breath sounds at the bases, and abdominal examination shows diffuse
tenderness without rebound or guarding.
Laboratory studies
BUN 45 mg/dL
Creatinine 2.6 mg/dL
AST 22 U/L
ALT 23 U/L
Total bilirubin 0.3 mg/dL
Alkaline phosphatase 102 U/L
Hgb 10.5 g/dL
WBC count 9200/μL
Platelets 200 000/μL
Which of the following is the most appropriate diagnostic test to perform next?
A. Antineutrophil cytoplasmic antibody (ANCA) titer
B. Peripheral blood smear to evaluate for schistocytes
C. Renal biopsy with direct immunofluorescence microscopy and serum anti–
glomerular basement membrane antibody
D. Renal ultrasound with Dopplers
E. Skin biopsy with direct immunofluorescence microscopy

15. A 38-year-old African American woman with no significant PMH other than
hypothyroidism presents to urgent care after a 10-day trip to England with a complaint of
several days of weakness, abdominal pain, nausea, and low-grade fever.
On physical examination, the patient appears fatigued and has scattered petechiae
over her bilateral shins. Her cardiac and pulmonary examinations are normal.
Laboratory studies
BUN 18 mg/dL
Creatinine 0.95 mg/dL
AST 22 U/L
ALT 23 U/L
Total bilirubin 1.8 mg/dL
Direct bilirubin 0.4 mg/dL
LDH 807 U/L
Hgb 9.2 g/dL
Direct antiglobulin (Coombs) test (DAT) Negative
WBC count 7200/μL
Platelets 12 000/μL
Peripheral smear Few platelets, 3 schistocytes per hpf
ADAMTS13 assay Pending
Which of the following is the most likely diagnosis?
A. Drug-induced thrombocytopenia
B. Evan syndrome
C. Hemolytic uremic syndrome
D. Primary immune thrombocytopenia
E. Thrombotic thrombocytopenic purpura

16. A 55-year-old woman with a recent admission 4 weeks ago for appendectomy, during
which she received prophylactic subcutaneous heparin, was admitted 2 days ago with
shortness of breath on exertion and chest tightness. She was found on workup to have
newly elevated troponin and ECG changes, consistent with non–ST elevation myocardial
infarction. She was started on a heparin drip and has been awaiting coronary angiogram.
This morning, she complained of new right lower extremity pain and swelling and
was found to have a deep vein thrombosis (DVT). On examination, she has bruising over
the bilateral arms, swelling of the right lower extremity, and pain with dorsiflexion of the
right foot.
Laboratory studies
BUN 18 mg/dL
Creatinine 0.95 mg/dL
Hgb 12.8 g/dL (13.2 g/dL on admission)

Freemedicalbooks4download
WBC count 7200/μL
Platelets 65 000/μL (155 000/μL on admission)
Which of the following is the most appropriate next step?
A. Continue heparin, send platelet factor 4 (PF4)-heparin enzyme-linked
immunosorbent assay (ELISA)
B. Continue heparin, send serotonin release assay
C. Stop heparin, send PF4-heparin ELISA, start bivalirudin
D. Stop heparin, send PF4-heparin ELISA, start warfarin
E. Stop heparin, send serotonin release assay, await results before resuming
anticoagulation

17. A 24-year-old man presents to his PCP’s office with 1 week of spontaneous nose and gum
bleeding. He otherwise feels well and recalls having a upper respiratory tract infection 3
weeks prior for which he took OTC nonsteroidal anti-inflammatory drugs (NSAIDs) for 1
to 2 days. He has never had similar symptoms in the past, and there is no family history of
any bleeding diathesis. On examination, he has petechiae in his oral mucosa. Two 0.5-cm
blood blisters are noted on his buccal mucosa.
CBC reveals WBC 7000/μL, normal differential, Hgb 14 g/dL, platelets 4000/μL.
Remaining blood work including renal and liver function tests (LFT) were within normal
limits. Peripheral blood smear shows normal RBCs, WBCs, and decreased albeit large
platelets.
What is the diagnosis in this case?
A. Acute lymphoid leukemia
B. Acute myelogenous leukemia
C. Ehrlichiosis
D. Immune thrombocytopenic purpura
E. NSAID-induced thrombocytopenia

18. How would you treat the patientin Question 17 in the acute setting?
A. Emergent splenectomy
B. Intravenous immunoglobulin (IVIG) + steroids
C. Platelet transfusion
D. Thrombopoietin agonist therapy (eg, romiplostim)
E. Watchful waiting for platelet recovery

19. A 54-year-old woman with hepatitis C and compensated cirrhosis is involved in a motor
vehicle accident (MVA) and suffers a left femoral fracture needing urgent surgical
exploration and fixation. Her labs show WBC 7000/μL (normal differential), Hgb 11
g/dL, and platelets 40 000/μL. International normalized ratio (INR) is 1.3. Both Hgb and
platelets have been stable on a few lab checks ranging back to the last 6 months.
What blood products does this patient need prior to proceeding with surgery?
A. 2U fresh frozen plasma and 10 mg IV vitamin K
B. 2U fresh frozen plasma and platelet transfusion with a platelet goal >50 000/μL
C. No blood products necessary. Proceed with surgery
D. Platelet transfusion with a platelet goal >50 000/μL
E. Thrombopoietin agonist: Romiplostin 10 mg/kg SC × 1
20. In which of the following situations will DDAVP (intranasal or intravenous) not be an
appropriate therapy for controlling bleeding?
A. End-stage renal disease
B. Mild hemophilia A
C. Mild type I von Willebrand disease (vWD)
D. Type 3 vWD

21. A previously healthy 45-year-old man with no PMH and taking no medications presented
to the ED with severe substernal chest pain and was diagnosed with an anterior ST
elevation myocardial infarction for which he underwent immediate percutaneous
intervention with a drug-eluting stent to the proximal left anterior descending coronary
artery. His course was complicated by cardiogenic shock requiring inotropes, and an intra-
aortic balloon pump placement. He was started on dual antiplatelet therapy with aspirin
and ticagrelor and an IV unfractionated heparin drip at the time of the percutaneous
coronary intervention. On day 2 of his admission, his platelet count was noted to be 70
000/μL, down from 180 000/μL on admission. Other blood counts were normal.
What is NOT a possible etiology of his thrombocytopenia?
A. Disseminated intravascular coagulation
B. Intra-aortic balloon pump
C. Type 1 heparin-induced thrombocytopenia (HIT)
D. Type 2 HIT

22. A 62-year-old woman presents with weakness, fatigue, recurrent epistaxis, and a recent
UTI currently being treated with ciprofloxacin. Physical examination shows dried blood in
bilateral nares and scattered ecchymoses on her trunk, arms, and legs. Labs are notable for
WBC 1200/μL, Hgb 8 g/dL, platelets 31 000/μL, prothrombin time (PT) 20 sec (normal
16 sec), INR 1.7 (normal 1.1), partial thromboplastin time (PTT) 41 sec (normal 25-35
sec), D-dimer >3.0 μg/mL (normal <0.5 μg/mL), and fibrinogen 2.2 g/L (normal >2 g/L).
Peripheral smear shows very rare large myeloid precursors with a high nucleus to
cytoplasmic ratio, fine chromatin, bilobed nuclei, prominent nucleoli, and cytoplasmic
granules.
Which of the following agents is most appropriate to administer?
A. Cryoprecipitate
B. IV vitamin K
C. PCC
D. Platelet transfusion

23. A 77-year-old man with atrial fibrillation on dabigatran presents with new right-sided
weakness. Labs are notable for normal creatinine and an elevated INR. A head CT shows
an intracerebral hemorrhage in the left frontal lobe. He last took dabigatran 3 hours before
presentation.
Which of the following agents is most appropriate to administer?
A. Andexanet α
B. Factor VIII inhibitor bypassing activity (FEIBA)
C. Fresh frozen plasma
D. Idarucizumab

Freemedicalbooks4download
24. A 17-year-old teenage boy with severe hemophilia A has been on prophylactic factor VIII
infusions over the last decade to reduce bleeding complications. However, over the past
year he had five episodes of spontaneous hemarthroses and was diagnosed with a high
titer of factor VIII inhibitor.
What is the best prophylactic therapy to reduce his risk of bleeding?
A. Aminocaproic acid
B. Emicizumab
C. High-dose recombinant factor VIII
D. Recombinant factor IX
E. Recombinant factor VIIa

25. A 38-year-old man presents with 2 days of intermittent periumbilical to right upper
quadrant pain with associated nausea and vomiting. He also reports a few weeks of
fatigue, a faint yellowing of his skin, and episodes of dark urine at night. Physical
examination reveals abdominal tenderness and distension.
Labs show WBC 2000/μL, hematocrit 24%, platelets 80 000/μL, reticulocytes 8%,
LDH 960 U/L, total bilirubin 4 mg/dL, direct bilirubin 0.3 mg/dL, and a negative Coombs
test. Abdominal CT with contrast shows a filling defect in the superior mesenteric vein
with mesenteric stranding and bowel wall enhancement with no evidence of
collateralization.
What diagnostic test will reveal the most likely underlying disorder?
A. Activated protein C resistance assay
B. Antithrombin III levels
C. Flow cytometry for CD55 and CD59
D. Protein C and S levels

26. A 25-year-old woman with hypothyroidism presents with new persistent weakness of her
right hand. She notes transient episodes of right hand weakness over the past few weeks as
well as a new rash on her lower extremities. Her current episode of right hand weakness is
similar to her prior episodes except that it has now lasted a few hours longer than the other
episodes, which lasted only minutes. Physical examination reveals 0/5 strength of her
right hand and reticular erythema on her bilateral lower extremities.
Labs are notable for WBC 8000/μL, 75% neutrophils, hematocrit 36%, and
platelets 110 000/μL. A head CT shows no acute intracranial hemorrhage, and a brain
magnetic resonance imaging (MRI) shows a small ischemic infarction in the left
precentral gyrus and multifocal white matter lesions. She is initially treated with
recombinant tissue plasminogen activator with improvement in her right hand strength.
She is subsequently started on a heparin drip and aspirin. Further testing reveals
significantly elevated anticardiolipin and β-2-glycoprotein antibodies that remain elevated
on repeat testing 3 months later.
Which long-term anticoagulation option is most appropriate for this patient?
A. Apixaban
B. Dabigatran
C. Enoxaparin
D. Warfarin

27. A 36-year-old man presents to a hematologist after discovering he has factor V Leiden,
which was diagnosed on a commercial genetic test. He asks about starting anticoagulation
to minimize his risk of developing a blood clot. He has no history of blood clots.
Which anticoagulation plan is indicated for this patient?
A. 6 months’ anticoagulation
B. 12 months’ anticoagulation
C. Lifelong anticoagulation
D. No anticoagulation

28. A 73-year-old man is referred to hematology for a persistently elevated WBC count of 20
000/μL for the past year with an absolute lymphocyte count of 9000/μL. He takes aspirin
and atorvastatin and is otherwise healthy. He has had no recent illnesses. His liver and
kidney function are normal on recent blood work. Lymph node examination is normal.
Flow cytometry and molecular diagnostics result in a diagnosis of chronic lymphocytic
leukemia.
What treatment is indicated at this time?
A. Ibrutinib
B. No treatment
C. Steroids
D. Venetoclax

29. A 69-year-old man presents to the hospital for acute myocardial infarction, for which he
undergoes emergent balloon angioplasty and subsequent stent placement. His presenting
WBC count was 13 000/μL with no eosinophils, but the day after his arrival, he is noted to
have eosinophils to 500/μL and a rise in his creatinine to 2.0 mg/dL from a baseline of 1.0
mg/dL. LFTs remain normal.
What is the most likely cause of this eosinophilia?
A. Drug rash with eosinophilia and systemic symptoms (DRESS)
B. Infection
C. Microscopic cholesterol emboli
D. Myeloproliferative neoplasm

30. A 52-year-old woman is sent to the ER after routine outpatient blood testing identified a
WBC count of 100 000/μL. Her kidney function, liver function, and electrolytes are all
normal. She is mentating well and breathing room air. Uric acid and coagulation tests are
normal. Examination of her blood under the microscope identifies basophilia and
numerous left-shifted neutrophils.
What is the most likely diagnosis?
A. Acute myelogenous leukemia
B. Chronic myelogenous leukemia
C. Strongyloides infection
D. Systemic mastocytosis

31. A 57-year-old man with polycystic kidney disease requiring a renal transplant last year on
tacrolimus, azathioprine, and prednisone presents with a diverticular bleed. Labs are
notable for Hgb 6.2 g/dL.
Which of the following types of RBCs is the most appropriate to administer?
A. Cytomegalovirus negative

Freemedicalbooks4download
B. Irradiated
C. Irradiated, CMV-negative, and leukoreduced
D. Leukoreduced

32. A 32-year-old man with sickle cell disease presents with faint jaundice, dark urine, and a
low-grade fever. A week earlier, he had been admitted for a vaso-occlusive pain crisis and
treated with pain medications, as well as a unit of packed RBCs for symptomatic anemia
below his usual baseline. Labs are notable for Hgb 6.1 g/dL (baseline 7-8 g/dL), direct
bilirubin 0.3 mg/dL, indirect bilirubin 2.5 mg/dL (normal <1 mg/dL), LDH 400 U/L
(normal 140-280 U/L), haptoglobin 20 mg/dL (normal 30-200 mg/dL), a positive Coombs
test, and urinalysis trace positive for RBCs. He is started on intravenous fluids.
What is the most appropriate next step?
A. Consult the blood bank
B. Discharge home
C. Obtain blood cultures
D. Transfuse 1 unit packed RBCs

33. An 86-year-old woman with chronic obstructive pulmonary disease (COPD) on 2L home
O2 and heart failure with preserved ejection fraction presents with coffee ground emesis.
Initial physical examination is notable for a weight of 89 lb and a height of 5 feet.
Baseline Hgb was 9.7 g/dL from recent clinic visit. At presentation, Hgb was 7.8 g/dL
with drop to 5.8 g/dL, so she was transfused 3 units packed RBCs with appropriate Hgb
response to 8.8 g/dL. Around 6 hours later, she developed respiratory distress with
desaturations to the 70s and hypertension with systolic blood pressures in the 180s. CXR
showed new perihilar fullness, indistinct pulmonary vasculature, and bibasilar patchy
opacities. She was treated with Nitropaste, IV Lasix, and bilevel positive air pressure
(BiPAP) but continued to desaturate and so was transferred to the ICU.
What is the most likely etiology of her respiratory distress?
A. Acute hemolytic reaction
B. Anaphylactic transfusion reaction
C. Transfusion-associated circulatory overload
D. Transfusion-related acute lung injury

34. A 66-year-old woman with hypertension presents with fatigue and a nonhealing wound on
her left arm. She reports tripping over furniture in her house and falling onto her left arm
with a resulting wound a few weeks ago. The wound initially started to heal with
antibiotics from her PCP. However, it reopened when she bumped it again a few days ago.
Physical examination reveals a laceration with a 2-cm area of fluctuance on her left arm
surrounded by erythema and draining a small amount of purulent serosanguineous fluid.
Labs are notable for hematocrit 24%, reticulocytes 2%, WBC 3500/μL, absolute
neutrophil count (ANC) 600/μL, and platelets 120 000/μL. Folate, vitamin B12, and
copper levels are normal, and an HIV test is negative. Peripheral smear reveals
ovalomacrocytosis, myelocytes, promyelocytes, and rare myeloblasts, as well as
neutrophils with bilobed nuclei. Bone marrow biopsy demonstrates hypercellularity, mild
fibrosis, increased abnormal megakaryocytes, and 2% myeloblasts. Cytogenetics shows
deletion 11q with no other abnormalities.
Which of the following is NOT an appropriate treatment for this patient?
A. Allogeneic stem cell transplant
B. Azacitidine
C. Epoetin alfa
D. Trimethoprim-sulfamethoxazole

35. For the patient in Question 34, which of the following is indicated for infection
prophylaxis?
A. Live attenuated zoster vaccine
B. Pegfilgrastim
C. Pneumococcal vaccine every 5 years
D. Prophylactic trimethoprim-sulfamethoxazole

36. A 61-year-old man who runs marathons presents with new dyspnea after running 3 to 5
miles for the past few weeks. He also reports more bruises on his legs and more frequent
episodes of epistaxis. Physical examination reveals clear lungs and scattered ecchymoses
on his lower extremities.
Labs are notable for hematocrit 22%, reticulocytes 1%, WBC 2900/μL, absolute
neutrophil count (ANC) 400/μL, and platelets 25 000/μL. Peripheral smear reveals
ovalomacrocytosis, myeloblasts, and neutrophils with bilobed nuclei. Bone marrow
biopsy demonstrates hypercellularity, moderate fibrosis, dysplastic erythrocytes, and 5%
myeloblasts. Cytogenetics shows deletions 3q and 7q with no other abnormalities.
Which of the following is an appropriate treatment for this patient?
A. Allogeneic stem cell transplant
B. Hypomethylating agents
C. Intensive chemotherapy
D. Targeted therapy with ivosidenib

37. A 45-year-old man presents to the hematology clinic after being diagnosed with an
unprovoked deep vein thrombosis (DVT), for which he is now on therapeutic low-
molecular-weight heparin. Initial testing was notable for low protein C and protein S
levels. In clinic, he describes a chronic headache and tinnitus. He also has a constant
burning sensation in his palms and soles.
Physical examination is remarkable for a red tint to his neck and face, a systolic
flow murmur, and a blood pressure of 170/90 mmHg. Labs are notable for WBC 12 000
K/μL, Hgb 17.5 g/dL, and hematocrit 50%. Erythropoietin levels are sent and return low.
Meanwhile, testing for JAK2 mutations has been sent, and results are pending.
Which of the following will NOT be indicated in the patient’s subsequent therapy?
A. Aspirin
B. Clopidogrel
C. Hydroxyurea
D. Phlebotomy

38. A 70-year-old woman with PMH significant for lupus presents to her PCP with fatigue
and abdominal pain.
On examination, she is diaphoretic and tender to right upper quadrant palpation.
Her labs are significant for Hgb 6 g/dL, platelets 450 000/μL, and WBC 10 000/μL. A
bone marrow biopsy identifies extensive fibrosis. No BCL-ABL translocation is

Freemedicalbooks4download
identified.
What is the next indicated course of treatment?
A. Allogeneic stem cell transplant
B. Hydroxyurea
C. Observation
D. RBC transfusion

39. A 43-year-old man is undergoing a preoperative evaluation for a knee surgery, during
which he has a CBC sent and is found to have a platelet count of 500 K/μL. Further work
up rules out secondary causes of thrombocytosis, and he is diagnosed with essential
thrombocytosis with a calreticulin mutation. He lives at home with his husband and
continues to work as an accountant. He plays golf every Sunday and sleeps comfortably
during the night.
What is the most appropriate treatment for the patient’s essential thrombocytosis?
A. Allogeneic stem cell transplant
B. Hydroxyurea
C. Low-dose aspirin
D. Observation

40. A 28-year-old woman presents to her PCP with 4 weeks of progressive fatigue, recurrent
upper respiratory infections, and frequent bruising. A CBC is performed and demonstrates
the following:
Hgb 8.2 g/dL
WBC count 2000/μL
ANC 320/μL
Platelets 38 000/μL
She is sent to the ED, where a peripheral smear is reviewed and demonstrates
circulating blasts, scarce platelets, and decreased RBC count. A bone marrow biopsy is
performed and demonstrates 38% blasts. Blasts were positive for CD13, CD33, and
myeloperoxidase on flow cytometry, and a diagnosis of acute myelogenous leukemia was
made.
Which of the following sets of laboratory results from the patient’s bone marrow
would be consistent with a favorable prognosis in this patient?
A. Cytogenetics: deletion 5, FLT3 internal tandem duplication (FLT3-ITD)
B. Cytogenetics: Monosomy 5, 7
C. Cytogenetics: t(9;22)
D. Cytogenetics: t(8;21), NPM1+

41. A 58-year-old woman with a history of breast cancer at age 49, treated with radiation and
chemotherapy, presents to the ED with fatigue, recurrent fevers, and bruising following
minimal injury over the past several weeks. On examination, she appears fatigued and has
ecchymoses over her bilateral lower extremities. Her lungs are clear and cardiac
examination is normal. A CXR shows clear bilateral lung fields without infiltrate or
mediastinal adenopathy.
Her laboratory workup is notable for CBC with absolute neutrophil count (ANC)
550/μL, platelets 45 000/μL, and Hgb 9.8 g/dL. The differential shows 10% circulating
blasts. A bone marrow biopsy confirms the diagnosis of acute myelogenous leukemia.
Cytogenetics and karyotype demonstrate intermediate prognosis.
Which of the following is the most appropriate choice for induction chemotherapy?
A. Liposomal cytarabine and daunorubicin on days 1, 3, and 5
B. Reduced intensity therapy with decitabine and 5-azacytadine
C. Standard induction therapy, “7 + 3” with cytarabine for 7 days and idarubicin or
daunorubicin for 3 days
D. Standard induction therapy, “7 + 3” with cytarabine for 7 days and idarubicin or
daunorubicin for 3 days, plus gemtuzumab ozogamicin on days 1, 4, and 7
E. Standard induction therapy, “7 + 3” with cytarabine for 7 days and idarubicin or
daunorubicin for 3 days, plus oral midostaurin on days 8 through 21

42. A 65-year-old man is noted on preoperative workup for hip replacement to have
neutropenia, anemia, and thrombocytopenia, with atypical promyelocytes on his
peripheral smear. A bone marrow biopsy is performed and demonstrates atypical
promyelocytes with Auer rods. Cytogenetic studies on the bone marrow are sent for rush
diagnostics and demonstrate(BUN) 60 mg/dL and creatinine 2.0 mg/dL,ranslocation of the
retinoic acid receptor, t(15;17), consistent with a diagnosis of acute promyelocytic
leukemia.
The patient is admitted to the hospital and treatment with all-trans-retinoic acid
(ATRA) and arsenic trioxide (ATO) is initiated within 24 hours. He is monitored closely
for disseminated intravascular coagulation over the next week and remains
hemodynamically stable. On the eighth day of treatment, he complains of increased
shortness of breath, and his examination is notable for fever to 38.5°C and hypotension
with blood pressure 85/45 mmHg. CXR is performed and demonstrates bilateral
pulmonary infiltrates.
What is the most likely diagnosis and most appropriate next step in management?
A. Acute respiratory distress syndrome; treat with antibiotics and diuresis
B. Acute respiratory distress syndrome; treat with intubation and low tidal-volume
ventilation
C. Differentiation syndrome; treat with daunorubicin plus ATRA consolidation therapy
and supportive care
D. Differentiation syndrome; treat with dexamethasone 10 mg twice daily and
supportive care
E. Differentiation syndrome; treat with supportive care alone

43. A 28-year-old woman with a 7-year history of chronic phase chronic myelogenous
leukemia is treated with imatinib at 400 mg/d. She feels fine, and you note that her
quantitative polymerase chain reaction (QT-PCR) has been undetectable for more than 3
years. She recently got married and now wants to try and conceive and asks you about
discontinuing the imatinib.
How would you advise her at this point?
A. If she discontinues the imatinib, she will have a 20% chance of recurrence in 24
months.
B. If she discontinues the imatinib, she will have a 60% chance of recurrence in 24
months.
C. If she discontinues the imatinib, she will have an 80% chance of recurrence in 24
months.

Freemedicalbooks4download
D. She can never safely discontinue the imatinib and must continue indefinitely because
she will surely relapse.
E. She can safely stop the imatinib without worry of recurrence because she is cured.

44. A 43-year-old woman was diagnosed with Ph-negative pre-B-cell acute lymphoblastic
leukemia (ALL) in 2014. The presenting WBC was 45 000/μL; and the cytogenetics were
normal (fluorescence in situ hybridization [FISH] and PCR for BCR-ABL negative).
Cerebrospinal fluid (CSF) studies were negative. She was treated with hyper-CVAD
(Cyclophosphamide, Vincristine, Adriamycin, Dexamethasone). She has two sibling
donors. She unfortunately relapses following completion of maintenance therapy.
A repeat bone marrow biopsy shows 75% lymphoblasts; cytogenetics remain
normal. Immunophenotype is CD45(dim), CD34, HLA-DR, TdT, CD123(dim; subset), B
lymphoid markers CD19 (100%), CD20(dim), CD10, CD22 (15%), aberrant expression
of CD33(dim), but negative for surface immunoglobulin light chain and other myeloid
markers, including myeloperoxidase.
How would you treat this patient with relapsed disease?
A. Blinatumomab
B. High-dose cytarabine (HiDAC)-containing regimen (eg, FLAG-Ida [fludarabine,
cytarabine, G-CSF, idarubicin], HAM [high-dose cytosine arabinoside and
mitoxantrone])
C. Inotuzumab ozogamicin
D. Liposomal vincristine plus corticosteroids
E. Tisagenlecleucel (anti-CD19 directed chimeric antigen receptor T-cell [CAR-T]
therapy)

45. A 32-year-old man presents to the local ED with 2 weeks of increasing fatigue, anorexia,
and fevers to 100.6°F. This morning, he noted bleeding from his gums and developed
dyspnea on walking. His symptoms progressed rapidly during the day, with worsening
dyspnea and new dizziness. His labs in the ED showed: WBC 210 000/μL, Hgb 7 g/dL,
platelets 11 000/μL; electrolytes showed a K of 5.9 mEq/L; blood urea nitrogen (BUN) 60
mg/dL and creatinine 2.0 mg/dL; LDH 1600 U/L; uric acid 18 mg/dL. Review of
peripheral blood smear shows abnormal cells with large nuclei, multiple nucleoli with fine
chromatin consistent with blasts without any granules. Peripheral blood flow cytometry is
50% blasts positive for CD13, CD117 and negative for CD3, CD4, CD19, CD20, and
TdT.
What is the diagnosis?
A. Acute myelogenous leukemia
B. B-cell acute lymphoblastic leukemia
C. Chronic myelogenous leukemia
D. T-cell acute lymphoblastic leukemia

46. What complications of acute leukemia could the patient in Question 45 likely have?
A. Leukostasis
B. Sepsis
C. Spontaneous tumor lysis syndrome
D. All of the above
47. A 34-year-old man with HIV is brought to the ED after having a new seizure. He has been
undomiciled for the past several years, and he has not had a recent CD4 check. CT of his
head identifies a mass, and further diagnostic tests confirm a central nervous system
(CNS) lymphoma.
What is the patient’s CD4 count (cells/mm3) most likely to be?
A. <10
B. <50
C. <100
D. <200

48. A 73-year-old woman presents to her PCP for routine evaluation, during which she is
found to have a white count of 23 000/μL with a lymphocyte count of 6000/μL. Peripheral
smear shows smudge cells, and further workup confirms a diagnosis of chronic
lymphocytic leukemia. Physical examination is unremarkable, and labs are otherwise
normal.
What is the most appropriate therapy for this patient’s chronic lymphocytic
leukemia ?
A. Chemotherapy
B. Ibrutinib
C. Observation
D. Splenectomy

49. A 73-year-old woman presents to the ED with fever, new weight loss, and dramatic
lymphadenopathy, most prominent in her right groin. She has known chronic lymphocytic
leukemia, for which she has been taking ibrutinib daily.
Labs show a WBC count of 50 000/μL, LDH 500 U/L, uric acid 9 mg/dL,
potassium 6 mEq/L, phosphate 6 mg/dL, calcium 6 mg/dL, and creatinine 3 mg/dL.
What is the most likely diagnosis?
A. Acute cytomegalovirus
B. Acute myelogenous leukemia
C. Diffuse large B-cell lymphoma (DLBCL)
D. Stage 4 chronic lymphocytic leukemia

50. A 29-year-old woman with a history of papillary thyroid carcinoma, for which she
underwent total thyroidectomy 1 year earlier, presents to her PCP with a new right
supraclavicular mass. A CT of the chest with contrast demonstrates a 1.5x3.2-cm enlarged
right supraclavicular lymph node and multiple enlarged mediastinal lymph nodes, each
measuring <2 cm without mass effect on the surrounding structures.
In addition to excisional lymph node biopsy with immunophenotyping and
cytogenetics and CBC, comprehensive metabolic panel, and LDH, which of the following
are required to complete staging and risk stratification?
A. CT abdomen/pelvis with contrast; erythrocyte sedimentation rate (ESR); Epstein-
Barr virus serologies; bone marrow biopsy
B. CT abdomen/pelvis with contrast; ESR; Epstein-Barr virus serologies; bone marrow
biopsy; head CT or MRI
C. Positron emission tomography-computed tomography (PET-CT) scan; Epstein-Barr
virus serologies; bone marrow biopsy

Freemedicalbooks4download
D. PET-CT scan; ESR; bone marrow biopsy
E. PET-CT scan; ESR; bone marrow biopsy; head CT or MRI

51. The 29-year-old woman in Question 50 is found to have nodular sclerosis type Hodgkin
lymphoma on her excisional right supraclavicular lymph node biopsy. PET-CT
demonstrates mediastinal involvement, with low-volume lymphadenopathy encasing
several vascular structures, including the right pulmonary artery, without evidence of
disease below the diaphragm. Bone marrow biopsy is negative for involvement with
lymphoma. She has confirmed stage 2 disease.
Which of the following would be the most appropriate initial step in management?
A. ABVD (doxorubicin, bleomycin, vinblastine, dacarbazine) and repeat PET-CT after 2
cycles
B. Brentuximab-vedotin (antibody-drug conjugate against CD30) × 6 cycles
C. Escalated BEACOPP (bleomycin, etoposide, doxorubicin, cyclophosphamide,
vincristine, procarbazine, and prednisone) for 6 cycles
D. Involved field radiation to neck
E. Pembrolizumab (anti-PD-1 monoclonal antibody [mAb]) × 6 cycles

52. A 55-year-old woman with a history of stage 4 Hodgkin lymphoma treated with 6 cycles
of ABVD (doxorubicin, bleomycin, vinblastine, dacarbazine) at age 25 presents to
establish care. She feels overall well, but complains of fatigue, slight dyspnea on exertion
walking uphill, and weight gain of 15 lb over the past 6 weeks.
In addition to CBC with differential, which of the following would be the most
appropriate initial workup specific to this patient’s risk factors?
A. CXR, echocardiogram, pulmonary function tests
B. CXR, echocardiogram, pulmonary function tests, 8 AM cortisol
C. CXR, echocardiogram, thyroid-stimulating hormone (TSH), and free thyroxine (T4)
D. Mammography, CXR, pulmonary function tests
E. Mammography, echocardiogram, pulmonary function tests, TSH and free T4, 8 AM
cortisol

53. A 76-year-old man presents to his PCP with fatigue, 20-lb unintentional weight loss, and
several weeks of dark stool. He is found to have anemia with Hgb of 8.8 g/dL. He is
referred for endoscopy, which reveals a nonbleeding mass in the stomach. Biopsy reveals
a clonal B-cell population with immunophenotype consistent with extranodal marginal
zone lymphoma. Helicobacter pylori testing on the sample is positive.
Which of the following is the most appropriate next step in management?
A. Quadruple therapy for treatment of the H. pylori (bismuth, metronidazole,
tetracycline, omeprazole)
B. Quadruple therapy for treatment of the H. pylori (bismuth, metronidazole,
tetracycline, omeprazole) with concurrent involved-site radiation
C. Quadruple therapy for treatment of the H. pylori (bismuth, metronidazole,
tetracycline, omeprazole) with concurrent rituximab plus bendamustine
D. Rituximab plus bendamustine followed by quadruple therapy for treatment of the H.
pylori (bismuth, metronidazole, tetracycline, omeprazole)
E. Rituximab plus quadruple therapy for treatment of the H. pylori (bismuth,
metronidazole, tetracycline, omeprazole)
54. A 78-year-old man presents to the ED with a complaint of severe left hip pain over the
past 2 weeks. His family also reports that he has had decreased urine output, fatigue, and
several episodes of confusion. On examination, the patient appears his stated age and is
lying in bed, leaning to his right side because of pain in the left hip.
Laboratory studies
BUN 45 mg/dL
Creatinine 3.5 mg/dL
Calcium 13.5 mg/dL
Hgb 8.2 g/dL
WBC count 7200/μL
Platelets 185 000/μL
Serum free light chains
κ 1500 mg/dL
λ 19.4 mg/dL
SPEP M-protein component 8 g/dL
Immunofixation Monoclonal IgG-κ chain component of M spike
Which of the following are the next steps in diagnosis?
A. Bone marrow biopsy with cytogenetics and gene mutation analysis; PET/CT
chest/abdomen/pelvis
B. Bone marrow biopsy with cytogenetics and gene mutation analysis; skeletal survey
C. Peripheral blood cytogenetics; skeletal survey; PET/CT chest/abdomen/pelvis
D. Repeat serum protein electrophoresis (SPEP), serum free light chain assay, and
immunofixation in 6 months
E. Repeat SPEP, serum free light chain assay, and immunofixation in 6 weeks

55. A 75-year-old man with a history of hypertension, T2DM, and hyperlipidemia undergoes
preoperative laboratory workup prior to hernia repair. The complete metabolic panel
demonstrates an elevated globulin gap, so his PCP sends serum free light chains, SPEP,
and urine protein electrophoresis with immunofixation. On the SPEP, the M-protein spike
is 0.8 g/dL; the serum free light chains show a κ:λ ratio of 5, with elevated free κ light
chain level. Urine protein electrophoresis shows total protein <167 mg/24 hours and urine
albumin <5 mg/dL. The creatinine is 0.8 mg/dL.
Which of the following most accurately represents this patient’s risk of progression
to multiple myeloma, Waldenstrom macroglobulinemia, or a malignant
lymphoproliferative disease?
A. 0.5% per year; 10% lifetime risk
B. 1% per year; 25% lifetime risk
C. 4% per year; 50% lifetime risk
D. 10% per year; 80% lifetime risk
E. 100% lifetime risk; the patient already has multiple myeloma

56. A 68-year-old woman with no significant PMH is brought in by her husband to the ED for
fatigue, mucosal bleeding, decreased appetite, new lower extremity rash, and decreased
perfusion to her distal fingers and toes over the preceding 2 weeks. In addition, over the 2
days preceding admission, she developed headache, dizziness, blurry vision, decreased
alertness, and shortness of breath. On physical examination, she is noted to have
hepatomegaly extending 3 cm below the right costal margin. She is fatigued with
decreased alertness and arousable only to loud sound or sternal rub. She is evaluated by

Freemedicalbooks4download
dermatology for the lower extremity rash because of concern for vasculitis and is
diagnosed with type I cryoglobulinemia.
Laboratory studies
SPEP with immunofixation IgM monoclonal spike 7 g/dL
UPEP Negative
Creatinine 1.0 mg/dL
Hgb 9 g/dL
Platelets 52 000/μL
Bone marrow biopsy shows increased plasmacytoid lymphocytes.
Which of the following is the most likely explanation for the patient’s symptoms of
confusion, headache, dizziness, blurred vision, and shortness of breath?
A. Amyloidosis secondary to immunoglobulin M (IgM) deposition
B. Anemia and hypercalcemia
C. Autoantibody activity of IgM
D. Hyperviscosity syndrome
E. Local tumor infiltration

57. A 75-year-old man with a history of CAD presents for follow-up after a recent diagnosis
of anemia. In addition to fatigue related to his anemia, he complains of a tingling
sensation and burning pain in the feet, extending up to the midcalf. This discomfort limits
his mobility, and he has difficulty sleeping because of the pain. Laboratory workup for his
anemia is shown below:
Laboratory studies
SPEP with immunofixation IgM monoclonal spike 7 g/dL
UPEP Negative
IgM 4895 mg/dL (normal 40-230 mg/dL)
Creatinine 0.8 mg/dL
Hgb 10.2 g/dL
Platelets 105 000/μL
Viscosity 1.7 cP (normal 1.4-1.8 cP)
A bone marrow biopsy is performed and demonstrates 65% bone marrow
involvement with monoclonal plasmacytoid lymphocytes. He is told that his overall
presentation is consistent with a diagnosis of Waldenstrom macroglobulinemia.
Which of the following is the next best step in management?
A. Observation with repeat labs and clinical examination every 3 months
B. Start bendamustine + rituximab
C. Start bortezomib + dexamethasone + rituximab
D. Stem cell collection in preparation for autologous stem cell transplant
E. Urgent plasmapheresis

58. A 72-year-old woman presents for preoperative evaluation prior to left total hip
arthroplasty. She has a long-standing history of osteoarthritis of the left hip after a skiing
accident many years ago, and her most recent left hip x-ray demonstrates severe
degenerative changes with joint space narrowing, without evidence of fracture. She has
not seen a PCP in 4 years, so a comprehensive metabolic panel and CBC are performed.
Pertinent results are shown below:
Laboratory studies
Creatinine 0.8 mg/dL
Calcium 8.8 mg/dL
Alkaline phosphatase 124 mg/dL
Total bilirubin 0.8 mg/dL
Albumin 4.0 g/dL
Total protein 9.5 g/dL
WBCs 5800/μL
Hgb 13.2 g/dL
Platelets 252 000/μL
Based on the elevated total protein, additional laboratory workup and bone marrow
biopsy are performed to evaluate for an underlying hematologic disorder. SPEP with
immunofixation demonstrates M-protein elevated at 4 g/dL, serum free κ light chain 255
mg/L, serum λ free light chain 2.5 mg/L, κ/λ ratio 102:1. Bone marrow biopsy
demonstrates 15% clonal plasma cells. Bone scan demonstrates no lytic lesions.
Which of the following is the most appropriate diagnosis?
A. Amyloid light-chain amyloidosis
B. Monoclonal gammopathy of unknown significance (MGUS)
C. Multiple myeloma
D. Nonsecretory multiple myeloma
E. Smoldering multiple myeloma

59. A 65-year-old man presents with new right shoulder pain for the past month. He is active
and works out at the local gym five times per week, so he initially attributed the
discomfort to muscle strain after lifting weights. However, the pain worsened over the
subsequent days. He is referred for CT of the shoulder, which demonstrates a large
destructive osseous lesion involving the upper part of the humerus with an enhancing
extraosseous soft tissue component. CT-guided biopsy of the lesion is performed, and
pathology demonstrates clonal proliferation of plasma cells. Bone scan demonstrates no
other bony abnormalities. bone marrow biopsy shows normal trilineage hematopoiesis
without abnormal plasma cell clonal proliferation.
Laboratory studies
Creatinine 0.8 mg/dL
Calcium 9.0 mg/dL
Albumin 4.0 g/dL
Hgb 12.9 g/dL
Platelets 198 000/μL
SPEP with immunofixation Normal, no M spike
Which of the following is the most appropriate next step in management?
A. Combination induction chemotherapy with lenalidomide and dexamethasone (RD)
B. Combination induction chemotherapy with lenalidomide, bortezomib, and
dexamethasone (RVD)
C. Observation and physical therapy referral
D. Referral to radiation oncology for radiation to the right humerus osseous lesion
E. Stem cell collection followed by high-dose melphalan and autologous stem cell
transplant

60. A 62-year-old man is admitted to the hospital from clinic for management of poorly
controlled bone pain. He has numerous lytic metastases throughout the lumbar and sacral
spine, the bilateral femoral heads, and the pelvic bones, related to his recently diagnosed
multiple myeloma. His pain is controlled with a patient-controlled analgesia (PCA), and
the decision is made to proceed with treatment for his multiple myeloma during the

Freemedicalbooks4download
hospitalization, in view of his pain.
Laboratory studies
Hgb 10.2 g/dL
Creatinine 1.8 mg/dL
Calcium 9.0 mg/dL (after 2 L normal saline)
Based on his age and functional status, it is determined that he would be a
transplant candidate.
In addition to denosumab, which of the following would be the most appropriate
initial therapy for the treatment of this patient’s multiple myeloma?
A. Bortezomib/lenalidomide/dexamethasone
B. Doxorubicin/lenalidomide/dexamethasone
C. Human leukocyte antigen typing to find a match, followed by allogeneic stem cell
transplant with myeloablative conditioning
D. Pomalidomide/cyclophosphamide/dexamethasone
E. Stem cell harvest, followed by high-dose melphalan conditioning and autologous
stem cell transplant

61. A 35-year-old previously healthy man was found to have acute myelogenous leukemia
with myelomonocytic differentiation. The karyotype was normal. Leukemic cells showed
a NPM1 mutation and a FLT3-ITD mutation. Complete remission has been achieved
following “3+7” (daunorubicin/cytarabine) induction chemotherapy. The CBC is now
normal.
Which of the following treatment approaches would you recommend now?
A. 1 to 2 cycles of High-dose cytarabine ( HiDAC), followed by myeloablative
allogeneic transplant from his sibling
B. 3 to 4 cycles of HiDAC plus FLT3 inhibitor
C. 3 to 4 cycles of HiDAC, then observation
D. Myeloablative conditioning followed by allogeneic transplant from his sibling
E. Reduced intensity conditioning followed by allogeneic transplant from his sibling

62. A 34-year-old woman who, two weeks earlier, underwent a matched unrelated peripheral
stem cell transplant (PSCT) for acute myelogenous leukemia develops a new fever to
102°F. She had previously been breathing normally on room air, but shortly after the
fever, her oxygenation falls, and she is soon on a nonrebreather at 15 L. CXR shows
diffuse bilateral opacities. Physical examination reveals a diffuse erythrodermatous rash
over her torso, arms, and upper thighs.
Labs are significant for a creatinine of 1.2 mg/dL (baseline 0.7), WBC 4000/μL
(absolute neutrophil count [ANC] 600/μL), Hgb 7.1g/dL, and platelets 73 000/μL. Blood
and urine cultures are drawn.
What is the most appropriate approach to therapy?
A. Granulocyte-colony stimulating factor (G-CSF)
B. Remove all lines and start broad-spectrum antibiotics
C. Steroids
D. Tocilizumab

63. A 53-year-old man received mismatched related peripheral stem cell transplant 4 months
earlier for chronic myelogenous leukemia blast crisis. He was discharged from the
hospital after a few posttransplant complications. He returns to clinic, however,
complaining of increasing diarrhea.
Inspection of his skin reveals a maculopapular rash on both extremities. Labs are
drawn and are significant for K 3.0 mEq/L, total bilirubin 2.3 mg/dL, and creatinine 1.1
mg/dL.
What is the most likely diagnosis?
A. Acute graft-versus-host disease (GVHD)
B. Chronic GVHD
C. Cytomegalovirus colitis
D. Engraftment syndrome

64. A 44-year-old man who underwent a matched related peripheral stem cell transplant 12
days ago begins to complain of abdominal distension. Examination reveals a new fluid
wave, peripheral edema, and tenderness to right upper quadrant palpation. Cardiac
examination is unremarkable. He is noted to have gained 5 lb since his last measurement
two days earlier. He has no skin rashes, and his stool output has remained formed.
Labs are significant for AST 220 U/L, ALT 330 U/L, bilirubin 6 mg/dL(direct 5.5),
WBC 9000/μL, Hgb 8 g/dL, platelets 70 000/μL, INR 1.3, and N-terminal pro b-type
natriuretic peptide (NT-proBNP) 100 pg/mL. Right upper quadrant ultrasound is
unremarkable.
What is the most likely diagnosis?
A. Acute graft-versus-host disease
B. Budd-Chiari syndrome
C. Congestive heart failure
D. Sinusoidal obstruction syndrome

65. A 62-year-old woman with a 35-pack-year smoking history, COPD, and CAD is admitted
with increased cough productive of bloody sputum, thought to be consistent with a COPD
exacerbation. She reports that she has been feeling unwell for the past month with
decreased appetite and 10-lb unintentional weight loss. On examination, she has decreased
breath sounds at the right lung base and expiratory wheezing throughout the bilateral lung
fields. She has temporal and supraclavicular wasting. A CT of the chest with contrast is
performed and demonstrates a right middle lobe tumor measuring 6 cm. A CT-guided
biopsy of the mass is performed and confirms the diagnosis of non–small cell lung cancer.
Which of the following are the most appropriate next steps in diagnosis and
management?
A. Fluorodeoxyglucose positron emission tomography/computed tomography (FDG
PET/CT), and brain MRI; pending results, follow with mediastinal lymph node
evaluation with endobronchial ultrasound, mediastinoscopy
B. Initiate chemotherapy with platinum doublet and adjuvant radiation therapy
C. Initiate concurrent pembrolizumab and radiation therapy
D. Initiate definitive chemotherapy/radiation therapy followed by adjuvant durvalumab
E. Surgical debulking followed by chemotherapy and radiation therapy

66. A 58-year-old man, nonsmoker, with no significant PMH, presents to the ED complaining
of fatigue, abdominal pain, and light-headedness over the past 48 hours. On physical
examination, he appears fatigued with supraclavicular and temporal wasting. His

Freemedicalbooks4download
infectious workup is negative. A cosyntropin stimulation test demonstrates adrenal
insufficiency, and the patient undergoes CT of the abdomen/pelvis, adrenal protocol, for
further evaluation. The scan demonstrates adrenal metastases of unknown primary with
intrahepatic and bony metastases. Further workup with CT of the chest reveals a large
primary lung mass. A biopsy of the adrenal lesion confirms lung adenocarcinoma. The
tumor specimen is sent for analysis for genetic mutations in epidermal growth factor
receptor (EGFR), anaplastic lymphoma kinase, ROS1, and BRAF and demonstrates an
exon 19 deletion in EGFR.
Which of the following is the most appropriate treatment plan for this patient?
A. Carboplatin, pemetrexed, and crizotinib
B. Carboplatin, pemetrexed, and osimertinib
C. Carboplatin, pemetrexed, and pembrolizumab
D. Osimertinib alone
E. Pembrolizumab alone

67. A 69-year-old woman with a 50-pack-year history of smoking and a diagnosis of COPD
presents to establish care with a PCP for increased cough productive of sputum over the
past several months. She has a friend who was recently diagnosed with lung cancer and
inquires about screening chest CT. Her CT of the chest is performed and demonstrates a
large cavitary lung mass with bony and intra-abdominal metastases. She undergoes CT-
guided biopsy and is diagnosed with squamous cell carcinoma, lung primary.
Which of the following is the most appropriate therapy for stage 4 squamous cell
carcinoma with PD-L1 staining <50%?
A. Carboplatin and paclitaxel alone
B. Carboplatin and pemetrexed alone
C. Carboplatin, paclitaxel, and pembrolizumab
D. Carboplatin, pemetrexed, and pembrolizumab
E. Pembrolizumab alone

68. A 63-year-old woman is brought into the hospital after a generalized seizure. She has no
history of neurologic illness, and this is her first seizure. Noncontrast head CT is negative
for acute processes or tumors. Her sodium is measured at 111 mEq/L. She does not drink
alcohol and has smoked one pack of cigarettes a day for the past 40 years.
In addition to sodium studies, what diagnostic imaging should be performed to
diagnose the cause of her hyponatremia?
A. Brain magnetic resonance angiography (MRA)
B. Brain MRI
C. Carotid ultrasound
D. Chest CT

69. A 50-year-old woman presents to her PCP for her annual physical. She has high blood
pressure and a history of depression but is otherwise in good health. Her PCP refers her
for her first screening colonoscopy. The patient asks if she should also be referred for lung
cancer screening. She has been smoking one pack per day for the past 30 years.
When should her PCP refer her for lung cancer screening?
A. Today
B. In 5 years
C. In 10 years
D. No referral indicated

70. A 41-year-old premenopausal woman identifies a lump in her right breast. She goes to her
PCP, who refers her for imaging and a subsequent biopsy. She is ultimately diagnosed
with a small (0.8 mm), node-negative, low-grade estrogen receptor–positive (ER+) breast
cancer with a low Oncotype DX score.
After surgery and radiation, what is the standard systemic therapy for this patient?
A. Anastrozole
B. Lapatinib
C. Tamoxifen
D. Trastuzumab

71. A 53-year-old woman with a history of hormone receptor–positive breast cancer treated
15 years ago presents to her PCP with abdominal pain. Imaging reveals multiple liver
lesions, the biopsy of which identifies metastatic ER+/PR+/HER2− breast cancer.
Which of the following is NOT a therapeutic option?
A. Anastrozole
B. Everolimus
C. Lapatinib
D. Palbociclib

72. A 52-year-old woman has a suspicious mass identified on mammogram. A stereotactically


guided biopsy identifies breast cancer, and further diagnostics confirm the cancer to be ER
−/PR−/HER2+. She undergoes a lumpectomy and sentinel lymph node biopsy. The tumor
is 3 cm, and the sentinel lymph nodes are negative.
Which is the most appropriate adjuvant therapy?
A. Docetaxel, cyclophosphamide, trastuzumab, pertuzumab
B. Letrozole plus ribociclib
C. Neratinib
D. T-DM1

73. A 40-year-old woman presents to breast clinic with a new ER+/PR+/HER2− breast
cancer. Her mother was diagnosed with breast cancer at age 38, her maternal aunt
developed ovarian cancer at age 63, and her maternal grandfather died of pancreatic
cancer.
What is the germline mutation most likely underlying these four cancers in the
same family?
A. BRCA1
B. BRCA2
C. MLH1
D. TP53

74. A 75-year-old woman presents to her PCP for her annual physical. She has been
undergoing biennial mammogram screenings since she was 50 years old, and her last was
at age 73. She has always had normal mammograms and is having no breast complaints.
When is her next mammogram due?

Freemedicalbooks4download
A. This year
B. 1 year
C. 5 years
D. No further mammograms indicated

75. A 53-year-old woman presents to the ER with abdominal pain and is found to have a liver
mass on CT. Subsequent biopsy identifies a new triple-negative breast cancer.
Mammogram identifies the primary tumor in her left breast. She presents to her oncologist
to discuss treatment plans. Meanwhile, PD-L1 testing on her tumor has returned positive.
She has seen advertising for immunotherapy on TV and would like to know if this is an
option for her.
What will her oncologist recommend?
A. Considering a clinical trial of immunotherapy, as there is currently no approved
immunotherapy agent for breast cancer
B. Considering immunotherapy in combination with chemotherapy if she fails first-line
therapy
C. Starting treatment with immunotherapy alone
D. Starting treatment with immunotherapy in combination with chemotherapy first line

76. A 77-year-old man with a history of hypertension, hyperlipidemia, CAD on medical


management, and mild chronic kidney disease presents to the ER with 2 to 3 months of
increasing back pain, decreased appetite and 10-lb weight loss. Examination reveals an
elderly man, with focal spinal tenderness on the lower back. Rectal examination reveals an
enlarged, hard, and nodular right prostate lobe. His labs are significant for a Hgb of 9 g/dL
Land a prostate-specific antigen (PSA) of 4000 ng/mL. His Eastern Cooperative Oncology
Group (ECOG) Performance Status (PS) is 2. A bone scan shows extensive metastatic
disease in the entire axial skeleton, and a CT scan shows enlarged pelvic and
retroperitoneal lymph nodes measuring up to 3 cm. A biopsy of the retroperitoneal lymph
node shows prostatic adenocarcinoma.
What is the upfront therapy for this patient?
A. 2 weeks of bicalutamide followed by leuprolide
B. 2 weeks of bicalutamide followed by leuprolide and docetaxel
C. Degarelix and abiraterone/prednisone
D. Docetaxel
E. Leuprolide

77. A 67-year-old very active and healthy man has a PSA checked, which returns at 6. On
repeat testing 3 months later, the PSA is 6.1. He meets with a urologist to discuss these
results. On rectal examination, he has a smooth but enlarged prostate without any nodules
or abnormalities.
What is the next best step in management? The patient is quite hesitant to have
invasive testing.
A. Likely secondary to benign prostatic hyperplasia (BPH). No further testing
B. Proceed with ultrasound-guided transrectal 12-core prostate biopsy
C. Prostate MRI
D. Repeat PSA in 6 months
78. The man in Question 77 undergoes a prostate MRI, which shows a small 4-mm lesion in
the right prostatic lobe, for which he undergoes two targeted core needle biopsies. 10% of
one of the two cores shows Gleason 6 (3+3) prostate cancer. What is the next best step for
management? Given his active lifestyle, he is very concerned about his quality of life and
treatment-related toxicities.
Which treatment is associated with the least amount of morbidity?
A. Active surveillance
B. Brachytherapy
C. External beam radiation therapy
D. Robotic or laparoscopic radical prostatectomy

79. An 18-year-old man presents to a new PCP after transitioning away from his pediatrician.
On speaking with the patient, the physician learns that he has a strong family history of
cancers: his father developed pancreatic cancer at age 50, his paternal aunt developed
colon cancer at age 40, and his paternal grandmother developed endometrial cancer at age
50. His germline mutation analysis reveals a MSH2 mutation.
At what age should this patient start colon cancer screening and at what frequency?
A. Colonoscopy at age 20, then repeat screen yearly
B. Colonoscopy at age 20, then repeat screen every 5 years
C. Colonoscopy at age 30, then repeat screen yearly
D. Colonoscopy at age 30, then repeat screen every 5 years

80. A 73-year-old man with a PMH of hypertension and hyperlipidemia presents to his PCP
with new fatigue. He gets winded walking upstairs and is losing his energy. He used to go
to the grocery store and was able to walk the aisles, but now he is getting too tired and
needs to sit on a motorized cart. Basic labs are sent, which are significant for a Hgb of 7.2
g/dL (baseline 10 g/dL).
After ruling out cardiac etiologies, he is sent for a colonoscopy, which identifies a
2-cm mass in his ascending colon. Biopsy confirms adenocarcinoma. He is referred for
surgical removal but prior to this step is sent for further testing.
Which of the following are indicated at this time?
A. Carcinoembryonic antigen (CEA) level
B. CT chest/abdomen/pelvis
C. MRI brain
D. A and B
E. All of the above

81. A 45-year-old man is diagnosed with resectable pancreatic adenocarcinoma. He is treated


appropriately and is seen for follow-up and survivorship. He is concerned about his two
children, ages 11 and 15, since he has a strong family history of cancer, including breast
cancer in his mother, breast cancer in his maternal aunt, ovarian cancer in his maternal
aunt, and prostate cancer in his maternal grandfather. He inquires about genetic testing in
order to better understand the risk of cancer for his children.
Which of the following is the most likely hereditary risk factor for pancreatic
cancer in this family?
A. Familial atypical multiple mole melanoma (CDKN2A/p16)
B. Familial breast/ovarian cancer (BRCA2)

Freemedicalbooks4download
C. Hereditary chronic pancreatitis (mutation in cationic trypsinogen gene PRSS1, or
SPINK1)
D. Hereditary nonpolyposis colorectal cancer
E. Peutz-Jeghers (LKB1) syndrome

82. The patient in Question 81 had BRCA testing performed and was found to be a carrier for
a known deleterious mutation. Unfortunately, he developed metastatic disease and was
started on FOLFIRINOX (folinic acid, fluorouracil, irinotecan, oxaliplatin). He had a near
complete response to this regimen and has received 6 months of therapy. However, he has
developed increasing peripheral neuropathy and does not want to continue with more
FOLFIRINOX. Otherwise, he still remains in good health with an ECOG PS 0.
What is the next best option for treating this patient?
A. FOLFOX (5FU/leucovorin + oxaliplatin)
B. Gemcitabine/abraxane
C. Hospice
D. Immunotherapy with PD-1 blocking mAb (eg, pembrolizumab)
E. Poly (ADP-ribose) polymerase (PARP) inhibitors

83. A 72-year-old man with a history of hypertension, heart failure with preserved ejection
fraction, and chronic kidney disease presents with a complaint of yellowing of the eyes
and 20-lb unintentional weight loss over the preceding 3 months. On physical
examination, he has temporal and supraclavicular wasting, scleral icterus, and jaundice.
His abdomen is distended with a positive fluid wave and tender to deep palpation at the
epigastric region.
Laboratory studies
Total bilirubin 9.0 mg/dL
Direct bilirubin 7.2 mg/dL
Alkaline phosphatase 655 U/L
Which of the following is the most appropriate next step in diagnosis?
A. Check CA 19-9 and consult gastroenterology for endoscopic retrograde
cholangiopancreatography (ERCP).
B. Check CA 19-9 and pancreatic protocol CT scan or MRI with contrast.
C. Check CA 19-9 and perform abdominal ultrasound.
D. Pancreatic protocol CT scan or MRI with contrast followed by endoscopic ultrasound
(EUS)-guided fine needle aspiration (FNA)

84. A 63-year-old man with a history of hypertension, hyperlipidemia, T2DM, and cirrhosis
secondary to chronic hepatitis C presents to his PCP for a routine visit. As part of
hepatocellular carcinoma (HCC) screening, he has been undergoing liver ultrasounds. A
liver ultrasound done prior to the visit shows concern for a 4-cm mass lesion in the right
lobe of the liver. A follow-up multiphasic liver protocol CT scan revealed a 4.5-cm mass
showing arterial enhancement followed by progressive washout in the venous and delayed
phase.
What is the next best step in the management of this patient?
A. Follow-up multiphasic CT in 3 months
B. Liver MRI with contrast
C. Percutaneous needle biopsy
D. Referral to multidisciplinary team for management of HCC
E. Serum α-feto protein level

85. A 57-year-old woman with obesity, T2DM, hyperlipidemia, and CAD presents with
increasing pain in the right upper abdomen. Investigations, including CT abdomen, reveal
cirrhotic liver morphology and an 8-cm large tumor in the right liver lobe with multiple
surrounding small satellite lesions in the right lobe. The tumor is seen invading into the
portal vein, leading to its thrombotic occlusion. A biopsy of the mass reveals
hepatocellular carcinoma. No other metastatic lesions are seen in the abdomen, and a CT
chest is normal. She is evaluated by a multidisciplinary oncology team and deemed not to
be a candidate for surgical resection.
Which of the following therapies would not be appropriate for this patient?
A. Clinical trial with pembrolizumab (PD-1 mAb)
B. Sorafenib
C. Stereotactic body radiation therapy (SBRT)
D. Transcatheter arterial chemoembolization (TACE)

86. A 53-year-old man with a history of alcohol cirrhosis undergoes routine ultrasonography
and α-fetoprotein (AFP) testing every 6 months. On his most recent ultrasound, he is
found to have a suspicious lesion. There is no change in his AFP level.
What next step is needed to make a diagnosis of hepatocellular carcinoma?
A. Abdominal PET scan
B. Liver biopsy
C. Liver MRI
D. Repeat AFP level in 3 months

87. A 52-year-old man with hypertension presents with dyspnea on exertion and bilateral
lower extremity edema for the past few months. Physical examination reveals decreased
breath sounds in bilateral lower lobes, massive splenomegaly at least 20 cm below the left
costal margin, and 3+ pitting edema bilaterally.
Labs are notable for Cr 1.7 mg/dL(normal baseline), K 5 mEq/L, Ca 8.6 mg/dL,
phosphorus 6.6 mg/dL, uric acid 14.7mg/dL, LDH 1810 U/L, WBC 641,000/μL,
hematocrit 21.5%, and platelets 71,000/μL. Peripheral smear reveals marked leukocytosis
with a predominance of left-shifted myeloid cells in all stages of differentiation, including
occasional blasts, as well as increased basophils and nucleated RBCs. CXR showed
diffuse bilateral fine reticular nodular opacities with no pleural effusions.
Which of the following is not an appropriate next step?
A. Bone marrow biopsy
B. Chest/abdomen CT with contrast
C. Hydroxyurea
D. Intravenous fluids
E. Rasburicase

88. A 29-year-old woman with acute myelogenous leukemia on cytarabine and daunorubicin
develops a new fever while she is neutropenic with an absolute neutrophil count (ANC) of
0. Her nurse reports one episode of nonbloody emesis and several watery stools. Vital
signs are notable for T 38.3°C, HR 105 beats/min, and BP 110/75 mmHg. Physical

Freemedicalbooks4download
examination reveals right lower quadrant tenderness to palpation with minimal guarding
and no rebound tenderness. Other notable labs include Cr 0.9 mg/dL, hematocrit 24%, and
platelets 20,000/μL. Abdominal CT with contrast shows cecal wall thickening with no
pneumatosis. Broad-spectrum antibiotics are initiated.
Which additional therapy should be considered?
A. Loperamide as needed for diarrhea
B. Morphine for pain control
C. Nasogastric suction
D. Prochlorperazine as needed for nausea
E. Surgical intervention

89. A 43-year-old woman presents to the ED after visiting her PCP for easy bruising and
being found to have a platelet count of 27 000/μL. On arrival, her white count is noted to
be 1.5 with an absolute neutrophil count (ANC) of 330/μL. Her Hgb is 9 g/dL. Her INR is
1.3. She is an otherwise healthy woman and takes only lisinopril for blood pressure.
Inspection of her peripheral blood reveals no schistocytes but rare scattered cells with high
nuclear-to-cytoplasmic ratios and overall immature appearances. These are presumed to
be blasts. Several of the blasts have red needle-shaped figures in the cytoplasm, suspicious
for Auer rods.
What treatment should this patient receive immediately?
A. 7+3 induction
B. Arsenic trioxide (ATO) and all-trans-retinoic acid (ATRA)
C. High-dose steroids
D. Hydroxyurea

90. A 63-year-old man with a history of metastatic melanoma presents for a follow-up
appointment. He is on a clinical trial medication, which he has been tolerating well.
Today, however, he is reporting new lower back pain. On subsequent neuro examination,
he is noted to have new right lower extremity weakness and distal numbness.
What is the next best step his providers should take?
A. Discontinue clinical trial medication
B. Lumbar puncture
C. Refer for emergency surgery
D. Stat MRI spine

91. A 61-year-old man with no PMH was recently diagnosed with sigmoid colon
adenocarcinoma and underwent laparoscopic resection with primary anastomosis.
Pathology showed a T3N2a moderately differentiated adenocarcinoma, with 4/17 lymph
nodes positive for malignancy—stage 3B. The tumor was microsatellite stable. His
medical oncologist recommended starting adjuvant chemotherapy with FOLFOX (5-
FU/leucovorin, oxaliplatin) to reduce the risk of tumor recurrence. He is brought to the ER
after the first cycle with febrile neutropenia. He has been having more than seven episodes
of diarrhea per day for the last 3 days, and his absolute neutrophil count (ANC) on
presentation is 250/μL.
What is the potential cause of his toxicity?
A. Dihydropyrimidine dehydrogenase deficiency
B. Error leading to incorrect dose administration
C. Expected for this chemotherapy regimen
D. Lack of G-CSF postchemotherapy
E. Oxaliplatin toxicity

92. A 53-year-old woman was recently diagnosed with metastatic melanoma (BRAF wild
type) and was started on dual checkpoint blockade with ipilimumab (CTLA4 mAb) and
nivolumab (PD-1 mAb). She tolerated treatment well, and the first set of restaging scans
done at 4 weeks showed a partial response. Six weeks after initiating therapy, she
developed abdominal pain, fevers, and profuse diarrhea up to eight times per day. Her
therapy was stopped, and she was admitted to the hospital, where initial workup, including
stool cultures and Clostridium difficile toxin assay, was negative. A flexible
sigmoidoscopy showed diffuse severe colitis with multiple shallow ulcerations, and a
biopsy confirmed an active colitis; stains for cytomegalovirus were negative on the
biopsy. Based on this information, she was started on treatment with intravenous
methylprednisolone but did not show much improvement.
What treatment would you consider next in this patient?
A. Antimotility agents
B. Broad-spectrum antibiotics with gram negative and anaerobic coverage
C. Infliximab
D. Surgical consultation

93. A 52-year-old woman with unresectable melanoma is being actively treated with
combination nivolumab and ipilimumab, which she started 2 months earlier. She is
brought into the ED with profound weakness, fatigue, and headache. Her blood pressure is
found to be 70/40 mmHg.
Physical examination is notable for vitiligo skin changes on both hands and feet, a
normal cardiac examination, and 1+ nonpitting edema to her midshins. Labs are notable
for TSH 0.1 mU/L (normal 0.5-4.7 mU/L), free T4 0.3 (normal >0.7), and cortisol and
adrenocorticotropic hormone (ACTH) levels below assay. Follicle-stimulating hormone
(FSH) and luteinizing hormone levels are subsequently sent and also found to be
markedly low.
The patient’s immunotherapy is held, and she is given stress dose steroids, after
which her blood pressure rises to 100/70 mmHg.
What test would likely confirm her diagnosis?
A. ACTH stimulation test
B. Blood and urine cultures
C. Brain MRI
D. Thyroid ultrasound

94. A 34-year-old woman with refractory acute lymphoblastic leukemia undergoes a chimeric
antigen receptor T-cell (CAR-T) infusion. One week later, she develops fever, chills, and
tachycardia to 120 bpm. Her baseline blood pressure is 110/70 mmHg. Once her fevers
start, her blood pressure drops to 88/60 mmHg. She is given 2 L of fluid, and her blood
pressure recovers.
The following evening, her blood pressure decreases again but this time does not
respond to fluids. She is transferred to the ICU and started on peripheral vasopressors and
broad-spectrum antibiotics.

Freemedicalbooks4download
What additional therapy should be added to the above?
A. Infliximab
B. Mycophenolate mofetil
C. Stress dose steroids
D. Tocilizumab

95. A 56-year-old woman with relapsed diffuse large B-cell lymphoma (DLBCL) is treated
with chimeric antigen receptor T-cell (CAR-T) therapy. Five days later, she becomes
agitated and confused. A preliminary infectious and metabolic workup is negative, and
ultimately she has a grand mal seizure and requires intubation and transfer to the ICU.
What therapy is indicated for this patient?
A. Levetiracetam
B. Lorazepam
C. Steroids
D. Tocilizumab
E. A, B, C
F. All of the above

ANSWERS

1. The correct answer is: B. Folate deficiency. This patient is an elderly woman with
malnutrition and a diet low in leafy green vegetables and fruits. Her presentation is
consistent with megaloblastic anemia from impaired DNA synthesis, resulting in
ineffective erythropoiesis and macrocytosis. Her absence of neurologic symptoms and
normal methylmalonic acid are consistent with folate deficiency rather than vitamin B12
deficiency. However, B12 should also be checked to make sure she is not borderline
deficient. Although methylmalonic acid is very sensitive, one would not want to treat with
folate and mask developing B12 deficiency.

2. The correct answer is: D. Vitamin B12 1 mg IM weekly and then monthly. This
patient has vitamin B12 deficiency, given his neurologic deficits, megaloblastic
macrocytic anemia, and increased methylmalonic acid. His vegan diet likely does not
include foods of animal origin rich in vitamin B12. Although less likely in light of his
dietary habits, testing for pernicious anemia, including autoantibodies to parietal cells and
intrinsic factor, should be performed to make sure he can absorb oral supplementation
after full repletion with IM B12.

3. The correct answer is: C. Iron/TIBC <18%, ferritin 5 ng/mL. This patient has iron
deficiency anemia based on his pica, angular cheilosis, atrophic glossitis, and guaiac
positive stool. He requires a colonoscopy to screen for colon cancer and initiation of oral
or IV iron.

4. The correct answer is: A. Allogeneic stem cell transplant. This patient most likely has
a new diagnosis of aplastic anemia. For young patients, allogenic stem cell transplant
offers 80% long-term survival and significantly decreased risk of malignant evolution,
although it has the risk of transplant-related morbidity and mortality. Immunosuppression,
such as cyclosporine or tacrolimus, is associated with a response rate of 80% to 90% and
5-year survival of 80% to 90% but is also associated with a 15% to 20% 10-year incidence
of clonal disorders, including myelodysplastic syndrome, acute myelogenous leukemia,
and paroxysmal nocturnal hemoglobinuria. Thrombopoietin mimetics are usually reserved
for refractory disease, and supportive care with transfusions, antibiotics, and growth
factors is more appropriate for an elderly or treatment refractory population.

5. The correct answer is: A. Anemia of chronic inflammation. This patient has anemia of
chronic inflammation based on her normocytic anemia and high ferritin. The best
management of her anemia is treatment of her lupus, as her worsening arthralgias and new
oral ulcers suggest a likely disease flare. Although iron deficiency can also coexist with
anemia of chronic inflammation, iron deficiency is unlikely in this patient given the
elevated ferritin. Her normal creatinine also argues against anemia of chronic kidney
disease, while her normal Hgb last year and lack of offending exposures (ie, alcohol,
isoniazid, chloramphenicol, lead) are not consistent with sideroblastic anemia.

6. The correct answer is: A. Autoimmune hemolytic anemia (AIHA). This patient has
hemolytic anemia, spherocytosis, and a positive Coombs test all consistent with warm
autoimmune hemolytic anemia. The patient’s lymphadenopathy and absolute
lymphocytosis point to chronic lymphocytic leukemia as the underlying etiology of the
autoimmune hemolytic anemia. Drug-induced hemolytic anemia from levofloxacin is less
likely, given the positive Coombs test. Similarly, there is no thrombocytopenia or
schistocytosis to suggest microangiopathic hemolytic anemia (MAHA). Finally,
hereditary spherocytosis would have presented at a much earlier age and is not
characterized by a positive Coombs test.

7. The correct answer is: E. Pure red cell aplasia. This history provided suggests that the
patient and her children were infected with parvovirus, which can be associated with
fevers and a “slapped cheek” appearance. Her laboratory workup demonstrates a
normocytic anemia without evidence of thrombocytopenia or leukopenia, making aplastic
anemia less likely. Pure red cell aplasia occurs when a patient develops destructive
antibodies or lymphocytes that target the bone marrow and lead to ineffective
erythropoiesis. This condition can be associated with parvovirus infection, as well as with
thymoma, chronic lymphocytic leukemia, autoimmune disease, and certain drugs. Bone
marrow biopsy would reveal lack of erythroid precursors with normal hematopoiesis of
the other cell lines. Importantly, in immunocompetent adults, parvovirus infection leads to
transient anemia, so this woman is likely to recover soon without intervention. In
immunocompromised patients, parvovirus can cause more prolonged anemia.

8. The correct answer is: C. α-Thalassemia minor. The long-standing, asymptomatic


nature of the patient’s anemia makes iron deficiency anemia, choice A, unlikely, despite
the low MCV. The correct diagnosis based on the low MCV, the normal transferrin
saturation >18%, and the Hgb electrophoresis results is α-thalassemia minor. α-
Thalassemia minor results from the loss of two α-chain genes. The genotype can be
heterozygous for the α-thalassemia-1 trait (aa/--, more common among Asian individuals)
or homozygous for the α-thalassemia-2 trait (a-/a-, more common among African

Freemedicalbooks4download
individuals). Patients typically have mild anemia, hypochromia, and microcytosis, without
other significant clinical manifestations, unlike α-thalassemia intermedia or major.

9. The correct answer is: C. Exchange transfusion. This patient has acute chest syndrome
based on her new pulmonary infiltrates on CXR and her cough, fever, tachypnea,
intercostal retractions, wheezing, rales, and dropping oxygen saturations. Acute chest
syndrome is a leading cause of death for patients with sickle cell disease, and this patient
rapidly evolves from moderate to severe acute chest syndrome. Therefore, exchange rather
than simple transfusion is the most appropriate management, as exchange transfusion will
allow rapid decrease in hemoglobin S (HgbS) percentage while avoiding the
hyperviscosity that could occur with a large-volume simple transfusion. Steroids are not
standard practice for acute chest syndrome management in adults, and albuterol nebulizers
will not reverse the underlying cause of her symptoms.

10. The correct answer is: B. Plasma exchange. This patient has thrombotic
thrombocytopenic purpura leading to microangiopathic hemolytic anemia (MAHA). He
requires emergent plasma exchange to remove inhibitory autoantibodies and replace low
levels of ADAMTS13 (a disintegrin-like and metalloproteinase with thrombospondin
type-1 motifs 13), the enzyme required to cleave von Willebrand factor multimers.
Uncleaved ultrahigh-molecular-weight von Willebrand factor multimers lead to platelet
thrombi and hemolytic anemia. Fresh frozen plasma will replace ADAMTS13 but will not
remove inhibitory autoantibodies. RBC and platelet transfusions will similarly not reverse
the underlying disorder.

11. The correct answer is: B. Glucose-6-phosphate dehydrogenase (G6PD) testing. G6PD
testing is required, as G6PD deficiency is more common in African Americans and
patients of Mediterranean descent. Trimethoprim-sulfamethoxazole is a frequent cause of
hemolytic anemia in patients with G6PD deficiency. If anemia is severe or the patient is
symptomatic, RBC transfusion should be given. Patients should be advised to avoid
oxidizing agents in the future. Note that G6PD levels may be normal in the setting of an
acute crisis if the patient has developed a significant reticulocytosis, because these
younger cells have a higher G6PD content.

12. The correct answer is: A. Administer 4-factor prothrombin complex concentrate
(4F-PCC). Warfarin-associated intracranial hemorrhage with prolonged prothrombin time
INR should be treated with 4F-PCC, rather than fresh frozen plasma. The rationale for this
is that the 4F-PCC is administered in a smaller volume and will lead to more rapid INR
reversal. 4F-PCC contains the vitamin K–dependent coagulation factors, which include II,
VII, IX, and X. Although vitamin K should also be administered, it should be given as 10
mg of an intravenous infusion. There is no indication to transfuse platelets in this patient
with a normal platelet count and no evidence of platelet dysfunction. With regard to RBC
transfusion, this is reserved for large-volume blood loss. In this patient, although the
intracranial hemorrhage produces a significant clinical effect, the true blood loss is likely
relatively small.

13. The correct answer is: A. Acquired platelet disorder; DDAVP (desmopressin
acetate). Bleeding in the setting of uremia is often multifactorial but includes platelet
dysfunction and abnormal platelet-endothelial interactions. This is considered an acquired
platelet disorder, given that there is a normal platelet count, but decreased platelet
function, in a patient without a known congenital etiology for platelet dysfunction and
with an identifiable acquired trigger. The factors thought to contribute to platelet
dysfunction in the setting of uremia include uremic toxins, anemia, and increased nitric
oxide production by endothelial cells (inhibit platelet aggregation). Administration of
desmopressin (DDAVP) acts by increasing the release of large factor VII: von Willebrand
factor multimers from endothelial cells and is a rapid-acting, low-risk intervention for
management of uremic bleeding. It may also potentiate platelet activation and granule
release. The other accepted initial intervention is dialysis to correct the uremia. Platelet
transfusion will not be effective, because the platelets will become dysfunctional in the
uremic plasma. Insulin does not contribute to bleeding.

14. The correct answer is: E. Skin biopsy with direct immunofluorescence microscopy.
Palpable purpura is a common manifestation of small vessel vasculitis. The erythematous
papules and plaques will eventually progress to raised, nonblanchable lesions. In this
young patient with palpable purpura, abdominal pain, and new renal failure, the most
likely diagnosis is immunoglobulin A (IgA) vasculitis. In patients with IgA vasculitis,
skin biopsy will show leukocytoclastic vasculitis on histopathologic evaluation and IgA
on direct immunofluorescence. The first step in the diagnosis of patients with defined skin
lesions is a skin biopsy. Renal biopsy presents increased risk of bleeding because of the
invasive nature of the procedure, although it would also demonstrate IgA deposits. Anti–
glomerular basement membrane antibody disease presents with rapidly progressive
glomerulonephritis and, in some cases, pulmonary alveolar hemorrhage, but would not be
expected to present with abdominal pain or palpable purpura. Although antineutrophil
cytoplasmic antibody (ANCA) vasculitis may produce a similar rash, affected patients are
typically older (age 60s) and do not present with abdominal pain.

15. The correct answer is: E. Thrombotic thrombocytopenic purpura. This patient is a
young woman who has microangiopathic hemolytic anemia (MAHA) (Coombs negative),
thrombocytopenia, schistocytes on peripheral smear (indicative of MAHA), elevated
LDH, and elevated indirect bilirubin, all consistent with thrombotic thrombocytopenic
purpura due to an acquired autoantibody. Thrombotic thrombocytopenic purpura is more
common in women and in patients of the African American race. Given this patient’s
history of hypothyroidism, most likely from chronic autoimmune hypothyroidism, also
known as Hashimoto thyroiditis, this patient may be at a higher risk for autoimmune
disorders. It is important to recognize the diagnosis of thrombotic thrombocytopenic
purpura early, because the treatment is emergent plasma exchange therapy. Primary
immune thrombocytopenia is less likely in this case because the patient also has hemolytic
anemia, and Evans syndrome (immune thrombocytopenia plus hemolytic anemia) is also
less likely because of the negative Coombs test. Drug-induced thrombocytopenia is not
likely with concurrent hemolytic anemia. Potential drug culprits for thrombocytopenia
include quinine, antimicrobials, quetiapine, and chemotherapy agents. The patient would
not have required antimicrobial prophylaxis for her trip to Europe.

16. The correct answer is: C. Stop heparin, send PF4-heparin ELISA, start bivalirudin.
This patient most likely has immune (antibody)-mediated heparin-induced

Freemedicalbooks4download
thrombocytopenia (HIT), which typically occurs after 4 to 10 days of heparin exposure
but can occur earlier if the patient has been exposed to heparin within the preceding 100
days because of persistent antibodies. Thrombotic events can occur in up to 50% of
patients. The 4-T score can be used clinically to assess the pretest probability of heparin-
induced thrombocytopenia and thrombosis in this patient. This patient has a very high 4-T
score based on the degree of platelet drop (>50% with nadir above 20k), timing of the
thrombocytopenia (within 1 day given prior heparin exposure in the last month),
associated deep vein thrombosis (DVT), and no other clear explanation of her
thrombocytopenia. Immediate discontinuation of heparin and initiation of alternative
anticoagulation (parenteral agents include argatroban, bivalirudin, danaparoid, or
fondaparinux; oral agents include the direct-acting oral anticoagulants) is indicated if there
is a high probability of antibody-mediated HIT. Warfarin is not an accepted therapy for
initial management of thrombosis in HIT, because there will be an initial depleti2 HIT of
the anticoagulant factors protein C and S, prior to depletion of the vitamin K–dependent
clotting factors, which can exacerbate thrombosis and lead to peripheral gangrene. In
contrast, type I HIT can occur after 1 to 4 days of heparin therapy without prior heparin
exposure and is mediated by the direct effect of heparin (nonimmune). Patients typically
do not develop thrombosis, have platelet count >100 K/μL, and can safely continue on
heparin with close observation.
For diagnosis of HIT, the PF4-heparin ELISA should be sent first. If the result is
positive, then HIT is confirmed. If the result is indeterminate, a functional HIT assay,
such as the serotonin release assay, is indicated. If the ELISA is negative, HIT is
extremely unlikely, and other causes of thrombocytopenia should be investigated.

17. The correct answer is: D. Immune thrombocytopenic purpura. This is a classic
presentation of immune thrombocytopenia that has a bimodal age distribution for disease
onset. In children and younger adults, the disease typically presents following a viral
syndrome with sequelae from severe thrombocytopenia. The large morphology of platelets
is also classic for immune thrombocytopenia. Although an aleukemic leukemia is a
possibility, the lack of constitutional symptoms, preserved Hgb, WBC, and absence of any
abnormal leukocytes in the peripheral blood all point against it. Tick-borne diseases such
as Ehrlichia often have associated thrombocytopenia, but the lack of a clinical syndrome
including constitutional symptoms, fevers, associated leukopenia, and LFT abnormalities
is not consistent with ehrlichiosis. Lastly, although drug-associated thrombocytopenia has
to be considered, this clinical syndrome is more consistent with immune
thrombocytopenia.

18. The correct answer is: B. Intravenous immunoglobulin (IVIG) + steroids. The patient
has spontaneous bleeding and a platelet count <10 000/μL, which requires urgent
treatment with the goal of increasing his platelet count into a safe range (generally >30
000/μL). In particular, the presence of blood blisters on mucosal surfaces can be indicative
of higher chances of spontaneous bleeding such as in intracranial or visceral locations.
Steroids form the backbone of immune thrombocytopenia treatment and are considered
first-line therapy in the acute setting and generally lead to a response in platelet count in 4
to 7 days after therapy is initiated. In cases of severe thrombocytopenia and high risk of
CNS bleed, IVIG can also be added. Platelet count response to IVIG is typically seen
within 1 to 2 days. Given the immune nature of platelet clearance in immune
thrombocytopenia, there is generally no role for platelet transfusion. Splenectomy or
thrombopoietin agonist therapies are affective treatment strategies for management of
immune thrombocytopenia in the chronic phase.

19. The correct answer is: D. Platelet transfusion with a platelet goal >50 000/μL. In
general, platelet goal >50 000/μL is recommended for most surgical procedures. This
patient likely has multifactorial thrombocytopenia from hepatitis C, chronic liver disease,
and splenomegaly and is unlikely to have immune thrombocytopenia. Hence, platelet
transfusion is the most effective strategy to rapidly bring the platelet count to goal.
Romiplostin is often used in the preprocedural setting for elective procedures where there
is time to titrate the medicine and allow for a platelet response, which may take up to 1 to
2 weeks. Romiplostin is advised for patients with immune thrombocytopenia in the case
where platelet transfusions are not an effective therapy. This patient has a mildly deranged
INR, likely secondary to hepatic dysfunction, which does not reflect a true coagulopathy,
although the patient should be given vitamin K to optimize coagulation factors.

20. The correct answer is: D. Type 3 vWD. DDAVP is a synthetic analogue of vasopressin
that causes a transient release of von Willebrand factor (vWF) from storage granules in
the vascular endothelium, leading to an increase in serum concentrations. Hence, it is a
reasonable strategy for prophylaxis and treatment of minor bleeding in patients with mild
type I vWD and mild hemophilia A (factor VIII deficiency), because vWF increases the
circulating half-life of factor VIII from approximately 30 minutes to 24 hours. It has also
been seen to be effective in platelet dysfunction of end-stage renal disease. Type 3 vWD is
a rare variant of vWD characterized by a severe deficiency or near absence of any vWF,
leading to a severe bleeding diathesis. DDAVP is ineffective in type 3 vWD because there
is no vWF present in storage granules (which are Weibel-Palade bodies).

21. The correct answer is: D. Type 2 HIT. Type 2 HIT is an immune-mediated disorder
with generation of platelets against the heparin-PF4 complex, which leads to
microvascular platelet thrombi and a massively procoagulant state. However, the
generation of antibodies against heparin-PF4 needs at least 5 days of exposure to IV
unfractionated heparin. In patients with prior exposure to heparin type 2, HIT may occur
sooner than 5 days, but this patient has no prior PMH and is on no medications. Type 1
HIT is a non–immune thrombocytopenia caused by direct effects of heparin binding to
platelets and causing clearance from the circulation and is seen within the first 2 days of
IV unfractionated heparin initiation. This phenomenon is not prothrombotic and improves
over time even with continuation of therapy with heparin. Intra-aortic balloon pump use is
associated with thrombocytopenia, which is thought to be mechanical in etiology.

22. The correct answer is: B. IV vitamin K. This woman is presenting with a peripheral
smear concerning for acute promyelocytic leukemia, which is associated with
disseminated intravascular coagulation (DIC), often leading to life-threatening bleeding.
However, she is not actively bleeding, and her labs are consistent with mild DIC. Platelets
should be administered for a platelet count <10 000/μL, and patients with serious bleeding
and a significantly prolonged PT or activated partial thromboplastin time or a low
fibrinogen level should receive coagulation factor replacement, such as fresh frozen
plasma or cryoprecipitate, the latter of which provides fibrinogen with less volume than

Freemedicalbooks4download
fresh frozen plasma Therefore, she currently has no indication for platelets,
cryoprecipitate, or fresh frozen plasma. Vitamin K is the most appropriate therapy at this
time, particularly in the setting of antibiotic therapy that may suppress vitamin K–
producing intestinal flora. PCCs are usually contraindicated in DIC, given the risk of
thrombosis.

23. The correct answer is: D. Idarucizumab. The patient last took dabigatran 3 hours prior
to presentation, and the half-life is around 12 hours without renal insufficiency. Since he
has a major potentially life-threatening bleed, reversal of dabigatran with idarucizumab is
indicated. If idarucizumab is not available, an activated prothrombin complex concentrate,
such as Factor VIII inhibitor bypassing activity (FEIBA), could be administered instead,
although this carries a significant prothrombotic risk. Fresh frozen plasma is generally
avoided in bleeding associated with direct oral anticoagulants, given associated risks,
including transfusion reactions, thrombosis, and volume overload, and the absence of data
supporting its use except in the case of a coexisting coagulopathy. Andexanet α is
indicated for the reversal of factor Xa inhibitors in patients with life-threatening bleeds
associated with factor Xa inhibitors, such as rivaroxaban, apixaban, and edoxaban.

24. The correct answer is: B. Emicizumab. Emicizumab is a recombinant humanized


bispecific mAb that bridges activated factors IX and X and hence replaces the function of
factor VIII. It is not inhibited by antibodies that bind to factor VIII and is now approved
for primary prophylaxis in patients with hemophilia A and high titer of inhibitors. Given
the high titers of inhibitors (alloantibodies produced in response to frequent and long-term
recombinant factor VIII infusion), a higher dose of factor VIII would not work.
Hemophilia A is a deficiency in factor VIII, and levels of factor IX are normal in these
patients; hence, recombinant factor IX therapy would be of no benefit. Recombinant
factor VIIa activates the extrinsic coagulation pathway, which does not need factor VIII,
and has been used in the past for patients with hemophilia and high titer of inhibitors in
the setting of acute bleeding events. However, this is associated with a high risk of
thrombotic complications and is not used for prophylaxis. In the context of efficacy and
approval for emicizumab, the utility of factor VIIa in patients with inhibitors and acute
bleeding is less clear. Please note that emicizumab does not work immediately: it requires
about a month of loading dose treatment.

25. The correct answer is: C. Flow cytometry for CD55 and CD59. This patient has
hemolytic anemia, pancytopenia, and mesenteric venous thrombosis, which together
suggest a diagnosis of paroxysmal nocturnal hemoglobinuria, a rare, acquired
hematopoietic stem cell disorder resulting from a somatic mutation in a gene on the X
chromosome. The diagnostic test of choice is flow cytometry for the
glycophosphatidylinositol-linked proteins CD55 and CD59, which are characteristically
decreased on RBCs and granulocytes. Protein C and S and antithrombin III levels are
affected by acute thrombosis and anticoagulation and are therefore best assessed more
than 2 weeks after completing anticoagulation. A positive activated protein C resistance
assay is consistent with factor V Leiden. These other inherited thrombophilias would not
explain this patient’s hemolytic anemia or pancytopenia.

26. The correct answer is: D. Warfarin. This patient presents with arterial thrombosis and
elevated antiphospholipid antibodies on two occasions, confirming the diagnosis of
antiphospholipid syndrome. Long-term anticoagulation with warfarin has been shown to
be more effective for secondary thrombosis prevention in patients with antiphospholipid
syndrome than direct oral anticoagulants, especially for those patients with a history of
arterial thrombosis. Lifelong use of enoxaparin imposes significant costs and burden on
patients and is therefore usually reserved for patients with antiphospholipid syndrome who
are or become pregnant.

27. The correct answer is: D. No anticoagulation. While factor V Leiden does carry a
slightly increased risk of venous thromboembolism, in the absence of a history of a clot or
a high-risk circumstance (eg, major surgery), there is no indication for anticoagulation in
these patients, although use of venous thromboembolism prophylaxis at times of surgery
should be considered.

28. The correct answer is: B. No treatment. This patient has stage 0 chronic lymphocytic
leukemia , where the only abnormality is an elevated WBC count. Stage 1 is diagnosed
when lymph nodes are enlarged, stage 2 when spleen is enlarged, stage 3 when the patient
becomes anemic, and stage 4 when thrombocytopenic. Treatment typically does not begin
until stage 3, or if symptomatic from lymphadenopathy and/or splenomegaly.

29. The correct answer is: C. Microscopic cholesterol emboli. Eosinophilia that develops
during a hospitalization is typically secondary to a drug exposure or procedure. Typical
drugs are antibiotics, and the eosinophilia will resolve after discontinuation.
Postcatheterization cholesterol emboli may also cause secondary eosinophilia. The
vasculature of the kidneys is most vulnerable when this occurs.

30. The correct answer is: B. Chronic myelogenous leukemia. Basophilia is not typically
an isolated finding but rather needs to be put into context. It may be seen in any of the
preceding conditions; hence clinical context is critical for distinguishing etiology. The
presentation with elevated WBC count with excess mature granulocytes and preserved
differentiation, without symptoms of leukostasis or tumor lysis, is classic for chronic
myelogenous leukemia.

31. The correct answer is: A. Cytomegalovirus negative. This patient requires
cytomegalovirus (CMV)-negative RBCs, because he is a solid organ transplant recipient
on immunosuppression at risk for CMV infection. Leukoreduction dramatically reduces
the risk of febrile nonhemolytic transfusion reactions, human leukocyte antigen
alloimmunization, and CMV transmission. However, CMV and other viruses may be
transmitted in plasma, so CMV-negative are preferred for renal transplant patients.
Furthermore, a few viable lymphocytes remain after leukoreduction, so patients at risk for
transfusion-associated graft-versus-host disease, such as recipients of hematopoietic stem
cell transplants (HSCTs), require irradiated RBC.

32. The correct answer is: A. Consult the blood bank. This patient is experiencing a
delayed hemolytic transfusion reaction, which usually occurs 1 or 2 weeks after a blood
transfusion. Most delayed hemolytic transfusion reactions are characterized by
extravascular hemolysis and are clinically silent. However, patients with sickle cell

Freemedicalbooks4download
disease are more likely to be symptomatic because of mild intravascular hemolysis.
Consulting the blood bank will prompt an antibody screen to identify the specific RBC
antigen to which the patient has become sensitized, so future reactions can be prevented.
Transfusing another unit of packed RBCs before the antigen is identified could result in
another hemolytic transfusion reaction and so should be avoided. Infection is a less likely
cause of fever and jaundice in this patient given the positive Coombs test, so blood
cultures are not necessary. Finally, discharge home would be premature, because the
patient requires serial blood checks to confirm that the hemolysis is not worsening.

33. The correct answer is: C. Transfusion-associated circulatory overload. This patient
has transfusion-associated circulatory overload, for which she has many risk factors,
including history of heart failure and COPD, as well as age >85 years, female sex, and
small body habitus. This could have been prevented by a slower transfusion rate, reduced
transfusion volume, and diuretic administration prior to transfusion. The other transfusion
reactions listed as answer options occur during or soon after a transfusion rather than
hours later and are more likely to cause hypotension rather than hypertension.

34. The correct answer is: A. Allogeneic stem cell transplant. This patient has
myelodysplastic syndrome with symptomatic anemia and neutropenia with recurrent
cellulitis. However, she has very good cytogenetics with a low blast count, so she has a
very low revised International Prognostic Scoring System (IPSS-R) score of 1.5, which
corresponds to a median overall survival approaching 9 years. She should therefore be
treated with supportive care measures, including antibiotics and erythropoietin-stimulating
agents, as well as low-intensity therapies, such as azacitidine and decitabine. Allogeneic
stem cell transplant is reserved for patients with high or very high IPSS-R scores with a
good performance status.

35. The correct answer is: C. Pneumococcal vaccine every 5 years. Patients with
myelodysplastic syndrome should receive age-appropriate vaccinations, including yearly
influenza vaccines and every 5-year pneumococcal vaccines. However, they should not
receive live vaccines. Similarly, prophylactic antibiotics and G-CSFs have not been shown
to provide benefit and are therefore not recommended by National Comprehensive Cancer
Network (NCCN) guidelines.

36. The correct answer is: A. Allogeneic stem cell transplant. This patient has
myelodysplastic syndrome with symptomatic anemia and thrombocytopenia. He has poor
cytogenetics and a very high IPSS-R score of 8, which corresponds to a median overall
survival of under 1 year. He is medically fit with good performance status, so he is a good
candidate for allogeneic stem cell transplant, which offers the only hope of a cure.
Hypomethylating agents are more appropriate for patients with poor performance status.
His disease does not have an IDH1 or IDH2 mutation that could be targeted with
ivosidenib or enasidenib, respectively. Based on his adverse cytogenetics, intensive
chemotherapy is unlikely to achieve long-term benefit and is associated with a high
toxicity rate.

37. The correct answer is: B. Clopidogrel. The patient most likely has polycythemia vera,
and, although all patients should be started on low-dose aspirin, he also meets criteria for
phlebotomy and hydroxyurea. Polycythemia vera is diagnosed in men with Hgb >16.5
g/dL or hematocrit >49%, and in women with Hgb >16 g/dL or hematocrit >48% who do
not have other explanations (eg, hypoxemia, erythropoietin-producing syndromes,
dehydration). Roughly 95% of patients will have the activating JAK2 V617F mutation.
Patients should also have erythropoietin levels measured, because low erythropoietin
levels make polycythemia vera more likely. Treatment entails phlebotomy to goal
hematocrit <45%, low-dose aspirin, and hydroxyurea if high risk, which includes age ≥60
or prior thrombus.
Of note, Protein C and S levels are often low after a clotting event, so measuring
these levels at that time is not indicative of a deficiency. Repeat testing would have to be
performed after the deep vein thrombosis (DVT), although in this case, the patient has a
better explanation for his presentation.

38. The correct answer is: D. RBC transfusion. The patient has primary myelofibrosis,
which is likely secondary to lupus. Primary myelofibrosis may also arise from
polycythemia vera, essential thrombocythemia, other hematologic and solid cancers, and
toxins. Symptoms are typically secondary to anemia and subsequent splenomegaly. If the
patient has no symptoms and is not anemic, there is no indication for treatment. If the
patient were younger and had a poor prognosis, she may attempt more aggressive
treatment with an allogeneic stem cell transplant, the only potential cure. For an elderly
symptomatic patient like this one, however, supportive care, including blood transfusions,
is standard.

39. The correct answer is: D. Observation. The patient is <60 years old, has no history of
thrombosis, and his platelet count is <1 500 000/μL, so he has no indication for aspirin or
hydroxyurea.

40. The correct answer is: D. Cytogenetics: t(8;21), NPM1+. In addition to CBC with
differential, the diagnostic evaluation for acute leukemia includes peripheral smear and
bone marrow biopsy with flow cytometry and cytogenetics. The peripheral smear will
demonstrate anemia, thrombocytopenia, and circulating blasts, the origin of which can be
determined by flow cytometry (acute lymphoblastic leukemia versus acute myelogenous
leukemia). In addition, the presence of specific cytogenetic abnormalities, including
t(15;17), t(8;21), inv(16), or t(16;16), is sufficient for a diagnosis of acute myelogenous
leukemia regardless of the blast count. This patient has acute myelogenous leukemia
based on blasts showing myeloid markers, including myeloperoxidase, CD13, and CD33.
Cytogenetics and molecular abnormalities are key in helping determine the risk of
relapse in acute myelogenous leukemia. In the question, only translocation (8;21) and
NPM1 gene mutation positivity confer a favorable prognosis. Other markers of favorable
prognosis include inv(16), t(16;16), and biallelic CEBPA mutations. t(15;17) leads to a
fusion PML-retinoic acid receptor α (PML-RARA) oncoprotein and is diagnostic of acute
promyelocytic leukemia, a unique subset of acute myelogenous leukemia that is
associated with a very high cure rate on modern therapies including all-trans-retinoic acid
(ATRA) and arsenic trioxide (ATO). Deletions 5 and 7, 3q26 aberrations, t(6;9); 11q23
aberrations, and complex karyotype confer an unfavorable prognosis, as do the gene
mutations FLT3-ITD, MLL-PTD, TP53, and RUNX1. t(9;22) signifies the presence of
BCR-ABL fusion transcript, which is not found in acute myelogenous leukemia but

Freemedicalbooks4download
identifies a high relapse risk subset of acute lymphoblastic leukemia.

41. The correct answer is: A. Liposomal cytarabine and daunorubicin on days 1, 3, and
5. This patient has most likely developed cytotoxic therapy-related acute myelogenous
leukemia based on her history of breast cancer requiring chemotherapy. Therefore, the
most appropriate induction therapy for this patient is liposomal cytarabine and
daunorubicin on days 1, 3, and 5. Standard induction therapy would be indicated for
patients who are under the age of 60 years, or who are over the age of 60 years but eligible
for intensive chemotherapy, and who do not have targetable mutations. Choice E, with the
addition of midostaurin, describes the appropriate induction therapy for patients with
FLT3-mutated acute myelogenous leukemia. Choice D, with the addition of gemtuzumab
ozogamicin, describes the appropriate induction therapy for patients with CD33-positive
acute myelogenous leukemia. The reduced intensity therapy is indicated for patients who
are over the age of 60 years and require nonintensive chemotherapy due to medical
comorbidities or functional status. For patients under the age of 45 years, induction
therapy includes standard therapy or high-dose cytarabine (HiDAC) for 6 days followed
by idarubicin or daunorubicin for 3 days.

42. The correct answer is: D. Differentiation syndrome; treat with dexamethasone 10 mg
twice daily and supportive care. The most likely explanation for the patient’s acute
presentation is differentiation (all-trans-retinoic acid [ATRA]) syndrome, which occurs in
approximately 25% of patients who are treated with ATRA for acute promyelocytic
leukemia. Differentiation syndrome is a life-threatening complication of therapy with
differentiating agents while leukemic blasts are present. Signs and symptoms of
differentiation syndrome include fever, pulmonary infiltrates, shortness of breath, edema,
hypotension, and acute kidney injury. Although acute respiratory distress syndrome may
also be characterized by shortness of breath, hypoxemia and bilateral pulmonary
infiltrates, the diagnosis of differentiation syndrome is more likely in this patient, who is
undergoing induction therapy for acute promyelocytic leukemia with ATRA.
The treatment for differentiation syndrome includes both dexamethasone 10 mg
twice daily and supportive care with diuresis, vasopressors, oxygen support, and dialysis
as needed. Supportive care alone is not sufficient for this life-threatening condition.
Treatment with additional chemotherapy agents, such as daunorubicin, is not indicated,
because this syndrome indicates a response to therapy, rather than resistance to therapy or
worsening disease. Antibiotics are indicated if an infectious source is identified, but the
differentiation syndrome alone does not require treatment with antibiotics.

43. The correct answer is: B. If she discontinues the imatinib, she will have a 60%
chance of recurrence in 24 months. This patient has achieved a complete molecular
remission with imatinib-based chemotherapy, which occurs in about one-fourth of
patients. The official recommendation is to remain on imatinib indefinitely, but she will
not be able to conceive as imatinib is teratogenic. There are many tyrosine kinase inhibitor
discontinuation trials that all show an approximate 40% freedom from molecular relapse
rate at 2 years. Should she decide to discontinue imatinib, she will require an intensive
monitoring strategy for the next 24 months (Mahon FX, Réa D, Guilhot J, et al. Lancet
Oncol. 2010;11:1029).
44. The correct answer is: A. Blinatumomab. Blinatumomab is approved for patients with
CD19 positive relapsed or refractory B-cell acute lymphoblastic leukemia and was shown
to be superior to standard chemotherapy options with respect to remission rates as well as
improvement in overall survival (Kantarjian HM, Stein A, Gökbuget N, et al. NEJM.
2017;376:836).
Inotuzumab and tisagenlecleucel are also new options for patients with relapsed or
refractory acute lymphoblastic leukemia, but only a subset of the lymphoblasts are CD22
positive, and the patient is over 26 years of age, making her ineligible to receive
tisagenlecleucel.

45. The correct answer is: A. Acute myelogenous leukemia. This patient has acute
leukemia based on the presence of >20% circulating blasts in the peripheral blood.
Chronic myelogenous leukemia leads to increase in myeloid cells all along the
differentiation spectrum of the myeloid lineage and does not have high blast counts
(unless it is accelerated phase or blast crisis). Assignment of lineage is critical to
expedient management of these patients and is done by flow cytometry on peripheral
blood (if there is evidence of circulating blasts) or bone marrow. Myeloid blasts express
markers myeloperoxidase, CD13, CD33, CD117. Lymphoid blasts can be either B or T
cells and are identified by respective markers. B-cell blasts: CD19, CD20, CD22. T-cell
blasts: CD3, CD4, CD8.

46. The correct answer is: D. All of the above. This case demonstrates that patients with
acute leukemia may present with several complications from their disease or related
sequelae. In particular, high burden of disease with a very high blast count increases the
risk of leukostasis due to visceral microvascular sludging and occlusion, leading to end
organ dysfunction. This classically presents with respiratory and CNS-related symptoms,
and its treatment includes urgent initiation of cytoreductive therapy, leukapheresis, and
intensive critical care. This patient also shows lab evidence of spontaneous tumor lysis
with hyperkalemia, renal dysfunction, and elevated LDH. This often occurs in patients
with high disease burden and rapidly proliferative disease, leading to spontaneous tumor
cell lysis. Given the replacement of the normal immune system by the leukemic clone,
patients with acute leukemia are at a very high risk for infection, and all fevers should be
urgently investigated and patients should receive empiric broad-spectrum antibiotic
therapy.

47. The correct answer is: B. <50. Primary CNS lymphoma typically occurs when a
patient’s CD4 count drops below 50/mm3. Non-Hodgkin lymphoma (NHL) is an AIDS-
defining malignancy. Patients are also typically infected with Epstein-Barr virus.
Treatment involves intrathecal methotrexate, steroids, and possible radiation and/or
temozolomide. The regimen may necessitate significant myeloablation, to the point of
considering an autologous HSCT.

48. The correct answer is: C. Observation. This patient has asymptomatic chronic
lymphocytic leukemia , which does not warrant treatment. Chronic lymphocytic leukemia
is not treated unless patients have disease-related symptoms (eg, pain from
lymphadenopathy or hepatosplenomegaly) or have developed cytopenias. First-line
therapy would typically be ibrutinib.

Freemedicalbooks4download
49. The correct answer is: C. Diffuse large B-cell lymphoma (DLBCL). The patient has
undergone a Richter Transformation, a progression of chronic lymphocytic leukemia to
DLBCL. This occurs in roughly 5% of chronic lymphocytic leukemia patients and carries
a poor prognosis. Patients are then treated for DLBCL, rather than for the initial chronic
lymphocytic leukemia . DLBCL treatment often includes R-CHOP (rituximab,
cyclophosphamide, doxorubicin = hydroxydaunorubicin, vincristine = oncovin,
prednisone) with or without radiation.

50. The correct answer is: D. PET-CT scan; ESR; bone marrow biopsy. The most
important next step in diagnostic workup is excisional lymph node biopsy (not fine needle
aspiration because this will not reveal the surrounding architecture) with
immunophenotyping and cytogenetics to confirm the diagnosis. Because staging requires
identification of the number of lymph node regions involved, the presence of disease on
one or both sides of the diaphragm, and the presence or absence of involvement of
extralymphatic organs, full-body imaging is needed. PET-CT scan is preferred, since CT
alone will not reliably detect spleen and liver involvement. In addition, PET response to
treatment can be prognostic and can at times guide treatment. Although CBC, complete
metabolic profile, and LDH are not needed for staging, these laboratory values are used in
predictive calculators to estimate the likelihood of response to therapy and overall
survival, as is ESR. Bone marrow biopsy is indicated, as this would be a site of extranodal
disease. Head CT and/or MRI is not indicated in the absence of symptoms. HIV, human
T-cell leukemia lymphoma virus, and Epstein-Barr virus serologies can be considered to
rule out alternative etiologies of lymphadenopathy but are not required for the diagnosis or
risk stratification.

51. The correct answer is: A. ABVD (doxorubicin, bleomycin, vinblastine, dacarbazine)
and repeat PET-CT after 2 cycles. This patient has classical Hodgkin lymphoma, stage
2, nonbulky. The first-line therapy for stage 1-2 Hodgkin lymphoma is ABVD with or
without radiation therapy. Although she would, in theory, be considered for involved-site
radiation based on the disease being confined to only two sites, the potential risk of
secondary malignancy in the breast and/or cardiomyopathy as a consequence of
mediastinal radiation outweighs the potential benefit, particularly since chemotherapy
alone is curative in more than half of patients. PET-CT after the first two cycles is
recommended for restaging. Patients with progressive disease at this time might be
considered for escalation of therapy to a more aggressive regimen such as BEACOPP,
while those with remarkable response to the first two cycles would be candidates for
elimination of bleomycin for the remaining four cycles in order to reduce pulmonary
toxicity. It would not be appropriate to plan for escalated BEACOPP prior to reimaging in
early-stage disease, making choice C incorrect. Both brentuximab-vedotin and
pembrolizumab are active agents for Hodgkin disease but are currently only approved for
relapsed disease and would not be appropriate for upfront therapy. Combination
chemotherapy with ABVD is standard upfront therapy for Hodgkin lymphoma, and there
is no role for radiation therapy alone (choice D) in this patient.

52. The correct answer is: A. CXR, echocardiogram, pulmonary function tests. The late
effects after treatment for Hodgkin lymphoma include increased risk for second cancers
(approximately 4.6x risk for up to 40 years), particularly breast (if received radiation
therapy), lung, and hematologic malignancies. Because this patient did not undergo chest
or neck radiation, her risk of breast cancer is relatively closer to average, while her risk of
lung cancer, acute leukemia, myelodysplastic syndrome, and non-Hodgkin lymphoma
remains elevated. CBC with differential is done to evaluate for hematologic malignancy.
CXR should be performed to evaluate for lung cancer given her symptoms. As she is at
increased risk for cardiac and pulmonary disease because she received 6 cycles of both
doxorubicin (cardiotoxic) and bleomycin (pulmonary toxicity), she should therefore
undergo echocardiogram and pulmonary function testing. Although hypothyroidism could
contribute to fatigue and shortness of breath in some cases, the risk of thyroid disease in
this patient is similar to that of the general population, because the increased risk among
patients with Hodgkin lymphoma applies only to those who receive neck radiation.

53. The correct answer is: A. Quadruple therapy for treatment of the H. pylori (bismuth,
metronidazole, tetracycline, omeprazole). This patient has a type of mucosa-associated
lymphoid tissue (MALT) lymphoma, in this case, H. pylori associated. More than 75% of
patients with H. pylori–associated early-stage gastric MALT lymphoma will have
regression of their disease and a durable long-term response with treatment of the H.
pylori alone. If the patient had persistent disease after eradication of H. pylori, radiation
therapy or rituximab monotherapy would be appropriate. Similarly, if the patient had no
evidence of H. pylori at diagnosis, it would be appropriate to proceed with radiation or
rituximab as first-line therapy.

54. The correct answer is: B. Bone marrow biopsy with cytogenetics and gene mutation
analysis; skeletal survey. This patient’s initial presentation with hypercalcemia, renal
disease, anemia, and bone pain along with a large monoclonal IgG κ is consistent with a
diagnosis of multiple myeloma. The criteria for diagnosing multiple myeloma include
clonal bone marrow plasma cells ≥10% and at least one myeloma-defining event, which
can include either myeloma-related organ or tissue impairment (lytic bone lesions,
calcium >11 mg/dL, creatinine >2 mg/dL, or Hgb <10 g/dL), or any one of several
biomarkers, including bone marrow plasma cells ≥60%, serum free light chain ratio
≥100:1, or more than one focal lesion on MRI studies. This patient has already undergone
serum testing consistent with multiple myeloma but requires a bone marrow biopsy with
cytogenetics and gene mutation analysis to help guide treatment and stratify risk. In
addition, his complaint of hip pain is concerning for lytic bone lesion that may be at risk
for pathologic fracture, and therefore a skeletal survey is indicated for further workup.
PET-CT is not a useful diagnostic test for multiple myeloma and is more commonly used
in diagnosing lymphoma. β2-microglobulin level and serum albumin should also be sent
for staging purposes. In this patient, who meets several diagnostic criteria for multiple
myeloma, it would not be appropriate to defer further workup and repeat testing in 6
weeks or 6 months.

55. The correct answer is: B. 1% per year; 25% lifetime risk. This patient has a diagnosis
of κ light chain monoclonal gammopathy of unknown significance (MGUS). This
diagnosis requires an abnormal free light chain ratio in the absence of lytic bone lesions,
renal failure, or hypercalcemia. This patient should undergo a skeletal survey to confirm
that there are no bony lesions. The prevalence of MGUS in the population is about 3% in
patients over age 50, 5% in patients over age 70, and 7.5% in patients over age 85. The

Freemedicalbooks4download
prognosis is favorable for most patients with MGUS, because there is a 1% per year risk
of progression to multiple myeloma, Waldenstrom macroglobulinemia, or a malignant
lymphoproliferative disease and a 25% lifetime risk. Patients should be followed closely
after diagnosis with repeat SPEP in 6 months and then yearly thereafter if stable.

56. The correct answer is: D. Hyperviscosity syndrome. This patient’s overall clinical
presentation with anemia, elevated monoclonal IgM spike, and cryoglobulinemia is most
consistent with a diagnosis of Waldenstrom macroglobulinemia, which is a B-cell
neoplasm (lymphoplasmacytic lymphoma) that secretes monoclonal IgM. Approximately
90% of these patients will have MYD88 L265P mutations. In addition, patients with
Waldenstrom macroglobulinemia should not have lytic bone lesions. Clinical
manifestations can include amyloidosis and glomerulopathy due to IgM deposition in the
skin, intestines, and kidney, although this patient does not have any signs consistent with
this. Patients can also have chronic autoimmune hemolytic anemia and peripheral
neuropathy due to autoantibody activity of IgM, although also not present in this patient.
Type I cryoglobulinemia, leading to Raynaud phenomenon and vasculitis, is also present
in this patient, but does not explain the neurologic and pulmonary symptoms. This
patient’s symptoms are most consistent with hyperviscosity syndrome, which occurs in
about 15% of patients with Waldenstrom macroglobulinemia. Symptoms will typically
present when the relative serum viscosity is >5 or 6. The symptoms include blurred vision,
headache, dizziness, change in mental status, congestive heart failure, and pulmonary
infiltrates. The management is plasmapheresis.

57. The correct answer is: B. Start bendamustine + rituximab. Indications for treatment in
patients with Waldenstrom macroglobulinemia include hyperviscosity, neuropathy,
organomegaly, amyloidosis, cold agglutinin disease, cryoglobulinemia, cytopenias related
to the disease, and bulky adenopathy. For asymptomatic patients with incidentally
diagnosed Waldenstrom macroglobulinemia, observation with close laboratory and
clinical follow-up is appropriate. However, this patient has symptoms of peripheral
neuropathy, most likely IgM-related, and requires treatment. Patients with Waldenstrom
macroglobulinemia and peripheral neuropathy can have antibodies to myelin-associated
glycoprotein or others that contribute to the development of neuropathy, unrelated to
serum viscosity. This patient does not have symptoms or laboratory evidence of
hyperviscosity, and therefore plasmapheresis, choice E, is not indicated. There are several
preferred regimens for initial treatment of Waldenstrom macroglobulinemia, including
bendamustine/rituximab, bortezomib/dexamethasone/rituximab, and
rituximab/cyclophosphamide/dexamethasone. Bortezomib would be an inappropriate
choice for this patient, because it can worsen symptoms of neuropathy. Autologous stem
cell transplant is recommended frequently for patients with multiple myeloma but is not
indicated as first-line treatment for Waldenstrom macroglobulinemia.

58. The correct answer is: E. Smoldering multiple myeloma. This patient has smoldering
multiple myeloma, which is diagnosed in patients with M protein >3 g/dL and/or 10% to
60% bone marrow clonal plasma cell infiltrate, without myeloma-related organ or tissue
impairment or amyloidosis. The workup for this patient demonstrates an elevated M
protein and >10% bone marrow plasma cell infiltration but absence of lytic lesions,
anemia, hypercalcemia, and renal insufficiency. The findings are inconsistent with
monoclonal gammopathy of unknown significance (MGUS), which requires M protein <3
g/dL and marrow plasmacytosis <10%. Nonsecretory multiple myeloma is diagnosed in
patients with no M protein, but marrow plasmacytosis and myeloma-related organ or
tissue impairment. The patient does not have clinical features or pathologic findings
consistent with amyloidosis.

59. The correct answer is: D. Referral to radiation oncology for radiation to the right
humerus osseous lesion. This patient has a solitary bone plasmacytoma, which is defined
as a lytic lesion without plasmacytosis or other myeloma-related organ or tissue
impairment, including other bone lesions, calcium >11 mg/dL, creatinine >2 mg/dL, or
Hgb <10 mg/dL, with SPEP negative for M spike. Because the disease is localized, the
most appropriate next step in management would be either radiation or surgical
reconstruction. In this case, radiation would be appropriate. Observation and referral to
physical therapy would be inappropriate, because the patient would be at risk for
worsening pain and possibly pathologic fracture. Although choices A, B, and E are
appropriate systemic therapies to consider in multiple myeloma, they are not indicated in
the case of a solitary plasmacytoma.

60. The correct answer is: A. Bortezomib/lenalidomide/dexamethasone. Induction


regimens with the best response rate for multiple myeloma combine proteasome inhibitors
(bortezomib, carfilzomib) with immunomodulators (lenalidomide). Other active drugs
include prednisone, dexamethasone, melphalan, and cyclophosphamide. A patient’s
candidacy for transplant is important to establish prior to induction, as the preferred
regimens differ for patients who may go on to receive a transplant. However, stem cell
transplant is not recommended as initial induction therapy for patients with multiple
myeloma. Autologous stem cell transplant is recommended for some patients, after they
undergo induction, in order to allow for high-dose consolidative therapy. Allogeneic stem
cell transplant is rarely used for patients with multiple myeloma. Common induction
regimens for transplant candidates include triplets, such as
bortezomib/lenalidomide/dexamethasone, bortezomib/cyclophosphamide/dexamethasone,
or carfilzomib/lenalidomide/dexamethasone, most of which combine proteasome
inhibitors with immunomodulators. Pomalidomide/cyclophosphamide/dexamethasone,
choice D, is indicated only for patients with previously treated, relapsed, or refractory
multiple myeloma.

61. The correct answer is: D. Myeloablative conditioning followed by allogeneic


transplant from his sibling. The patient has a high risk of relapse based on the presence
of the FLT3-ITD mutation. Given the high risk of disease relapse and availability of a
fully matched sibling donor, an allogeneic transplant is the best treatment strategy for him.
In a young patient with high-risk acute myelogenous leukemia without any comorbidities,
myeloablative conditioning is tolerable and reduces the risk of leukemia recurrence in
addition to the antileukemic effects from graft versus leukemia effect of the allogeneic
transplant itself. Option B is a reasonable approach for consolidation therapy for patients
with good-risk acute myelogenous leukemia. FLT3 inhibitors have shown improvement in
overall survival when added to standard induction chemotherapy followed by maintenance
dosing. However, given the patient’s age, availability of fully matched sibling donor, and
high risk of disease recurrence, an allogeneic transplant would be the best choice for

Freemedicalbooks4download
consolidation therapy.

62. The correct answer is: C. Steroids. This patient has developed engraftment syndrome,
which occurs typically 1 to 4 days after absolute neutrophil count (ANC) >500. Major
characteristics are fever, noncardiogenic pulmonary edema, and erythrodermatous rash. If
only two of the prior criteria are present, diagnosis can be supported by renal dysfunction,
hepatic dysfunction, encephalopathy, or unexplained weight gain. Treatment of
engraftment syndrome entails steroids 1 mg/kg, rapidly tapered over 3 to 4 days.

63. The correct answer is: A. Acute graft-versus-host disease (GVHD). The combination
of diarrhea, maculopapular rash, and liver dysfunction makes acute GVHD the most likely
diagnosis. Acute GVHD occurs typically within 6 months of transplant and is
characterized by involvement of these three systems. Severity is graded based on
percentage of body area involved by rash, level of bilirubin elevation, and volume of
diarrhea. The mismatched related peripheral stem cell transplant increased the patient’s
risk for GVHD. This patient’s symptoms and labs are all mild (grade 1), but he will likely
need a biopsy to support the gastrointestinal (GI) GVHD diagnosis, which will then be
managed with increased immunosuppression. (Of note, if <25% of his skin were involved,
he could be initially managed with topical steroids alone.)

64. The correct answer is: D. Sinusoidal obstruction syndrome. This patient is presenting
with hepatic sinusoidal obstruction syndrome. The diagnosis is typically made based on a
clinical syndrome of serum bilirubin >2 mg/dL, hepatomegaly or right upper quadrant
pain, and sudden weight gain >2% baseline body weight. Although several conditions
predispose patients to the risk of sinusoidal obstruction syndrome, patients who have
undergone hematopoietic cell transplants are at increased risk, particularly within the first
3 weeks. Diagnosis must also exclude other causes of hepatic failure, including infection,
graft-versus-host disease, Budd-Chiari, and ischemia.

65. The correct answer is: A. Fluorodeoxyglucose (FDG) PET/CT, and brain MRI;
pending results, follow with mediastinal lymph node evaluation with endobronchial
ultrasound, mediastinoscopy. The key information required to make a treatment
determination here is the stage of the cancer. Hence, the initial workup focuses on
evaluation of distant metastatic disease with a PET/CT and brain MRI and, if negative, to
fully evaluate local extent of tumor within the mediastinum. The latter is critical in
helping decide the approach to treating locally advanced tumors, including the role of
radiation and feasibility of surgical approaches.

66. The correct answer is: D. Osimertinib alone. Deletions in epidermal growth factor
receptor (EGFR) exon 19 are activating mutations and these are predominantly found in
nonsmokers with lung cancer. Presence of EGFR mutations is predictive of response to
EGFR inhibitors which are more efficacious and less toxic than standard cytotoxic
chemotherapy. In this patient, with stage 4 non–small cell lung cancer with EGFR-
positive mutation analysis, the most appropriate first-line therapy is osimertinib, a third-
generation EGFR tyrosine kinase inhibitor. Osimertinib has shown more efficacy than
earlier generation inhibitors (erlotinib, gefitinib) in the upfront setting, and this is thought
to be related to its activity against both canonical EGFR mutations found in treatment-
naïve patients (L858R and exon 19 deletions) and T790M resistance mutations found in
almost 50% of patients who progress on early generation EGFR inhibitors. Additionally,
osimertinib has better brain penetration than earlier generation EGFR inhibitors.
Osimertinib should not be combined with carboplatin and pemetrexed as first-line therapy.
Crizotinib is also a tyrosine kinase inhibitor, but targets ROS1 and should be used in
patients with ROS1 mutation. Pembrolizumab alone and
carboplatin/pemetrexed/pembrolizumab can be used for patients with PD-L1 staining
>50% without other targetable mutations.

67. The correct answer is: C. Carboplatin, paclitaxel, and pembrolizumab. For patients
with metastatic squamous cell carcinoma, PD-L1 staining is recommended. For those with
>50% staining, pembrolizumab alone is the first-line therapy. For those with PD-L1
<50%, the first-line therapy is carboplatin/paclitaxel plus pembrolizumab.

68. The correct answer is: D. Chest CT. This patient has a smoking history and significant
hyponatremia. Although sodium studies are indicated, this patient also has a concerning
story for a paraneoplastic syndrome secondary to a lung small cell carcinoma, so a chest
CT is warranted.

69. The correct answer is: B. In 5 years. Currently, US Preventive Services Task Force
(USPSTF) recommends annual low-dose CT for individuals aged 55 to 80 with a 30-pack-
year smoking history who currently smoke or quit within the past 15 years.

70. The correct answer is: C. Tamoxifen. Tamoxifen is the standard systemic therapy for
low-risk, early-stage ER+ breast cancer in premenopausal women. An aromatase inhibitor
such as anastrozole would be first-line therapy for postmenopausal women. In
premenopausal women, however, aromatase inhibitors do not block ovarian production of
estrogen and so can only be used with concurrent medical or surgical ovarian suppression.
Combined aromatase inhibition and ovarian suppression is more effective than tamoxifen
alone in younger women with high risk of disease (ie, involved lymph nodes, high tumor
grade, large tumor size, high Oncotype DX score). Adjuvant endocrine therapy for 10
years is generally superior to treatment for 5 years. However, the disease-free and overall
survival benefits of extended therapy are small for patients, such as this woman, with low-
risk disease, so therapy could be stopped before 10 years if she experiences adverse
effects, such as intolerable hot flashes or thrombosis.

71. The correct answer is: C. Lapatinib. Options for metastatic hormone receptor–positive,
HER2-negative breast cancer include aromatase inhibitors (eg, anastrozole), CDK 4/6
inhibitors (eg, palbociclib), and mammalian target of rapamycin (mTOR) inhibitors (eg,
everolimus) added to aromatase inhibitors (eg, exemestane). Lapatinib is a reversible
HER2 inhibitor, which is not indicated in HER2-negative disease.

72. The correct answer is: A. Docetaxel, cyclophosphamide, trastuzumab, pertuzumab.


HER2+ breast cancer is treated with HER2-targeted therapy. These agents include the
monoclonal antibodies trastuzumab and pertuzumab, tyrosine kinase inhibitors, including
lapatinib and neratinib, and the antibody-drug conjugate trastuzumab emtansine (T-DM1).
In the adjuvant setting, docetaxel (Taxotere), cyclophosphamide, trastuzumab (Herceptin),

Freemedicalbooks4download
pertuzumab (TCHP) is now considered standard therapy based on the Adjuvant
Pertuzumab and Herceptin in Initial Therapy of Breast Cancer (APHINITY) trial (NEJM
2017). Letrozole, an aromatase inhibitor, plus ribociclib, a CDK4/6 inhibitor, is an
appropriate first-line therapy for metastatic hormone receptor–positive, HER2-negative
breast cancer.

73. The correct answer is: B. BRCA2. Although both BRCA1 and BRCA2 increase the risk
of breast and ovarian cancer, there is a greater association of pancreatic cancer with
BRCA2 mutations. Of note, BRCA mutated cancers are often more sensitive to therapies
that target the homologous recombination repair pathway, including platinum therapy and
poly (ADP-ribose) polymerase (PARP) inhibitors. PARP inhibitors are approved for
metastatic breast cancer with germline BRCA1/2 mutations.

74. The correct answer is: D No further mammograms indicated. The USPSTF
recommends biennial mammogram screening for women aged 50 to 74. In the absence of
prior abnormal mammograms and no current breast complaints, there is no indication for
this patient to continue regular mammogram screening.

75. The correct answer is: D. Starting treatment with immunotherapy in combination
with chemotherapy first line. In March 2019, the Food and Drug Administration (FDA)
granted accelerated approval for first-line atezolizumab in combination with
chemotherapy for PD-L1-positive patients with triple-negative breast cancer.

76. The correct answer is: C. Degarelix and abiraterone/prednisone. This patient has
newly diagnosed hormone-sensitive metastatic prostate cancer. The backbone of therapy
for hormone-sensitive metastatic prostate cancer is androgen deprivation therapy, either
with a long-acting luteinizing hormone–releasing hormone (LHRH) agonist (eg,
leuprolide) or LHRH antagonist (eg, degarelix). The choice of androgen deprivation
therapy is variable between experts. However, leuprolide alone is associated with a
transient androgen burst, given its agonist activity on the LHRH receptor, and can hence
worsen symptoms or precipitate crises in patients with high-volume disease. Hence, either
it is phased into treatment after pretreating with an androgen receptor blocker
(bicalutamide) for a couple of weeks or treatment is initiated with a direct antagonist of
the LHRH receptor, which is not associated with the androgen surge. Several would favor
the latter approach, particularly in patients with a high volume of disease such as this
patient, although this is a more expensive therapy.
We have recently learned from several clinical trials that the addition of newer
generation anti androgen abiraterone (CYP17 lyase inhibitor) or chemotherapy in the
form of docetaxel to androgen deprivation therapy improves overall survival in patients
with hormone-sensitive metastatic prostate cancer (especially those with high disease
burden). Given this patient’s age, multiple comorbidities, and poor performance status, he
is not a good candidate for docetaxel chemotherapy. Hence, degarelix with
abiraterone/prednisone is the best option among those listed.

77. The correct answer is: C. Prostate MRI. The patient has a persistently elevated PSA,
which could be secondary to prostate cancer. He is 67 years and, without any major
medical issues, he has a long life expectancy. In such patients, diagnosing early prostate
cancer can help prevent future cancer-related mortality. A few years ago, the only way to
investigate this PSA further was a transrectal ultrasound (TRUS)-guided 12-core biopsy.
Recent advances in prostate imaging with MRI now allow visual identification of any
potential pathologic lesions. In a recent study (Kasivisvanathan V, Rannikko, A., Borghi,
M, et al. NEJM. 2018;378:1767) prostate MRI was compared head to head with standard
TRUS-guided prostate biopsy in patients with a similar epidemiologic profile as our
patient. Approximately 30% of patients in the study were able to forego a prostate biopsy
because the MRI revealed no concerning lesions. Patients with visualized lesions on the
MRI underwent a more focused biopsy rather than the blind 12-core biopsy. Not
surprisingly, a higher percentage of patients undergoing prostate MRI were diagnosed
with clinically significant prostate cancer (Gleason grade ≥7) compared with standard
biopsy given the targeted approach.

78. The correct answer is: A. Active surveillance. Active surveillance is a strategy to avoid
or delay treating patients with low-risk prostate cancer that is clinically insignificant and
unlikely to pose a health issue to them for a long time. Active surveillance is, as the name
suggests, an active strategy with close follow-up and testing, including repeat PSA
measurements, prostate biopsies, and/or MRI, to identify patients with early progression
for treatment and hence not reduce the chance of cure while avoiding therapy in patients
with truly low-risk disease. Choices C and D are standard of care localized clinically
significant prostate cancer but have associated toxicity. In a patient with low-risk Gleason
6 prostate cancer from a targeted MRI-guided biopsy, active surveillance is a reasonable
approach as long as he understands that if the disease is seen to be progressing (elevation
in PSA, change to higher Gleason grade), then treatment with choices C or D would be
appropriate.

79. The correct answer is: A. Colonoscopy at age 20, then repeat screen yearly. This
patient likely has Hereditary nonpolyposis colorectal cancer (HNPCC) based on his family
history: ≥3 family members with HNPCC-associated cancers, two generations affected,
and at least one diagnosis before age 50. This was confirmed by germline mutation
testing, which showed a mutation in PMS2, one of the components of the mismatch repair
pathway. Other components of the mismatch repair pathway that can be mutated in
HNPCC include MLH1, MSH6, PMS2. Patients with this familial syndrome should
undergo genetic counseling, begin screening at age 20 to 25 years, and continue with
screening colonoscopies every 1 to 2 years.

80. The correct answer is: D. A and B. In patients with a known colon cancer diagnosis,
Carcinoembryonic antigen (CEA) is measured in order to subsequently measure response
to therapy. Of note, CEA is not a screening tool. Patients then undergo CT
chest/abdomen/pelvis to look for metastatic disease. This disease preferentially
metastasizes to the liver, lungs, and peritoneum in that order. Brain metastases are less
common, so head imaging is not indicated unless neurologic symptoms are present.

81. The correct answer is: B. Familial breast/ovarian cancer BRCA2. Although all
syndromes listed can lead to a hereditary form of pancreatic cancer, this particular pattern
of cancers of the breast, ovary, and prostate point toward BRCA2 mutations. The
penetrance for developing pancreatic cancer in patients with BRCA2 inherited mutations

Freemedicalbooks4download
is lower than that of breast and ovarian cancers.

82. The correct answer is: E. Poly (ADP-ribose) polymerase (PARP) inhibitors. PARP
inhibitors target poly (ADP-ribose) polymerase. BRCA2 functions in the homologous
recombination DNA repair pathway and tumors that have lost BRCA2 (and hence
homologous recombination deficient tumors) are exquisitely sensitive to loss of PARP,
which is normally involved in repairing single-strand breaks. This exquisite sensitivity of
BRCA null tumor cells to PARP inhibition is known as synthetic lethality, a phenomenon
where a defect in either of two genes has little effect on the cell or organism but a
combination of defects in both genes results in death. Hence, nontumor cells in the patient
(which have not lost the second copy of BRCA2) do not suffer the same toxicity to PARP
inhibition as tumor cells. Recently, maintenance PARP inhibitor therapy with olaparib
was shown to prolong disease progression in metastatic pancreatic cancer patients who
had a genomic BRCA2 mutation. Because this patient’s main toxicity is neuropathy
(which is secondary to oxaliplatin), continuing chemotherapy with FOLFOX (5FU/LV +
oxaliplatin) or a regimen consisting of abraxane (also causes neurotoxicity) would likely
worsen these symptoms. Regarding option D, immunotherapy, specifically checkpoint
blockade with PD-1 inhibitors, has not shown efficacy in pancreatic cancers except for a
very small subset of patients (~1% or less) who have pancreas cancer secondary to Lynch
syndrome, leading to MSI-H (microsatellite unstable). Because the patient has a good
performance status and an excellent response to upfront FOLFIRINOX, the option
(Option C) for therapy hospice at this point would not be appropriate.

83. The correct answer is: D. Pancreatic protocol CT scan or MRI with contrast
followed by endoscopic ultrasound (EUS)-guided fine needle aspiration (FNA).
Pancreatic protocol CT scan with intravenous contrast, including arterial and venous
phase imaging or MRI with contrast, is the most important next step in diagnosis. If no
lesion is seen, the next step would be EUS, or magnetic resonance
cholangiopancreatography (MRCP). Following imaging to define the lesion size and
location, EUS-guided FNA is the preferred modality to obtain tissue diagnosis. Although
it is useful to check CA 19-9 preoperatively and trend postoperatively to assess for
recurrence, this is not an essential test in diagnosis. Abdominal ultrasound is useful for
assessing the liver, gallbladder, and kidneys but less sensitive for pancreatic lesions.

84. The correct answer is: D. Referral to multidisciplinary team for management of
HCC. Hepatocellular carcinoma is one of the rare tumors that can be diagnosed without a
biopsy in patients with liver cirrhosis and characteristic findings on multiphasic liver CT
scan or contrast-enhanced liver MRI. The American College of Radiology has established
LI-RADS (Liver Imaging Reporting and Data System) to standardize the interpretation of
surveillance liver imaging for hepatocellular carcinoma. LI-RADS category 5 implies
definitive Hepatocellular carcinoma, and these patients do not need a biopsy for
confirmation. LI-RADS 5 criteria for masses >2 cm include arterial phase
hyperenhancement and one of the following: (1) Washout appearance (nonperipheral). (2)
Enhancing capsule. (3) Threshold growth = size increase of a mass by ≥50% in ≤6
months.

85. The correct answer is: D. Transcatheter arterial chemoembolization (TACE). TACE
is a minimally invasive technique of locally delivering chemotherapy (coated on gel
beads) into the hepatic artery branches feeding the tumor and represents a reasonable
approach to palliate unresectable locally advanced nonmetastatic hepatocellular
carcinoma. However, TACE is generally contraindicated in patients with portal vein
thrombosis of the main vein or first order right or left branches because of high risk of
hepatic insufficiency from post-TACE ischemic liver injury. Patients with hepatocellular
carcinoma and associated portal vein thrombosis have a worse overall prognosis and, in
general, are not considered candidates for liver transplantation, surgical resection, and
TACE. Stereotactic body radiation therapy (SBRT), a type of highly focused radiation
therapy, can still be used in such cases, but is unlikely to be helpful given the multiple
satellite nodules.

86. The correct answer is: C. Liver MRI. A diagnosis of hepatocellular carcinoma can be
made via a 3-phase contrast-enhanced abdominal CT or MRI. CT or MRI is indicated if
either a mass is found on ultrasound or if α-fetoprotein levels are found to be increasing.

87. The correct answer is: B. Chest/abdomen CT with contrast. This patient has a new
diagnosis of chronic myelogenous leukemia with increased creatinine, phosphorous, and
uric acid, as well as borderline high potassium, all consistent with impending tumor lysis
syndrome. A chest/abdomen CT will not help diagnose chronic myelogenous leukemia,
and contrast should be avoided in the setting of renal dysfunction associated with tumor
lysis syndrome. Instead, a bone marrow biopsy is necessary to confirm the diagnosis of
chronic myelogenous leukemia. Rasburicase is appropriate to treat this patient’s elevated
uric acid, and hydroxyurea is helpful for cytoreduction while awaiting diagnostic
confirmation, although it requires close monitoring for worsening tumor lysis syndrome.
Intravenous fluids are helpful for tumor lysis syndrome prevention.

88. The correct answer is: C. Nasogastric suction. This patient has neutropenic
enterocolitis, in which microbial infection in the setting of severe neutropenia has led to
necrosis of various layers of the bowel wall. Nasogastric suction should be considered as a
supportive therapy, as well as bowel rest, intravenous fluids, and nutritional support.
Morphine, loperamide, and prochlorperazine are opioid, antidiarrheal, and anticholinergic
agents, respectively, that may aggravate ileus and so should be avoided. Although a
surgery consult may be appropriate, surgical intervention should similarly be avoided in
this neutropenic and thrombocytopenic patient, unless she clinically deteriorates or
develops perforation with free air or persistent GI bleeding despite medication
interventions.

89. The correct answer is: B. Arsenic trioxide (ATO) and all-trans-retinoic acid (ATRA).
This patient has a presentation concerning for acute promyelocytic leukemia, given the
easy bruising and the Auer rods. Such patients have a very high cure rate, but mortality
typically occurs early if the condition is not identified. Roughly 90% of these patients will
present with disseminated intravascular coagulation with evidence of bleeding, and the
primary cause of death is intracranial hemorrhage. This patient should receive ATO and
ATRA, and her labs should be monitored closely to keep her platelets >10 000/μL,
fibrinogen >100 mg/dL, and normal PT/PTT/INR.

Freemedicalbooks4download
90. The correct answer is: D. Stat MRI spine. The patient is exhibiting signs of spinal cord
compression, an oncologic emergency. Although surgery may ultimately be indicated, this
cannot be performed before imaging confirmation. The patient must thus undergo
immediate spine MRI. He may also receive high-dose steroids to manage sequelae.

91. The correct answer is: A. Dihydropyrimidine dehydrogenase deficiency. Up to 80%


of 5-FU (and its oral prodrug capecitabine) is degraded by liver enzyme
dihydropyrimidine dehydrogenase into inactive metabolites. Polymorphisms in
dihydropyrimidine dehydrogenase that lead to partial or complete deficiency of enzyme
activity are associated with increased exposure to 5-FU and its active metabolites and can
present with serious hematologic, GI, CNS, and dermatologic toxicity. Uridine triacetate,
a pyrimidine derivative, competitively inhibits incorporation of 5-FU in RNA and is an
approved agent to be given within 96 hours of completion of treatment with either 5-FU or
capecitabine in cases of severe and unexpected toxicity. Symptoms present are not
routinely seen after FOLFOX, which is given on an outpatient basis without G-CSF
support in most cases. Although option B is possible, dihydropyrimidine dehydrogenase
deficiency needs to be considered first.

92. The correct answer is: C. Infliximab. This patient has severe checkpoint inhibitor-
induced colitis, which is refractory to steroids. Checkpoint inhibitors de-repress the
immune system to generate antitumor effect. However, autoimmune side effects can occur
with these agents. In particular, use of dual checkpoint inhibitors targeting both the
CTLA4 and the PD-1 axis can have more severe toxicities and more unusual toxicities not
seen with monotherapy. In this case, the timing of the diarrhea in relation to the treatment
duration fits well with occurrence of checkpoint inhibitor–induced colitis. Infection is the
most important differential diagnosis and has been ruled out with stool cultures, and C.
difficile toxin. Cytomegalovirus colitis can often occur in these patients who tend to be
immunocompromised and can often be missed. Cytomegalovirus colitis was also
evaluated on the colonic biopsy specimen and was negative. Hence, given the timing of
the symptoms, and findings on sigmoidoscopy and active colitis on biopsy, there is very
little doubt as to this being checkpoint inhibitor–induced colitis. The first-line treatment
for severe checkpoint-induced colitis is parenteral steroids. A subset of patients can be
refractory to steroids and are treated with anti–tumor necrosis factor-α antibody such as
infliximab.

93. The correct answer is: C. Brain MRI. Although there are many infectious and
autoimmune causes of hypotension in a patient on immunotherapy, the combination of
thyroid, adrenal, and gonadal abnormalities suggests a problem high in their respective
axes. Hypophysitis is a rare but serious side effect of ipilimumab and nivolumab, the
chances of which are increased with combination therapy. The clinical syndrome raises
suspicion, which can be confirmed with MRI. Meanwhile, immunotherapy agents should
be held, and, given the severe hemodynamic risk, the patient should also receive steroids.

94. The correct answer is: D. Tocilizumab. When the patient’s blood pressure first dropped
but was fluid responsive, she had developed grade 2 cytokine release syndrome. When she
was no longer fluid responsive, this became grade 3 cytokine release syndrome. Currently,
the interleukin-6 inhibitor tocilizumab is indicated at grade 3 cytokine release syndrome,
although practices are moving toward grade 2. Steroids are avoided if possible, given they
have not been definitely shown to not interfere with the efficacy of chimeric antigen
receptor T-cell. Tumor necrosis factor (TNF)-α inhibitors like infliximab are sometimes
given for colitis, and mycophenolate mofetil is sometimes given for hepatitis.

95. The correct answer is: E. A, B, C. The patient has developed chimeric antigen receptor
T-cell–related encephalopathy syndrome (CRES), which is characterized by cerebral
edema, delirium, aphasia, seizures, and, potentially, death. If patients progress to the
severity of seizure and intubation, they should be treated with antiepileptics and steroids.
Tocilizumab is indicated for cytokine release syndrome but not for CRES.

Freemedicalbooks4download
6
INFECTIOUS DISEASES

QUESTIONS

1. A 78-year-old woman with a history of chronic obstructive pulmonary disease (COPD),


hypertension, and gastroesophageal reflux disease presents with 4 days of productive
cough, fever, and malaise. She feels short of breath when she walks. Her granddaughter
recently had a cold, but she cannot recall any other sick contacts in the assisted living
facility where she resides. She has not received any antibiotics in the last 3 months. On
physical examination, her temperature is 37.6°C, pulse 90/min, respirations 32
breaths/min, and blood pressure (BP) 150/87 mmHg. Her examination is notable for
diminished lung sounds and crackles in the right lower base, without egophony or
wheezing. Her laboratory findings demonstrate a white blood cell (WBC) count of 16 800/
μL with 89% neutrophils, 1.2% bands, 8% lymphocytes, erythrocyte sedimentation rate
(ESR) 48 mm/h, C-reactive protein 33 mg/L, platelets 90 000/μL, and creatinine 0.6
mg/dL. Chest radiograph (CXR) has stable emphysematous changes and evidence of right
lower lobe and right middle lobe opacification.
What is the most likely diagnosis?
A. Acute exacerbation of bronchiectasis
B. Aspiration pneumonitis
C. COPD exacerbation
D. Severe community-acquired pneumonia

2. If the patient in Question 1 were immunocompromised, which of the following tests


should NOT be used?
A. Blood cultures
B. Procalcitonin
C. Streptococcus pneumoniae urinary antigen
D. Sputum Gram stain and culture
E. Viral respiratory testing

3. What would be the appropriate empiric therapy for the patient in Question 1?
A. Ceftriaxone plus azithromycin
B. Doxycycline
C. No antibiotics, provide supportive care
D. Vancomycin plus cefepime

4. Which vaccines should you ensure the patient in Question 1 has received?
A. Influenza vaccine
B. Pneumococcal 13-valent conjugate vaccine
C. Pneumococcal 23-valent polysaccharide vaccine
D. Zoster recombinant vaccine
E. All of the above

5. If the patient in Question 1 had recently been in the hospital, within the last 1 month, for
treatment of a urinary tract infection (UTI), what would be the most appropriate empiric
treatment for her pneumonia?
A. Cefepime and vancomycin
B. Ceftriaxone and metronidazole
C. Ceftriaxone and vancomycin
D. Ciprofloxacin

6. If influenza testing was positive in the patient in Question 1, what would be the
appropriate treatment?
A. Amantadine 200 mg once daily for 5 days
B. Oseltamivir 75 mg once daily for 5 days
C. Oseltamivir 75 mg twice daily for 5 days
D. Supportive measures only

7. A 54-year-old woman with a history of ischemic cardiomyopathy, status post heart


transplant 12 days ago, has developed new fevers. She has evidence of waxing and
waning delirium, but otherwise denies any localizing infectious symptoms such as
headache, neck stiffness, photophobia, cough, shortness of breath, abdominal pain,
dysuria, or diarrhea. She has a stage 1 pressure ulcer on her sacrum and one peripherally
inserted central catheter line in her right upper extremity. She has an external pacemaker
with a clean, dry incision site. On physical examination, her temperature has peaked to
39.4°C, pulse 115/min, respirations 18 breaths/min, and BP 126/82 mmHg. Her
examination is notable for decreased alertness with orientation to person, place, but not
time. She has decreased lung sounds bilaterally; cardiac examination with S1, S2, and S4
heart sounds; no murmurs, rubs, or gallops. Her abdomen has positive bowel sounds and
is non tender. Her extremities are symmetrically edematous. She has no rashes. Her labs
are notable for creatinine 1.8 mg/dL and WBC count 19 300/μL with 93% neutrophils. A
CXR demonstrates atelectasis at the bases and mild pulmonary edema. Urinalysis has 1+
glucose mg/dL, 1+ protein mg/dL, and specific gravity 1.016 (reference range 1.001–
1.035), with 0 to 2 WBC/high-power field (hpf). Two sets of blood cultures result in the
growth of yeast in two out of four bottles at 72 hours; 1,3-β-D glucan and galactomannan
serum tests are pending.
Which of the following factors does not increase the risk for invasive fungal
infection?
A. Central venous catheters
B. Foley catheter

Freemedicalbooks4download
C. Hematologic malignancy
D. Total parenteral nutrition
E. Tumor necrosis factor (TNF)-α inhibitors

8. All of the following may result in a positive 1,3-β-D glucan serum test except for which?
A. Hemodialysis with cellulose membranes
B. Invasive aspergillosis
C. Invasive mucormycosis
D. Recent administration of albumin
E. Recent administration of intravenous immunoglobulins (IVIGs)

9. Which species of fungi can be detected by galactomannan serum testing?


A. Aspergillus species
B. Histoplasma species
C. Penicillium (Talaromyces) species
D. A and C
E. All of the above

10. Which diagnostic test is necessary in the patient in Question 7?


A. Ophthalmology consult
B. Repeat blood cultures
C. Transthoracic echocardiogram
D. All of the above

11. What is your empiric management for the patient in Question 7?


A. Initiate an echinocandin
B. Initiate fluconazole
C. Remove all central venous catheters
D. A and C
E. B and C

12. A 56-year-old man presents 8 months after an allogenic stem cell transplant with
increased cough and shortness of breath. He has been on a prolonged course of steroids
for complications of graft-versus-host disease of the skin. He is on trimethoprim-
sulfamethoxazole (TMP-SMX) for Pneumocystis prophylaxis. On examination, he is
afebrile, pulse 92/min, respirations 22 breaths/min, BP 122/85 mmHg, and O2 saturation
90% on room air. A computerized tomography (CT) scan of the chest is notable for
multiple scattered pulmonary nodules with nodular consolidations and surrounding ground
glass opacities (“halo sign”). Serum galactomannan is elevated.
What is the most likely diagnosis?
A. Bronchiolitis obliterans
B. Invasive pulmonary aspergillosis
C. Methicillin-resistant Staphylococcus aureus (MRSA) pneumonia
D. Respiratory syncytial virus (RSV)

13. A 36-year-old woman with a history of recurrent nephrolithiasis and poorly controlled
diabetes presents with urinary burning, frequency, and suprapubic tenderness. The patient
is sexually active with one male partner and does not use condoms. On physical
examination, her temperature is 37.4°C, pulse 70/min, respirations 16 breaths/min, and BP
118/79 mmHg. Her examination is notable for a nontoxic appearance, normal
cardiovascular evaluation, and suprapubic tenderness. She does not have costovertebral
tenderness.
What is the most likely diagnosis?
A. Acute pyelonephritis
B. Acute uncomplicated cystitis
C. Neurogenic bladder from diabetes
D. None of the above

14. Which of the following should NOT be considered for workup of the patient in Question
13?
A. Complete blood count (CBC)
B. CT abdomen and pelvis
C. Neisseria gonorrhoeae and Chlamydia trachomatis testing
D. Urinalysis and urine culture

15. Which of the following could be considered for empiric treatment in the patient in
Question 13?
A. Fosfomycin
B. Nitrofurantoin
C. TMP-SMX
D. All of the above

16. A 36-year-old man with a remote history of renal transplant presents with burning,
frequency, and foul-smelling urine for several days. He received TMP-SMX last month
for an extended β-lactamase E. coli UTI. His symptoms resolved for a few weeks and now
have returned. He is sexually active and does not use condoms. On physical examination
his temperature is 38°C, pulse 80/min, respirations 16 breaths/min, and BP 118/79 mmHg.
His examination is notable for a nontoxic appearance, normal cardiovascular evaluation,
and suprapubic tenderness. No urethral discharge or genital lesions are noted. He does not
have costovertebral tenderness.
What is the most likely diagnosis?
A. Complicated UTI
B. Transplant rejection
C. Uncomplicated UTI
D. None of the above

17. What is the appropriate workup for the patient in Question 16?
A. CT abdomen and pelvis
B. N. gonorrhoeae and C. trachomatis testing
C. Prostate examination
D. Urinalysis and urine culture
E. All of the above

18. What is the empiric treatment for the patient in Question 16?

Freemedicalbooks4download
A. Ciprofloxacin
B. IV carbapenem
C. Nitrofurantoin
D. TMP-SMX

19. A 73-year-old man with a history of gout, hypertension, diabetes, and alcohol use disorder
presents with a right plantar foot wound of 1 year duration that has become acutely
painful. One week prior to admission, at an urgent care clinic, the patient received a dose
of IV antibiotics and had a foot radiograph, which was not suggestive of osteomyelitis. He
has received ongoing wound care with a local podiatrist. His home medications include
allopurinol, indomethacin, and lisinopril. He recently completed a prednisone taper for a
gout flare. On physical examination, his temperature is 37.6°C, pulse 72/min, respirations
16 breaths/min, and BP 160/92 mmHg. His extremities are notable for a 4 × 3 cm
ulcerated lesion on the right plantar foot at the base of the great toe. There is a black
necrosis at the center of the lesion, with erythematous, well-defined edges. The erythema
extends beyond the edges by 4 cm. With a probe, the subcutaneous tissue is identified;
however, the probe does not reach the bone.
What is the most likely diagnosis?
A. Chronic osteomyelitis
B. Diabetic foot infection
C. Gout flare
D. Necrotizing fasciitis

20. Which of the below characteristics would support a diagnosis of acute osteomyelitis?
A. ESR >70 mm/h
B. Probing to bone or grossly visible bone
C. Ulcer duration longer than 1 to 2 weeks
D. Ulcer size >2 cm2
E. All of the above

21. What is the optimal empiric treatment strategy for the patient in Question 19?
A. Ciprofloxacin and metronidazole
B. Piperacillin-tazobactam
C. Vancomycin and ertapenem
D. Vancomycin and levofloxacin
E. Vancomycin, cefepime, and metronidazole

22. A 39-year-old woman with active IV drug use and untreated hepatitis C presents with
acute-onset fevers and chills. She last injected IV heroin into her arm 12 hours before
arrival. On examination, she is in acute distress, febrile to 40.3°C, pulse 118/min, BP
97/70 mmHg, respirations 18 breaths/min, oxygen saturation 99% on room air. She is alert
and oriented x3; however, she is unable to turn her neck or lift her right arm. Brudzinski
and Kernig testing are both positive. The remainder of the examination is notable for track
marks along bilateral antecubital fossa. Labs are notable for WBC count of 20 000/μL
with 87% neutrophils. Blood cultures result in methicillin-sensitive Staphylococcus
aureus (MSSA), and she is initiated on oxacillin. A transthoracic echocardiogram was
negative for endocarditis. Despite appropriate antistaphylococcal antibiotics, the patient
has persistent bacteremia with neck pain and focal neurologic symptoms.
What is the most likely diagnosis?
A. Aortic root abscess not observed on a transthoracic echocardiogram
B. Epidural abscess
C. MSSA meningitis
D. Recurrent IV drug use while hospitalized
E. None of the above

23. A 22-year-old healthy woman presents after cutting her left foot on a boat propeller while
swimming in a lake. The accident occurred 24 hours ago, with only a small abrasion along
the dorsum of her foot. The site became increasingly painful; therefore, she presented to
the hospital. Her examination is notable for a young woman in acute distress, diaphoretic,
with an edematous, erythematous right foot. Erythema reaches her ankle and the tissue
appears boggy. There is no drainage, no crepitus on examination. She is febrile at 40.1°C,
pulse 112/min, respirations 16 breaths/min, and BP 100/62 mmHg. Labs are notable for
Cr 1.7 mg/dL, WBC count 22 400/μL with 98% neutrophils, 6% bands, and lactate 3.4.
What is the most likely diagnosis?
A. Necrotizing fasciitis
B. Pyoderma gangrenosum
C. Pyomyositis
D. Severe cellulitis
E. Shark bite

24. A 52-year-old man, who has been training for his first marathon, presents with 1 week of
left-sided facial numbness, left facial paralysis, left ear tinnitus, and painful tongue
lesions. He presented to the emergency department (ED) 4 days prior and was told he has
viral-associated Bell’s palsy and should take 60 mg of prednisone daily. He did as
instructed and his symptoms progressed. Patient re-presented to the ED when he
developed new severe left-sided headaches and hearing loss. On physical examination, he
is afebrile, pulse 70/min, respirations 12 breaths/min, and BP 115/78 mmHg. He has
obvious left-sided facial paralysis without evidence of rash. Inspection of his mouth
reveals white coating along his tongue and vesicles lining his palate. He is oriented but
has decreased arousal. Serum laboratory findings are unremarkable.
What is the most likely diagnosis?
A. Acute human immunodeficiency virus (HIV) infection
B. Facial nerve paresis secondary to Lyme disease
C. Herpes zoster ophthalmicus
D. Melkersson-Rosenthal syndrome
E. Ramsay Hunt syndrome

25. Of the studies you may send on the patient in Question 24, which has the lowest
sensitivity for diagnosing active infection?
A. Direct fluorescent antibody testing on scrapings from vesicular skin lesions
B. Varicella zoster virus immunoglobulin G (IgG)
C. Varicella zoster virus polymerase chain reaction (PCR) from lesion
D. Varicella zoster virus viral culture

Freemedicalbooks4download
26. The patient in Question 24 became increasingly delirious over the next 12 hours. Lumbar
puncture (LP) is performed, which shows WBC 320/mm3 with 70% polymorphic nuclear
cells, glucose 110 mg/dL, and total protein 100 mg/dL.
What is the most likely etiology given his history and these results?
A. Bacterial superinfection meningitis
B. Herpes simplex virus (HSV)-1 encephalitis
C. Stroke
D. Varicella zoster encephalitis

27. A 68-year-old man with a history of hypertension, gout, obesity, cervical stenosis status
post recent cervical laminectomy, and spinal fixation 5 days ago is brought in by his wife
for confusion. There were no complications with the surgery and the patient was
discharged on postoperative day 3 with oxycodone. At home, his pain steadily escalated;
therefore, the patient increased his doses of oxycodone, gabapentin, and ibuprofen. On
physical examination, his temperature is 39.6°C, pulse 106/min, respirations 24
breaths/min, and BP 170/96 mmHg. He is oriented and able to hold a conversation but
appears lethargic. Pupils are equally round and reactive. Cervical collar is in place. The
incision site appears clean, dry, and intact without induration or tenderness. He is able to
move all four limbs without focal deficits. Cardiac pulmonary evaluation is normal.
Laboratory findings demonstrate Cr 1.9 mg/dL, K 5.4 mEq/L, WBC count 20 800/μL with
87% neutrophils, 3.2% bands, and platelets 306 000/μL; urinalysis with 1+ protein, 1+
urobilinogen, and 10 to 20 WBC/hpf.
What is the most likely diagnosis?
A. Bacterial meningitis
B. Cerebral vascular event
C. Medication-induced altered mental status
D. Viral encephalitis

28. Which of the following scenarios does not require head imaging prior to LP?
A. Epilepsy patients
B. Focal neurologic findings
C. New-onset seizure
D. Papilledema
E. Primary central nervous system (CNS) lymphoma

29. The patient in Question 27’s LP shows WBC 1200/mm3 with 80% polymorphonuclear
cells, glucose 25 mg/dL, and total protein 600 mg/dL.
What is the most likely etiology given his history and these results?
A. Bacterial meningitis
B. Drug-induced aseptic meningitis
C. HSV encephalitis
D. Tuberculous (TB) meningitis

30. What is the most appropriate empiric treatment regimen for the patient in Question 27?
A. Hold on antibiotics until further culture data are obtained
B. IV acyclovir
C. Liposomal amphotericin-B
D. Vancomycin and cefepime

31. A 34-year-old woman with a history of IV drug use presents to ED with 5 days of fever,
shaking chills, and malaise. She has no other significant past medical history and takes no
medications. She endorses daily use of IV heroin. On examination, she is febrile at
39.2°C, pulse 107/min, BP 105/67 mmHg, respirations 16 breaths/min, and oxygen
saturation 97% on room air. Examination is notable for a 2/6 systolic murmur heard best
in the left lower sternal border and scattered crackles on pulmonary examination. She has
multiple injection site marks noted on her arms and legs, as well as a few erythematous
macules on her bilateral palms. Labs are notable for a WBC count of 18 000/μL with 92%
neutrophils. Her basic metabolic panel and liver function tests (LFTs) are within normal
limits. A CXR is notable for diffuse bilateral nodular densities.
What are the immediate next steps in the workup and management of this patient?
A. Initiate empiric broad-spectrum antibiotics
B. Obtain an echocardiogram
C. Obtain an electrocardiogram (ECG)
D. Obtain blood cultures
E. All of the above

32. The patient in Question 31’s blood cultures return with four out of four bottles growing
MSSA. Transthoracic echocardiogram is notable for an 8 mm mobile vegetation on the
tricuspid valve and a transesophageal echocardiogram (TEE) is ordered for better
assessment.
What additional steps should be taken at this time?
A. Narrow antibiotics to a β-lactam (ie, nafcillin or cefazolin)
B. Obtain CT chest to assess for pulmonary emboli
C. Screen for HIV and hepatitis C
D. B and C
E. A, B, and C

33. The patient in Questions 31 and 32 clears her blood cultures after 48 hours of antibiotic
therapy. Transesophageal echocardiogram (TEE) is negative for any significant valvular
abscess or perforation and cardiac surgery is not felt to be indicated. Chest CT is notable
for several subpleural nodular lesions and a few wedge-shaped densities without necrosis
caused by septic infarcts.
What are the next steps for management?
A. Continue antibiotics for 2 weeks from date of first negative blood cultures and follow
along with weekly labs
B. Continue antibiotics for 6 weeks from date of first negative blood cultures and check
weekly labs
C. Continue antibiotics for 2 weeks from date of first negative blood cultures; no
indication to check weekly labs
D. Continue antibiotics for 6 weeks from date of first negative blood cultures; no
indication to check weekly labs

34. Which of the following are indications for antimicrobial prophylaxis for dental
procedures?

Freemedicalbooks4download
A. History of infective endocarditis
B. History of ventricular septal defect repaired as a child
C. Presence of a bioprosthetic aortic valve
D. A and C
E. A, B, and C

35. A 45-year-old man presents to establish primary care. He moved to the United States from
Eastern Europe 6 months ago and reports that he has no prior medical issues and takes no
medication. He received the bacillus Calmette-Guérin (BCG) vaccine as a child. On
physical examination, he is afebrile and his vital signs are within normal limits. He is well
appearing with clear lung sounds bilaterally and no lymphadenopathy. You decide to
screen him for TB.
Which test should you proceed with?
A. CXR
B. Induced sputum for mycobacterial culture and smear for acid-fast bacilli (AFB)
C. Interferon-γ release assay
D. Tuberculin skin test

36. If the patient in Question 35’s TB interferon-γ release assay returns indeterminate, what
do you do next?
A. Order a tuberculin skin test for comparison
B. Reassure—this is expected with the BCG vaccine
C. Repeat interferon-γ release assay
D. Treat empirically for latent TB infection

37. The patient in Question 35’s TB interferon-γ release assay returns positive. He denies any
recent fever, chills, night sweats, or weight loss. He has no pulmonary symptoms. He has
no known history of contacts who had TB. His HIV testing is negative. He undergoes a
CXR, which is clear.
What is the next best step in the management and treatment of this patient?
A. Initiate isoniazid daily (with vitamin B6) for 9 months
B. Initiate rifampin daily for 4 months
C. Initiate rifampin, isoniazid, pyrazinamide, and ethambutol for 2 months followed by
rifampin with isoniazid for 4 months
D. Either A or B

38. Suppose instead the patient in Questions 35 and 37 endorses several months of
intermittent fever and cough. His CXR was normal.
What would be your next steps?
A. Monitor and recheck interferon-γ release assay in 1 year
B. Obtain CT chest imaging
C. Send three induced sputum for AFB and mycobacterial culture
D. B and C

39. A 37-year-old man presents to clinic requesting an HIV test. He reports that a former
sexual partner of his recently notified him that he was diagnosed with HIV. The patient
reports that he has been sexually active with multiple male partners in the past and reports
that he does not always use condoms. Medical history is notable for one prior episode of
gonococcal urethritis 2 years ago and an episode of primary syphilis 5 years ago. He
reports that he had a negative HIV test about 10 years ago. On examination, he is afebrile,
heart rate (HR) 87 beats/min, and BP 124/85 mmHg. He is anxious appearing. Head and
neck examination are normal. He has a regular rate and rhythm and lungs are clear. His
abdominal examination is benign and he has no rashes. A HIV screen test for HIV-1/2
antibody/antigen is sent and returns positive. The HIV-1/2 differentiation assay is positive
for HIV-1.
In addition to CD4 cell count, HIV viral load and genotype, CBC with differential,
basic metabolic panel and LFTs, of the laboratory workup listed below, which is NOT
recommended in those with a new HIV diagnosis?
A. Fasting lipids
B. Glucan and galactomannan
C. HbA1c
D. Hepatitis A, B, C serologies
E. Interferon-γ release assay

40. The patient in Question 39’s CD4 cell count returns at 172 cells/μL and his HIV-1 viral
load is 364 000 copies/mL. His CBC, basic metabolic panel, and liver enzymes are all
within normal limits. Further history and examination are unrevealing for any evidence of
opportunistic infection.
When should antiviral treatment be started?
A. At this visit, if patient is agreeable to start
B. Begin with Pneumocystis jiroveci prophylaxis and, if tolerated, start antiretroviral
therapy 1 week later
C. Once the results of his genotype return
D. When CD4 cell count is <100 cells/μL
E. When the patient develops signs/symptoms of an opportunistic infection

41. The patient in Questions 39 and 40 is started on antiretroviral treatment as well as TMP-
SMX for Pneumocystis jiroveci pneumonia prophylaxis. At a follow-up visit 6 months
later, he is feeling well and has a CD4 cell count of 300 cells/μL and HIV viral load is
now nondetectable.
Which of the following is correct?
A. He can stop TMP-SMX at this time.
B. He should continue on TMP-SMX until CD4 cell count remains above 200 cells/μL
for 12 months.
C. He should continue on TMP-SMX until he has been on antiretroviral therapy for 12
months.
D. He will need to remain on lifelong TMP-SMX.

42. A 24-year-old man presents to you seeking consideration for HIV preexposure
prophylaxis (PrEP). He reports that he has multiple male sex partners but always uses
condoms. He has a history of rectal gonorrhea 1 year ago. Routine sexually transmitted
infection screening, including HIV screening, is done and negative. Additional laboratory
workup shows a normal CBC and a comprehensive metabolic panel.
What would you offer this patient, at this time, for HIV preexposure prophylaxis?

Freemedicalbooks4download
A. No indication for preexposure prophylaxis since patient is using condoms regularly
B. Start tenofovir and emtricitabine as needed after unprotected sex
C. Start tenofovir and emtricitabine daily
D. Start tenofovir, emtricitabine, and dolutegravir daily

43. A 62-year-old nurse in the intensive care unit presents after having had a needlestick
exposure a few hours ago. The patient she was drawing labs on has a history of IV drug
use and HIV infection. His most recent viral load is unknown and viral load result on this
admission is pending.
What would you offer the nurse at this time, for HIV postexposure prophylaxis?
A. Await results of the patient’s HIV viral load and hold off on treatment unless it is
detectable
B. Await results of the patient’s HIV viral load and only start postexposure prophylaxis
if detectable and once the genotype can be confirmed
C. Start tenofovir and emtricitabine daily for 4 weeks
D. Start tenofovir and emtricitabine with an integrase inhibitor (dolutegravir or
raltegravir) daily for 4 weeks

44. A 23-year-old man from Massachusetts presents after passing out on the football field
during practice. On examination, he is afebrile, HR 48 beats/min, and BP 110/83 mmHg.
His ECG is notable for a new third-degree heart block. The patient denies any significant
family history of cardiac issues, takes no medications, and denies any known tick bites.
On further review of systems, he does report that he had a nonpainful, nonpruritic rash on
his right side a few weeks ago that resolved on its own.
Which of the following is true regarding management?
A. Blood cultures would be useful to confirm the diagnosis.
B. The condition is reversible with administration of antibiotics.
C. The patient will need a permanent cardiac pacer.
D. The patient will need lifelong suppressive antibiotics.
E. The underlying infection can be sexually transmitted.

45. A 65-year-old man with a remote history of splenectomy presents with 1 week of
progressive fatigue and weakness, followed by 2 days of fever, malaise, and dark-colored
urine. He lives in Cape Cod, Massachusetts, and enjoys spending time outdoors. He denies
any known tick bites though does report that he had a circular red rash with central
clearing on the back of his leg, about 1 week ago, that has since resolved. On examination,
his temperature is 38.6°C, HR 118 beats/min, and BP 116/62 mmHg. He appears
jaundiced. Labs are notable for WBC 4000/μL, hemoglobin 8.6 g/dL, platelets 110 000/
μL, total bilirubin 7.2 mg/dL, alanine aminotransferase (ALT) 68 U/L, and aspartate
aminotransferase (AST) 72 U/L.
Given his presentation and risk factors, what would be the next best step?
A. Check Lyme ELISA (enzyme-linked immunosorbent assay) screen, blood parasite
smear, human granulocytic anaplasmosis PCR
B. Check Lyme ELISA screen, blood parasite smear, human monocytic ehrlichiosis
PCR
C. No testing indicated; start empiric azithromycin
D. No testing indicated; start empiric doxycycline
46. Blood parasite smear for the patient in Question 45 shows intraerythrocytic forms and it is
estimated that he has 8% parasitemia. His other labs tests are pending.
What treatment should be initiated?
A. Azithromycin and atovaquone
B. Azithromycin, atovaquone, and doxycycline
C. Ceftriaxone and azithromycin
D. Doxycycline alone
E. Vancomycin and cefepime

47. A 28-year-old man presents with intermittent fevers for the last week. Patient reports that
every few days he develops chills, headaches, and muscle pain. He has no past medical
history and takes no medications. He is sexually active with one partner, denies IV drug
use, and drinks alcohol on occasion. He works for the Peace Corps, but his last trip out of
the country was 1 month ago to Ethiopia. On examination, his temperature is 37.1°C, HR
80 beats/min, and BP 128/62 mmHg. He appears fatigued, but in no acute distress. Labs
are notable for WBC count 12,600/μL, hemoglobin 10.4 g/dL, platelets 106 000/μL, ALT
72 U/L, AST 65 U/L, international normalized ratio (INR) 1.3 (reference range 0.9–1.1),
and creatinine 1.4 mg/dL.
Given his risk factors and presentation, which condition could he have that would
be a medical emergency that must be presumed until proven otherwise?
A. Acute HIV infection
B. Dengue
C. Epstein-Barr virus
D. Malaria
E. Typhoid fever

48. A 72-year-old woman presents with 2 weeks of headache and fever. The headache is
mostly on the left side of her face and worsened with chewing. She reports no known sick
contacts or recent travel. On examination, her temperature is 38.4°C, HR 92 beats/min,
and BP 128/67 mmHg. Cranial nerves, including visual acuity, are intact, and she has no
focal neurological deficits and no meningeal signs. Labs, including CBC, basic metabolic
panel, and LFTs, are all within normal limits. Her ESR is elevated to 82 mm/h.
Which of the following is the next best step in management?
A. Obtain brain magnetic resonance imaging (MRI)
B. Obtain LP
C. Obtain temporal artery biopsy
D. Start high-dose prednisone

ANSWERS

1. The correct answer is: D. Severe community-acquired pneumonia. This patient likely
has community-acquired pneumonia. The potential etiologies include a bacterial infection,
such as S. pneumoniae, Haemophilus influenzae, or Moraxella catarrhalis, or viral causes,
such as influenza, rhinovirus, or respiratory syncytial virus. Infectious Diseases Society of
America (IDSA) 2019 guidelines for community-acquired pneumonia define severe

Freemedicalbooks4download
community-acquired pneumonia as having a respiratory rate greater than 30 breaths/min,
pO2/FiO2 (P/F) ratio of less than 250, multilobular infiltrates, confusion, uremia,
leukopenia, thrombocytopenia, hypothermia, or hypotension as minor criteria. Three or
more of these criteria qualify for severe community-acquired pneumonia. Two major
criteria include septic shock and respiratory failure. This patient has a respiratory rate of
32 breaths/min, thrombocytopenia, and multilobular infiltrates, classifying her as severe
community-acquired pneumonia.

2. The correct answer is: B. Procalcitonin. Additional information that is useful for
patients presenting with community-acquired pneumonia includes sputum Gram stain and
culture (adequate sample if >25 PMN/lpf and <10 squamous cells/lpf), blood cultures, S.
pneumoniae urinary antigen, viral testing, and procalcitonin. Procalcitonin is a biomarker
that is upregulated in acute respiratory infections caused by bacterial, but not viral,
etiologies. The use of procalcitonin varies across hospital systems and could be considered
to help direct the use of antibiotics at some institutions; however, it is NOT validated in
immunocompromised hosts and therefore would not be advised.

3. The correct answer is: A. Ceftriaxone plus azithromycin. For community-acquired


pneumonia, the local S. pneumoniae resistance patterns should be considered.
Monotherapy with azithromycin or doxycycline should not be used if prevalence of
resistance is >25%. Additionally, for patients with comorbidities, such as chronic heart,
lung, liver, or renal disease, diabetes mellitus, alcoholism, malignancy, or asplenia,
guidelines suggest either combination therapy (cephalosporin + macrolide/doxycycline) or
monotherapy (levofloxacin). Of the choices listed, ceftriaxone and azithromycin would be
appropriate in this patient. Oral monotherapy with levofloxacin (not listed as an option
here) could also be considered. Living in an assisted living facility does not qualify her to
be treated for resistant organisms encountered with hospital-acquired infections. She also
has not received antibiotics in the last 3 months, which lowers her risk for resistant
organisms; therefore, vancomycin and cefepime would not be indicated. She should be
placed on droplet precautions while viral testing is pending. Duration of therapy for
community-acquired pneumonia is 5 to 7 days with clinical improvement.

4. The correct answer is: E. All of the above. After 65 years of age, all immunocompetent
adults should receive 1 dose of PCV13 followed by 1 dose of PPSV23 at least 1 year after
the PCV13. If patients aged 19 through 64 years have chronic medical conditions (chronic
heart, lung, liver disease, diabetes, alcoholism, cigarette smoking), then 1 dose of PPSV23
should be administered. Every person should receive the influenza vaccine annually. For
those with an egg allergy, with respiratory distress or angioedema, the influenza vaccine
should still be administered under the supervision of a healthcare provider in a medical
setting. Zoster recombinant vaccine (Shingrix) is recommended in all adults aged 50 years
or older. It is a 2-dose series administered at least 2 to 6 months apart, regardless of a
previous herpes zoster or previously received zoster vaccination. In this case, all of the
vaccines listed would be indicated for this patient if she has not received them already.

5. The correct answer is: A. Cefepime and vancomycin. Empiric treatment for hospital-
acquired pneumonia depends on the presence or absence of risk factors for multidrug-
resistant pathogens, knowledge of predominant pathogens within the healthcare setting,
and the individual’s prior microbiology data. The most important predictive factor that
places an individual at risk for multidrug-resistant organisms is if they have received IV
antibiotics within the past 90 days. Two organisms that are necessary to consider in
hospital-acquired pneumonia are Pseudomonas aeruginosa and MRSA. For this reason,
the only antibiotic option that would treat both of those organisms would be a
combination of cefepime (for Pseudomonas) and vancomycin (for MRSA).

6. The correct answer is: C. Oseltamivir 75 mg twice daily for 5 days. Antiretroviral
therapy should be initiated as soon as possible and is most likely to provide benefit when
initiated within the first 48 hours of illness. Treatment should NOT be held in patients
with indications for therapy who present greater than 48 hours after the onset of
symptoms, especially those requiring hospitalization. Oseltamivir 75 mg twice daily for 5
days would be recommended in this patient if she were diagnosed with influenza as the
etiology of her community-acquired pneumonia. The once-daily dose of oseltamivir is
recommended for influenza chemoprophylaxis or if patients have renal dysfunction.
Amantadine is no longer used for the treatment of influenza due to high resistance rates.

7. The correct answer is: B. Foley catheter. This patient is immunosuppressed given her
recent heart transplant and this puts her at increased risk for invasive fungal infections.
Other risk factors for fungemia include administration of total parenteral nutrition,
hematologic malignancy, solid organ transplant, biologic therapies (including TNF-α
inhibitors), and IV drug use. In this case, the source was likely her peripherally inserted
central catheter line, which should be removed immediately. The Foley catheter is
unlikely to be a source of her candidemia.

8. The correct answer is: C. Invasive mucormycosis. The 1,3-β-D glucan assay detects a
cell wall component in many fungi, including Aspergillus species, Candida species, and
Pneumocystis jiroveci; however, it is typically negative in patients with mucormycosis or
cryptococcosis. The 1,3-β-D glucan assay can be falsely positive with IVIGs, albumin,
and surgical gauze, certain types of dialysis membranes, and infections with certain
bacteria that contain cellular β-glucans (Pseudomonas aeruginosa).

9. The correct answer is: E. All of the above. Galactomannan is a major constituent of
Aspergillus cell walls. An enzyme immunoassay that detects the galactomannan antigen is
available for use in blood and bronchoalveolar lavage fluid as an adjuvant test for the
diagnosis of aspergillosis. Serum galactomannan assay is established in the setting of
suspected invasive aspergillosis in patients with hematologic malignancies. The test can
be positive in the setting of infections with Fusarium spp, Penicillium spp, and
Histoplasma capsulatum due to cross-reactivity, and there can be false positives in the
setting of IVIG, and historically with piperacillin-tazobactam (rarely the case now with
new preparation of piperacillin-tazobactam).

10. The correct answer is: D. All of the above. Blood cultures with yeast are NEVER
contaminants. It is important to collect daily blood cultures until they are negative. Other
important diagnostic tests for fungemia include an ophthalmology consult to evaluate for
endophthalmitis and a transthoracic echocardiogram to evaluate for endocarditis,
particularly in cases of persistent fungemia or if any evidence of embolic phenomena is

Freemedicalbooks4download
present.

11. The correct answer is: D. A and C. The most likely cause of yeast in this patient’s blood
is Candida species. The first step is to remove any potential sources of infection (in this
case, the peripherally inserted central catheter line) and to obtain an Infectious Disease
consult. There is high mortality associated with candidemia in immunocompromised
hosts. Empiric treatment for Candida species is an echinocandin, such as micafungin,
because Candida glabrata and Candida krusei can have resistance against azole
antifungals. For Candida albicans, there is little resistance in the absence of prior
exposure to azoles; therefore, this patient could be transitioned to fluconazole after
susceptibility is confirmed.

12. The correct answer is: B. Invasive pulmonary aspergillosis. This patient most likely
has invasive pulmonary aspergillosis. Risk factors for this include prolonged and profound
neutropenia or immunosuppression. In this case, the patient has history of stem cell
transplant and also was recently on steroids. The presence of a positive serum
galactomannan and CT findings make aspergillosis most likely.

13. The correct answer is: B. Acute uncomplicated cystitis. This is most likely acute
uncomplicated cystitis. Her symptoms are confined to the bladder without evidence of
upper tract involvement or systemic signs. It is important to note that a history of
nephrolithiasis, poorly controlled diabetes, HIV, urinary strictures with stents, and
immunocompromised patients are not automatically considered complicated UTIs;
however, there should be a lower threshold to treat these individuals as complicated
infections.

14. The correct answer is: B. CT abdomen and pelvis. Further workup includes a
urinalysis, urine culture, and CBC. Pyuria on urinalysis is suggestive of infection. In
sexually active patients with UTI symptoms and “sterile pyuria,” you should also test for
sexually transmitted diseases such as N. gonorrhoeae and C. trachomatis. Consider CT
imaging only when the patient is severely ill, symptoms persist for 72 hours despite
antibiotics, and there is suspicion for obstruction from stone, renal abscess, or recurrent
symptoms within a few weeks of treatment. This patient does not require a CT abdomen
and pelvis at this time.

15. The correct answer is: D. All of the above. If a patient has uncomplicated acute cystitis
and no prior UTIs with multidrug-resistant organisms, empiric treatment options include
nitrofurantoin monohydrate 100 mg twice daily for 5 days, TMP-SMX 1 DS tablet orally
twice per day for 3 to 5 days, or fosfomycin 3 g orally for one single dose. For
complicated cystitis, a longer course of 7 to 14 days with either a fluoroquinolone or
TMP-SMX may be given.

16. The correct answer is: A. Complicated UTI. This is a complicated UTI. Individuals
with renal transplants and pregnant women are treated as complicated UTIs regardless of
whether the infection extends beyond the bladder.

17. The correct answer is: E. All of the above. All men with UTIs should have a prostate
examination to test for prostatitis. The diagnostic studies include urinalysis, urine culture,
and CBC. Pyuria on urinalysis indicates infection. CT imaging would be appropriate in
this patient given his recurrent symptoms within a few weeks of completing treatment. In
sexually active patients with UTI symptoms, consider also testing for sexually transmitted
infections such as N. gonorrhoeae and C. trachomatis.

18. The correct answer is: B. IV carbapenem. For a complicated UTI, the treatment
strategy depends on the patient’s risk for multidrug-resistant organisms. If, in the last 3
months, the patient had an multidrug-resistant gram-negative bacilli infection, an inpatient
stay, used a fluoroquinolone, TMP-SMX, or a broad-spectrum β-lactam, then cefepime or
a carbapenem is the correct first-line medication for a complicated UTI while awaiting
results of susceptibility testing. This patient has recently received TMP-SMX for a UTI
and has a history of drug-resistant organisms, qualifying him for a carbapenem as empiric
therapy. If there are no multidrug-resistant risk factors, then the drug of choice is a
fluoroquinolone, TMP-SMX, or a β-lactam.

19. The correct answer is: B. Diabetic foot infection. This patient likely has a diabetic foot
infection. The risk factors for diabetic foot infections include peripheral neuropathy,
which leads to decreased awareness of injury; peripheral vascular disease, which impairs
necessary blood flow for healing; and poor glycemic control, which impairs neutrophil
function. If the patient had chronic osteomyelitis, you would expect to see findings on the
foot radiograph, which can typically demonstrate abnormalities by 2 weeks after infection.

20. The correct answer is: E. All of the above. Clinical findings that help support
osteomyelitis include (1) probing to bone or grossly visible bone; (2) ulcer size >2 cm; (3)
ulcer duration longer than 1 to 2 weeks; (4) ESR >70 mm/h. Ultimately, MRI is the most
sensitive imaging study to detect osteomyelitis if the plain radiograph is negative.

21. The correct answer is: E. Vancomycin, cefepime, and metronidazole. This is a
moderately severe diabetic foot infection. Antibiotic treatment should target gram-positive
organisms (vancomycin, linezolid, daptomycin), gram-negative bacilli (fluoroquinolones,
advanced-spectrum β-lactams), and anaerobic organisms (metronidazole, unless a
carbapenem, piperacillin-tazobactam, or ampicillin-sulbactam is included in the regimen).
Pseudomonas aeruginosa is a common pathogen with diabetic foot infections; therefore,
ertapenem would not be an appropriate agent. Other treatments including limb elevation,
non–weight-bearing status, wound care, tight glycemic control, and venous insufficiency
are critical for wound healing. Surgical consultation is necessary for debridement and
revascularization (or, if these measures are unsuccessful, amputation).

22. The correct answer is: B. Epidural abscess. Patients who present with bacteremia,
fevers, and back pain should raise concern for an epidural abscess. Classic symptoms
include focal back pain, electric shock–like pain, motor and/or sensory weakness, and
paralysis. It is important to recognize this clinical presentation, obtain prompt imaging,
and involve neurosurgery as necessary for early decompression. Blood cultures can help
identify the pathogen in 60% of cases. LP for examination of cerebrospinal fluid (CSF) is
often not performed because diagnostic yield is low.

Freemedicalbooks4download
23. The correct answer is: A. Necrotizing fasciitis. Necrotizing fasciitis involves deep soft
tissues and results in rapid destruction of several fascial planes. It has a high mortality if
not recognized early. The infection should be suspected in patients with soft tissue
infection (edema, erythema) that progresses rapidly (hours), and if any signs of systemic
illness are present (fever, hemodynamically unstable). Other skin manifestations include
crepitus or severe pain out of proportion to examination. Treatment includes early and
aggressive surgical exploration with debridement of necrotic tissue as well as broad-
spectrum antibiotic therapy. This patient’s presentation is more consistent with β-
hemolytic strep infection instead of polymicrobial necrotizing fasciitis, but regardless,
empiric treatment should target gram-positive, gram-negative, and anaerobic organisms,
and in addition, add the antitoxin antibiotic clindamycin. In this patient, who has had
water exposure, it is important to consider Aeromonas and Pseudomonas species.

24. The correct answer is: E. Ramsay Hunt syndrome. Ramsay Hunt syndrome, otherwise
known as herpes zoster oticus, is varicella zoster virus reactivation within the eighth
cranial nerve that involves ipsilateral facial paralysis, hearing abnormalities (ear pain,
decreased hearing, tinnitus, hyperacusis), and often vesicles within the auditory canal.
Patients can also experience altered taste perception, tongue lesions, and lacrimation.

25. The correct answer is: D. Varicella zoster virus viral culture. Viral PCR yields the
highest sensitivity for diagnosing varicella zoster virus reactivation, with direct
fluorescent antigen testing from scrapings of an active vesicular lesion the second-best
diagnostic study. Isolating the virus and culture takes 1 week for results and yields only
50% to 75% compared with PCR-positive samples. Varicella zoster virus IgG would have
no utility in diagnosing active infection in this patient.

26. The correct answer is: D. Varicella zoster encephalitis. The LP results, along with
delirium, are consistent with viral encephalitis. Herpes zoster–associated encephalitis
typically presents with delirium within days following vesicular abruption but may occur
prior to the onset of rash. Varicella zoster virus encephalitis is more common in
immunocompromised patients, but it is also seen in a healthy host, like this patient. When
cranial or cervical dermatomes are involved with the varicella zoster virus reactivation or
patients develop disseminated herpes zoster, they are at a higher risk for developing
encephalitis.

27. The correct answer is: A. Bacterial meningitis. Given the patient’s confusion, fever,
and recent neurosurgery, there is a high likelihood his presentation is due to post–
neurosurgical bacterial meningitis. Infections from gram-positive skin flora are the most
likely organisms in the setting of CSF shunts, recent neurosurgery, or penetrating head
trauma and should be considered in this case. Patients with meningitis commonly present
with fever, headache, and meningeal signs. Although they may appear lethargic, they will
have a normal sensorium. This is different from patients with encephalitis, who have
mental status changes consistent with delirium. Patients can have features of both
diseases; however, in this patient, it would be unusual for the patient to develop viral
encephalitis following neurosurgery. Additionally, his examination is not consistent with
encephalitis.
28. The correct answer is: A. Epilepsy patients. Head imaging prior to an LP is considered
if the patient has a history of CNS disease (primary CNS lymphoma), new-onset seizure,
focal neurologic findings, or papilledema; therefore, patients with a known diagnosis of
epilepsy do not require head imaging before the procedure.
This patient does not meet the criteria for head imaging. When approaching
bacterial meningitis as a possible diagnosis, you should first and foremost obtain blood
cultures and then initiate appropriate antibiotics. Steroids may also be given in some
circumstances and, if done, should be given before the first dose of antibiotics. LP should
also be performed as soon as possible but should not delay initiation of antibiotics and/or
steroids. Note the yield of CSF culture is unlikely to be changed if the LP is obtained
within 4 hours of antibiotic initiation. Additional CSF studies that can be ordered on the
basis of clinical suspicion include AFB smear and culture, fungal culture and stain,
cryptococcal antigen, Venereal Disease Research Laboratory (VDRL), HSV/varicella
zoster virus/enteroviral PCR, and cytology.

29. The correct answer is: A. Bacterial meningitis. Given the elevated WBC count, low
glucose, and high total protein, the CSF studies suggest bacterial, fungal, or tuberculous
etiologies. Given that the WBC count is particularly high and predominantly
polymorphonuclear cells, a bacterial meningitis would be most likely. Given his recent
neurosurgical procedure, this patient is at risk for Staphylococcus aureus, coagulase-
negative staphylococci, and gram-negative bacilli.

30. The correct answer is: D. Vancomycin and cefepime. This patient likely has bacterial
meningitis and should be initiated on vancomycin and cefepime empirically while
awaiting Gram stain and culture results from the CSF.

31. The correct answer is: E. All of the above. This patient’s presentation and examination
are highly concerning for infectious endocarditis, especially given her risk from IV drug
use. She has murmur on examination and evidence of vascular phenomena with palmar
Janeway lesions and possible septic pulmonary emboli on CXR. This patient should have
blood cultures drawn immediately, prior to the initiation of antibiotics. At least two sets of
blood cultures should be drawn with two bottles per set. After blood cultures are drawn,
she should be started on empiric broad-spectrum antibiotics, including coverage of
MRSA. IV vancomycin and cefepime would be a good empiric option. An ECG should be
obtained to assess for any conduction abnormalities and an echocardiogram should be
obtained to assess for valvular vegetations. It would be reasonable to start with a
transthoracic echocardiogram, although she may also need a transesophageal
echocardiogram (TEE), depending on the transthoracic echocardiogram quality and
findings.

32. The correct answer is: E. A, B, and C. This patient has MSSA bacteremia with tricuspid
valve endocarditis. Infectious Diseases should be consulted immediately. A CT scan of
the chest can be done, given the abnormal CXR, to assess for septic pulmonary emboli
and rule out any sites of abscess or empyema that could require drainage. She should be
narrowed from empiric MRSA coverage (ie, vancomycin) to a β-lactam (cefazolin 2 g IV
q6h or nafcillin 2 g IV q4h), which is superior to vancomycin for treatment of MSSA
infections. Blood cultures should be checked every 24 to 48 hours until negative. She

Freemedicalbooks4download
should be screened for hepatitis C and HIV.

33. The correct answer is: B. Continue antibiotics for 6 weeks from date of first negative
blood cultures and check weekly labs. She should continue on IV antibiotics treatment
with either nafcillin or cefazolin for 6 weeks from date of first negative blood cultures.
During this time, she should be monitored with weekly labs (basic metabolic panel, LFTs,
and CBC with differential) for possible side effects from long-term antibiotics (ie, acute
renal failure, liver injury, eosinophilia, neutropenia).

34. The correct answer is: D. A and C. Antibiotic prophylaxis is recommended prior to
dental procedures (which is defined by any manipulation of gingival tissue or periapical
region of teeth or perforation of oral mucosa) in patients with a history of infective
endocarditis, prosthetic heart valves, prosthetic valve repair material, cardiac transplants
with valvulopathy, and unrepaired or incompletely repaired congenital heart disease.
Additionally, it is indicated for patients with repaired congenital heart disease if within 6
months of the initial surgery. It would not be indicated for someone with a remote
ventricular septal defect repaired as a child. The prophylactic regimen is typically
amoxicillin 2 g orally 30 to 60 minutes before the procedure. Alternatively, clindamycin
can be used in patients with a penicillin allergy.

35. The correct answer is: C. Interferon-γ release assay. A serum interferon-γ release assay
should be checked as the next step for this patient. Prior receipt of the BCG vaccine,
particularly later than infancy, can result in a reactive tuberculin skin test. The interferon-γ
release assays do not cross-react with BCG and therefore have better specificity in BCG
recipients. Induced sputum and CXR would not be done prior to TB screening if the
patient does not have any symptoms concerning for active TB.

36. The correct answer is: C. Repeat interferon-γ release assay. The two most common
interferon-γ release assay tests include QuantiFERON and T spot. If the interferon-γ
release assay results are indeterminate, from either a strong negative control response or a
weak response to positive control, then the interferon-γ release assay should be repeated.
If the repeat interferon-γ release assay results are indeterminate, then you can proceed
with the tuberculin skin test. This is not an expected result from the BCG vaccine and
reassurance would be incorrect in this scenario.

37. The correct answer is: D. Either A or B. This patient does not have any signs or
symptoms concerning for active pulmonary TB (fever, cough, night sweats, weight loss),
is not immunocompromised, and has a clear CXR. He likely has a latent TB and has no
evidence of active TB. He also has no known or suspected contacts with multidrug-
resistant TB; therefore, treatment options for latent TB in this case would include rifampin
daily for 4 months or isoniazid daily (with vitamin B6) for 9 months. Isoniazid and
rifapentine weekly for 12 weeks is also an option. Treatment for active TB (answer choice
C) would not be indicated in this scenario.

38. The correct answer is: D. B and C. In this case, the patient requires further evaluation
for active TB. Airborne infection isolation should be instituted. He should have three
sputum samples, induced if necessary, for AFB stain and mycobacterial culture. TB PCR,
which is more sensitive than AFB smear, should be checked on one of the samples.
Negative sputum testing does not necessarily exclude the diagnosis of active pulmonary
TB. The patient should also undergo chest CT imaging since his CXR was negative.
Patients who are diagnosed with pulmonary TB (and are not suspected to have multidrug-
resistant TB) should be initiated on 4-drug therapy with rifampin, isoniazid, pyrazinamide,
and ethambutol while awaiting susceptibility testing.

39. The correct answer is: B. Glucan and galactomannan. All patients with a newly
diagnosed HIV infection should have the following baseline labs checked: CD4 cell count,
HIV viral load and genotype (of the reverse transcriptase and protease genes, to assess for
drug resistance), CBC with differential, basic metabolic panel, LFTs, fasting lipids,
HbA1c, syphilis screen, hepatitis (A, B, and C) serologies, and routine gonorrhea and
chlamydia screening. Depending on the patient’s CD4 cell count, toxoplasmosis IgG may
be done, as this informs options for opportunistic infection prophylaxis in patients with
CD4 cell count <200/μL and, if negative, provides the opportunity on counseling for
prevention of infection. In patients with profound immunocompromise, cytomegalovirus
IgG is also useful to detect those at risk for cytomegalovirus retinitis and primary
infection. Patients should also have a tuberculin skin test or an interferon-γ release assay,
with an appropriate review of symptoms, to assess for TB infection. Neither glucan nor
galactomannan is advised in the routine workup for new diagnosis of HIV.

40. The correct answer is: A. At this visit, if patient is agreeable to start. In the absence of
certain severe opportunistic infections, patients should be counseled and started on early
antiretroviral therapy immediately. This can even be done the same day as or prior to his
full set of labs or genotype returning. First-line regimens include two nucleoside reverse
transcriptase inhibitors with either an integrase inhibitor or a protease inhibitor boosted
with ritonavir or cobicistat. Additionally, he should be started on P. jiroveci pneumonia
prophylaxis with TMP-SMX since his CD4 cell count is below 200 cells/μL. If he is
unable to take TMP-SMX, other options include atovaquone, dapsone (need to check
glucose-6-phosphate dehydrogenase status), or aerosolized pentamidine.

41. The correct answer is: A. He can stop TMP-SMX at this time. Pneumocystis jiroveci
pneumonia prophylaxis can be stopped once a patient’s CD4 cell count is >200 cells/μL
for 3 to 6 months while on suppressive antiretroviral therapy. This patient can therefore
stop TMP-SMX at this time.

42. The correct answer is: C. Start tenofovir and emtricitabine daily. Preexposure
prophylaxis is highly effective at preventing HIV transmission in HIV sero-discordant
couples and consists of the HIV-uninfected partner taking tenofovir and emtricitabine
(TDF-FTC) daily. Some studies have also suggested on-demand use may be effective, in
which the person takes two tablets of TDF-FTC prior to the sex act and one tab every 24
hours after for two doses (thus four tablets total per sex act). Taking preexposure
prophylaxis only after a sexual encounter is not currently advised. Even though the patient
reports using condoms, he is at high risk of acquiring HIV infection and preexposure
prophylaxis should be advised.

43. The correct answer is: D. Start tenofovir and emtricitabine with an integrase

Freemedicalbooks4download
inhibitor (dolutegravir or raltegravir) daily for 4 weeks. A needlestick exposure from a
patient with known HIV warrants postexposure prophylaxis. Triple drug therapy, typically
consisting of tenofovir and emtricitabine with an integrase inhibitor (dolutegravir or
raltegravir), should be started ASAP (<2 hours), but at most 48 to 72 hours of possible
exposure. You should not await the results of the patient’s viral load or genotype to
initiate treatment. If the patient has known HIV resistance, then a different regimen might
be considered, but treatment should never be delayed for a genotype or viral load testing.

44. The correct answer is: B. The condition is reversible with administration of
antibiotics. This patient has Lyme carditis, a manifestation of early disseminated infection
from the spirochete Borrelia burgdorferi. This is transmitted from the deer tick Ixodes
scapularis, common throughout New England, the mid-Atlantic states, and the Midwest.
It is not sexually transmitted. Lyme carditis can result in first-, second-, or third-degree
heart block. While patients may sometimes need a temporary pacer, the condition is
reversible with antibiotic treatment. A permanent cardiac pacer should be avoided. Lyme
will not grow readily in culture, so blood cultures would not be useful to confirm the
diagnosis. This patient should be initiated on IV ceftriaxone and can be switched to an
oral agent (ie, doxycycline) upon clinical improvement. Treatment is typically for 14 to 21
days. He will not need lifelong antibiotics for this condition.

45. The correct answer is: A. Check Lyme ELISA screen, blood parasite smear, human
granulocytic anaplasmosis PCR. There is a high concern for tick-borne illness,
especially given his geographical location. His recent rash is typical for erythema migrans,
which occurs in up to 80% of patients with early Lyme infection and can be seen 1 to 2
weeks after a tick bite. His other lab abnormalities, however, raise concern for potential
coinfection with another tick-borne infection. Specifically, his anemia with evidence of
hemolysis raises concern for infection with Babesia microti, the primary agent of human
babesiosis in the United States. An older patient with asplenia is at high risk for severe
infection and should be hospitalized. A blood smear should be sent to examine for
intracellular parasites and, if present, to measure parasitemia. He should also be tested for
coinfection with other tick-borne diseases that may also cause pancytopenia (without
hemolysis) and transaminitis, particularly anaplasmosis (Anaplasma phagocytophilum).
Ehrlichiosis (Ehrlichia chaffeensis) is relatively uncommon in Massachusetts and while
testing is considered, it would be more useful to test for coinfection with anaplasmosis of
the options listed. While empiric treatment is indicated, testing should be done and none
of the regimens listed above would provide adequate coverage.

46. The correct answer is: B. Azithromycin, atovaquone, and doxycycline. He should be
started on treatment for babesiosis with IV azithromycin and atovaquone. Given that he
also likely had primary Lyme infection based on his history of erythema migrans, he
should receive treatment with doxycycline as well. Evidence of erythema migrans is
sufficient to clinically diagnose early Lyme disease even without further testing. Note that
doxycycline would also cover empirically for ehrlichiosis and anaplasmosis while tests are
pending. Infectious Diseases should also be consulted in patients who have severe
symptoms, who are asplenic or otherwise immunocompromised, or if the parasitemia is
4% or higher.
47. The correct answer is: D. Malaria. The epidemiology of fever among returned travelers
is highly dependent on geography. In a recent traveler to sub-Saharan Africa, malaria
should be of the highest concern, even if afebrile at the time of the evaluation. Fevers in
malaria wax and wane and physical examination may be normal. Notably, the incubation
time for malaria is usually 7 to 30 days, and can be longer.
A thorough history and physical should always guide diagnostic workup. In this
scenario, laboratory testing should include basic metabolic panel, CBC and differential,
LFTs, blood cultures, and immediate performance of rapid diagnostic testing for malaria.
While the other conditions are also possible, the patient should be presumed to have
malaria until proven otherwise, and an Infectious Disease consult should be obtained.
Malaria is a medical emergency with high risk or rapid deterioration and death,
particularly with falciparum malaria. Prompt initiation of treatment is essential.

48. The correct answer is: D. Start high-dose prednisone. Remember that the majority of
cases of fever of unknown origin are from a noninfectious etiology. In this case, the
patient’s symptoms of temporal headache with jaw pain and elevated ESR are most
concerning for giant cell arteritis. While temporal artery biopsy is helpful for diagnosis,
steroids should be started immediately when this condition is suspected because if left
untreated, this patient is at risk of loss of vision. Neither brain MRI nor LP would be
indicated as the next best step.

Freemedicalbooks4download
7
ENDOCRINOLOGY

QUESTIONS

1. A 28-year-old woman presents to the emergency department (ED) with severe headache
and visual field disturbances. A computed tomography (CT) scan of the head is
performed, which identifies a 3-cm sellar mass as well as intralesional hyperdensities
concerning for hemorrhage. On examination, the patient is in moderate distress due to
pain, with temperature (T) 37.5°C, heart rate (HR) 100 beats/min, blood pressure (BP)
90/60 mmHg, and respiratory rate (RR) is 22 breaths/min. She is alert and oriented, and
extraocular movements are intact with no diplopia. The following labs are acquired:

Sodium: 132 mEq/L


Potassium: 4.2 mEq/L
Creatinine: 0.8 mg/dL

What is the next best step in her management?


A. Administer hydrocortisone 50 mg IV
B. Assessment of thyroid function and antidiuretic hormone
C. Refer for brain magnetic resonance imaging (MRI) pituitary protocol
D. Refer for neurosurgical consultation

2. The patient in Question 1 receives IV hydrocortisone and is evaluated by neurosurgery


for emergent surgery. She receives transsphenoidal surgery in the evening with
preliminary pathology of the lesion showing pituitary adenoma with necrosis. The patient
is feeling well the following day.
What is the expected result of the cosyntropin stimulation test the morning
following the surgery, assuming she received stress dose dexamethasone prior and after
the surgery?
Cortisol levels with cosyntropin stimulation test:
A. Baseline: 0.2 30 min: 1.3 60 min: 2.2
B. Baseline: 0.5 30 min: 15.3 60 min: 20.5
C. Baseline: 2.5 30 min: 4.5 60 min: 12.2
D. Baseline: 6.3 30 min: 17.1 60 min: 22.3
3. A 45-year-old woman presents to endocrinology clinic for evaluation of weight gain. The
patient reports sudden-onset weight gain 6 months ago, increasing 40 lb in weight. She
has also noted easy bruising and mild leg swelling. On examination, the patient has a BP
of 145/90 mmHg, HR 80 beats/min, and RR 15 breaths/min. There is evidence of facial
rounding and plethora. She exhibits central adiposity with adipose collections on the back
of her neck. She has 1+ pitting edema in her legs bilaterally. HbA1c was recently noted as
6.4%, increased from 5.3% a year prior. The patient denies snoring, headache, or visual
field changes.
What would be the next best step in her management?
A. Order a low-dose dexamethasone suppression test (DST)
B. Order fasting insulin-like growth factor (IGF-1) level
C. Order morning cortisol with adrenocorticotropic hormone (ACTH)
D. Refer for pituitary MRI

4. A dexamethasone suppression test (DST) is performed in the patient in Question 3, which


shows a morning cortisol of 9.5 μg/mL. Concomitant adrenocorticotropic hormone
(ACTH) level was 15 μg/mL. The patient performed two separate 24-h urine cortisol
collections, both of which were three times the upper limit of normal. The patient received
a pituitary-protocol MRI, which showed a 4-mm pituitary lesion concerning for
microadenoma.
What is the next best step in her management?
A. Initiate ketoconazole 200 mg twice daily
B. Perform late-night salivary cortisol measurements two times
C. Refer to neurosurgery for microadenoma resection
D. Refer to radiology for inferior petrosal sinus sampling

5. A 40-year-old man presents to his primary care physician (PCP) office due to joint pain.
He reports increasing stiffness in his knees and shoulders over the past 2 years. He also
has noted increasing swelling in his hands and feet. The swelling has required him to
purchase new shoes as the old ones no longer fit. He also reports that his wife states that
his snoring has gotten worse. On examination, the patient exhibits thickened digits on his
hands and feet. He has multiple skin tags on his chest. He has an enlarged jaw and brow
when compared to his ID photo taken 4 years ago. His oral and nasal examinations show
no lesions or malformations.
Which of the following would be the next best step in his diagnosis?
A. Checking growth hormone after an oral glucose load
B. Fasting morning growth hormone level
C. Glucagon stimulation test
D. Insulin tolerance test

6. The patient in Question 5’s IGF-1 level is noted to be 1034 ng/mL (normal: 125-320
ng/mL). He receives an oral glucose tolerance test with 75 g glucose, and postglucose
growth hormone level is 2.1 ng/mL (normal: <1.0 ng/mL). A pituitary MRI shows a 9-mm
lesion within the pituitary. No invasion of the cavernous sinus or compression of the optic
chiasm is noted.
What is the next best step in his management?
A. Initiate lanreotide 30 mg monthly

Freemedicalbooks4download
B. Initiate pegvisomant 10 mg daily
C. Refer to neurosurgery for resection
D. Refer to radiation oncology for radiosurgery

7. A 30-year-old woman, who is otherwise healthy, presents for secondary amenorrhea. Six
months ago, she noted a lack of menses. Pregnancy test was negative at that time and 1
month ago. She has also noted occasional breast tenderness and milky discharge from
both breasts. She has developed an intermittent headache over the past 2 weeks that
improves with tylenol. She denies dizziness or changes in her vision.
What is the most likely source of her symptoms?
A. Adrenal hyperplasia
B. Ovarian tumor
C. Pituitary mass
D. Surreptitious medication use

8. A prolactin level is checked in the patient in Question 7, which is elevated to 120 ng/mL
(normal: <22 ng/mL). Thyroid-stimulating hormone (TSH) and free thyroxine (fT4) levels
are normal. Pituitary-protocol MRI is performed, which identifies a 1-cm intrasellar lesion
concerning for a macroadenoma. The patient is not interested in fertility at this time.
What is the next best step in her management?
A. Initiate cabergoline 0.5 mg weekly
B. Initiate combined oral contraceptive pill (OCP)
C. Initiate levothyroxine 88 μg daily
D. Refer to neurosurgery for lesion removal

9. A 21-year-old woman presents to the ED for evaluation of fatigue and frequent urination.
The patient reports experiencing constant fatigue and brain fog over the past year. She
required an emergency room visit 6 months ago due to abdominal pain and was diagnosed
with a kidney stone. She has also developed a worsening headache as well as a lack of
menses over the past year. She is currently taking no medications. She denies alcohol or
tobacco use. She is adopted and does not know her family history. On examination, the
patient appears tired with dry mucous membranes. Her vital signs are as follows: BP
100/70 mmHg, HR 85 beats/min, T 36.8°C, and RR 18 breaths/min. Her abdomen is soft
and nontender to palpation. She has trace nonpitting edema in her legs bilaterally.
Laboratory evaluations include:

Calcium: 11.6 mg/dL (normal: 8.5-10.5 mg/dL)


Creatinine: 0.9 mg/dL
Parathyroid hormone: 75 pg/mL (normal: 10-65 pg/mL)
25-OH vitamin D: 30 ng/dL (normal: 20-80 ng/dL)
TSH: 0.4 mU/L (normal: 0.4-5.0 mU/L)
fT4: 0.7 ng/dL (normal: 0.9-1.8 ng/dL)
Prolactin: 10 ng/mL (normal: <22 ng/mL)

The patient receives aggressive hydration in the ED with reduction in her calcium.
A brain MRI is performed due to headache, and a 1-cm pituitary lesion is noted that abuts
the cavernous sinus but does not compress the optic chiasm. A thyroid ultrasound is
performed, which shows a normal-sized thyroid with enlarged masses in each lobe
concerning for parathyroid enlargement.
What is the most important screening test for this patient after management of her
acute presentation?
A. Abdominal MRI
B. Colonoscopy
C. Echocardiogram
D. Pelvic ultrasound

10. A 36-year-old woman with no chronic medical conditions presents to her PCP with weight
gain, fatigue, and dry skin.
What is the appropriate screening test(s) for hypothyroidism?
A. TSH
B. TSH and fT4
C. TSH, fT4, and free triiodothyronine (T3)
D. TSH, fT4, and total T3

11. A 39-year-old man with a medical history including obesity is diagnosed with primary
hypothyroidism after presenting to his PCP with fatigue and constipation. His TSH is 29
mU/L (ref. 0.4-5 mU/L) and fT4 is 0.6 ng/dL (ref. 0.9-1.8 ng/dL). His weight is 80 kg.
How should his hypothyroidism be managed?
A. Levothyroxine 50 μg
B. Levothyroxine 88 μg
C. Levothyroxine 125 μg daily
D. Levothyroxine 150 μg daily

12. A 79-year-old man with a medical history including obesity, heart failure, and coronary
artery disease (CAD) is diagnosed with primary hypothyroidism after presenting to his
PCP with fatigue and constipation. His TSH is 29 mU/L (ref. 0.4-5 mU/L) and fT4 is 0.6
ng/dL (ref. 0.9-1.8 ng/dL). His weight is 80 kg.
How should his hypothyroidism be managed?
A. Levothyroxine 25 μg
B. Levothyroxine 50 μg
C. Levothyroxine 88 μg daily
D. Levothyroxine 120 μg daily

13. A 35-year-old woman with medical history of obesity and hypothyroidism is admitted to
the medical intensive care unit with altered mental status, hypothermia, and bradycardia.
Laboratory evaluation is notable for creatinine 0.8 mg/dL, TSH 30.4 mU/L (ref. 0.4-5
mU/L), fT4 0.2 ng/dL (ref. 0.9-1.8 ng/dL), and white blood cell (WBC) count 15 500/μL.
What is the most appropriate next best step in her management?
A. Restart home levothyroxine
B. Start broad-spectrum antibiotics
C. Start hydrocortisone 100 mg every 8 hours and 100 μg T4 IV daily
D. Start 200 μg T4 IV and T3 10 μg IV

14. A 55-year-old woman with medical history of multiple sclerosis was admitted to the

Freemedicalbooks4download
medical intensive care unit with Staphylococcus aureus pneumonia complicated by
bacteremia and infective endocarditis. One week into her hospitalization, thyroid function
studies are checked and she is found to have a TSH of 0.3 mU/L (ref. 0.4-5 mU/L) with
fT4 of 0.7 ng/dL (ref. 0.9-1.8 ng/dL).
What is the appropriate management?
A. Repeat thyroid tests after recovery
B. Start levothyroxine 25 μg IV daily
C. Start levothyroxine 25 μg IV daily and T3 10 μg IV daily
D. Start levothyroxine 50 μg oral daily

15. A 45-year-old woman presents to the ED complaining of palpitations. She reports heat
intolerance and unintended 5-lb weight loss in the past month. She has no chronic medical
conditions and denies ingestions. Her laboratory evaluation is notable for undetectable
TSH with fT4 of 4.1 ng/dL (ref. 0.9-1.8 ng/dL). On examination, thyroid is not enlarged,
is nontender, and no nodules are appreciated.
What is the next best diagnostic step?
A. Order radioiodine uptake scan
B. Order thyroid ultrasound
C. Order thyrotropin receptor antibodies
D. Repeat TSH and fT4

16. A 55-year-old man presents for evaluation of hyperthyroidism discovered after evaluation
for palpitations. Laboratory evaluation was notable for undetectable TSH with fT4 of 3.8
ng/dL (ref. 0.9-1.8 ng/dL). Radioiodine uptake scan is performed, revealing absent
radioiodine uptake.
Which of the following is the most likely diagnosis?
A. Graves’ disease
B. Painless thyroiditis
C. Toxic adenoma
D. TSH-producing pituitary adenoma

17. A 35-year-old woman presents for treatment of newly diagnosed Graves’ disease with eye
irritation without vision changes, diplopia, or restriction of extraocular movements. She is
the sole caregiver for a 12-month-old child and desires pregnancy in the next 6 to 12
months.
Which of the following would be the best treatment option?
A. Methimazole for 1 year
B. Propranolol alone
C. Radioiodine
D. Surgery

18. A 56-year-old man presents after CT scan with contrast with agitation, tachycardia,
hypotension, and fever to 40.5°C. Laboratory evaluation is notable for TSH 0.05 mU/L
(ref. 0.4-5 mU/L), fT4 of 3.7 ng/dL (ref. 0.9-1.8 ng/dL), and total T3 360 ng/dL (ref. 60-
181 ng/dL).
All except for which of the following are recommended as initial treatment?
A. β-blocker
B. Glucocorticoids
C. Selenium
D. Thionamide

19. A 65-year-old woman presents to primary care clinic after a thyroid nodule is seen
incidentally on a CT scan while she was hospitalized for pulmonary nodular amyloidosis.
She has history of well-controlled hypertension. She underwent a thyroid ultrasound that
revealed a single 10-mm hypoechoic nodule with microcalcifications. Her TSH is within
normal limits.
What is the most appropriate management?
A. No further evaluation indicated
B. Refer for fine needle aspiration
C. Repeat TSH and ultrasound in 6 months
D. Repeat ultrasound in 6 months

20. A 70-year-old man undergoes fine needle aspiration of 1.5 cm solid, hypoechoic nodule
with irregular margins. Cytologic results show atypia of undetermined significance
(Bethesda III).
What is the next appropriate step?
A. Perform surgery
B. Repeat fine needle aspiration with sample for molecular testing
C. Repeat TSH and ultrasound in 6 months
D. Repeat ultrasound in 6 months

21. A 79-year-old man with atrial fibrillation is started on amiodarone.


Which of the following is NOT true regarding amiodarone and thyroid function?
A. Amiodarone can result in destructive thyroiditis.
B. Amiodarone can result in hypo- and hyperthyroidism.
C. Amiodarone inhibits T4 to T3 conversion.
D. Each 100 mg amiodarone contains half of iodine contained in average diet daily.

22. A 35-year-old man with newly diagnosed hypertension presents to clinic with ongoing
elevated BPs at home despite maximum dose triple therapy. He is currently taking
metoprolol, amlodipine, and hydrochlorothiazide with minimal effect. He has no family
history of hypertension and denies symptoms. On examination, he is well-appearing. BP is
165/95 mmHg, HR 80 beats/min, T 37°C, and RR 16 breaths/min. Laboratory evaluation
is notable for creatinine 0.8 mg/dL, potassium 3.6 mmol/L, and sodium 141 mmol/L. He
denies weight gain, bruising, muscle weakness, or skin changes.
Evaluation for the most likely secondary cause would involve which of the
following?
A. Dexamethasone suppression test (DST)
B. Morning plasma renin and aldosterone
C. Renal ultrasound
D. Twenty-four hours urinary fractionated metanephrines and catecholamines

23. An aldosterone/renin ratio is checked in the patient in Question 22 with appropriate


technique and was found to be 25 (<10) with a plasma aldosterone concentration of 13

Freemedicalbooks4download
ng/dL (ref. <21 ng/dL).
What is the next appropriate step in his management?
A. Abdominal CT adrenal protocol
B. Adrenal venous sampling
C. Initiate spironolactone 100 mg daily
D. Salt suppression test

24. The patient in Questions 22 and 23 receives a salt suppression test via saline infusion in
clinic and has an aldosterone level of 12 ng/dL (ref. <5 ng/dL) after 4 hours. An
abdominal CT is performed, which shows the right adrenal gland with a 6-mm nodule and
the left adrenal gland with mild hypertrophy and a 1.1-cm nodule. Hounsfield units are
consistent with adrenal adenomas.
What is the next best step in his management?
A. Adrenal venous sampling
B. Initiate spironolactone 100 mg daily
C. Left-sided adrenalectomy
D. Right-sided adrenalectomy

25. A 67-year-old woman presents to the clinic because of 1 year of weight gain. The patient
had mild weight gain after menopause but noted a sudden increase in weight of 20 lb over
the past year. She has also noted increased fatigue and difficulty climbing the stairs. On
examination, she exhibits mild facial plethora and rounding as well as prominent central
adiposity. She has 1+ pitting edema in her legs with occasional bruises on her arms. Vital
signs show a BP 150/95 mmHg, HR 78 beats/min, RR 16 breaths/min, and body mass
index (BMI) of 32 kg/m2. Laboratory values show a creatinine of 1.1 mg/dL, potassium of
3.5 mmol/L, and sodium of 138 mmol/L. Cushing syndrome is suspected and the patient
receives a dexamethasone suppression test (DST). A morning cortisol level after a 1-mg
dexamethasone dose the night before is 6.2 μg/mL (ref. <1.8 μg/mL).
What is the next best step in her management?
A. Refer for abdominal CT scan adrenal protocol
B. Refer for high-dose (8 mg) DST
C. Refer for inferior petrosal sinus sampling
D. Repeat DST with dexamethasone level and obtain adrenocorticotropic hormone
(ACTH)

26. The patient in Question 25 receives a repeat 1-mg dexamethasone suppression test (DST).
which shows a morning cortisol of 7.3 μg/dL (<1.8 μg/dL) and an adrenocorticotropic
hormone (ACTH) level of 4 pg/mL (<0.6 pg/mL), with an appropriate dexamethasone
level. She receives an abdominal CT scan, which shows a nodular right adrenal gland
containing an 8-mm and 1.2-cm lesion and a nodular left adrenal gland containing a 1.4-
cm lesion.
What is the next best step in her management?
A. Adrenal venous sampling
B. Bilateral adrenalectomy
C. Initiation of ketoconazole 200 mg twice daily
D. Initiation of metyrapone 500 mg three times daily
27. A 53-year-old man with hypertension and obesity is admitted to the hospital for an
episode of dizziness, nausea, and vomiting. On examination, he appears in mild distress,
BP 170/100 mmHg, HR 100 beats/min, RR 20 breaths/min, and T 36.6°C. His abdomen is
soft and mildly tender in the epigastric region. In the course of his workup, he has a CT of
his abdomen and pelvis that is notable for a 4.2 × 3.5-cm left adrenal mass with smooth
borders.
What is the next best step in management of this finding?
A. Abdominal MRI with adrenal protocol
B. No further evaluation required given benign features of nodule
C. Twenty-four-hour urine fractionated metanephrines now, with plans for aldosterone,
renin, and overnight 1 mg dexamethasone suppression test (DST) in the outpatient
setting
D. Twenty-four-hour urine fractionated metanephrines, aldosterone, renin, and overnight
1 mg DST now

28. The patient in Question 27 is successfully discharged home and receives hormonal testing.
Urine fractionated metanephrines are within normal range, an aldosterone/renin ratio of 35
(ref. <10) with aldosterone level of 10 ng/dL and a cortisol level of 5.7 μg/dL (ref. <1.8
μg/dL) after dexamethasone suppression test (DST).
What is the next best step in his management?
A. Repeat abdominal CT adrenal protocol in 6 months
B. Repeat abdominal CT adrenal protocol in 12 months
C. Repeat imaging and biochemical testing in 6 months
D. Surgical referral

29. A 25-year-old woman with no chronic medical conditions presents to primary care with
intermittent headache, associated sweating, and palpitations over the past 3 months. She
states that these episodes occur randomly and are not associated with hunger, anxiety
symptoms, or panic. She does not experience any flushing, diarrhea, or abdominal pain
with the episodes. She does not use alcohol or illicit drugs and is not on any medications.
In the office, she is afebrile with HR of 90 beats/min and BP 135/90 mmHg. Her abdomen
is soft with no masses palpated. Laboratory findings show a creatinine of 1.0 mg/dL and
normal electrolytes.
What is the next best step in her management?
A. Insulin and glucose check during the next episode
B. Renin/aldosterone ratio
C. Twenty-four-hour urinary fractionated metanephrines and catecholamines
D. Urinary 5-hydroxyindoleacetic acid, chromogranin, and gastrin levels

30. The patient in Question 29’s urinary metanephrines were measured and were 3000 μg/24
h (normal: <1000 μg/24 h in hypertensive patients). The patient received an abdominal CT
scan, which revealed normal adrenal glands and otherwise was unremarkable.
What is the next best step in her management?
A. Initiation of terazosin
B. Metaiodobenzylguanidine (MIBG) I-123 scan
C. Serial monitoring of urinary metanephrines
D. Whole body CT scan

Freemedicalbooks4download
31. Metaiodobenzylguanidine (MIBG) scan was performed in the patient in Questions 29 and
30 that identified a lesion on the left neck. The patient was referred to an endocrine
surgeon and is pending surgery.
What is the next best step in her management?
A. Initiate metoprolol 50 mg twice daily
B. Initiate metyrapone 250 mg four times daily
C. Initiate nifedipine 30 mg every 8 hours
D. Initiate prazosin 1 mg every 4 hours

32. The patient in Questions 29-31 successfully receives surgery and a 2.3-cm paraganglioma
is resected from the left neck, with negative lymph nodes and local margins. The patient is
now pending discharge from the hospital.
What is the recommended follow-up?
A. Benign tumor, no follow-up indicated
B. Genetics referral, repeat imaging, and catecholamines in 1 year
C. Repeat imaging and catecholamine levels in 1 year
D. Repeat imaging in 1 year

33. A 23-year-old woman with history of hypothyroidism presents to clinic with symptoms of
extreme fatigue. The patient has noted markedly low energy levels and is barely able to
get out of bed. She is experiencing dizziness when standing and walking. She also has
been craving very salty foods like pretzels and chips. She denies illness, sick contacts,
traveling abroad, or camping. She denies alcohol, tobacco, or illicit drug use. She has a
history of hypothyroidism diagnosed at age 21 and has been on LT4 100 μg daily with
stable TSH. On examination, the patient appears ill, BP 85/60 mmHg, HR 110 beats/min,
T 37.6°C, and RR 15 breaths/min. Her abdomen is soft and nontender. She has dry, tan
skin, which she reports is darker than usual. Her examination is otherwise unremarkable.
What is the next best step in her management?
A. Cosyntropin stimulation test and thyroid function studies
B. Dexamethasone 2 mg oral dose and IV saline
C. Hydrocortisone 20 mg oral dose and encourage fluids
D. Referral to the emergency room

34. The patient in Question 33 is sent to the ED where she receives IV hydrocortisone and 3 L
normal saline. She was noted to have a potassium of 5.6 mmol/L, which recovered to 4.0
mmol/L after the hydrocortisone. She is feeling much better.
What is the next appropriate step in her management?
A. Check aldosterone and renin levels
B. Check antiadrenal antibodies
C. Check baseline adrenocorticotropic hormone (ACTH) and perform cosyntropin
stimulation test
D. Refer for abdominal CT scan adrenal protocol

35. The patient in Question 33 and 34’s morning adrenocorticotropic hormone (ACTH)
returns at 300 pg/mL (ref. 10-60 pg/mL). A cosyntropin stimulation is performed, which
shows maximal cortisol stimulation to 1.3 μg/dL (ref. >18 μg/dL).
What is the appropriate treatment for her Addison disease?
A. Dexamethasone 1 mg daily
B. Fludrocortisone 0.1 mg daily
C. Hydrocortisone 10 mg twice daily
D. Prednisone 5 mg daily and fludrocortisone 0.1 mg daily

36. A 29-year-old man, who is a carpenter, broke his radius after a fall from a ladder. His
calcium was found to be elevated at 11.3 mg/dL (normal: 8.5-10.5 mg/dL), parathyroid
hormone 75 pg/mL (normal: 10-65 pg/mL), and 25(OH)D 20 ng/mL (normal: 20-80
ng/mL).
What is the next best diagnostic step?
A. Check dual-energy x-ray absorptiometry (DXA)
B. Check neck ultrasound and sestamibi scan
C. Check serum and 24-hour urine calcium and creatinine
D. Check serum phosphate and magnesium

37. A 70-year-old man with history of hypertension, remote history of Graves’ disease and
total thyroidectomy, and a 25-pack-year history of smoking presents with increasing
fatigue, thirst, frequent urination, weight loss, and confusion. He drinks several liters of
water every day. On examination, he has double vision and is not oriented to time.
Which of the following diagnoses is highest in the differential?
A. Hypercalcemic crisis
B. Hyperosmolar hyperglycemic state
C. Hyperthyroid crisis
D. Stroke

38. TSH in the patient in Question 37 was found to be 0.9 mU/L (ref. 0.4-5 mU/L), calcium
15.0 mg/dL (ref. 8.5-10.5 mg/dL), albumin 3.0, albumin-corrected calcium 15.8 mg/dL
(ref. 8.5-10.5 mg/dL), creatinine 2.0 mg/dL, head CT normal, and glucose 80 mg/dL. His
calcium was normal a year ago at a routine blood check.
What is the most likely etiology of the hypercalcemia?
A. Graves’ disease
B. Lung cancer
C. Multiple endocrine neoplasia type 1 (MEN1)
D. Primary hyperparathyroidism

39. Chest x-ray and subsequent chest CT in the patient in Questions 37 and 38 show a central
4-cm spiculated tumor with cavitation.
How would you treat the hypercalcemia?
A. Calcitonin 200 mg IM every 12 hours
B. Infusion of parathyroid hormone/parathyroid hormone–related protein (PTHrP)
receptor antibody
C. Normal saline IV
D. Treatment of the underlying lung tumor

40. A 40-year-old African-American woman presents with dry cough for several months,
fatigue, and mild dyspnea on exertion. Chest x-rays and subsequent CT show bilateral
hilar lymphadenopathy and micronodules. Laboratory evaluation shows normal renal

Freemedicalbooks4download
function, calcium 11.5 mg/dL (8.5-10.5 mg/dL), and albumin 3.9 g/dL. Parathyroid
hormone is 5 pg/mL (10-60 pg/mL), parathyroid hormone–related protein (PTHrP) is
undetectable, 25(OH)D is 30 ng/mL (20-80 ng/mL), and 1,25(OH)D is 110 pg/mL (19-70
pg/mL).
What is the most likely diagnosis?
A. Humoral hypercalcemia of malignancy
B. Primary hyperparathyroidism
C. Sarcoidosis
D. Secondary hyperparathyroidism

41. A 51-year-old woman presents with tingling in her hands and feet and numbness around
her mouth several times a week. She has had these symptoms for about 2 months and she
has not identified any particular triggers. She is worried about multiple sclerosis and
would like to be tested for it. She only takes a multivitamin and levothyroxine since a total
thyroidectomy 3 months ago. There is no family history of any neurologic disease. Her
recent TSH was normal.
What tests would you order?
A. Calcium and albumin
B. Complete blood count (CBC)
C. Lumbar puncture with cerebrospinal fluid (CSF) evaluation
D. Urinalysis

42. A 45-year-old man who is a healthy computer engineer has a serum calcium of 8.6 mg/dL
(ref. 8.5-10.5 mg/dL), phosphorus of 1.7 mg/dL (2.5-4.5 mg/dL), and normal renal
function. He is asymptomatic but worried about his low phosphorus.
What is the next best diagnostic test?
A. Check vitamin D and parathyroid hormone
B. Genetic test for PHEX mutation
C. Measure fasting blood glucose and insulin
D. Measure FGF23

43. A 35-year-old woman with chronic postsurgical hypoparathyroidism is pregnant and is in


her second trimester. Her serum calcium is low at 6.8 mg/dL. She is on calcium 500 mg
TID and calcitriol 0.25 μg QD.
What is the next best step?
A. Check serum calcium and albumin
B. Check serum phosphate and magnesium
C. Increase her calcitriol to 0.25 μg BID
D. Increase her calcium to 1000 mg TID
E. C & D and recheck calcium 2-3 days later

44. A 65-year-old asymptomatic woman with osteoporosis (T-score in the femoral neck −2.8)
is found to have a calcium of 10.9 mg/dL (normal: 8.5-10.5 mg/dL), albumin of 3.7 g/dL,
parathyroid hormone of 70 pg/mL (normal: 10-65 pg/mL), estimated glomerular filtration
rate (eGFR) of 70 mL/min/1.73 m2, and 25(OH)D 32 ng/mL (normal: 20-80 ng/mL).
Calcium-to-creatinine clearance ratio (CCCR) is 0.04 (ref. >0.02).
What is the next best step?
A. Recommend genetic testing
B. Recommend parathyroid surgery
C. Recommend watchful waiting

45. A 65-year-old man comes into clinic for follow-up of his type 2 diabetes mellitus
(T2DM). He has been on metformin 1000 mg twice daily for 1 year and his HbA1c
recently was 8.2%. He has modified his diet and has lost 10 lb. His past medical history is
notable for CAD and hypertension. He underwent percutaneous coronary intervention
with placement of drug-eluting stent (DES) to right coronary artery 6 months ago.
What would be the next best drug to add?
A. Basal insulin
B. Glipizide 2.5 mg daily
C. Liraglutide daily
D. Rosiglitazone 4 mg daily

46. A 75-year-old man with history of T2DM and hypertension is admitted to the hospital for
a urinary tract infection and acute kidney injury with creatinine increased to 3.5 mg/dL
from baseline of 0.85 mg/dL. His home diabetes medications include metformin 1000 mg
BID and empagliflozin 25 mg daily with blood glucose levels ranging from 150 to 200
mg/dL at home. His most recent HbA1c was 8.2% 1 month ago. On examination, he is in
mild distress from abdominal pain. He weighs 100 kg, and BMI is 35 kg/m2. His blood
sugar level is persistently elevated to over 300 mg/dL in the hospital after holding home
medications.
In order to improve glycemic control, you make which of the following changes to
his current therapy?
A. Initiate 5 U regular insulin TID with regular sliding scale
B. Initiate sliding scale aspart insulin
C. Resume home medications and start sliding scale regular insulin
D. Start 25 U insulin glargine nightly with sliding scale aspart insulin

47. A 35-year-old woman with T2DM and morbid obesity presents to clinic for routine
follow-up. Her laboratory evaluation reveals HbA1c of 8.5%, creatinine 0.75 mg/dL, and is
otherwise unremarkable. Her current medications include metformin 1000 mg twice daily
and daily multivitamin. Since her last visit 3 months ago, she has modified her diet and
increased her physical activity to 30 minutes of moderate exercise three times weekly. She
has lost 2 lb.
What changes would you recommend?
A. Continue current medical therapy and encourage lifestyle modification
B. Start glargine 20 U nightly
C. Start glipizide 2.5 mg daily
D. Start liraglutide 0.6 mg subcutaneous (SC) once daily

48. A 27-year-old man with type 1 diabetes mellitus (T1DM) presented to ED with nausea,
vomiting, and altered mental status after missing 2 days of insulin due to insurance issues.
He received fluid resuscitation, potassium repletion to maintain serum potassium between
3.3 and 5.3 mEq/L, and was started on insulin IV infusion after receiving IV bolus of 0.1
U per kg. Initial labs with venous pH of 7.15, anion gap of 30, and bicarbonate of 12

Freemedicalbooks4download
improved after 12 hours to pH of 7.38, anion gap of 12, and bicarbonate of 21. The insulin
IV infusion was discontinued and home regimen of insulin glargine daily in the morning
with insulin Humalog at meals was restarted.
What parameters, in addition to blood glucose <200 mg/dL, do the American
Diabetes Association (ADA) recommend be met when tapering IV insulin and
overlapping with SC insulin?
A. Able to eat and hemodynamically stable
B. Hemodynamically stable, venous pH >7.30
C. Serum anion gap <12 mEq/L and serum bicarbonate ≥15 mEq/L
D. Venous pH >7.3, K > 3.3 and <5.3 mEq/L

49. A 54-year-old man with history of hypertension presents with episode of confusion and
slurred speech. In the ED, he is found to have fingerstick glucose of 33 mg/dL. His
confusion improves after he drinks 8 ounces of orange juice. Additional history gathered
from the patient’s wife reveals the patient has had episodes of confusion often in the
morning for about 1 month.
Which of the following labs drawn during hypoglycemia are most consistent with
endogenous hyperinsulinemia?
Insulin (pmol/L) Proinsulin (pmol/L) C-peptide (ng/mL) β-hydroxybutyrate (mmol/L) Oral hypoglycemic agent screen
A. 3 4 0.4 6.5 negative
B. 20 14 0.6 <2.7 positive
C. 36 <2 <0.2 <2.7 negative
D. 38 25 0.8 <2.7 negative

50. A 75-year-old woman with unknown medical history presents to the emergency room
with altered mental status. Her family states she has been visiting for Thanksgiving and
has been feeling unwell for several days. On examination, her BP is 105/75 mmHg, HR is
115 beats/min, and RR is 18 breaths/min. She is stuporous with dry mucous membranes
and tachycardia, but otherwise the examination is unremarkable. Her laboratory
evaluation is most notable for venous blood gas (VBG) with pH of 7.35, basic metabolic
panel (BMP) with glucose of 738 mg/dL, serum bicarbonate of 22 mEq/L, and anion gap
of 12. Serum osmolality is 330 mOsm/kg and β-hydroxybutyrate is negative.
Initial management of hyperosmolar hyperglycemic state involves all BUT which
of the following?
A. Electrocardiogram (ECG)
B. Infectious evaluation
C. Intravenous fluid resuscitation
D. Transthoracic echocardiogram

51. A 56-year-old man with T2DM on metformin and stage 3 chronic kidney disease presents
to clinic for follow-up of diabetes care. His most recent HbA1c is 8.0%. He is exercising
30 minutes, five times per week, and has made dietary changes to improve his glycemic
control. He is interested in the medications he has heard about that protect the kidneys and
treat diabetes.
All of the following are true regarding diabetic nephropathy and sodium-glucose
cotransporter 2 (SGLT2) inhibitors EXCEPT?
A. Diabetic nephropathy is the most common cause of end-stage renal disease
worldwide.
B. SGLT2 inhibitors have been shown to reduce urinary albumin-to-creatinine ratio.
C. SGLT2 inhibitors increase hyperfiltration.
D. Studies evaluating SGLT2 inhibitors in patients with moderate renal impairment have
shown an initial decrease in eGFR, but then a lesser decrease over time compared
with placebo.

52. A 65-year-old woman presents to clinic for follow-up of T2DM. She is currently taking
metformin at maximum dose and dulaglutide 1.5 mg weekly. Her most recent HbA1c is
9.5% while on this regimen regularly for 5 months. She wakes up several times during the
night to urinate and is willing to start daily injections for improved glycemic control and
to improve nocturia.
What are the appropriate instructions for starting insulin?
A. Start long-acting insulin at 0.1 to 0.2 IU/kg/d and prandial insulin at 4 U per meal
B. Start long-acting insulin at 0.1 to 0.2 IU/kg/d with instructions to increase 2 U every
3 days until reaching fasting glucose target
C. Start long-acting insulin at 20 U per day and continue this dose until follow-up while
keeping glucose log
D. Start long-acting insulin at 20 U per day and reduce metformin dose to 500 mg twice
daily

53. A 58-year-old man with T2DM and heart failure with ejection fraction of 35% presents
for follow-up. He is physically active, and his diabetes is fairly well-controlled with HbA1c
of 7.5% on maximally tolerated metformin. He has no history of kidney disease or
atherosclerotic heart disease.
What changes would be appropriate to make to antihyperglycemic agents today?
A. Continue metformin and add dapagliflozin
B. Continue metformin and add sitagliptin
C. Stop metformin and start dapagliflozin
D. Stop metformin and start basal insulin

54. A 19-year-old woman presents to clinic after recent hospitalization for diabetic
ketoacidosis during which she was diagnosed with T1DM.
What additional screening tests are recommended at the time of diagnosis?
A. Lipid profile
B. Lipid profile and TSH
C. Lipid profile, tissue transglutaminase, and TSH
D. TSH

55. A 58-year-old man with T2DM on basal-bolus insulin regimen presents for urgent visit
after 1 week of worsened hyperglycemia and variable response to prandial insulin
injections (he uses vial and syringe). He otherwise feels well and has changed his injection
sites. He had left his insulin vial overnight in the car on a cold night.
What should have been done with this insulin vial?
A. Allow insulin to thaw in refrigerator
B. Allow insulin to warm to room temperature before administration
C. Discard vial and open new vial

Freemedicalbooks4download
D. Shake insulin thoroughly before use

56. A 35-year-old man with history of T1DM managed with insulin pump presents to the
emergency room obtunded after a motor vehicle accident (MVA). His temperature is
36°C, BP is 80/55 mmHg, HR is 120 beats/min, and RR is 22 breaths/min. The initial
finger stick glucose is 150 mg/dL and his labs are not consistent with diabetic
ketoacidosis.
What is the appropriate management of this patient’s diabetes?
A. Continue insulin pump at current settings
B. Continue insulin pump at current settings and provide correctional with SC insulin on
sliding scale
C. Discontinue insulin pump and administer long-acting insulin based on weight
D. Discontinue insulin pump and transition to IV insulin infusion

57. A 67-year-old man with history of hypertension and CAD presents for evaluation
regarding lipid management. He experienced a myocardial infarction (MI) 6 months ago
requiring two drug-eluting stents. He was placed on atorvastatin 80 mg at that time, which
he has been taking consistently. His pretreatment low-density lipoprotein (LDL) was 162
mg/dL, and a lipid panel checked 2 weeks ago showed an LDL of 85 mg/dL. He currently
feels well with no chest pain, myalgias, or leg cramping.
What is the next best step in his management?
A. Continue atorvastatin 80 mg daily and add ezetimibe 10 mg daily
B. Continue atorvastatin 80 mg daily and add fenofibrate 145 mg daily
C. Discontinue atorvastatin and initiate evolocumab 140 mg every 2 weeks
D. Switch to pravastatin 40 mg

58. A 28-year-old man presents to lipid clinic due to concerns regarding family history. His
father experienced MI at 40 years of age and died at age 48 from cardiovascular causes.
His 34-year-old brother also recently experienced an MI requiring stent placement. He
feels well and has no chest pain. On examination, there is no evidence of tendon
xanthomas. A lipid panel is performed, which shows a total cholesterol of 350 mg/dL,
LDL of 250 mg/dL, high-density lipoprotein (HDL) 35 mg/dL, and triglycerides of 200
mg/dL.
What would be recommended treatment at this time?
A. Atorvastatin 80 mg daily
B. Evolocumab 140 mg every 2 weeks
C. No treatment indicated
D. Simvastatin 10 mg daily

59. A 45-year-old man with history of T2DM, hypertension, and nonalcoholic fatty liver
disease presents with acute-onset abdominal pain radiating to his back with nausea and
vomiting. A CT scan is performed in the ED, which shows evidence of acute pancreatitis.
On examination, the patient is in distress due to abdominal pain; T 37.7°C, HR 115
beats/min, RR 25 breaths/min, and BP 110/75 mmHg. Laboratory work shows a creatinine
of 1.5 mg/dL, blood glucose of 230 mg/dL, and a WBC of 20 000/μL. A lipid panel is
performed, which shows a triglyceride level of 2000 mg/dL.
Other than aggressive hydration and NPO status, what is the next best step in
management for this patient’s triglycerides?
A. Initiate apheresis
B. Initiate fenofibrate 145 mg daily
C. Initiate fenofibrate 145 mg daily and atorvastatin 80 mg daily
D. Initiate intravenous insulin and dextrose

ANSWERS

1. The correct answer is: A. Administer hydrocortisone 50 mg IV. This patient is


experiencing pituitary apoplexy. The patient will require evaluation by neurosurgery to
determine management of the pituitary lesion and the hemorrhage; however, the patient
will require medical stabilization first. This patient has hypotension, tachycardia, and mild
hyponatremia, thus exhibiting symptoms concerning for adrenal insufficiency. Acute
adrenal insufficiency can be life-threatening and requires immediate attention to prevent
hemodynamic collapse. Administration of IV fluids is often needed but was not offered as
a choice here. Evaluations for adrenal insufficiency are important but should occur after
medical stabilization to prevent delay in the administration of glucocorticoids.

2. The correct answer is: B. Baseline: 0.5 30 min: 15.3 60 min: 20.5. The patient is
experiencing acute-central adrenal insufficiency, whereby sudden loss of pituitary
function prevents the release of adrenocorticotropic hormone (ACTH). Under these
conditions, the adrenal glands have not yet atrophied from lack of stimulation and thus
remain sensitive to ACTH. Answer B is correct because the patient should exhibit no
stimulation at baseline with a low-morning cortisol, however she is responsive to
cosyntropin (an ACTH analog). Answer A suggests primary adrenal pathology due to
profound lack of cortisol production. Answer C suggests chronic secondary adrenal
insufficiency with some evidence of adrenal atrophy. Answer D represents a normal
stimulation test. It is important to remember that cosyntropin stimulation tests can be
falsely reassuring in situations of acute-central adrenal insufficiency as the adrenal glands
have not had time to atrophy.

3. The correct answer is: A. Order a low-dose dexamethasone suppression test (DST).
This patient is exhibiting classic signs and symptoms of Cushing syndrome, including
hypertension, hyperglycemia, abdominal adiposity, easy bruising, and edema. She
warrants confirmation of these clinical findings and assessment of the etiology of her
hypercortisolism. Screening examinations for Cushing syndrome include the DST, the
late-night salivary cortisol test, and the 24-h urinary cortisol collection. It is important to
note that a random or morning cortisol is not a helpful tool in diagnosing Cushing
syndrome. Imaging should be performed only after biochemical confirmation of disease
and testing to suggest a pituitary source.

4. The correct answer is: D. Refer to radiology for inferior petrosal sinus sampling.
This patient has clinical and biochemical evidence of Cushing syndrome. Once the
evidence of hypercortisolemia is confirmed, the diagnostic evaluation focuses on whether
the disease is due to a central or peripheral cause. The adrenocorticotropic hormone

Freemedicalbooks4download
(ACTH) level measured during the dexamethasone suppression test (DST) was detectable,
suggesting an ACTH-dependent process. This could be either a pituitary source
(adenoma) or Cushing syndrome from an ectopic ACTH-producing tumor. The method to
determine if the ACTH production localizes to the pituitary gland is through inferior
petrosal sinus sampling, which measures ACTH values in the venous beds of the pituitary.
Moving forward with medical or surgical therapy might lead to misdiagnosis of the type
of Cushing this patient experiences. Of note, the presence of a lesion on pituitary MRI
does not guarantee that it is the source of hypercortisolism, as incidental lesions are not
uncommon.

5. The correct answer is: A. Checking growth hormone after an oral glucose load. This
patient is exhibiting classic signs of acromegaly, including enlargement of the hands and
feet, soft tissue edema, changes in facial structure, and development of skin tags. The
patient has strong clinical findings of acromegaly; however, they require clinical
confirmation. One component is to check an IGF-1 level, which if elevated suggests
acromegaly. However, this test can be affected by obesity, illness, and medications. The
gold standard for the diagnosis of acromegaly is the growth hormone suppression test,
which involves checking growth hormone levels after an oral glucose tolerance test. Lack
of suppression after a glucose load is diagnostic for acromegaly. Random growth hormone
levels are not diagnostic in acromegaly due to their large physiologic variability. The
glucagon stimulation test and insulin tolerance test are used for the diagnosis of growth
hormone deficiency.

6. The correct answer is: C. Refer to neurosurgery for resection. Acromegaly is almost
always due to excess growth hormone secretion from a pituitary adenoma. The first-line
therapy is surgical resection as it may lead to cure. If the tumor is inoperable due to size or
involvement of central nervous system (CNS) structures, or incomplete resection is
achieved, then medical therapy can be added for control of symptoms. Pegvisomant is a
growth hormone receptor blocker and lanreotide is a somatostatin analog, both of which
can successfully reduce IGF-1 levels in uncontrolled acromegaly.

7. The correct answer is: C. Pituitary mass. The patient is exhibiting signs and symptoms
of hyperprolactinemia, including amenorrhea, galactorrhea, and headaches.
Hyperprolactinemia is frequently due to prolactin-secreting pituitary adenomas:
“prolactinomas.” Other pituitary pathologies that can produce elevations in prolactin
include empty sella syndrome and stalk compression/deviation. Ovarian tumors and
adrenal hyperplasia are more commonly associated with hyperandrogenism, rather than
hyperprolactinemia. Medication-induced prolactin elevations are common; however, there
is no evidence this patient is taking such a medication. It is always important to evaluate
for use of dopamine antagonists as antipsychotics as these frequently raise prolactin.

8. The correct answer is: A. Initiate cabergoline 0.5 mg weekly. This patient has evidence
of a macroprolactinoma. Prolactin-secreting adenomas frequently respond excellently to
dopamine agonist therapy and do not require surgical management as first-line therapy.
The patient’s symptoms of amenorrhea and galactorrhea are suggestive that she is
experiencing symptomatic hyperprolactinemia and should receive treatment. Cabergoline
is a once-weekly dopamine agonist that is a well-tolerated treatment for
hyperprolactinemia. If the patient was experiencing only amenorrhea as a symptom and
was not interested in conceiving, then an OCP would be a reasonable choice.

9. The correct answer is: A. Abdominal MRI. This patient’s presentation is consistent
with hyperparathyroidism, with symptomatic hypercalcemia in the context of elevated
parathyroid hormone levels. She also is exhibiting headache and amenorrhea, which is
concerning for a pituitary lesion. This is further supported by the evidence of
hypothyroidism that is central in origin with low TSH and low fT4. The combination of
hyperparathyroidism and pituitary mass is concerning for multiple endocrine neoplasia
type 1 (MEN1). Patients with MEN1 are at risk for developing neuroendocrine tumors of
the pancreas as well and should be screened for pancreatic neuroendocrine tumors at the
time of diagnosis. This is best done through the use of abdominal MRI. Pelvic ultrasound
and colonoscopy are important for screening tumors in other oncogenic syndromes but are
not indicated here, nor is an echocardiogram.

10. The correct answer is: A. TSH. In most patients with signs or symptoms of
hypothyroidism, TSH should be the initial test. At some hospitals, TSH can be ordered
with reflexive testing, meaning that fT4 is checked only if the TSH is elevated or TSH can
be ordered alone and can be repeated with fT4 if the initial TSH is elevated. T3, either
total or free, is not helpful in the management of outpatient hypothyroidism.

11. The correct answer is: C. Levothyroxine 125 μg daily. The formula 1.6 μg/kg body
weight per day is used to estimate the initial replacement dose in adults. The full estimated
dose can be used in young and healthy patients without history of cardiac disease.

12. The correct answer is: A. Levothyroxine 25 μg. Initial levothyroxine dosing for older
adults with history of CAD or cardiopulmonary disease should be low (12.5-25 μg daily)
with gradual increases based on symptoms and TSH levels. Full calculated daily
levothyroxine dose based on weight is not recommended to minimize the risk of
precipitating cardiac events by increasing myocardial demand.

13. The correct answer is: C. Start hydrocortisone 100 mg every 8 hours and 100 μg T4
IV daily. Myxedema coma is an endocrine emergency. Until coexisting adrenal
insufficiency is ruled out, patients should be empirically treated for adrenal insufficiency
while treating severe hypothyroidism. As myxedema coma is associated with high
mortality (~40%), treatment should be initiated based on clinical suspicion, even prior to
return of laboratory results. Supportive care and evaluation for triggers are fundamental
for the management of myxedema coma, in addition to treatment of hypothyroidism and
possible adrenal insufficiency.

14. The correct answer is: A. Repeat thyroid tests after recovery. In critically ill patients,
assessment of thyroid function is difficult, as nonthyroidal illness can cause acquired
central hypothyroidism that resolves with treatment of primary illness. Thyroid studies
should be checked in critically ill patients only if there is high suspicion for thyroidal
illness. This patient’s thyroid studies likely reflect nonthyroidal illness (sick euthyroid)
and will resolve with treatment of her infection.

Freemedicalbooks4download
15. The correct answer is: A. Order radioiodine uptake scan. For the evaluation of
hyperthyroidism, indicated in this patient by decreased TSH and elevated fT4, radioiodine
uptake scan is the appropriate first test. The most common etiology of hyperthyroidism is
Graves’ disease. Radioactive iodine uptake in Graves’ disease shows diffuse
homogeneous uptake.

16. The correct answer is: B. Painless thyroiditis. Painless thyroiditis is a transient
hyperthyroidism resulting from release of stored thyroid hormone and is thought to be a
form of Hashimoto thyroiditis. Radioiodine uptake indicates synthesis of thyroid hormone
within the gland, so causes of hyperthyroidism resulting from excess stimulation or de
novo synthesis of thyroid hormone have high radioiodine uptake. Low radioiodine uptake
is seen in hyperthyroidism caused by destruction or inflammation of the thyroid and
release of preformed hormone.

17. The correct answer is: D. Surgery. The American Thyroid Association (ATA)
emphasizes discussing all treatment options for Graves’ disease with patients as there is
no best treatment. In this patient, radioiodine would be contraindicated as she is the sole
caregiver of a child and would not be able to follow radiation precautions. Antithyroid
drugs are teratogenic and should be avoided when possible in women desiring pregnancy.
Surgery provides cure of hyperthyroidism and would be the best option for this patient.

18. The correct answer is: C. Selenium. Thyroid storm is an endocrine emergency that
requires prompt treatment. The American Thyroid Association (ATA) recommends
multimodal treatment with β-adrenergic blockade, thionamide, inorganic iodide,
corticosteroids, cooling with acetaminophen and cooling blankets, volume resuscitation,
nutritional support, and respiratory care.

19. The correct answer is: B. Refer for fine needle aspiration. Fine needle aspiration
should be performed on nodules ≥1 cm if they are solid or hypoechoic and they have one
or more suspicious sonographic characteristics: irregular margins, microcalcifications,
taller than wide shape, and/or rim calcifications. In this patient, who has a 10-mm
hypoechoic nodule with microcalcifications, fine needle aspiration should be pursued.

20. The correct answer is: B. Repeat fine needle aspiration with sample for molecular
testing. When the results of an fine needle aspiration are indeterminate (atypia of
unknown significance, follicular lesion of undetermined significance), additional
evaluation is required. After a biopsy with indeterminate cytology, the American Thyroid
Association (ATA) recommends either molecular testing, if an extra sample was collected
during initial biopsy, or repeat fine needle aspiration at a 6- to 12-week interval. If the
repeat cytology is indeterminate, molecular testing is performed.

21. The correct answer is: D. Each 100 mg amiodarone contains half of iodine contained
in average diet daily. Amiodarone is ~37% iodine by weight. Each 100 mg tablet
contains 3 mg of inorganic iodine, which is 10 times the amount of iodine consumed daily
by the average American.

22. The correct answer is: B. Morning plasma renin and aldosterone. Hyperaldosteronism
is present in 11% of patients with hypertension refractory to three drugs. In this patient,
with resistant hypertension without a family history and without symptoms to suggest
hypercortisolism or a pheochromocytoma, the most likely etiology of secondary
hypertension is primary hyperaldosteronism. The first step in the evaluation of
hyperaldosteronism is measurement of morning plasma renin and aldosterone as a
screening test.

23. The correct answer is: D. Salt suppression test. This patient exhibits an
aldosterone/renin ratio that is elevated; a ratio >20 is suspicious of primary
hyperaldosteronism. In the rare cases where the diagnosis is unequivocal (frank
hypokalemia on presentation and aldosterone >20 μg/mL), no further testing is required.
In this case, however, confirmation of primary aldosteronism is required. This can be done
with the salt suppression test. Salt suppression test can be performed in the ambulatory
setting with salt tablets or via infusion of IV saline in clinic. The addition of saline over 4
hours should suppress aldosterone levels <5 ng/dL in normal individuals. Lack of
suppression is diagnostic of primary hyperaldosteronism.

24. The correct answer is: A. Adrenal venous sampling. Imaging reveals multiple adrenal
adenomas of unclear significance. Assuming that the larger lesion or more
hypertrophied/nodular adrenal gland is the source of the aldosterone may lead to
erroneous diagnosis. The correct approach would be to perform adrenal venous sampling
to determine the source of the aldosterone. Once the source is localized, then the patient
can be referred to surgery for subtotal or total adrenalectomy. Medical therapy is utilized
in situations where surgical management is technically difficult or refused.

25. The correct answer is: D. Repeat DST with dexamethasone level and obtain
adrenocorticotropic hormone (ACTH). The patient’s DST is concerning for lack of
suppression, as dexamethasone would expect to bring morning cortisol levels <1.8 μg/dL.
It is important to evaluate whether the patient appropriately took, absorbed, and
metabolized dexamethasone, however, as compliance or hypermetabolism may explain
the failed test. If cortisol levels are truly not suppressed, then determine whether the
hypercortisolism is ACTH-dependent or not. A nonsuppressed ACTH level would suggest
an ACTH-dependent process such as a pituitary mass or ACTH-producing tumor. A
suppressed ACTH would suggest an autonomous source of cortisol production such as an
adrenal adenoma.

26. The correct answer is: A. Adrenal venous sampling. The patient exhibited
adrenocorticotropic hormone (ACTH)-independent Cushing syndrome and received
abdominal imaging to look for adrenal sources. Bilateral nodularity and lesions in this
patient make it difficult to localize the main source for the glucocorticoids. As with
mineralocorticoid excess, adrenal venous sampling can be helpful to determine which
adrenal gland is most involved in excess production of glucocorticoids. If the source is not
lateralized, then the other three options (one surgical, two medical) are reasonable to
manage the hypercortisolism.

27. The correct answer is: C. Twenty-four-hour urine fractionated metanephrines now,
with plans for aldosterone, renin, and overnight 1 mg dexamethasone suppression

Freemedicalbooks4download
test (DST) in the outpatient setting. All patients with adrenal incidentalomas should be
evaluated for hormonal hyperfunction. Hormonal evaluation includes workup for
pheochromocytoma, subclinical Cushing syndrome, and, if hypertensive or with
unexplained hypokalemia, hyperaldosteronism. In this scenario, given the high degree of
clinical suspicion for pheochromocytoma and the seriousness of the condition, it would be
recommended to perform evaluation for pheochromocytoma as soon as possible. The rest
of the workup should be performed in the outpatient setting when the patient is at their
baseline health, as acute illness may induce false positives. An abdominal MRI does not
add further information at this time.

28. The correct answer is: D. Surgical referral. Imaging characteristics that suggest
malignancy include irregular shape, heterogeneous density, diameter >4 cm, and
calcifications. For nodules with benign appearance (<10 HU or relative washout >40%),
repeat imaging after 6 to 12 months should be performed. In this case, the tumor is >4 cm,
making the risk for adrenocortical carcinoma much higher and warranting prophylactic
adrenalectomy to prevent progression to adrenocortical carcinoma. Of note, the patient
failed the dexamethasone suppression test (DST) and had elevated aldosterone/renin ratio,
suggesting overproduction of glucocorticoids and mineralocorticoids. Adrenocortical
carcinomas can produce multiple hormones, unlike adenomas which usually secrete only
one.

29. The correct answer is: C. Twenty-four-hour urinary fractionated metanephrines and
catecholamines. The patient’s presentation is most concerning for catecholamine excess.
While sweating and palpitations may be associated with hypoglycemia, the patient is not
describing Whipple triad. The symptoms are also less likely due to a gut neuroendocrine
tumor or carcinoid syndrome as there is no flushing or gastrointestinal (GI) symptoms.
The patient warrants evaluation for pheochromocytoma and paraganglioma by measuring
urinary fractionated metanephrines and catecholamines. A positive value warrants further
evaluation with imaging.

30. The correct answer is: B. Metaiodobenzylguanidine (MIBG) I-123 scan. Elevated
catecholamines can also be produced by tumors of the sympathetic chain known as
paragangliomas. They are extra-adrenal and require broader imaging to identify.
Frequently multiple modalities are required until they are identified, with MIBG,
fluorodeoxyglucose-positron emission tomography (FDG-PET), and octreotide scans
having different sensitivities for identifying paragangliomas. As catecholamine excess has
been confirmed, serial monitoring or prophylactic treatment is inappropriate at this time.

31. The correct answer is: D. Initiate prazosin 1 mg every 4 hours. The current
preoperative practice for catecholamine-secreting tumors is to provide α-blockade,
preventing catastrophic α-adrenergic stimulation during manipulation of the tumor. This
can be performed with nonselective α-blockers, such as phenoxybenzamine, or with
selective agents such as prazosin. Patients need dose titration until they develop
orthostatic hypotension as a marker for sufficient blockage. Exclusive β-blockade should
never be performed in patients with catecholamine-secreting tumors to prevent
cardiovascular collapse from unopposed α-stimulation. Metyrosine prevents the
production of catecholamines and is used in the management of metastatic
pheochromocytoma.

32. The correct answer is: B. Genetics referral, repeat imaging, and catecholamines in 1
year. As the field of genetics has advanced, more gene mutations have been identified that
increase the risk for paraganglioma/pheochromocytoma. It is estimated that >20% of
pheos are genetic/familial in origin; therefore, genetic counseling and testing are of high
value in this patient population. Patients with catecholamine-secreting tumors should be
regularly screened with imaging and catecholamine levels to ensure no evidence of
recurrence, particularly in patients with known genetic mutations such as multiple
endocrine neoplasia 2A (MEN2A), Von Hippel-Lindau syndrome, and succinate
dehydrogenase mutations.

33. The correct answer is: D. Referral to the emergency room. The patient is presenting
with signs of adrenal insufficiency, including fatigue, hypotension, and orthostasis. The
presence of hyperpigmentation and salt cravings suggest both a glucocorticoid and
mineralocorticoid deficit. In the context of a young woman with hypothyroidism, the most
likely cause would be Addison disease. The patient is exhibiting significant hemodynamic
symptoms and may also have underlying severe electrolyte abnormalities such as
hyperkalemia and hyponatremia. Her presentation is concerning for adrenal crisis and
requires prompt management with IV hydrocortisone, fluid resuscitation, and correction
of electrolyte abnormalities.

34. The correct answer is: C. Check baseline adrenocorticotropic hormone (ACTH) and
perform cosyntropin stimulation test. The clinical picture is strongly suggestive of
Addison disease; however, the patient requires biochemical confirmation of adrenal
insufficiency. An elevated ACTH (frequently in the hundreds) is consistent with adrenal
failure. A cosyntropin stimulation test would be expected to show a minimal response to
cosyntropin. Once the diagnosis is confirmed, the patient will require discussions
regarding appropriate glucocorticoid and mineralocorticoid counseling. 21-Hydroxylase
antibodies can also be checked, as their presence is associated with Addison disease but is
not necessary for diagnosis. An abdominal CT scan can be used to identify other
etiologies of adrenal failure but is of low utility for known autoimmune adrenalitis.

35. The correct answer is: D. Prednisone 5 mg daily and fludrocortisone 0.1 mg daily.
With her new diagnosis, the patient will require replacement of both glucocorticoids and
mineralocorticoids for life. Prednisone and fludrocortisone cover this requirement. While
hydrocortisone has both glucocorticoid and mineralocorticoid effects, the replacement
dose of hydrocortisone is unlikely to provide sufficient mineralocorticoid coverage and is
typically supplemented with fludrocortisone. The lowest possible dose of both is
recommended to prevent the complications of excess glucocorticoid use and
aldosteronism.

36. The correct answer is: C. Check serum and 24-hour urine calcium and creatinine.
Primary hyperparathyroidism is unusual in young men and raises the possibility of a
familial form of hypercalcemia. Familial hypocalciuric hypercalcemia needs to be ruled
out, as parathyroidectomy should be avoided in familial hypocalciuric hypercalcemia. All
parathyroid cells have a defect in the calcium-sensing receptor, and removing a

Freemedicalbooks4download
parathyroid gland will not change the disease. Currently, calculating the calcium-to-
creatinine clearance ratio (CCCR) is the standard test, and a 24-hour urine calcium,
creatinine, and concomitant serum calcium and creatinine measurements are needed.
Magnesium and phosphate, while typically checked, will not contribute much to the
diagnostic question above. Imaging tests for parathyroid gland localization may be
performed after familial hypocalciuric hypercalcemia is ruled out. He suffered a traumatic,
but not a fragility fracture (defined as a fall from standing height). Dual-energy x-ray
absorptiometry (DXA) is not indicated at this stage of diagnostic workup.

37. The correct answer is: A. Hypercalcemic crisis. This patient’s presentation with acute
neurologic symptoms and signs requires emergent evaluation. While his presentation
could be associated with hyperthyroidism, the history of thyroidectomy without
mentioning medication changes makes this diagnosis unlikely. The fact that he is able to
drink a lot of water makes hyperosmolar hyperglycemic state less likely. Thirst, frequent
urination, and weight loss are not characteristics of a stroke. Hyponatremia should be
ruled out, but this was not a choice given. Hypercalcemia can cause all of the symptoms
and is high on the differential diagnosis. Lethargy and confusion can be seen in severe
hypercalcemia, and weakness of extraocular muscles can lead to diplopia.

38. The correct answer is: B. Lung cancer. Severe hypercalcemia, or hypercalcemic crisis,
can have several causes, but hypercalcemia of malignancy is most likely in this patient
with a strong history of tobacco use. A normal calcium a year ago practically excludes
primary hyperparathyroidism, including multiple endocrine neoplasia type 1 (MEN1)-
associated hyperparathyroidism. His TSH is normal, presumably through appropriate
therapy with levothyroxine, excluding hyperthyroidism (from Graves’ disease or other
etiologies).

39. The correct answer is: C. Normal saline IV. Severe hypercalcemia needs to be treated
immediately. Aggressive hydration with IV saline is the most effective intervention. Large
doses are often needed. Calcitonin is sometimes used in the acute treatment of
hypercalcemia, but its effect is relatively weak and does not last more than a few days
because of tachyphylaxis. Bisphosphonates (if renal function allows) and denosumab are
effective in the treatment of hypercalcemia of malignancy but do not work immediately.
Parathyroid hormone (PTH)/PTH-related protein (PTHrP) receptor antibodies are not
clinically available. The management of the underlying lung tumor needs to be addressed,
but should not delay the acute treatment of the hypercalcemia.

40. The correct answer is: C. Sarcoidosis. Sarcoidosis, an inflammatory disease


characterized by the presence of noncaseating granulomas, can affect almost all organs,
can be asymptomatic, or is associated with a wide range of symptoms and presents a
diagnostic challenge. Black women have a higher incidence of sarcoidosis. This patient’s
presentation is classic for pulmonary sarcoidosis, but a diagnostic evaluation that typically
includes a tissue biopsy is needed. Hypercalcemia in sarcoidosis is due to the parathyroid
hormone–independent conversion of 25-hydroxy vitamin D to the active 1,25-dihydroxy
vitamin D. Primary or secondary hyperparathyroidism presents with elevated parathyroid
hormone (and serum calcium is not elevated in secondary hyperparathyroidism). Humoral
hypercalcemia of malignancy is caused by elevated parathyroid hormone–related protein
(PTHrP).

41. The correct answer is: A. Calcium and albumin. This is a typical presentation of
hypocalcemia with neuromuscular irritability. If her (albumin-adjusted) serum calcium
turns out to be low and her parathyroid hormone is low or inadequate, she has postsurgical
hypoparathyroidism and not multiple sclerosis.

42. The correct answer is: A. Check vitamin D and parathyroid hormone. This patient
had an undetectable vitamin D level with secondary hyperparathyroidism, a common
condition that is characterized by low-to-normal serum calcium (due to low calcium
absorption) and low serum phosphorus (due to the elevation of parathyroid hormone).
Treatment with vitamin D supplementation normalizes these findings. PHEX is mutated in
XLH, a hereditary form of rickets. FGF23 is elevated in several rare types of
hypophosphatemia. Vitamin D deficiency is much more common and should be ruled out
first. Insulin can lead to an intracellular shift of phosphate, but that would play a role in
glucose infusions.

43. The correct answer is: A. Check serum calcium and albumin. During pregnancy,
serum albumin is typically decreased due to an increase in plasma volume. Total serum
calcium has to be corrected for albumin. Her albumin is 2.0 g/dL and her adjusted calcium
is 8.4 mg/dL and, therefore, within range for hypoparathyroid patients of low to low-
normal. In the absence of symptoms, no change in treatment is needed.

44. The correct answer is: B. Recommend parathyroid surgery. She has primary
hyperparathyroidism, a disease with an incidence of >100 000 new cases annually in the
United States. It is typically seen in postmenopausal women. In familial hypocalciuric
hypercalcemia, calcium-to-creatinine clearance ratio (CCCR) typically is <0.01; genetic
testing is, therefore, not indicated. Criteria for the surgical treatment of asymptomatic
primary hyperparathyroidism include any of the following: age <50 years, serum calcium
>11.5 g/dL, CrCl <60 or dual-energy x-ray absorptiometry (DXA) T-score <−2.5, or
history of fragility fracture. She has osteoporosis and parathyroidectomy is recommended.

45. The correct answer is: C. Liraglutide daily. In this patient with known cardiovascular
disease, according to the 2018 American Diabetes Association (ADA) guidelines when
adding a second medication to metformin and lifestyle management to improve glycemic
control, it is recommended to add therapy validated to improve heart health. Of the listed
options, the glucagon-like peptide-1 receptor agonist liraglutide is the only agent approved
by the Food and Drug Administration (FDA) to reduce the risk of cardiovascular death in
adult patients with T2DM.

46. The correct answer is: D. Start 25 U insulin glargine nightly with sliding scale aspart
insulin. This patient appropriately had his home antihyperglycemic medications held on
admission, given his acute illness and kidney injury. The extent of his underlying insulin
resistance, both due to metabolic factors and from his acute illness, suggest that he will
require insulin during the hospitalization. For patients that you anticipate a significant
insulin requirement, initiating standing insulin will provide better glycemic control than
correctional doses alone. Starting basal insulin with insulin glargine or neutral protamine

Freemedicalbooks4download
Hagedorn (NPH) would be a reasonable approach in someone with the glycemic pattern
above.

47. The correct answer is: D. Start liraglutide 0.6 mg subcutaneous (SC) once daily. This
patient is not currently meeting her HbA1c goal of 7.0%, so it is appropriate to add an
additional therapy for improved glycemic control. Given her comorbid obesity, liraglutide
is the most appropriate option, as liraglutide is Food and Drug Administration (FDA)-
approved for the treatment of obesity. Sulfonylureas and insulin are associated with
weight gain.

48. The correct answer is: C. Serum anion gap <12 mEq/L and serum bicarbonate ≥15
mEq/L. The American Diabetes Association (ADA) guidelines for diabetic ketoacidosis
recommend IV insulin be tapered and a multiple-dose SC insulin schedule be started when
blood glucose is <200 mg/dL and at least two of the following parameters are met: (1)
serum anion gap <12 mEq/L (or at the upper limit of normal for the local laboratory), (2)
serum bicarbonate ≥15 mEq/L, and (3) venous pH >7.30. In addition, it is preferred for a
patient to remain on IV infusion if unable to eat. The IV infusion should be continued for
1 to 2 hours after initiation of SC insulin.

49. The correct answer is: D. Insulin = 38 pmol/L, proinsulin = 25 pmol/L, C-peptide =
0.8 ng/mL, β-hydroxybutyrate <2.7 mmol/L, oral hypoglycemic agent screen =
negative. In the evaluation of hypoglycemia in an adult without diabetes, it is necessary to
document Whipple triad and collect the following labs at the time of symptoms: insulin,
proinsulin, C-peptide, β-hydroxybutyrate, and oral hypoglycemia agent screen. In cases of
endogenous hyperinsulinemia (insulinoma), there will be a low glucose with elevated
insulin, proinsulin, and C-peptide, with suppressed β-hydroxybutyrate and negative oral
hypoglycemic agent screen. The laboratory findings in hypoglycemia due to sulfonylureas
will be identical, except the oral hypoglycemia agent screen will be positive.

50. The correct answer is: D. Transthoracic echocardiogram. The initial management of
hyperosmolar hyperglycemic state (HHS), like the management of diabetic ketoacidosis,
involves replacing volume and electrolyte deficits, insulin, and identification of any
underlying illness that may have precipitated HHS. Common precipitants include
infection, medication noncompliance, MI, and stroke. Although MI can be a precipitant of
HHS, this would be evaluated initially with an ECG rather than an echocardiogram.

51. The correct answer is: C. SGLT2 inhibitors increase hyperfiltration. Animal models
have suggested that SGLT2 inhibition reduces hyperfiltration and reduces albuminuria. In
clinical trials, SGLT2 inhibitors cause a slight initial decrease in eGFR, but then lessen the
decrease in eGFR over time compared with placebo. Canagliflozin (Invokana) has been
approved to reduce the risk for end-stage renal disease, worsening of kidney function,
cardiovascular death, and hospitalization for heart failure among adults with T2DM and
diabetic kidney disease.

52. The correct answer is: B. Start long-acting insulin at 0.1 to 0.2 IU/kg/d with
instructions to increase 2 U every 3 days until reaching fasting glucose target. The
American Diabetes Association (ADA) recommends early introduction of insulin if there
is evidence of ongoing catabolism (weight loss), if symptoms of hyperglycemia are
present, or when A1c level is >10%. In this patient, her nocturia likely reflects
symptomatic hyperglycemia. Once initiated, metformin should be continued as long as
tolerated with additional agents added as necessary. The ADA recommends initial basal
insulin dosing of 0.1 to 0.2 IU/kg/d or 10 U per day with instructions for patient self-
titration. Prior to initiation of prandial insulin, it would be reasonable to monitor response
to basal insulin to simplify regimen and promote adherence.

53. The correct answer is: A. Continue metformin and add dapagliflozin. In individuals
with T2DM with heart failure as the predominant comorbid condition, sodium-glucose
cotransporter 2 (SGLT2) inhibitors are recommended as the second agent (in addition to
metformin). SGLT2 inhibitors have been shown to significantly lower the risk of
hospitalization due to heart failure. As this patient is still above HbA1c target,
dapagliflozin should be added to current regimen.

54. The correct answer is: C. Lipid profile, tissue transglutaminase, and TSH. At the time
of diagnosis of T1DM, screening for celiac disease, hypothyroidism, and dyslipidemia is
recommended.

55. The correct answer is: C. Discard vial and open new vial. Unopened insulin should be
stored in the refrigerator. Most insulin vials expire ~30 days after first use and do not
require refrigeration after opening. Insulin should not be exposed to extremes of
temperature as it can lead to significant changes in insulin action. It is recommended that
insulin be stored at room temperature in a place away from direct heat and light, as long as
the temperature is not >30°C. If a vial of insulin freezes or is exposed to extreme heat, it
should be discarded.

56. The correct answer is: D. Discontinue insulin pump and transition to IV insulin
infusion. The American Diabetes Association (ADA) advocates allowing patients who are
physically and mentally able to continue to use their pumps when hospitalized. In this
setting in which the patient is unable to manage his insulin pump due to altered mental
status and appears critically ill, it would be appropriate to transition to IV insulin infusion.
The ADA recommends transitioning to basal-bolus regimen in noncritically ill patients
admitted to hospital but unable to safely operate the pump.

57. The correct answer is: A. Continue atorvastatin 80 mg daily and add ezetimibe 10
mg daily. The IMPROVE-IT trial showed the addition of ezetimibe to statin therapy
decreased LDL and improved cardiovascular outcomes in patients with known CAD. As
this patient has a more stringent LDL goal (<70 mg/dL), further therapy would be
beneficial. He is currently on a high-potency statin and so switching to a moderate
potency would not improve his LDL. It is unclear if the addition of the fibrate would add
concomitant cardiovascular benefit and can increase the risk of myalgias and
rhabdomyolysis with statins. It is reasonable to start by adding ezetimibe and see the
resultant LDL cholesterol (LDL-C). US guidelines recommend adding a PCSK9 inhibitor
if the LDL-C remains 70 mg/dL or above; European guidelines recommend targeting an
LDL-C <55 mg/dL.

Freemedicalbooks4download
58. The correct answer is: A. Atorvastatin 80 mg daily. The current recommendations are
to initiate high-intensity statins as first-line therapy in patients thought to be heterozygote
carriers of familial hypercholesterolemia. High-intensity statin with or without the
addition of ezetimibe would be a good first option to reduce LDL levels by >50%. If the
patient shows evidence of cardiovascular disease or does not respond sufficiently to a
statin, the addition of a PCSK9 inhibitor such as evolocumab would be reasonable.

59. The correct answer is: D. Initiate IV insulin and dextrose. This patient is experiencing
severe hypertriglyceridemia and shows concerning symptoms such as tachycardia and
leukocytosis in association with his pancreatitis. Rapid management of the metabolic
insults associated with hypertriglyceridemia is needed. Insulin promotes expression of
lipoprotein lipase, encouraging clearance of triglycerides from circulation. As this patient
is hyperglycemic and can tolerate insulin therapy, initiation of IV insulin with sufficient
glucose to preserve euglycemia would assist in decreasing triglycerides. If this patient
does not respond to more conservative measures, then apheresis would be a reasonable
next step.
8
RHEUMATOLOGY

QUESTIONS

1. A 42-year-old woman with an unremarkable medical history presents to her primary care
physician (PCP) with discomfort along the radial side of her right wrist that started 1 week
ago. She reports no injury or trauma to the hand or wrist and is otherwise healthy. She has
a 6-month-old healthy boy. Physical examination reveals tenderness over the dorsal radial
side of the wrist with fullness over the first dorsal compartment of the right wrist. Pain is
reproduced with ulnar deviation of the wrist with the thumb grasped in her palm.
Palpation of the first carpometacarpal joint is not painful. A radiograph of the wrist is
normal.
Which of the following is the most likely diagnosis?
A. Carpal tunnel syndrome
B. De Quervain tenosynovitis
C. Early presentation of inflammatory arthritis
D. Hand osteoarthritis

2. A 64-year-old man with a past medical history (PMH) of hypertension, obesity, non-
alcoholic fatty liver disease (NAFLD), and dyslipidemia presents to rheumatology clinic
in November with a 2-month history of bilateral hand pain. He denies any rashes, oral or
nasal ulcers, Raynaud phenomenon, proximal weakness, dyspnea, or pleuritic chest pain.
Medications include amlodipine 10 mg QD, simvastatin 20 mg QD, and aspirin 81 mg
QD. Examination is notable for bilateral second to fourth metacarpal phalangeal (MCP)
synovitis and proximal interphalangeal synovitis. Labs show normal complete blood count
(CBC). Aspartate aminotransferase (AST) is 130 U/L and alanine aminotransferase (ALT)
is 150 U/L, which is at baseline. Renal function is normal. Hepatitis serologies are
negative, and a QuantiFERON-TB test is negative. X-rays of hands do not show any
erosive disease.
Which of the following is the most likely diagnosis?
A. Ankylosing spondylitis
B. Parvovirus-associated arthritis
C. Polyarticular gout

Freemedicalbooks4download
D. Rheumatoid arthritis

3. Positive tests for which of the following will help to establish the diagnosis in the patient
in Question 2?
A. Acute parvovirus serologies
B. Dual-energy computed tomography (CT) scan
C. Human leukocyte antigen (HLA)-B27
D. Magnetic resonance imaging (MRI) of hand
E. Rheumatoid factor and anti–cyclic citrullinated peptide (anti-CCP) antibodies

4. The patient in Questions 2 and 3 is started on sulfasalazine (SSZ) 500 mg BID and
hydroxychloroquine 400 mg QD. Three weeks later, during travel to Florida, the patient
presents to the local emergency department (ED) with fever, cough, rash, myalgias, and
diffuse arthralgias. On examination, there is a confluent maculopapular rash over the trunk
and arms. He is noted to have bilateral metacarpal phalangeal (MCP) and proximal
interphalangeal synovitis. Labs are notable for AST of 400 U/L, ALT of 350 U/L, and
alkaline phosphatase of 800 IU/L. He has mild leukopenia. A chest radiograph (CXR) is
negative for any consolidation. A rapid flu test is negative.
Which of the following is the next most appropriate step in the management of this
patient?
A. Conservative management for viral syndrome with hydration, Tylenol, and rest
B. Start oseltamivir for flu, given high index of suspicion
C. Start prednisone 0.5 mg/kg for flare of rheumatoid arthritis
D. Stop hydroxychloroquine
E. Stop SSZ and start prednisone 0.5 mg/kg

5. A 32-year-old woman was recently diagnosed with nonerosive seropositive rheumatoid


arthritis. She is unable to tolerate methotrexate due to gastrointestinal (GI) intolerance
even when switched to SQ formulation and addition of folinic acid. Her PMH is notable
for Stevens-Johnson syndrome that developed after a course of
sulfamethoxazole/trimethoprim for the treatment of urinary tract infection 3 years ago.
She is married and wants to have children. She is started on hydroxychloroquine but does
not tolerate the medication due to GI intolerance, and on follow-up 3 months later
continues to have active synovitis.
Which of the following is the next aopriate agent?
A. Biologic disease-modifying antirheumatic drug (DMARD)
B. Leflunomide
C. Penicillamine
D. Sulfasalazine

6. You decide to start the patient on a biologic disease-modifying antirheumatic drug


(DMARD).
Which of the following agents is least likely to achieve good disease control?
A. Abatacept (cytotoxic T-lymphocyte–associated protein 4 [CTLA4]-Ig)
B. Anakinra (interleukin [IL]-1 receptor antagonist)
C. Infliximab (anti–tumor necrosis factor [TNF])
D. Tocilizumab (anti-IL-6)
7. A 28-year-old otherwise healthy man presents to urgent care with a 6-week history of
polyarticular pain involving his wrists, elbows, knees, and feet. He experiences daily
febrile episodes up to 38.9°C occurring toward the end of the day. His mother, who
accompanies him, notes a rash over his trunk during these episodes. A thorough review of
systems (ROS) is notable for a weight loss of 6 lb that is unintentional over the last 4
weeks, as well as night sweats. He denies any sore throat, oral/nasal ulcers, cough,
abdominal pain, diarrhea, or dysuria. He denies any recent travel or exposure to known
sick contacts. On examination, he is afebrile and vital signs are normal. No rash is noted.
There is tenderness to palpation of both wrists, without evidence of effusion, redness, or
warmth. Firm, nontender, matted cervical lymph nodes are palpable, the largest of which
measure up to 2 cm in greatest diameter. The remainder of the examination is
unremarkable. A CBC shows white blood cell (WBC) count 14 000 cells/μL, with
neutrophil predominance. His AST and ALT are mildly elevated at 78 U/L and 98 U/L,
respectively. Rheumatoid factor, cyclic citrullinated peptide (CCP), and antinuclear
antibody (ANA) are negative. X-rays of the wrists are normal. Blood cultures show no
growth after 5 days.
Which of the following is NOT an appropriate step in evaluation and management?
A. CT of chest, abdomen, and pelvis
B. Open biopsy of cervical lymph node
C. Start naproxen 500 mg BID with omeprazole 20 mg QD
D. Start prednisone 0.5 mg/kg

8. A 28-year-old man with PMH of hypertension, obesity, and dyslipidemia presents to the
ED with a 1-day history of severe right knee and left ankle pain. On examination, he is
afebrile, with normal vital signs. Examination is notable for right knee and left ankle
synovitis. Left knee examination is also notable for tenderness along the infrapatellar
tendon, as well as small, firm nodules in the left olecranon bursa. The remainder of the
examination is normal. Labs in the ED show a normal CBC, normal comprehensive
metabolic panel, and elevated erythrocyte sedimentation rate (ESR, 56 mm/h) and C-
reactive protein (80 mg/L). He reports two similar previous episodes of acute-onset knee
pain and swelling that were treated with nonsteroidal anti-inflammatory drugs (NSAIDs)
with complete resolution within 1 week. He denies any history of rash, inflammatory eye
disease, low back pain, conjunctivitis, dysuria, heel pain, abdominal pain, or diarrhea. He
does not recall any antecedent GI, genitourinary (GU), or ear, nose, and throat (ENT)
infections.
Which of the following is the most likely diagnosis?
A. Acute gouty arthritis
B. Rheumatoid arthritis
C. Septic arthritis
D. Seronegative spondyloarthritis
E. Systemic lupus erythematosus

9. Which of the following is the next best step to establish the diagnosis in the patient in
Question 8?
A. Arthrocentesis with cell count, differential, and crystal examination
B. Dual-energy CT scan
C. HLA-B27 testing

Freemedicalbooks4download
D. Musculoskeletal ultrasound examination
E. Sacroiliac (SI) joint x-ray

10. A 50-year-old man with PMH of hypertension, hyperlipidemia, type 1 diabetes mellitus
(T1DM) (on insulin pump), peptic ulcer disease, stage 3 chronic kidney disease, ischemic
cardiomyopathy, and gout is admitted to the hospital with a 2-day history of worsening
dyspnea on exertion. He is found to be in acute decompensated heart failure and is treated
with intravenous furosemide. On the third day of hospitalization, the patient develops
severe joint pain and swelling of multiple joints, including the wrists, elbows, knees,
ankles, and metatarsal phalangeals. On examination, he is afebrile. You note multiple
subcutaneous painless hard masses within both olecranon bursae. There is active synovitis
of his second and third metacarpal phalangeals (MCPs); bilateral knees; and bilateral first,
second, and third metatarsal phalangeal joints. Arthrocentesis of the right knee is
performed, with preliminary studies showing 36 000 cells/μL with 85% neutrophils.
Crystal examination and Gram stain are pending. A CBC shows normal WBC and red
blood cell (RBC) count. His glomerular filtration rate (GFR) is 18 mL/min. One month
ago, his GFR was 39 mL/min. His uric acid level is 5.7 mg/dL. A recent HbA1c was 9%.
Which of the following is the best treatment for his arthritis?
A. Administer subcutaneous adrenocorticotropic hormone (ACTH)
B. Start anakinra 100 mg QD SQ
C. Start colchicine 1.2 mg followed by 0.6 mg 1 hour later
D. Start indomethacin 50 mg TID
E. Start prednisone 40 mg QD

11. A 65-year-old man with PMH of hypertension, hyperlipidemia, and tophaceous gout is
being seen in follow-up for hypertension. His gout is stable, and he has not had any acute
gouty arthritis in 2 years. His most recent uric acid level is 4.8 mg/dL. His blood pressure
(BP) on multiple visits is elevated and you plan to make adjustments to the current
regimen of amlodipine 10 mg QD.
Which antihypertensive medication adjustment would you recommend?
A. Continue amlodipine and start lisinopril
B. Continue amlodipine and start losartan
C. Do not change antihypertensive regimen as this may worsen renal function and
precipitate a gout attack
D. Stop amlodipine and start hydrochlorothiazide
E. Stop amlodipine and start lisinopril

12. An 80-year-old man with a history of hypertension, stage 4 chronic kidney disease,
diabetes mellitus, and atrial fibrillation on Coumadin presents to the ED with a 2-day
history of right knee pain and swelling. He denies any prior episodes of joint pain. He
reports a recent fall 1 week earlier but does not recall injuring the knee. He denies any
fevers, chills, rashes, abdominal pain, diarrhea, dysuria, or eye pain. He has not received
any antibiotics in the preceding 3 months. On examination, he is afebrile. BP is 130/85
mmHg. There is a moderate-sized knee effusion, redness, and warmth. Range of motion is
limited by pain, but you are able to passively flex the knee to 45 degrees. X-ray
demonstrates moderate effusion and medial compartment joint space narrowing with
calcification of the articular cartilage. His CBC is normal. Basic metabolic panel is
notable for GFR of 25 mL/min and serum uric acid level is 6.7 mg/dL.
Which of the following is NOT a likely cause of his presentation?
A. Acute gouty arthritis
B. An acute presentation of rheumatoid arthritis
C. Hemarthrosis
D. Pseudogout
E. Septic arthritis

13. What is the next best step in the management of the patient in Question 12?
A. Arthrocentesis with cell counts, crystal examination, Gram stain, and culture
B. Dual-energy CT scan
C. MRI of right knee
D. Serum uric acid level measurement
E. Start prednisone 40 mg daily

14. Arthrocentesis is performed in the patient in Question 12, and 20 mL cloudy, yellow fluid
is aspirated. Cell count shows 35 000 cell/μL and 75% neutrophils. Gram stain is negative
for organisms, and crystal analysis shows rhomboid-shaped crystals with weakly
positively birefringence. Joint fluid culture shows no growth after 3ays.
Which of the following is the most likely diagnosis?
A. Calcium hydroxyapatite arthritis
B. Cholesterol arthritis
C. Gout
D. Pseudogout

15. What is the next best step in the management of the patient in Questions 12-14?
A. Intra-articular steroid injection with 80 mg methylprednisolone
B. Repeat arthrocentesis
C. Start ceftriaxone and vancomycin
D. Start colchicine
E. Start naproxen 500 mg BID

16. A 28-year-old man from India is referred to rheumatology for the evaluation of a 3-month
history of low back pain. The patient notes the insidious onset of low back pain associated
with 60 minutes of morning stiffness. Pain improves with activity and is worsened with
rest. He has tried naproxen 440 mg BID with good effect, but with recurrence of pain
upon discontinuation of therapy. The patient denies any rashes, visual blurring, red eyes,
sensation of eye grittiness, dysuria, increased urinary frequency, abdominal pain, or
diarrhea. There is no history of antecedent GU or GI infections. His examination is
notable for tenderness over the lower spine and paraspinal region, mostly over sacroiliac
(SI) joints. Flexion, abduction, and external rotation (FABER) of hips reproduce low back
pain.
What is the most likely diagnosis?
A. Ankylosing spondylitis
B. Inflammatory bowel disease (IBD)–associated axial spondyloarthropathy
C. Reactive arthritis
D. Rheumatoid arthritis

Freemedicalbooks4download
17. What is the next best diagnostic test to evaluate the low back pain in the patient in
Question 16?
A. Colonoscopy
B. Rheumatoid factor and anti–cyclic citrullinated peptide (CCP) antibodies
C. Sacral MRI
D. Sacroiliac (SI) joint radiographs

18. Sacroiliac (SI) joint x-rays with Ferguson views in the patient in Question 16 show
evidence of bilateral grade 4 SI joint disease (total ankylosis). Laboratory evaluations
show a normocytic, normochromic anemia, normal renal and liver function, and elevated
inflammatory markers. He is HLA-B27 positive. You decide to start the patient on
adalimumab 40 mg SQ every 2 weeks.
Which of the following testing is necessary prior to starting adalimumab in this
patient?
A. Hepatitis C antibody
B. Human immunodeficiency virus (HIV) Ag/Ab testing
C. Interferon γ release assay (IGRA)
D. Treponemal-specific antibodies

19. A 28-year-old man presents to the ED with a 3-day history of right knee pain and
swelling. His PMH is notable for history of cocaine abuse complicated by levamisole-
induced vasculitis as well as a recent episode of gonococcal urethritis 3 weeks ago treated
with IM ceftriaxone and azithromycin. The patient reports a history of bilateral eye
redness at the time of diagnosis with gonococcal urethritis that has since resolved, a rash
on the glans penis, and low back pain of 3 weeks’ duration that is worse in the morning
and improves with activity. He denies any other rashes, dactylitis, and ongoing symptoms
of dysuria, frequency, or urgency. He denies any heel pain. There is no personal history of
psoriasis or IBD. Family history is unremarkable. On examination, he is afebrile. There is
right knee and left ankle synovitis. He has an erythematous circinate rash over the glans
penis without penile discharge. There are no other rashes. Sacroiliac (SI) joint palpation is
not painful. Flexion, abduction and external rotation (FABER) test is negative; right-sided
FABER is limited by knee pain and reduced range of motion. Labs are notable for WBC
of 12 700/μL and hemoglobin (Hgb) 9.9 g/dL. His C-reactive protein is 127 mg/L and
ESR is 98 mm/h. A urinalysis (UA) is negative for WBC, RBC, and nitrites. Urine
toxicity screen is negative. Synovial fluid analysis is performed in the ED, which shows
27 000 cells/μL with 71% neutrophils. Both Gram stain and crystal examination are
negative. Culture is pending.
What is the most likely diagnosis?
A. Disseminated gonococcal arthritis
B. Gout
C. Reactive arthritis
D. Rheumatoid arthritis
E. Septic arthritis

20. Which of the following is the next most appropriate step for the patient in Question 19?
A. Administer IV Toradol and then transition to PO NSAIDs
B. Do nothing and wait for results of culture
C. Start ceftriaxone and azithromycin
D. Start ceftriaxone and vancomycin
E. Start PO prednisone 40 mg QD

21. A 23-year-old woman presents to the ED with a 2-day history of left ankle and right hand
pain. She reports feeling warm a few days earlier but has not checked her temperature.
She denies any rashes, oral or nasal ulcers, chest pain, abdominal pain, dysuria, frequency,
eye redness, grittiness, or pain. She denies any history of trauma. She is sexually active
with a single male partner. On examination, she is afebrile. Her left ankle is swollen,
tender, and warm with severely limited range of motion. There is no synovitis on the
remainder of her examination, but passive extension of her right, second finger reproduces
pain along the second flexor tendon. A few pustular papules are noticed along the palms
of her hands bilaterally. A CBC is notable for mild leukocytosis, and her comprehensive
metabolic panel is normal. Synovial fluid analysis shows a WBC count of 35 000 cells/μL,
84% neutrophil predominant. UA shows 20 to 50 WBCs.
Which of the following is the most likely diagnosis?
A. Disseminated gonococcal infection (DGI)
B. Gout
C. Reactive arthritis
D. Rheumatoid arthritis
E. Syphilis

22. Which of the following tests would have the highest yield for diagnostic confirmation in
the patient in Question 21?
A. Blood cultures
B. MRI
C. Nucleic acid amplification testing (NAAT)
D. Serologic testing

23. A 69-year-old man with T2DM, rheumatoid arthritis, and crystal-proven gouty arthritis
presents to the ED with a 2-day history of right knee pain. He denies any fever, chills,
rashes, or other joint pain. He is historically seropositive for rheumatoid factor and anti–
cyclic citrullinated peptide (CCP) antibodies. He is on methotrexate and infliximab
therapy for his rheumatoid arthritis and allopurinol for his gout. He notes good symptom
control of his rheumatoid arthritis (typically with metacarpal phalangeal [MCP] proximal
interphalangeal, and wrist pain). His last gout flare was in his left first metatarsal
phalangeal joint 2 years ago, and he has since discontinued his allopurinol. On
examination, he is afebrile. His right knee is swollen, tender, and erythematous with
significantly reduced range of motion. A CBC is notable for leukocytosis at 13 500 cells/
μL. His renal function is reduced with a GFR of 35 mL/min. X-ray of his knee shows
evidence of chondrocalcinosis.
Which of the following is the LEAST likely cause of his symptoms?
A. Gout
B. Pseudogout
C. Rheumatoid arthritis
D. Septic arthritis

Freemedicalbooks4download
24. Which of the following is the next best step in diagnosis for the patient in Question 23?
A. Arthrocentesis
B. Blood cultures
C. MRI of right knee
D. X-ray of right knee

25. Arthrocentesis is performed in the patient in Question 23 with complete evacuation of 40


mL of cloudy synovial fluid. Results of synovial fluid analysis show a WBC count of 49
000 cells/μL with 98% neutrophils. Crystal examination is positive for extracellular
negatively birefringent, needle-shaped crystals. Gram stain is negative, and cultures are
pending.
What is the next best step in his management?
A. Administer intra-articular steroid injection
B. Obtain blood cultures and start intravenous vancomycin/ceftriaxone
C. Resume allopurinol
D. Start indomethacin
E. Start oral prednisone

26. A 36-year-old woman presents to urgent care in Massachusetts in the fall with a 3-week
history of bilateral hand and wrist pain. She notes pain worse in the morning, associated
with 45 minutes of morning stiffness. Pain improves with activity and is associated with
worsening and stiffness after rest. She has not noted any new rashes, oral/nasal ulcers,
chest pain, abdominal pain, back pain, diarrhea, or dysuria. She denies any recent GU or
GI infections. She is a mother of two healthy boys. The youngest, 2 years old, has been
recovering from a febrile illness with upper respiratory infection (URI) symptoms and a
malar rash, after a family vacation to Florida. She denies any other known sick contacts.
On examination, there is synovitis of bilateral second through fifth metacarpal phalangeals
(MCPs), proximal interphalangeals, and both wrists. Both CBC and comprehensive
metabolic panel are normal.
Which of the following tests is most likely to establish the diagnosis?
A. Antinuclear antibody (ANA) and double-stranded (ds)-DNA
B. Chikungunya virus polymerase chain reaction (PCR)
C. Hepatitis B surface antibody immunoglobulin G (IgG)
D. Lyme antibody testing with reflex western blot
E. Parvovirus B19 IgM/IgG
F. Rheumatoid factor and anti–cyclic citrullinated peptide (CCP)

27. A 46-year-old woman with a history of hypertension, hyperlipidemia, T2DM, and


Raynaud phenomenon presents to the ED with headache and blurred vision for the past 8
hours. On examination, she is noted to be afebrile with heart rate (HR) 100 beats/min, BP
170/95 mm Hg, respiratory rate (RR) 18 breaths/min, and oxygen saturation of 97% on
room air. Skin examination is notable for small, blanching dilated blood vessels on her
hands and face, and examination of her nails reveals dilated capillary loops with dropout.
Labs are notable for the following:

CBC: WBC 7000 μL, Hgb 8.6 g/dL, platelets 54 000 μL


Blood smear: schistocytes and thrombocytopenia
Basic metabolic panel: Na 144 mEq/L, K 4.0, Cl 108 mEq/L , bicarbonate 32 mEq/L,
creatinine 3.9 mg/dL, blood urea nitrogen (BUN) 78 mg/dL
Liver function tests (LFTs): total bilirubin 4.5 mg/dL, direct bilirubin 0.4 mg/dL, AST 35
U/L, ALT 42 U/L, and alkaline phosphatase 86 IU/L
UA: neg protein, neg blood, neg leukocyte esterase, neg nitrites
ADAMTS13 activity: normal

What is the next best step in management?


A. Emergent hemodialysis
B. Observation and supportive care
C. Plasmapheresis
D. Start captopril
E. Start IV steroids

28. Which of the following autoantibodies puts the patient in Question 27 more at risk for
developing this complication of her disease?
A. Antinuclear antibody (ANA)
B. Anti-dsDNA
C. Anti-histone antibody
D. Anti-RNA polymerase III
E. Anti-U1 ribonucleoprotein

29. A 53-year-old man with a history of CAD, hypertension, hyperlipidemia, and


gastroesophageal reflux disease (GERD) presents to clinic with a chief complaint of
progressive weakness over the past 4 weeks. He first noted increased fatigue when
climbing the stairs to his second-floor apartment and recently developed difficulty
combing his hair. On examination, the patient is noted to be afebrile with HR 63 beats/min
and BP 130/72 mm Hg. His examination is notable for 4/5 strength in his proximal
muscles, poikiloderma of the upper chest and back, and fissured scaly plaques of the
radial index fingers. The patient is started on goal-directed medical therapy but
unfortunately develops progressive shortness of breath over the next few months.
Which of the following antibodies is most closely associated with an increased risk
of developing interstitial lung disease for patients with this condition?
A. Antinuclear antibody (ANA)
B. Anti-dsDNA
C. Anti-histone antibody
D. Anti-Jo1
E. Anti-RNA polymerase III
F. Anti-topoisomerase I

30. Given the patient in Question 29’s new underlying diagnosis, which of the following
screening tests is indicated?
A. Barium swallow
B. Bone marrow biopsy
C. Cardiac ultrasound
D. Colonoscopy
E. Retinal examination

Freemedicalbooks4download
31. A 34-year-old woman with a history of obesity, hyperlipidemia, T2DM, and Raynaud
phenomenon presents to clinic with an 8-week history of arthralgias of the bilateral wrists,
hands, and knees. She denies any recent fevers, chills, cough, nasal congestion, nausea,
vomiting, diarrhea, or rash. The patient drinks two to three glasses of wine every night and
denies smoking. Her grandmother has an unspecified history of arthritis. On examination,
the patient is noted to be afebrile with an HR of 75 beats/min and BP of 107/53 mmHg.
Her oxygen saturation is 98% on room air. Joint examination reveals no evidence of
synovitis, but she is noted to have puffy fingers on examination. On further ROS, the
patient notes hair thinning and some difficulty putting on her shirts and jackets in the
morning. Initial laboratory tests reveal the patient is antinuclear antibody (ANA) positive
at 1:1280, SSA positive, SSB negative, Scl-70 negative, Jo1 negative, U1
ribonucleoprotein positive, cyclic citrullinated peptide (CCP) negative, and rheumatoid
factor positive.
Which of the following is the most likely diagnosis?
A. Dermatomyositis
B. Mixed connective tissue disease
C. Rheumatoid arthritis
D. Scleroderma
E. Sjögren syndrome
F. Systemic lupus erythematosus

32. The patient is most likely to develop which of the following complications of her
condition?
A. Acute renal failure
B. Erosive arthritis
C. Lymphoma
D. Pulmonary hypertension
E. Vision loss

33. A 65-year-old man with a history of rheumatoid arthritis, CAD, hyperlipidemia, atrial
fibrillation, hepatic steatosis, chronic kidney disease, and benign prostatic hyperplasia
presents to clinic with 3 weeks of low-grade fever, arthralgias, rash, and right-sided chest
pain on inspiration. Prior to this episode, he was in his usual state of health and had been
taking all his home medications as prescribed. ROS is negative for recent sick contacts,
chills, upper respiratory symptoms, nausea/vomiting, or diarrhea. Vital signs show
temperature 37.9°C, HR 93 beats/min, and BP 143/89 mmHg. On examination, the patient
is noted to have faint, erythematous, macular rash overlying the sun-exposed areas of his
arms with no evidence of synovitis. Notable labs include WBC 4.3 × 109/L, Hgb 11.2
g/dL, platelets 95 × 109/L, creatinine 0.65 mg/dL, antinuclear antibody (ANA) positive
1:640, anti-dsDNA positive, anti-Smith negative, anti-histone positive, and normal
complement levels.
Which of the following medications from the patient’s home medication list may
be responsible for his clinical presentation?
A. Adalimumab
B. Aspirin
C. Atorvastatin
D. Methotrexate
E. Rivaroxaban

34. Which of the following is most likely to differentiate the patient in Question 33’s
presentation from a diagnosis of new-onset systemic lupus erythematosus?
A. Antinuclear antibody (ANA) pattern
B. Anti-histone antibody status
C. Clinical history
D. Skin biopsy

35. A 32-year-old woman with a history of systemic lupus erythematosus manifesting in the
past with pericarditis and deep vein thrombosis (DVT) on warfarin presents with
increasing fatigue, low-grade fevers, rash, and bilateral wrist pain and swelling for the
past 2 weeks. The patient has been adherent to her home hydroxychloroquine regimen but
continues to smoke on a regular basis. On examination, the patient is noted to be febrile to
38.2°C with HR 98 beats/min and BP 128/68 mmHg. Joint examination reveals symmetric
bilateral synovitis of the wrists with associated edema, and she has been taking ibuprofen
600 mg TID with little effect on her symptoms. The patient is also noted to have a bright
red rash overlying her nose and bilateral cheeks. Labs are notable for creatinine 0.8
mg/dL, UA with no proteinuria or hematuria, weakly positive dsDNA, normal C3 and C4,
and elevated ESR and C-reactive protein.
What is the next best step in her management?
A. Provide reassurance
B. Renal biopsy
C. Start methotrexate
D. Start steroids
E. Stop hydroxychloroquine

36. The patient unfortunately goes on to develop renal involvement and is noted to have a
creatinine 2.8 mg/dL, UA with 3+ protein and 2+ blood with microscopic analysis
showing dysmorphic RBCs and RBC casts, and a spot urine total protein to creatinine
ratio of 2.7 g. Renal biopsy reveals evidence of class IV lupus nephritis.
In addition to high-dose steroids, which of the following would be the most
appropriate treatment for a patient newly diagnosed with lupus nephritis?
A. Azathioprine
B. Belimumab
C. Cyclophosphamide
D. Methotrexate
E. Plasmapheresis
F. Rituximab

37. A 56-year-old woman with a history of systemic lupus erythematosus complicated by


lupus nephritis requiring hemodialysis, antiphospholipid syndrome with recurrent deep
vein thrombosis (DVT)/pulmonary embolism on anticoagulation, pericarditis, lupus
arthritis, fibromyalgia, and anemia presents with a chief complaint of vision changes. The
patient has noted increasing reading difficulty over the past 3 to 4 weeks and describes
worsening night vision for the past 6 months that has led her to stop driving at night.
Fundus examination is notable for bilateral parafoveal retinal pigment atrophy, and further

Freemedicalbooks4download
ophthalmologic testing confirms these findings consistent with “bull’s eye maculopathy.”
The patient is on appropriate medical therapy for her conditions.
Which of the following medications is most likely responsible for her retinopathy?
A. Azathioprine
B. Hydroxychloroquine
C. Methotrexate
D. Prednisone
E. Warfarin

38. How could this medication-related adverse event have been prevented?
A. Combination of antioxidant and zinc vitamins
B. Intraocular pressure monitoring
C. Regular monitoring of medication levelE
D. Screening ophthalmologic examinations
E. Tight blood glucose control

39. A 75-year-old man with a history of hypertension, hyperlipidemia, T2DM, CAD,


congestive heart failure, obstructive sleep apnea (OSA), asthma, stage 3 chronic kidney
disease, osteoarthritis, GERD, and allergic rhinitis presents to the ED with a chief
complaint of hemoptysis for the past 6 hours. He describes three episodes where he
coughed up approximately one tablespoon of bright red blood. He also notes 2 to 3 weeks
of recurrent low-grade fevers at home, 15 lb of unintentional weight loss, and difficulty
writing due to new-onset weakness of his right hand and wrist. Vital signs show
temperature 38.1°C, HR 105 beats/min, BP 160/83 mm165 000Hg, RR 20 breaths/min,
and SpO2 91%. Examination is notable for a petechial rash of both lower extremities and
wrist drop on the right. Laboratory workup is shown below:

CBC: Hgb 10.9 g/dL, platelets 165 000/μL, WBC 16 900/μL (Differential: neutrophils 61%,
lymphocytes 22%, eosinophils 12%, and basophils 0.2%)
Basic metabolic panel: Na 142 mEq/L, K 3.8 mEq/L, creatinine 0.9 mg/dL
LFT: AST 32 U/L, ALT 25 U/L, total bilirubin 1.0 mg/dL, direct bilirubin 0.3 mg/dL,
alkaline phosphatase 91 IU/L
UA: neg blood, 1+ glucose, neg leuk esterase, neg protein, specific gravity 1.010

Which of the following is the most likely cause of his current symptoms?
A. Cryoglobulinemic vasculitis
B. Eosinophilic granulomatosis with polyangiitis 15 800
C. Granulomatosis with polyangiitis
D. Microscopic polyangiitis
E. Polyarteritis nodosa (PAN)

40. Which of the following studies is most likely to yield a diagnosis?


A. Allergy testing
B. Blood smear
C. Bronchoscopy
D. Electromyography (EMG) and nerve conduction study (NCS)
E. PET-CT imaging
41. A 54-year-old homeless man who is not well known to the medical system presents to the
ED with 1 week of diffuse abdominal pain and difficulty ambulating. His abdominal pain
generally worsens with meals, and while he denies nausea/vomiting, he has experienced
three episodes of hematochezia in the past 24 hours. The patient also reports left ankle
tingling and weakness for 2 weeks and has fallen twice after losing his balance. Review of
systems is positive for subjective fever, malaise, myalgias, weight loss, and rash. Social
history is notable for recent incarceration and IV drug use. Vital signs are as follows:
temperature 37.6°C, HR 112 beats/min, BP 170/95 mm Hg, RR 16 breaths/min, and SpO2
98% on room air. Examination reveals mild-to-moderate diffuse abdominal pain, 3/5
strength on foot dorsiflexion, and palpable purpura over both lower extremities. Initial
laboratory results show WBC 15 800/μL, Hgb 7.5 g/dL, platelets 115 000/μL, ESR 61
mm/hr, C-reactive protein 15.8 mg/L , creatinine 2.3 mg/dL (baseline 0.8 mg/dL), and UA
with 1+ protein and 1+ blood without RBC casts. The patient is admitted to the hospital
and undergoes renal artery angiography, which reveals multiple renal artery aneurysms
with constrictions in the large vessels and occlusion of several smaller arteries.
What is the most likely diagnosis?
A. Cryoglobulinemic vasculitis
B. Eosinophilic granulomatosis with polyangiitis
C. Granulomatosis with polyangiitis
D. Microscopic polyangiitis
E. Polyarteritis nodosa (PAN)

42. Which of the following infections is most closely associated with the patient in Question
41’s condition?
A. Epstein-Barr virus (EBV)
B. Hepatitis B
C. Hepatitis C
D. HIV
E. Tuberculosis

43. A 69-year-old man with a history of chronic obstructive pulmonary disease (COPD),
former tobacco use, alcohol use disorder, recurrent falls, and traumatic subdural
hematoma (SDH) in the setting of intoxication, dementia, and leaking abdominal aortic
aneurysm requiring endovascular repair presents with rash, joint pain, and abdominal
pain. The patient was in his usual state of health until approximately 3 weeks prior to his
presentation when he reports a sore throat, fevers, and mild neck pain to palpation. His
symptoms resolved over the course of 3 to 4 days. Two days ago, the patient noted rash on
his lower extremities and went on to develop joint pain affecting both ankles, knees, and
hips and colicky abdominal pain. Vital signs are as follows: temperature 36.8°C, HR 94
beats/min, BP 115/65 mmHg, RR 14 breaths/min, and SpO2 90% on room air.
Examination reveals violaceous purpura overlying the lower legs and distal thighs,
inflammation of the ankles and knees, and mild diffuse abdominal pain. Labs reveal WBC
12 000/μL, Hgb 13 g/dL, platelets 170 000/μL, and creatinine 0.5 mg/dL, and UA was
negative for both protein and blood. Skin biopsy demonstrates leukocytoclastic vasculitis
with IgA deposition.
What is the most likely diagnosis?

Freemedicalbooks4download
A. Anti–glomerular basement membrane disease
B. Cryoglobulinemic vasculitis
C. Granulomatosis with polyangiitis
D. IgA vasculitis
E. Microscopic polyangiitis

44. What is the next best step in the management of the patient in Question 43?
A. Abdominal CT angiogram
B. Colchicine
C. Observation
D. Renal artery ultrasound
E. Steroids

45. A 65-year-old man with a history of hypertension, hyperlipidemia, COPD, current


smoking, abdominal aortic aneurysm, nephrolithiasis, and benign prostatic hyperplasia
presents to clinic with 1 week of painless jaundice. The patient denies fever, chills, weight
loss, or night sweats and is otherwise in his usual state of health. The patient has no
history of alcohol use or smoking. Vital signs are as follows: temperature 36.4°C, HR 75
beats/min, BP 150/93 mm Hg, RR 16 breaths/min, and SpO2 93% on room air.
Examination reveals axillary lymphadenopathy and jaundice. Labs show WBC 8000/μL,
Hgb 12 g/dL, platelets 274 000/μL, lipase 130 U/L, AST 110 U/L, ALT 85 U/L, alkaline
phosphatase 218 IU/L, total bilirubin 4.3 mg/dL, direct bilirubin 3.8 mg/dL, creatinine 0.8
mg/dL. CT scan of the chest, abdomen, and pelvis reveals a 3-cm mass in the pancreatic
head with mediastinal, hilar, axillary, and retroperitoneal lymphadenopathy. Endoscopic
retrograde cholangiopancreatography (ERCP) shows dilation of the bile duct, but no
gallstones. Serum IgG subclass testing reveals elevated levels of IgG4.
What is the next best step in his management?
A. Endoscopic ultrasound with biopsy
B. Magnetic resonance cholangiopancreatography
C. Observation
D. Radiation therapy
E. Smoking cessation counseling
F. Whipple procedure

46. Further workup of the patient in Question 45 reveals findings consistent with IgG4-related
disease.
What is the first step in his treatment?
A. Adalimumab
B. Methotrexate
C. Radiation therapy
D. Rituximab
E. Steroids
F. Surgical resection

47. A 45-year-old man with a history of cirrhosis due to alcohol use disorder, alcohol
withdrawal complicated by seizures, IV drug use, hepatitis C, methicillin-resistant
Staphylococcus aureus (MRSA) tricuspid valve endocarditis, and lumbar osteomyelitis
presents to the ED with a chief complaint of new-onset rash involving both lower
extremities for the past 2 days. The patient first began feeling unwell approximately 1
week ago when he developed generalized malaise, fatigue, arthralgias, and paresthesia
with mild weakness in both lower legs. The patient was recently released from prison and
has not been followed by a regular medical provider for several years. Vital signs are as
follows: temperature 37.3°C, HR 97 beats/min, BP 93/64 mmHg, RR 16 breaths/min, and
SpO2 95% on room air. Examination is notable for violaceous palpable purpura involving
lower extremities, 2/6 systolic murmur best heard at the lower sternal border, nodular
liver, splenomegaly, 4/5 strength to dorsiflexion and plantar flexion of both feet, and
decreased sensation to light touch over the lower legs to mid-shin. Labs reveal WBC 11 ×
109/L, Hgb 8.5 g/dL, platelets 69 × 109/L, AST 60 U/L, ALT 45 U/L, alkaline phosphatase
115 IU/L, total bilirubin 1.5 mg/dL, direct bilirubin 0.8 mg/dL, creatinine 0.75 mg/dL, UA
2 normal, negative antinuclear antibody (ANA), positive rheumatoid factor, normal C3,
and low C4 Blood cultures x 2 are no growth to date.
What is the next best step in his management?
A. Anti–cyclic citrullinated peptide (CCP) testing
B. Anti–glomerular basement membrane testing
C. Cardiac ultrasound
D. Cryoglobulin testing
E. Kidney biopsy

48. What is the patient in Question 47’s main risk factor for developing this condition?
A. Alcohol use disorder
B. Cirrhosis
C. Hepatitis C
D. History of endocarditis
E. IV drug use

49. A 65-year-old woman with a history of carpal tunnel, stress urinary incontinence, and
anxiety presents to clinic as a triage visit with a chief complaint of bilateral lower
extremity pain for the past 6 weeks. She describes the pain as a “tingling, electricity-like
sensation” affecting both feet and both ankles. The pain is always present, but she reports
that she notices it most frequently at night. The patient is followed regularly by her PCP,
and she had not had any recent medical issues until she developed right-sided carpal
tunnel syndrome approximately 6 months ago. Since that time, the patient describes
various complaints, including worsening bilateral shoulder pain for the past 3 months,
easy bruising, and a 10-lb weight gain. Vital signs are as follows: temperature 36.9°C, HR
65 beats/min, BP 120/70 mmHg, RR 14 breaths/min, and SpO2 98% on room air.
Examination is notable for hepatosplenomegaly, fullness of the anterior shoulder
bilaterally, positive Phalen and Tinel sign of the right wrist, decreased sensation to light
touch of the feet and ankles, 2+ pitting lower extremity edema to the knees, and periorbital
bruising with additional scattered bruising over both the upper and lower extremities. Labs
reveal WBC 8000/μL, Hgb 12 g/dL, platelets 95 000/μL, creatinine 1.6 mg/dL, and UA
3+ protein.
What is the next best step in her management?
A. Coronary angiogram

Freemedicalbooks4download
B. EMG
C. Hepatitis B testing
D. Liver biopsy
E. Serum protein electrophoresis (SPEP) and urine protein electrophoresis (UPEP) with
immunofixation
F. Venous ultrasound lower extremities

50. After pursuing the testing above for the patient in Question 49, the decision is made to
perform a fat pad biopsy.
What is pathologic evaluation of the tissue most likely to show?
A. Dense lymphoplasmacytic infiltrate with “storiform fibrosis”
B. Excessive organized collagen deposition with expansion of the dermis
C. Focal lymphocytic sialadenitis
D. Interface dermatitis
E. Positive Congo red staining with “apple green” birefringence

ANSWERS

1. The correct answer is: B. De Quervain tenosynovitis. The clinical history of atraumatic
radial wrist pain, fullness and tenderness over dorsal radial wrist, and examination
findings of a positive Finkelstein maneuver are suggestive of de Quervain tendinopathy.
The etiology is not completely understood; however, observational data suggest repetitive
activities that maintain the thumb in extension and abduction predispose to this condition.
The tendinopathy affects the abductor pollicis longus and the extensor pollicis brevis as
they pass through the first dorsal compartment from the forearm into the hand. Treatment
is conservative with application of ice to the area, immobilization with a forearm-based
thumb spica splint, and NSAIDs. Glucocorticoid injection may be considered in patients
who do not respond to these measures, while surgery may be offered in resistant cases.

2. The correct answer is: D. Rheumatoid arthritis. The patient’s presentation is notable
for a chronic, symmetric, small-joint predominant inflammatory arthritis consistent with
rheumatoid arthritis. Parvovirus arthritis may have a similar presentation, but the duration
of symptoms is longer than would be expected for parvovirus infection. Gout typically
presents with an acute inflammatory arthritis, which may be polyarticular, but a chronic
small-joint polyarthritis is a much less likely presentation for gout. Ankylosing spondylitis
can be associated with an inflammatory arthritis that is typically oligoarticular,
asymmetric, and large-joint predominant. A polyarticular small-joint arthritis without
inflammatory-type low back pain does not suggest ankylosing spondylitis.

3. The correct answer is: E. Rheumatoid factor (RF) and anti–cyclic citrullinated
peptide (anti-CCP) antibodies. The next best step in the evaluation is to check serologic
tests for rheumatoid factor and anti-CCP antibodies.

4. The correct answer is: E. Stop SSZ and start prednisone 0.5 mg/kg. The constellation
of fever, maculopapular rash, arthralgias, myalgias, and transaminitis occurring 2 to 3
weeks after initiation of SSZ should raise concern for an allergic hypersensitivity
syndrome related to SSZ, and the medication should be discontinued immediately.
Systemic steroids may be used to control the acute inflammatory state and to allow for
resolution of hypersensitivity symptoms. Although symptoms and findings are nonspecific
and may suggest a viral exanthem, empirically treating for a viral etiology without
discontinuing SSZ is not the correct answer, particularly given the severe and potentially
fatal consequences of SSZ allergy. The patient’s symptoms are unlikely to be caused by
hydroxychloroquine and cannot be simply explained by a flare of his underlying
rheumatoid arthritis, which would not be expected to cause her other symptoms of fever,
rash, myalgias, and transaminitis.

5. The correct answer is: A. Biologic disease-modifying antirheumatic drug (DMARD).


DMARD options in rheumatoid arthritis include conventional DMARDs (including
sulfasalazine, hydroxychloroquine, methotrexate, and leflunomide), targeted synthetic
DMARDs (JAK/STAT inhibitors), and biologic DMARDs (such as anti-TNF and anti-IL-
6). Given her intolerance of methotrexate and persistent disease activity, she requires
adjustment of therapy. Her history of a severe reaction to a sulfa-containing medication
precludes the use of SSZ, and her desire to have children would make the use of
leflunomide undesirable given its teratogenicity. Of the choices provided, the next best
step would be to initiate a biologic therapy.

6. The correct answer is: B. Anakinra (interleukin [IL]-1 receptor antagonist). Anakinra
is least likely to achieve symptom control in this patient with rheumatoid arthritis.
Anakinra is a recombinant human IL-1 receptor antagonist that is effective in the
treatment of autoinflammatory conditions, which involve, in part, the overproduction of
IL-1 through dysregulated or aberrant activation of the inflammasome—a multimeric
platform that converts pro-IL-1 to IL-1. Although anakinra is available for the treatment
of rheumatoid arthritis, it is significantly less effective for rheumatoid arthritis when
compared with other biologic agents, including anti-TNF therapies. Anakinra is approved
for use in combination with methotrexate for the treatment of rheumatoid arthritis in the
European Union but not in the United States.

7. The correct answer is: D. Start prednisone 0.5 mg/kg. The constellation of daily fevers,
night sweats, unintentional weight loss, and lymphadenopathy in an otherwise healthy
adult should prompt a broad evaluation for infectious, malignant, and inflammatory
noninfectious etiologies. Although adult-onset Still disease, an inflammatory disease of
unknown etiology, may account for many of the features including the rash concurrent
with fevers, polyarthralgias and transaminitis, the diagnosis requires exclusion of
malignant, infectious, and other inflammatory conditions. Appropriate next steps in the
evaluation include additional imaging of chest, abdomen, and pelvis to evaluate for
localizing pathology. An excisional biopsy of a cervical lymph node or other accessible
lymph nodes to evaluate for a malignancy such as lymphoma, as well as atypical
infectious etiologies, would be appropriate. Judicious use of NSAIDs may be pursued for
symptomatic control while additional evaluation is pursued. Empiric treatment for adult-
onset Still disease with steroids would not be appropriate at this stage, given the potential
for steroids to obscure underlying pathology.

Freemedicalbooks4download
8. The correct answer is: A. Acute gouty arthritis. The recurrent acute intermittent
inflammatory asymmetric oligoarthritis that resolves completely with anti-inflammatory
therapy as well as evidence of firm nodules in the olecranon bursa is suggestive of crystal-
induced arthritis, specifically gout due to the presence of tophi, on examination. The
intermittent resolving nature of these episodes and the absence of extra-articular or axial
features argue against peripheral arthritis associated with seronegative
spondyloarthropathies.

9. The correct answer is: A. Arthrocentesis with cell count, differential, and crystal
examination. Confirmation of the diagnosis with crystal examination of a synovial fluid
sample would be the next most appropriate step. If arthrocentesis is not possible, or crystal
examination is negative, evaluation for the presence of monosodium urate crystal
deposition with imaging may be pursued; however, this would not be the first step in
evaluation. As the symptoms are not consistent with a seronegative spondyloarthropathy,
sacroiliac (SI) joint films or HLA-B27 testing is not necessary.

10. The correct answer is: B. Start anakinra 100 mg QD SQ. The patient has developed an
acute inflammatory polyarthritis in the setting of multiple comorbidities, including chronic
kidney disease, T1DM, and ischemic cardiomyopathy. The differential diagnosis for an
acute polyarthritis includes crystalline arthritis, viral arthritis, immune complex arthritis,
or acute presentation of a chronic inflammatory arthritis. The patient has multiple risk
factors for hyperuricemia, and the clinical finding of bilateral tophi suggests a
considerable burden of chronic hyperuricemia. Diuresis with loop diuretics is a risk factor
for fluctuation in serum uric acid levels that can precipitate an acute gouty arthritis. A uric
acid level within normal range does not exclude an acute gouty arthritis due to the
uricosuric effect of inflammatory cytokines. For these reasons, the most likely diagnosis is
acute polyarticular gouty arthritis.
Treatment of gout involves (1) the acute management of gouty arthritis and (2)
determination of whether the patient will benefit from urate-lowering therapy. Options for
the acute management of gout include NSAIDs, intra-articular steroids, colchicine,
systemic steroids, or anti-IL-1 therapy, and the choice is often determined by severity of
symptoms, number of involved joints, and presence of comorbidities that may preclude
the safe use of certain therapies. NSAIDs are contraindicated due to chronic kidney
disease and peptic ulcer disease, intra-articular or systemic steroids will cause acute blood
sugar elevations from his T1DM, and colchicine is relatively contraindicated due to
severe renal insufficiency. Anakinra, an IL-1R antagonist, has been demonstrated to be
efficacious for the treatment of severe gouty arthritis flares, and its use is not precluded by
the patient’s comorbidities.

11. The correct answer is: B. Continue amlodipine and start losartan. Elevated uric acid
levels and gout have been linked to multiple comorbidities, including hypertension,
hyperlipidemia, T2DM, obesity, and congestive heart failure. The presence of
hyperuricemia has also been linked to adverse cardiovascular outcomes; thus,
modification of risk factors, including appropriate control of hypertension, is necessary.
Losartan is the only angiotensin-II receptor blocker (ARB) that has been shown to have
uricosuric effects and reduce uric acid levels. This effect is hypothesized to be related to
the inhibition of URAT1 transporters in the kidney that are involved in reabsorption of
filtered uric acid. Hydrochlorothiazide would increase serum uric acid levels through
promoting net uric acid reabsorption. Angiotensin-converting enzyme (ACE) inhibitors,
including lisinopril, have not been shown to reduce uric acid levels.

12. The correct answer is: B. An acute presentation of rheumatoid arthritis. The
differential diagnosis for an acute monoarthritis is limited and includes crystal-induced
inflammatory arthritis, including gout and pseudogout, bacterial septic arthritis, or an
acute monoarticular presentation of a chronic inflammatory arthritis. However, an acute
presentation of RA is the least likely diagnosis.

13. The correct answer is: A. Arthrocentesis with cell counts, crystal examination, Gram
stain, and culture. Arthrocentesis MUST be performed to exclude an infectious etiology;
clinical features alone, including absence of fever, examination findings (degree of pain,
swelling, loss of range of motion), and absence of leukocytosis, are not sensitive enough
to exclude an infectious process, particularly in elderly patients with immunosuppressive
comorbidities.

14. The correct answer is: D. Pseudogout. The synovial fluid cell count of 35 000 cell/μL is
reassuring; however, it does not rule out a septic process. Gram stain is ~30% sensitive for
the detection of septic arthritis. Calcium pyrophosphate dihydrate (CPPD) crystal–induced
arthritis is the most likely diagnosis, given the presence of chondrocalcinosis on x-ray,
knee osteoarthritis, age, and first presentation with knee arthritis. Crystal examination
showing rhomboid-shaped crystals with weakly positive birefringence would establish the
diagnosis.

15. The correct answer is: A. Intra-articular steroid injection with 80 mg


methylprednisolone. The patient’s advanced chronic kidney disease would preclude the
safe use of NSAIDs. Colchicine may be used if adjusted for renal function; however, it is
less likely to be effective given the duration of the patient’s symptoms. Colchicine may be
started while waiting for results of synovial fluid analysis to rule out a septic etiology. In
this case, based on the cell count, Gram stain, and culture data, an infectious etiology is
unlikely, and an intra-articular steroid injection is likely to provide the most relief.

16. The correct answer is: A. Ankylosing spondylitis. This 28-year-old patient from India is
presenting with low back pain with insidious progression, prolonged morning stiffness
improving with activity, and a good response to NSAID therapy, suggestive of
inflammatory-type low back pain. The differential for this includes the axial
spondyloarthritis, including ankylosing spondylitis, reactive arthritis, IBD-associated
spondyloarthritis, psoriatic arthritis, nonradiographic spondyloarthritis, and
undifferentiated spondyloarthritis. In this patient, there are no historical or examination
findings to suggest the presence of underlying IBD or psoriasis, or preceding infectious
etiologies to suggest another type of spondyloarthritis. Inflammatory involvement of the
lower axial skeleton is not typically associated with rheumatoid arthritis.

17. The correct answer is: D. Sacroiliac (SI) joint radiographs. The next best step in the
evaluation is a plain radiograph of the SI joints to evaluate for radiographic involvement
of the SI joints. SI joint abnormalities are typically graded from 0 to 4 based on severity

Freemedicalbooks4download
and to determine the degree of confidence that the changes seen reflect sacroiliitis. MRI of
the SI joints may be considered if radiographs do not suggest sacroiliitis or when findings
are uncertain.

18. The correct answer is: C. Interferon γ release assay (IGRA). Anti-TNF therapy is
associated with a high risk of Mycobacterium tuberculosis (MTb) reactivation in patients
with a history of latent TB. Therefore, excluding latent TB infection prior to anti-TNF
initiation is critical. This patient from India, where TB is endemic, is at high risk for TB
infection. Several tests are available to assess for immunologic response to MTb that
indicates exposure to MTb in the past. These tests include the tuberculin skin test, which
involves the intradermal injection of purified protein derivative, which stimulates a
delayed-type hypersensitivity response mediated by T-lymphocytes at the injection site
within 48 to 72 hours. Other tests include the interferon-gamma release assay (IGRA),
which measures T-cell release of interferon-γ following stimulation by antigens from
MTb. Both assays can produce false-positive and false-negative results. The other tests are
not required prior to initiation of anti-TNF therapy, but may be checked depending on
additional risk factors or age-appropriate screening.

19. The correct answer is: C. Reactive arthritis. NSAIDs are considered first-line treatment
for reactive arthritis. Most cases tend to resolve within 1 year of diagnosis. Conventional
or biologic disease-modifying antirheumatic drugs (DMARDs) may be added depending
on clinical response. Prednisone may be considered in patients who have an inadequate
response to NSAIDs, although clinical response to steroids is usually suboptimal.

20. The correct answer is: A. Administer IV Toradol and then to PO NSAIDs. The
clinical picture of an acute oligoarticular inflammatory arthritis with asymmetric large-
joint involvement and inflammatory-type low back pain in the setting of recent suspected
GU infection and symptoms compatible with conjunctivitis and circinate balanitis is
consistent with a diagnosis of reactive arthritis.
Although septic arthritis is a possibility given the acuity of the presentation, an
oligoarticular presentation of bacterial septic arthritis is less likely. Although a synovial
fluid cell count of 27 000 cells/μL does not exclude a septic process, the clinical picture of
oligoarticular involvement, extra-articular features (conjunctivitis, balanitis), and negative
Gram stain reduce the pretest probability that this is a bacterial septic process.
Disseminated gonococcal infection must always be considered in any sexually active
individual; however, the absence of evidence of tenosynovitis or pustular rash, the clinical
picture of balanitis, conjunctivitis, inflammatory low back pain, and recent history of
appropriate treatment for suspected gonococcal urethritis with IM ceftriaxone make a
diagnosis of disseminated gonococcal infection unlikely.

21. The correct answer is: A. Disseminated gonococcal infection. The clinical picture is
suggestive of an acute inflammatory monoarthritis of the left ankle and flexor
tenosynovial involvement of the second flexor tendon. Other notable findings include a
pustular eruption on the palms, leukocytosis, and pyuria. In a sexually active person, this
constellation is highly concerning for disseminated gonococcal infection. Reactive
arthritis can present with an acute monoarthritis; however, reactive arthritis would not
account for her pustular rash.
22. The correct answer is: C. Nucleic acid amplification testing (NAAT). NAAT on bodily
fluid from multiple sources, including oropharynx, rectum, urethra, and synovial fluid, has
the highest diagnostic yield. Serologic testing does not have a role in the diagnosis, and
MRI does not show specific findings for disseminated gonococcal infection. Blood
cultures should be obtained, but culture has a lower diagnostic yield than NAAT.

23. The correct answer is: C. Rheumatoid arthritis. The differential diagnosis of an acute
monoarthritis is rather limited and includes crystal-associated inflammatory arthritis (gout
and pseudogout), bacterial infectious arthritis, or an acute presentation of a chronic
inflammatory arthritis (eg, rheumatoid arthritis pseudosepsis). Risk factors for septic
arthritis in this patient include immunosuppressed status (diabetes, immunosuppressive
medications, age). Risk factors for gouty arthritis include prior history of crystal-proven
gout, cessation of urate-lowering therapy, and renal dysfunction. The presence of
chondrocalcinosis on x-ray suggests calcium pyrophosphate crystal deposition. While a
flare of his rheumatoid arthritis is possible, he notes symptoms being well-controlled on
his current regimen of methotrexate and infliximab, and an acute monoarthritis should
raise suspicion of an alternative process.

24. The correct answer is: A. Arthrocentesis. The next best step would be an arthrocentesis
with cell counts/differential, crystal analysis, Gram stain, and culture.

25. The correct answer is: B. Obtain blood cultures and start intravenous
vancomycin/ceftriaxone. Septic arthritis due to most bacterial organisms is typically
associated with high synovial fluid WBC counts, often >50 000 cells/μL. Lower cell
counts may be observed, and thus the results of the fluid must be interpreted in the clinical
context. In this patient, the pretest probability for septic arthritis is high, so a cell count of
49 000 cells/μL does NOT rule out a septic process. Furthermore, a neutrophil percentage
>90% is concerning for a septic process. Gram stain has a sensitivity of only 30% to 50%.
The presence of crystals in the fluid does NOT rule out the presence of a concomitant
septic process. Given the overall highpretest probability of a septic bacterial arthritis and
the results of the synovial fluid analysis, the next most appropriate step would be to start
empiric antibiotics after obtaining blood cultures. If a septic process is excluded on the
basis of cultures, then antibiotics can be discontinued, and the patient can be treated for a
flare of gouty arthritis.

26. The correct answer is: E. Parvovirus B19 IgM/IgG. The patient is presenting with an
acute symmetric small-joint predominant polyarthritis occurring in the absence of extra-
articular features, known comorbidities, or antecedent infections. The differential
diagnosis for such a presentation includes acute viral arthritis, including parvovirus, acute
hepatitis B or C infections, cytomegalovirus, Epstein-Barr virus, or chikungunya viral
infections. An acute presentation of a chronic process, such as rheumatoid arthritis, a
seronegative spondyloarthropathy with peripheral arthritis, or a connective tissue disease
with arthritis is possible. Of these, a viral arthritis is favored, given the acuity, absence of
extra-articular features, and history of likely viral infection in her son.
The son’s exanthema presenting as a febrile illness with malar rash is consistent
with acute parvovirus B19 infection, or fifth disease. In adults, acute parvovirus infection
is often associated with an acute polyarthritis that may mimic rheumatoid arthritis in the

Freemedicalbooks4download
pattern of joint symptoms. Serologic testing with IgM/IgG is likely to lead to correct
diagnosis; presence of positive IgM antibodies is suggestive of an acute infection. The
pattern of joint symptoms in this patient is not consistent with a presentation of Lyme
arthritis. Hepatitis B surface antibody IgG would document presence of protective
immunity and not acute infection. Although chikungunya virus arthritis can present with
this clinical picture, pain is often more severe, and no cases of chikungunya infection have
been documented to be acquired in the United States without foreign travel.

27. The correct answer is: D. Start captopril. This patient with a history of Raynaud
phenomenon presents with hypertension and is found to have an acute kidney injury with
evidence of thrombotic microangiopathy (TMA) on laboratory testing with anemia,
thrombocytopenia, and an elevated indirect bilirubin. Her presentation is most consistent
with scleroderma renal crisis (SRC). SRC occurs in 10% to 15% of patients with systemic
sclerosis and is more frequent in diffuse cutaneous systemic sclerosis (DcSSc) compared
with limited cutaneous systemic sclerosis (LcSSc). It often occurs early in the disease and
generally occurs within a median duration of 7.5 months from the first non-Raynaud
clinical manifestation of the disease. In this case, the patient’s telangiectasias and dilated
capillary loops with dropout on examination suggest the diagnosis of scleroderma. UA is
usually normal in SRC, but may show proteinuria if severe hypertension is present. SRC
is a TMA, and it is important to consider other causes of TMA, such as thrombocytopenic
purpura (nonspecific symptoms, fever, mild acute kidney injury, with or without
neurologic symptoms, low ADAMTS13 activity), hemolytic uremic syndrome (exposure
history, severe abdominal pain, nausea and/or vomiting, diarrhea), and drug-induced TMA
(exposure to quinine in particular, or to gemcitabine or oxaliplatin). SRC is a
rheumatologic emergency and should be treated without delay, and these patients are most
commonly managed in the intensive care unit. An ACE inhibitor, such as captopril, is the
mainstay of therapy for SRC. Steroids are thought to provoke SRC and should be avoided
in patients with scleroderma. Patients with SRC often require hemodialysis, but our
patient has no current indication for emergent hemodialysis. Plasmapheresis is used to
treat thrombotic thrombocytopenic purpura, which could present similarly.

28. The correct answer is: D. Anti-RNA polymerase III. Scleroderma patients with
antibodies against RNA polymerase III are at an increased risk of developing scleroderma
renal crisis (SRC) when compared to patients with either anti-Scl-70 or anti-centromere
antibodies. Antinuclear antibody (ANA) is often positive in scleroderma, as it is a
connective tissue disease, but a positive ANA does not increase the risk of SRC. Anti-
dsDNA antibodies are associated with lupus, anti-histone antibodies are found in patients
with drug-induced lupus, anti-Jo1 antibodies can be seen in dermatomyositis, and anti-U1
ribonucleoprotein antibodies are seen in mixed connective tissue disease (MCTD) and in
some patients with lupus.

29. The correct answer is: D. Anti-Jo1. The patient presents with progressive weakness and
is found to have a history and physical examination that are consistent with proximal
muscle weakness. This combined with the patient’s skin findings is highly characteristic
of dermatomyositis. Proximal muscle weakness is the most common presentation of
dermatomyositis or polymyositis (PM). Patients often have difficulty performing tasks
with their arms above their heads (eg, combing their hair, putting on shirts/jackets), and
proximal leg weakness may manifest as difficulty standing from a chair or climbing up
stairs. Skin findings commonly seen in dermatomyositis include poikiloderma of the neck
(V-sign), back (shawl sign), and hips (holster sign); small purple or red papules on the
extensor surfaces of the joints of the hand and elbows (Gottron papules); and a violaceous
rash affecting the upper eyelids (heliotrope sign).
A subset of patients may also have fissured scaly plaques on the radial surfaces of
the fingers and palms with hyperpigmentation of the palmar creases known as mechanic’s
hands. Mechanic’s hands are commonly seen in an aggressive form of dermatomyositis or
PM known as anti-synthetase syndrome. Patients with anti-synthetase syndrome may
develop rapid-onset interstitial lung disease and can present with shortness of breath as
their initial symptom in the absence of prominent muscle weakness or rash. Anti-Jo1 is
the most common antibody associated with anti-synthetase syndrome, and it is often
ordered after a diagnosis of dermatomyositis or PM is made to assess the risk of
developing rapidly progressive lung disease. Antinuclear antibodies (ANAs) are
nonspecific and can be seen in a variety of different connective tissue diseases. Anti-
dsDNA is associated with lupus, and anti-histone antibodies are seen in drug-induced
lupus. Anti-topoisomerase I antibodies are also known as anti-Scl-70 antibodies and are
seen in certain types of scleroderma. Anti-RNA polymerase III antibodies are also
associated with scleroderma and can increase the risk of developing scleroderma renal
crisis.

30. The correct answer is: D. Colonoscopy. Due to its strong association with malignancy,
all patients with a new diagnosis of dermatomyositis should undergo thorough cancer
screening. This includes low-dose chest CT (as indicated for lung cancer screening),
mammogram, Pap smear, and colonoscopy. Many rheumatology providers will also obtain
a transvaginal ultrasound to screen for ovarian cancer.

31. The correct answer is: B. Mixed connective tissue disease (MCTD). MCTD is a unique
condition that represents an overlap of lupus, systemic sclerosis, and inflammatory
myopathy. Patients often present with vague symptoms, including fatigue, arthralgias,
myalgias, Raynaud phenomenon, hand edema, puffy fingers, synovitis, and low-grade
myositis with muscle weakness. Patients rarely present with clear overlap from multiple
diseases initially but often develop these features over several years. MCTD falls into the
connective tissue disease family and patients generally have a positive antinuclear
antibody (ANA) and may have SSA or SSB positivity as well. Rheumatoid factor is a
nonspecific marker and may be seen in up to 50% of patients with MCTD. The diagnosis
requires positive anti-U1 RNP antibodies, by definition.

32. The correct answer is: D. Pulmonary hypertension. Pulmonary hypertension is the
major cause of death in these patients.

33. The correct answer is: A. Adalimumab. The patient presents with fevers, arthralgia,
rash, and chest pain upon inspiration, consistent with pleuritis. Laboratory testing is
notable for a positive antinuclear antibody, positive anti-dsDNA, and a positive anti-
histone antibody. His symptoms and labs are most consistent with a diagnosis of drug-
induced lupus. Drug-induced lupus is a relatively uncommon disorder caused by an
autoimmune response triggered by certain medications. Common medications known to

Freemedicalbooks4download
be associated with drug-induced lupus include isoniazid, chlorpromazine, hydralazine,
methyldopa, procainamide, TNF-α inhibitors, minocycline, and penicillamine. From this
patient’s potential medication list described in the question above, adalimumab (TNF-α
inhibitor) is the only medication known to be associated with drug-induced lupus.

34. The correct answer is: C. Clinical history. While it shares many features in common
with lupus (systemic lupus erythematosus), drug-induced lupus occurs with equal
frequency in males and females, often occurs in older adults due to increased exposure to
causative medications, and generally has a more abrupt onset. The most common
symptoms of drug-induced lupus include fever, malaise, arthritis, rash, and serositis. The
classic malar and discoid rashes seen in systemic lupus erythematosus are uncommon in
drug-induced lupus, and renal disease is rare. The skin biopsy for both systemic lupus
erythematosus and drug-induced lupus would show interface dermatitis. Laboratory
testing is generally positive for anti-histone antibodies in patients with drug-induced lupus
(>90%), but anti-histone antibody positivity rates can be as high as 80% in patients with
systemic lupus erythematosus. While the exception of anti-TNF induced lupus, anti-
dsDNA positivity rates are much lower in drug-induced lupus than in systemic lupus
erythematosus and may serve to differentiate the two diseases. The diagnosis of drug-
induced lupus relies on a plausible clinical syndrome, consistent laboratory testing, and a
clear medication exposure.

35. The correct answer is: D. Start steroids.

36. The correct answer is: C. Cyclophosphamide. This patient with a known history of
lupus presents with fatigue, low-grade fevers, rash, and synovitis. Her symptom
constellation is most consistent with a mild-to-moderate lupus flare. In addition to
symptoms observed in our patient, lupus flares can also present with malaise, headache,
oral or nasal ulcers, photosensitivity, shortness of breath, chest pain, decreased urine
output, hematuria, confusion, and memory loss. Some lupus flares are asymptomatic and
may only be suggested by abnormal results of laboratory testing, such as acute kidney
injury or worsening anemia. Her symptoms certainly warrant treatment, and a short course
of prednisone is most appropriate. Hydroxychloroquine is the foundation of lupus
treatment and should not be stopped in the setting of an acute flare. If her symptoms were
to persist despite prednisone therapy, or if the patient were to develop organ involvement,
IV steroid therapy and/or the addition of other immunosuppressive agents may be
warranted.
Lupus nephritis is a serious complication of systemic lupus erythematosus. The
diagnosis is made by kidney biopsy, and aggressive treatment is generally warranted.
Pending the patient’s age, race, and medication adherence history, new-onset lupus
nephritis is generally treated with either oral mycophenolate mofetil or cyclophosphamide
therapy.

37. The correct answer is: B. Hydroxychloroquine. Hydroxychloroquine is the foundation


of lupus treatment, and it is the only medication that provides a significant survival and
morbidity benefit across the lupus spectrum. hydroxychloroquine therapy is known to
cause retinopathy, and fundoscopic examination often reveals parafoveal retinal pigment
atrophy and “bull’s eye maculopathy.” The risk of retinopathy is dependent on both the
cumulative dose of hydroxychloroquine and duration of exposure.

38. The correct answer is: D. Screening ophthalmologic examinations. Patients taking
hydroxychloroquine should have a baseline screening fundoscopic examination with an
ophthalmologist to rule out preexisting retinal disease and then regular screening with
fundoscopic examination and visual field testing. Spectral-domain optical coherence
therapy (SD OCT) should be performed, especially after 5 years of receiving
hydroxychloroquine therapy. Regular ophthalmologic examinations help identify
hydroxychloroquine-induced retinopathy before vision changes occur, and further damage
can often be prevented by stopping hydroxychloroquine therapy, although
hydroxychloroquine retinopathy may also progress despite stopping therapy.

39. The correct answer is: B. Eosinophilic granulomatosis with polyangiitis.

40. The correct answer is: D. Electromyography (EMG) and nerve conduction study
(NCS). The patient presents with fevers, unintentional weight loss, hemoptysis, petechial
rash, and mononeuritis multiplex, concerning for development of a new, small-vessel
vasculitis. Given his history of allergic rhinitis, asthma, and the peripheral eosinophilia
seen on his WBC differential, his clinical presentation is most consistent with eosinophilic
granulomatosis with polyangiitis (formerly Churg-Strauss syndrome). Eosinophilic
granulomatosis with polyangiitis is a rare form of antineutrophil cytoplasmic antibody
(ANCA) vasculitis that causes granulomatous inflammation of small- and medium-sized
vessels in the body. The disease most commonly affects the lung and the skin but can
cause damage to any organ in the body. Laboratory testing for p-ANCA is only positive in
30% to 60% of patients, and the majority with positive p-ANCA testing have an MPO
antigen-specific perinuclear staining pattern. Patients with nerve involvement from ANCA
vasculitis may present with wrist or foot drop due to mononeuritis multiplex. A combined
EMG and NCS provides valuable diagnostic information and can help identify this
specific type of progressive sensory and motor peripheral neuropathy. EMG/NCS
combined with ANCA serologic testing and biopsy data is often used to make a diagnosis
of ANCA vasculitis. Biopsy showing perivascular necrotizing eosinophilic granulomas or
eosinophilic infiltration remains the gold standard for the diagnosis of eosinophilic
granulomatosis with polyangiitis, but nerve biopsy can cause permanent nerve damage
and would likely be avoided in a patient with a clinical syndrome compatible with
eosinophilic granulomatosis with polyangiitis and EMG/NCS evidence of mononeuritis
multiplex. A significant history of asthma or rhinosinusitis with peripheral eosinophilia on
laboratory testing can help differentiate eosinophilic granulomatosis with polyangiitis
from other forms of ANCA vasculitis, such as granulomatosis with polyangiitis and
microscopic polyangiitis.

41. The correct answer is: E. Polyarteritis nodosa (PAN). The patient presents with fever,
malaise, weight loss, abdominal pain, hematochezia, and myalgias and is found to have
purpura and mononeuritis multiplex on examination. The patient is also noted to be
hypertensive with elevated inflammatory markers and an acute kidney injury on
laboratory testing. His clinical presentation is concerning for a medium-vessel vasculitis,
and the presence of renal artery aneurysms with large-vessel constrictions and occlusion
of the small arteries confirms a diagnosis of PAN. PAN is a medium-vessel vasculitis that

Freemedicalbooks4download
often presents with nonspecific systemic symptoms, hypertension due to renin-
angiotensin-aldosterone system (RAAS) activation with renal artery disease, acute kidney
injury, abdominal pain especially after meals (mesenteric arteritis), GI bleeding,
mononeuritis multiplex, and myalgias. When attempting to differentiate PAN from the
other causes of vasculitis, PAN importantly tends to spare the lungs.

42. The correct answer is: B. Hepatitis B. The majority of cases of polyarteritis nodosa
(PAN) are idiopathic, but hepatitis B is also known to cause PAN in up to one-third of
cases. Hepatitis C and hairy cell leukemia are also less common causes of secondary
polyarteritis nodosa (PAN).

43. The correct answer is: D. IgA vasculitis. The patient presents with petechial rash, lower
extremity arthritis, and abdominal pain 3 weeks after a streptococcal pharyngitis infection.
Skin biopsy shows leukocytoclastic vasculitis with positive IgA staining, confirming the
diagnosis of IgA vasculitis (formerly Henoch-Schönlein purpura). IgA vasculitis is
typically a self-limited disease and often resolves spontaneously without intervention
within approximately 1 month. More severe disease can sometimes lead to an acute
kidney injury requiring temporary dialysis and GI complications, including
intussusception (generally in children), GI bleeding, bowel ischemia and necrosis, and
bowel perforation. Thankfully, serious complications are relatively rare, and this patient
appears to have a milder case of the disease.

44. The correct answer is: C. Observation. Patients without renal involvement are generally
managed symptomatically, and NSAIDs can be used to treat arthritis in the absence of
renal disease. The use of steroids to treat IgA vasculitis is controversial. In adults, a
prolonged steroid taper is generally only considered in patients with severe renal disease
or if the patient has a contraindication to receiving NSAIDs.

45. The correct answer is: A. Endoscopic ultrasound with biopsy. This patient presents
with painless obstructive jaundice and is found to have a 3-cm mass in the pancreatic head
with widespread lymphadenopathy and elevated serum IgG4 levels. While a similar
presentation could be seen in pancreatic cancer, the elevated IgG4 levels and widespread
lymphadenopathy without additional evidence of metastatic disease suggest IgG4-related
disease. Biopsy remains the gold standard for diagnosis, and endoscopic ultrasound is the
preferred method for accessing lesions within the pancreas. Tissue slides in IgG4-related
disease typically show a dense plasma cell infiltrate with storiform fibrosis and
immunohistochemical stains that are positive for IgG4 deposition. IgG4-related disease
was only recently recognized as a distinct rheumatic condition. Patients may present with
painless obstructive jaundice from a mass in the pancreas, but recurrent episodes of
autoimmune pancreatitis are the most common presentation. Patients also typically have
lymphadenopathy and may develop disease in several other organ systems, including, but
not limited to, sclerosing cholangitis, retroperitoneal fibrosis, sclerosing sialadenitis of the
salivary glands, and orbital disease.

46. The correct answer is: E. Steroids. Steroids are the first line of treatment for IgG4
disease, and a good response to steroid administration helps to confirm the diagnosis.
47. The correct answer is: D. Cryoglobulin testing. A patient presents with general malaise,
arthralgias, palpable purpura, and new-onset peripheral neuropathy. His symptom
constellation is concerning for a small-vessel vasculitis, and his history of hepatitis C
raises suspicion for mixed cryoglobulinemia syndrome. Other supporting lab testing for
mixed cryoglobulinemia includes a positive rheumatoid factor and low C4 levels. Mixed
cryoglobulinemia refers to a small-vessel vasculitis caused by immune complex
deposition. The disease can present in a number of ways but is most commonly associated
with palpable purpura, joint pain (arthralgia or arthritis), and weakness (Meltzer triad). It
may also be associated with peripheral neuropathy, renal disease, or a variety of other
systemic manifestations. Cryoglobulin testing and biopsy can confirm the diagnosis of
mixed cryoglobulinemia. Cryoglobulins can precipitate at room temperature, so blood
samples must be collected in warmed tubes that remain warm until processed by the lab to
prevent false-negative cryoglobulin testing. Leukocytoclastic vasculitis is generally seen
on skin biopsy, and most patients with renal disease present with membranoproliferative
glomerulonephritis. Since the patient is not exhibiting signs of renal disease, kidney
biopsy would not be appropriate at this time.

48. The correct answer is: C. Hepatitis C. Hepatitis C is a recognized risk factor for mixed
cryoglobulinemia.

49. The correct answer is: E. Serum protein electrophoresis (SPEP) and urine protein
electrophoresis (UPEP) with immunofixation. The patient presents with peripheral
neuropathy, recent carpal tunnel, volume overload, and easy bruising. Examination is
notable for enlargement of the anterior shoulder (“shoulder pad sign”),
hepatosplenomegaly, and periorbital bruising. Laboratory testing reveals an elevated
creatinine with significant proteinuria. The patient’s presentation is most concerning for
amyloid light-chain (AL) amyloidosis. AL amyloidosis is a primary amyloidosis due to an
overproduction of monoclonal light-chain antibodies that cause complications when
deposited in various tissues of the body. The presenting symptoms of AL amyloidosis
differ depending on which organs are most affected. If the kidneys or heart is primarily
involved, AL amyloidosis can present with volume overload due to nephrotic syndrome or
restrictive cardiomyopathy. The majority of patients with AL amyloidosis have hepatic
involvement, which can present with hepatomegaly or elevated liver enzymes in a
cholestatic pattern. Patients may also have musculoskeletal involvement, leading to
findings such as macroglossia, tongue scalloping, muscular pseudohypertrophy, shoulder
arthropathy (“shoulder pad sign”), carpal tunnel, or involvement of other joints or
connective tissues. Other potential findings include splenomegaly, anemia,
thrombocytopenia, easy bruising or bleeding, peripheral neuropathy, and periorbital
purpura (“raccoon eyes”). In this patient with renal involvement, both serum and urine
free light chains are likely to be positive on SPEP and urine protein electrophoresis,
respectively. In addition to SPEP and urine protein electrophoresis testing, biopsy of
several sites can also lead to a diagnosis of AL amyloidosis. The most common and least
invasive procedure used to obtain a tissue diagnosis is an abdominal fat pad biopsy. A
bone marrow biopsy is also commonly performed.

50. The correct answer is: E. Positive Congo red staining with “apple green”
birefringence. Pathology of the affected tissue in patients with AL amyloidosis will show

Freemedicalbooks4download
infiltration of an amorphous, waxy substance that will demonstrate positive staining with
Congo red. Although liver biopsy may be considered if the initial testing is equivocal, it
would likely be considered only after SPEP, urine protein electrophoresis, and the less
invasive abdominal fat pad biopsy were obtained. Interface dermatitis is associated with
cutaneous lupus, focal lymphocytic sialadenitis is seen on salivary gland biopsy in Sjögren
syndrome, a dense lymphoplasmacytic infiltrate with “storiform fibrosis” is seen in IgG4-
related disease, and excessive organized collagen deposition with expansion of the dermis
is associated with scleroderma.
9
NEUROLOGY

QUESTIONS

1. A 78-year-old woman with a history of Alzheimer’s disease, hypertension, atrial


fibrillation on apixaban, and type 2 diabetes on insulin presents with a sudden change in
mental status. At baseline, she has short-term memory difficulty and with remembering
names. Her family helps her with medications and preparing meals, but she feeds and
dresses herself, and has been able to remain in her home. Her daughter came to bring
dinner and found her sitting at the kitchen table in her nightgown, disheveled-appearing,
withdrawn, and unable to answer questions appropriately. She was brought to the
emergency room. On examination, she was found to have a low-grade fever with
otherwise normal vital signs, dry mucous membranes, clear lungs, and a soft, nontender,
and nondistended abdomen. She was drowsy, often falling asleep unless constantly
stimulated. She was able to give her name, but she stated that she was at home and was
not oriented to month or year. She was able to name the days of the week forward, but not
backward. The remainder of the neurologic examination was normal.
What is the next best step in her management?
A. Basic metabolic panel and complete blood count
B. Blood glucose fingerstick
C. Noncontrast head computed tomography (CT)
D. Routine electroencephalogram (EEG)
E. Urinalysis and urine culture

2. The patient in Question 1’s labs return and are notable for a normal blood glucose; serum
white blood cell (WBC) count of 12 000/μL and urinalysis showing 50 to 100 WBC/hpf,
positive leukocyte esterase, and positive nitrites. She has a head CT that shows stable,
moderate atrophy most predominant in the temporal and parietal lobes, without any
hemorrhage or other acute findings.
Which of the following is NOT true about the cause of her mental status change?
A. Advanced patient age and underlying neurologic disease are risk factors.
B. Inattention is a hallmark feature.
C. It is associated with increased mortality.

Freemedicalbooks4download
D. Symptoms are slowly progressive.
E. There is often an identifiable trigger.

3. A 56-year-old man presents for evaluation of behavioral changes. His wife reports that
the patient has become a “different person” over the past 2 years. He used to be a very
calm and pleasant person, but over that period of time he has become “nasty.” He curses
in public, spits on the bus, and makes unwanted sexual advances toward women. He has
also been less able to complete tasks at work and was recently given a formal warning
from his employer that he is at risk of losing his job if he does not change his behavior and
meet deadlines. During the neurologic examination, he calls the neurologist “honey” and
“gorgeous” and asks if he can use the reflex hammer to “hit the doctor back.” He
undergoes brain imaging, which is depicted below:
A: Sagittal view. B: Axial view. Images courtesy of Dr. Omar Al-Louzi, Harvard Neurology Residency Program,

Freemedicalbooks4download
Brigham and Women’s & Massachusetts General Hospitals.

What is the most likely diagnosis?


A. Alzheimer’s disease
B. Behavioral variant of frontotemporal dementia
C. Corticobasal degeneration
D. Lewy body disease
E. Primary progressive aphasia

4. A 25-year-old woman presents to the hospital after a first-time seizure.


Which of the following could have contributed to her seizure risk?
A. Alcohol withdrawal
B. Bupropion
C. Hypoglycemia
D. Hyponatremia
E. All of the above

5. Which of the following is the first step if you believe someone is in status epilepticus?
A. Administer intravenous lorazepam
B. Administer intravenous phenytoin
C. Administer stat naloxone
D. Ensure proper oxygenation
E. Refer for head CT

6. A 23-year-old man with no past medical history presents after losing consciousness at the
supermarket. He recalls feeling flushed and slightly anxious in the moments before the
event, and his next memory is of being on the ground attended to by a bystander. This
bystander reported that the patient had lost consciousness for roughly 30 seconds, during
which time he had occasional symmetric convulsions. He was drowsy and slightly
confused for 10 to 15 minutes following this event, after which he returned to baseline. He
notes that his tongue is sore. He is brought to the hospital and testing is ordered to
investigate for possible seizure versus syncope.
Which of the following statements is FALSE?
A. A normal routine EEG excludes the possibility of an underlying seizure disorder.
B. Echocardiogram should be performed to assess for structural cardiac disease.
C. Syncope is frequently accompanied by convulsions.
D. The patient should be counseled to refrain from driving for as long as is required by
the driving regulations of his state.
E. Tongue laceration increases the probability of seizure.

7. A 39-year-old woman with history of migraine, alcohol use disorder, and major
depressive disorder with a prior suicide attempt presents to the emergency department
(ED) with a first-time seizure, characterized by sudden loss of consciousness, rightward
head turn, and rhythmic convulsions of the right arm lasting 2 minutes with a prolonged
period of somnolence and confusion following the event. Laboratory studies reveal Na
132 mEq/L, aspartate transaminase (AST) 178 U/L, alanine aminotransferase (ALT) 98
U/L, and platelets 118 × 109/L. Electrocardiogram (ECG) reveals first-degree heart block
and is otherwise normal. Brain magnetic resonance imaging (MRI) is unremarkable. EEG
shows infrequent epileptiform discharges originating from the left temporal lobe.
Which of the following is a contraindication to starting levetiracetam in this
patient?
A. First-degree heart block
B. Hepatic disease
C. History of major depression
D. Hyponatremia
E. Thrombocytopenia

8. Deficiency in which of the following vitamins results in a syndrome characterized by


ophthalmoparesis, ataxia, and altered mental status?
A. Cyanocobalamin
B. Pantothenic acid
C. Pyridoxine
D. Riboflavin
E. Thiamine

9. A 55-year-old man with a history of depression and alcohol abuse presents to the hospital
for a planned surgery. About 72 hours after the surgery, the patient tells his nurse that he
is hearing his friends tell him that he has been abducted by aliens. There is no one else in
his room. He is shaky, hypertensive, and tachycardic on examination. His blood work is
normal.
What is the most likely diagnosis?
A. Delirium tremens
B. Depression
C. Posttraumatic stress disorder
D. Schizoaffective disorder

10. A 48-year-old man with a history of chronic alcohol use disorder presents 36 hours after
his last drink. He is tremulous, diaphoretic, and anxious. Heart rate (HR) is 110-120
beats/min and blood pressure (BP) is 177/93 mmHg.
All of the following medications would be appropriate for treating this patient’s
alcohol withdrawal EXCEPT:
A. Alprazolam
B. Chlordiazepoxide
C. Diazepam
D. Lorazepam
E. Phenobarbital

11. A 25-year-old woman presents to the ED with dizziness and vomiting since the morning.
She reports that she was sick with a cold about a week ago, but otherwise had been
healthy. On examination, you notice she has horizontal gaze-evoked nystagmus beating to
the right that is worse when looking to the right and goes away when looking to the left. A
catch-up saccade was seen with left head thrust. No skew was noted on the cover-uncover
test. Her hearing and the rest of her neurologic examination are normal.
What is the most likely diagnosis?

Freemedicalbooks4download
A. Benign paroxysmal positional vertigo
B. Labyrinthitis
C. New posterior circulation stroke
D. Vestibular neuritis

12. A 78-year-old woman comes to clinic with repeated episodes of falls. On history she tells
you that every time when she first wakes up to get out of bed, she has an episode of
feeling very dizzy and she is unable to walk for 30 seconds. She has a normal neurologic
examination.
What is the most likely diagnosis?
A. Benign paroxysmal positional vertigo
B. Concussion
C. Migraine
D. Vertebrobasilar insufficiency
E. Vestibular neuritis

13. A 53-year-old woman with type 2 diabetes presents with sudden onset of dizziness and
nausea, which began when she was eating breakfast. She has a constant feeling of
spinning toward the right. The symptoms did not resolve after a period of lying down and
she presented to the ED for evaluation.
All of the following findings would be concerning for a posterior circulation stroke
EXCEPT:
A. Diminished hearing
B. Direct-changing nystagmus
C. Double vision
D. Dysarthria
E. Limb ataxia

14. Each of the following is a component of the HINTS test for sudden-onset dizziness
EXCEPT:
A. Finger-to-nose coordination test
B. Head impulse test
C. Nystagmus
D. Test of skew

15. A 65-year-old man with a history of hypertension and dyslipidemia was brought to the
hospital when his family found him unable to speak, eyes deviated to his left side, with
right facial drooping, and inability to move his right arm.
The most likely location of his stroke is in the distribution of which of the
following?
A. Left middle cerebral artery (MCA)
B. Left posterior cerebral artery
C. Right anterior cerebral artery
D. Right MCA
E. Right posterior inferior cerebellar artery

16. Which of the following patients is best qualified to receive tissue plasminogen activator?
A. A 40-year-old woman with no past medical history who presents with left-sided
whole-body weakness, which she noticed when she woke up. She last felt normal at 8
PM the night before.
B. A 55-year-old man with orthopedic surgery 1 week ago who presents with whole-
body weakness on the right side that started 3 hours earlier.
C. A 60-year-old man with an intracerebral hematoma 4 weeks ago who presents with
acute onset of incoordination and dizziness that started 30 minutes prior to arrival.
D. A 68-year-old woman with a history of hypertension and diabetes who presents to the
hospital with sudden onset of left face and arm weakness that started 4 hours ago.
E. A 92-year-old man with a history of hypertension who presents with 5 hours of
difficulty seeing things off to his left side.

17. A 68-year-old woman with history of hypertension, hyperlipidemia, type 2 diabetes, and
coronary artery disease presents with sudden onset of expressive aphasia and right face
and arm weakness. She is brought to the ED 60 minutes after symptom onset. Her NIHSS
(National Institutes of Health Stroke Scale) score is 17. CT with angiography of the head
and neck is ordered.
This test does all of the following EXCEPT:
A. Assess for a proximal large-vessel occlusion that could warrant endovascular
thrombectomy
B. Assess the cervical and intracranial vasculature for atherosclerotic disease as a
possible stroke etiology
C. Determine the extent of an acute infarct
D. Is contraindicated in patients with end-stage renal disease (ESRD) or allergy to
iodine
E. Rules out intracranial hemorrhage

18. A 74-year-old man with hypertension and hyperlipidemia presents with sudden onset of
left face, arm, and leg weakness, which began suddenly while the patient was on the toilet.
He is found by his wife to be dysarthric, weak throughout the left side, and unable to
stand. The patient is brought to the ED, where noncontrast head CT reveals a 2 × 3 × 2 cm
hemorrhage in the right basal ganglia.
Which of the following is the most likely etiology?
A. Aneurysm rupture
B. Arteriovenous malformation
C. Cerebral amyloid angiopathy
D. Hemorrhage of a neoplastic lesion
E. Hypertension

19. A 55-year-old woman with no significant past medical history had fever, malaise, and sore
throat that was treated with antibiotics and analgesics. Ten days later, her legs “buckled”
underneath her, she had difficulty climbing stairs, and noticed tingling in both hands. She
awoke the next morning and could not stand. She was brought to the hospital, where vital
signs were normal. Neurologic examination was notable for weakness of eye closure
bilaterally with otherwise intact cranial nerves, symmetric proximal more than distal
weakness (neck flexion 4+/5, shoulder abduction 4/5, 5−/5 more distally in the upper
extremities, hip flexion 3/5, 4+/5 more distally in the lower extremities), diffuse areflexia,

Freemedicalbooks4download
decreased sensation to light touch and pinprick in both hands, and inability to sit or stand
independently. Lumbar puncture was performed and showed 0 WBCs x 106/L, 4 red blood
cells x 106/L, protein of 86 mg/dL, and glucose of 65 mg/dL.
What is the next best step in her management?
A. Electromyography (EMG)/nerve conduction studies
B. Intravenous corticosteroids
C. Intravenous immunoglobulin (IVIG)
D. MRI of the lumbar spine
E. Vancomycin, ceftriaxone, and acyclovir

20. A 57-year-old man with history of poorly controlled type 2 diabetes presents to clinic
complaining of a burning pain in his feet, which is worse at night and sometimes prevents
him from falling asleep. He endorses some numbness in the bilateral toes but denies any
weakness or sensory symptoms in the hands. On examination, he has mildly decreased
sensation to vibration at the bilateral great toes, with normal joint position sense.
Vibration is normal at the ankles. Strength and reflexes are normal. Labs show
hemoglobin A1c 8.2%, vitamin B12 723 ng/mL, thyroid-stimulating hormone (TSH) 1.16
mU/L, and serum protein electrophoresis (SPEP) with immunofixation normal without M-
spike. You diagnose him with a diabetic peripheral neuropathy and plan to optimize his
glucose control. You would also like to address his pain.
Which of the following is not an effective option for treating neuropathic pain?
A. Nonsteroidal anti-inflammatory drugs (NSAIDs)
B. Nortriptyline
C. Pregabalin
D. Topical capsaicin
E. Venlafaxine

21. A 60-year-old day laborer had difficulty climbing stairs and getting up from the toilet seat
for the last 5 months, particularly in the morning on waking up, which improves slightly
throughout the day. He also complained of dry mouth and fatigue. There was no
dysphagia or dysarthria. He had borderline diet-controlled diabetes and angina. He had
smoked two packs of cigarettes per day for 30 years. He consumed several alcoholic
drinks a day. The neurologic examination showed that the cranial nerves were intact,
except for questionable sluggish pupils. Neck flexion was weak, deltoids were 4/5, and
hip flexion was 3/5. He could hardly walk on his heels and toes. Deep tendon reflexes
(DTRs) were trace with an absent ankle jerk bilaterally. His gait was cautious and slightly
wide-based. Nerve conduction studies showed diffusely reduced motor amplitudes
(compound muscle action potentials). After rapid repetitive nerve stimulation (30 to 50
Hz) or brief (10 sec), intense contractions, a marked increase of the compound muscle
action potential amplitude was seen.
What treatment would be most effective in reducing symptoms in this condition?
A. 3,4-Diaminopyridine (3,4-DAP)
B. IVIG
C. Pyridostigmine
D. Rituximab
E. Steroids
22. A 63-year-old right-handed retired masonry worker presented to the clinic for evaluation
regarding gait dysfunction with neck, shoulder, and leg weakness over the past 2 years. He
reported experiencing a gradual deterioration in his gait as well as difficulty going up and
down stairs. On examination, his vital signs were normal, and he was breathing on room
air without excessive work of breathing or usage of accessory muscles. There was
evidence of moderate erythema and papules over the extensor surfaces of his fingers and
an erythematous rash on his upper chest and back. Motor examination was notable for
mild atrophy of the periscapular muscles without apparent fasciculations. Neck flexion
and extension were mildly weak (4+/5). There was evidence of proximal greater than
distal weakness of the upper and lower extremities (4/5 in shoulder abduction, 4/5 in
shoulder external rotation, and 4+/5 in hip flexion and abduction). Deep tendon reflexes
(DTRs) were 2+ throughout with down-going toes. He had a stooped posture with
standing. His creatine kinase (CK) level was mildly elevated. Nerve conduction studies
were normal, and EMG showed prominent spontaneous activity (fibrillation potentials and
positive sharp waves).
Which of the following findings on muscle biopsy would be most consistent with
his diagnosis?
A. Nodular collections of immune cells and degeneration with rimmed vacuoles
B. Perifascicular atrophy and perivascular inflammation
C. Prominent myofiber necrosis
D. Ragged red muscle fibers
E. Type 2 muscle fiber atrophy

23. A 28-year-old man with a history of smoking and heavy alcohol use comes to clinic
complaining of daily headaches. Every evening at about 10 PM, he develops a left-sided
stabbing pain in the front of his head that intensifies over 5 minutes to an excruciating
level. The pain lasts about 30 minutes and then resolves. He also reports tearing and
redness in the left eye and rhinorrhea. He has had these symptoms nightly for the last 2
weeks, although he recalls a similar period of headaches 1 year ago that resolved.
This patient’s symptoms are most consistent with which of the following?
A. Cluster headache
B. Secondary headache
C. Short-lasting unilateral neuralgiform pain with conjunctival injection and tearing
D. Tension headache
E. Trigeminal neuralgia

24. A 28-year-old woman with no past medical history on oral contraceptive pills comes to
clinic complaining of daily headaches. For the past 3 months, she has had a continuous
daily left-sided headache. The headache intensity is typically moderate, although she does
have exacerbations with severe pain that she likens to an icepick stabbing pain behind the
left eye. During these exacerbations, she has rhinorrhea and left-sided tearing and redness
in the eye. Her neurologic examination is normal and she has had an MRI of the brain
with a venogram that did not show a structural lesion or venous sinus thrombosis.
What is the best initial treatment?
A. Indomethacin
B. Sumatriptan
C. Topiramate

Freemedicalbooks4download
D. Tricyclic antidepressant
E. Verapamil

25. A 47-year-old woman is seen in clinic for evaluation of multiple subcortical ischemic
strokes and abnormal brain imaging. She has experienced three separate subcortical
strokes over the past 5 years, despite no known stroke risk factors and good adherence to
healthy lifestyle habits. Her personal medical history is notable for migraine with aura
dating back to adolescence; family history is remarkable for severe migraines in her sister
and early strokes and cognitive impairment in her mother. Brain MRI is shown below:

Image courtesy of Dr. Omar Al-Louzi, Harvard Neurology Residency Program, Brigham and Women’s &
Massachusetts General Hospitals.

Genetic testing is most likely to show abnormalities in which of the following


genes?
A. α-galactosidase A gene (GAL)
B. COL4A1 mutation
C. HtrA serine peptidase 1 (HTRA1) gene
D. NOTCH3
E. TREX1 gene

26. A 24-year-old woman with a history of polycystic ovarian syndrome on oral contraceptive
pills presents with headache of 3 weeks’ duration. She reported experiencing pressure-
like, right occipital headaches for the prior 2 to 3 weeks. They were intermittent in nature,
occurring every 2 to 3 days and lasting a couple of hours. However, over the past 5 days,
her headaches have become constant and progressively worsened in their severity. They
wake her up at night. They worsen in intensity with lying flat and she describes sitting up
as the best position. Examination shows evidence of bilateral papilledema. Cranial
imaging is obtained and shown below:

Image courtesy of Dr. Omar Al-Louzi, Harvard Neurology Residency Program, Brigham and Women’s &
Massachusetts General Hospitals.

What would be the next best step in her management?


A. Administer mannitol 1 g/kg IV
B. Maintain BP <140 mmHg
C. Obtain blood cultures and start broad-spectrum intravenous antibiotics
D. Obtain lumbar puncture to rule out meningitis
E. Send thrombophilia workup and start therapeutic anticoagulation with heparin

27. A 56-year-old man with a history of hypertension and obesity presents with weakness and
numbness in the right foot. He has trouble fully flexing at the ankle and has noticed that
he catches his toes when walking, causing him to trip. He also reports shooting pain from
the low back down the right lateral thigh and calf into the great toe. His symptoms came
on suddenly after he helped his friend move. Initially he thought he had a muscle strain
but decided to seek medical attention given the difficulty walking. On physical
examination, he has 4+/5 weakness in right ankle dorsiflexion, foot inversion, and foot
eversion. He has decreased sensation to pinprick over the right lateral calf and great toe.
Reflexes are normal. When the patient is in a supine position and right leg is raised to 60°,
it elicits a shooting pain from the low back down the right leg.

Freemedicalbooks4download
What is the localization of his symptoms?
A. L4 nerve root
B. L5 nerve root
C. Peroneal nerve
D. S1 nerve root
E. Thoracic spinal cord

28. A 71-year-old woman with history of hypertension, hyperlipidemia, and coronary artery
disease presents for evaluation of leg pain. The pain starts in her back and radiates down
to both legs. It is worse with walking and better with rest, although she notes that she is
able to use a stationary bicycle without much pain. She also reports pain and cramping in
her calves, as well as numbness and tingling in both feet. On examination, she has full
strength throughout, 1+ patellar reflexes, and absent ankle jerks. Pulses are normal.
Which feature(s) of her presentation are less consistent with neurogenic
claudication, as opposed to a vascular cause?
A. Absent ankle jerks
B. Calf cramping
C. Numbness and paresthesias in the feet
D. Radiating pain down the legs
E. Both A and B

29. A 51-year-old woman with a history of non–small cell lung cancer presents to the ED with
bilateral lower extremity weakness and numbness. Two hours prior to presentation, she
noted sudden onset of worsening of her thoracic back pain, in addition to new numbness
and tingling in her thighs bilaterally. This was associated with difficulty in walking and
bilateral leg weakness. She was still able to walk but felt her legs were wobbly. She did
not notice any urinary or stool incontinence. On examination, strength was full in her
upper extremities. There was mild weakness in hip flexion bilaterally (4+/5). She was
hyperreflexic at the patellae bilaterally (3+) with crossed adduction. Her toes were
upgoing bilaterally. She has altered sensation to pinprick at the level of the nipples
anteriorly. Imaging of the spine was obtained and is shown below:
A: Sagittal view. B: Axial view. Images courtesy of Dr. Omar Al-Louzi, Harvard Neurology Residency Program,
Brigham and Women’s & Massachusetts General Hospitals.

What is the most appropriate next step in her management?


A. Administer dexamethasone 10 mg IV, with urgent evaluation for thoracic spine
decompression
B. Administer mannitol 1 g/kg IV
C. Immobilization with a hard cervical collar
D. Obtain blood cultures and start broad-spectrum intravenous antibiotics
E. Supplementation with vitamin B12

ANSWERS

1. The correct answer is: B. Blood glucose fingerstick. This patient presents with acute
change in mental status, superimposed on chronic cognitive decline due to Alzheimer’s
disease. The next best step in management is to check her blood glucose. She is at risk for
hypoglycemia because she is on insulin, and this is a fast point-of-care test that can be
done immediately at the bedside. Basic metabolic panel, complete blood count, and
urinalysis should also be checked as part of the workup of encephalopathy, but would not
be the next best test. Initial labs should also include liver function tests and a toxicology
screen; the list can be expanded based on the clinical picture. Patients with Alzheimer’s
disease are at increased risk of seizures due to encephalomalacia, so EEG may be
appropriate for some patients. Noncontrast head CT is often indicated for acute change in
mental status, particularly in patients on anticoagulation, but is not the next best test.

2. The correct answer is: D. Symptoms are slowly progressive. This patient has delirium
secondary to a urinary tract infection. Patients have waxing and waning mental status
changes, rather than slowly progressive symptoms. Inattention, as demonstrated by her
inability to name days of the week backward, is a hallmark feature. Delirium is a symptom
and should prompt a workup for underlying medical causes, including infection, metabolic
abnormalities, toxin exposure/medications, and physiologic stressors (such as pain or
constipation). An underlying dementia makes patients more vulnerable to delirium. Other
risk factors include advanced age, medical and psychiatric comorbidities poor functional
baseline, and sensory impairment. Delirium is easier to prevent than to treat and is
associated with increased mortality, so it is important to actively prevent delirium in
hospitalized patients.

3. The correct answer is: B. Behavioral variant of frontotemporal dementia. The patient
presents with a progressive behavioral syndrome manifested by disinhibition. His
personality dramatically changed from calm and pleasant to inappropriate. The symptoms
are clearly impairing both his social and occupational functioning. Frontotemporal
dementia causes atrophy that typically involves the frontal and temporal lobes, as
illustrated in the figure. This is a tauopathy, and, hence, pathology typically demonstrates
rounded intracytoplasmic structures with immunoreactivity to tau immunostains.
Alzheimer’s disease can certainly manifest with behavioral changes; this typically occurs

Freemedicalbooks4download
during the later stages of the disease, and early frontal atrophy is not characteristic of this.
Lewy body disease and corticobasal degeneration can manifest with psychosis but not
dramatic behavioral changes without other Parkinsonian features. There is no language
involvement in this patient to suggest primary progressive aphasia.

4. The correct answer is: E. All of the above. All of the above causes put the patient at risk
for seizures. Generally, withdrawal and intoxications from medications and alcohol can
put patients at risk for seizures. Laboratory abnormalities, particularly low glucose and
sodium, are seizure risk factors. Bupropion is one of many medications known to lower
the seizure threshold.

5. The correct answer is: D. Ensure proper oxygenation. The first step of managing
seizure activity is to ensure the patient’s ABCs (airway, breathing, and circulation) are
addressed. Secondary testing, including head CT and lumbar puncture, may be considered
to understand the etiology of the seizures, but the first step is to ensure the patient is
protecting their airway. Lorazepam is the first medication to be given in status epilepticus,
not phenytoin. Naloxone is typically given to reverse opioid withdrawal.

6. The correct answer is: A. A normal routine EEG excludes the possibility of an
underlying seizure disorder. An interictal EEG (an EEG obtained when a patient is not
manifesting symptoms of a possible seizure) has roughly 50% sensitivity for an
underlying seizure disorder. Therefore, an abnormal EEG can rule in a seizure disorder if
epileptiform discharges are seen, but a normal EEG cannot rule out the possibility of
seizure disorder. If suspicion remains high for seizures after an inconclusive workup,
further investigation with long-term EEG monitoring can be pursued in an effort to
capture a clinical event during EEG monitoring and assess for an electrographic correlate.
The remainder of the answer choices are all true for the workup of patients presenting
with syncope versus seizure.

7. The correct answer is: C. History of major depression. In 10% to 20% of patients,
levetiracetam causes mood symptoms ranging from depression to irritability and, in severe
cases, rage and agitation. Given the patient’s history of major depressive disorder with a
prior suicide attempt, this medication should be avoided. Thrombocytopenia and hepatic
disease are relative contraindications to valproic acid. First-degree heart block is a
contraindication to lacosamide. Hyponatremia is a contraindication to carbamazepine and
oxcarbazepine.

8. The correct answer is: E. Thiamine. Ataxia, ophthalmoparesis, and altered mental status
compose the hallmark triad of Wernicke encephalopathy. Wernicke encephalopathy is an
acute life-threatening condition from thiamine deficiency. Many vitamin deficiencies can
occur in patients with alcohol use disorder; however, riboflavin deficiency typically
causes stomatitis. Pyridoxine deficiency typically causes skin findings, neuropathy, and
sometimes somnolence or confusion. Cyanocobalamin deficiency typically causes anemia
and/or neurologic symptoms, including subacute combined degeneration of the spinal
cord.

9. The correct answer is: A. Delirium tremens. The patient’s history of alcohol abuse puts
him at risk for alcohol withdrawal. Delirium tremens is usually seen around 3 days after
the patient’s last alcoholic drink. It typically manifests with hallucinations, hypertension,
shivering, sweating, and tachycardia. Schizoaffective disorder, posttraumatic stress
disorder, and depression would not be expected to have these vital sign abnormalities.

10. The correct answer is: A. Alprazolam. Alprazolam is a short-acting benzodiazepine,


and, owing to its short half-life and potential for causing rebound seizures, it should not be
used for the management of alcohol withdrawal. The remainder of the answer choices are
all appropriate to consider.

11. The correct answer is: D. Vestibular neuritis. This patient has all the examination
features of a peripheral cause of her dizziness (positive head thrust, no skew, and
unilaterally beating nystagmus). Given that she has been experiencing symptoms for
hours, with no reported association with head movement, this is unlikely to be benign
paroxysmal positional vertigo. Given that hearing is normal, this is not labyrinthitis.
Generally, central nystagmus is direction-changing and the patient also has no other
localizing symptoms to suggest a posterior circulation stroke.

12. The correct answer is: A. Benign paroxysmal positional vertigo. Benign paroxysmal
positional vertigo causes brief (seconds to minutes long) episodes of dizziness that are
often noticed when changing head position. This is unlikely to be vestibular neuritis given
how brief the episodes are. This is unlikely to be migraine given no reported associated
headache. There is no trauma history and the episodic nature makes concussion less likely.
Though vertigo can occur with vertebrobasiliar insufficiency, there are typically other
neurologic symptoms and signs on examination.

13. The correct answer is: A. Diminished hearing. Diminished hearing accompanying
dizziness suggests an otologic etiology and is unlikely to be due to a central lesion. The
remainder of the response choices are all concerning for a possible lesion in the brainstem
or cerebellum, and any of these would warrant brain MRI to assess for an acute stroke in
the posterior circulation.

14. The correct answer is: A. Finger-to-nose coordination test. The HINTS test consists of
the Head Impulse test, Nystagmus, and Test of Skew. The head impulse test involves
rapid, passive head turns while the patient maintains visual fixation. The test of skew
involves alternately covering and uncovering each eye to assess for vertical misalignment
of the eyes. If the HINTS testing reveals absence of a correctional saccade on head
impulse testing, presence of direction-changing nystagmus, or presence of a vertical skew
deviation, MRI of the brain should be pursued to assess for a central etiology. The finger-
to-nose coordination test assesses ipsilateral cerebellar function and may be abnormal in a
posterior circulation stroke, but is not a component of the HINTS test.

15. The correct answer is: A. Left middle cerebral artery (MCA). The left MCA supplies
the area of the brain responsible for language in the majority of people. The left MCA also
supplies the left motor strip, which is responsible for controlling the right-side body,
which explains why the patient is unable to move the right side. In strokes, the eyes
deviate toward the lesion, when compared to seizures, in which the eyes deviate away

Freemedicalbooks4download
from the lesion. The right anterior cerebral artery would present with predominantly left
leg more than left arm weakness, would not typically have an impact on language, and
would have gaze deviation in the opposite direction. A right MCA infarction would
present with weakness of the left side, not the right side, and also would have evidence of
neglect on further testing. A left posterior cerebral artery infarction would predominantly
cause visual symptoms, in particular, right homonymous hemianopia. A right posterior
inferior cerebellar artery stroke is also known as a lateral medullary syndrome and would
typically cause vertigo, ipsilateral (in this case right-sided) hemiataxia, dysarthria, ptosis,
and miosis (ie, ipsilateral Horner’s syndrome).

16. The correct answer is: D. A 68-year-old woman with a history of hypertension and
diabetes who presents to the hospital with sudden onset of left face and arm
weakness that started 4 hours ago. The patient in answer D is presenting within 4.5
hours of last known well time and does not have any exclusion criteria. Choices A and E
are incorrect because the patients are presenting outside the 4.5-hour tissue plasminogen
activator window. Choice B is incorrect because the patient’s recent surgery is a relative
contraindication. Choice C is incorrect because the patient’s recent intracerebral
hemorrhage is a contraindication.

17. The correct answer is: C. Determine the extent of an acute infarct. While CT is highly
sensitive for acute intracranial hemorrhage, radiographic changes are typically absent for
up to 6 hours following an ischemic stroke. As such, it is a poor tool for determining the
infarct burden within 6-24 hours following an ischemic stroke. MRI is the preferred
imaging modality for determining the extent of an acute infarct. CT angiography is an
important component of the acute stroke assessment, as it detects the presence or absence
of a proximal large-vessel occlusion that would indicate endovascular thrombectomy and
detects atherosclerotic disease in the aortic arch, cervical and intracranial vessels that
could contribute to the underlying stroke etiology. Though patients with end-stage renal
disease (ESRD) or allergy to iodine cannot receive CT contrast, they may have vessel
imaging with magnetic resonance angiography (MRA) time-of-flight protocol, which does
not require IV contrast with gadolinium or a carotid duplex ultrasound.

18. The correct answer is: E. Hypertension. Hemorrhages of the deep gray matter structures
such as the basal ganglia and pons are typically the result of hypertension. This is due to
rupture of small perforator lenticulostriate arteries that arise from the middle cerebral
arteries. Most perforator arteries originate from the MCA, but not necessarily the large
arteries of the circle of Willis. Hemorrhage of an underlying neoplastic lesion is possible
but less likely in a patient with no known history of malignancy. Arteriovenous
malformations and cerebral amyloid angiopathy typically give rise to more peripheral
lobar hemorrhages. Aneurysm rupture typically results in subarachnoid hemorrhage.

19. The correct answer is: C. Intravenous immunoglobulin (IVIG). This patient has acute
inflammatory demyelinating polyneuropathy or Guillain-Barré syndrome. Symptoms of
symmetric proximal and distal limb weakness, areflexia, and distal paresthesias/sensory
loss develop over a few days, with peak severity at 2 to 4 weeks. Up to two-thirds of
patients can also present with back pain. Lumbar puncture shows cytoalbuminologic
dissociation or elevated protein without cerebrospinal fluid (CSF) pleocytosis. These
patients are at high risk for respiratory compromise (neck flexion weakness is a proxy for
respiratory muscle involvement), so require very close monitoring, including tests of
respiratory mechanics. The clinical history, examination, and CSF are sufficient to begin
treatment for acute inflammatory demyelinating polyneuropathy with either IVIG or
plasmapheresis. EMG/nerve conduction studies can be helpful, but may be normal early in
the disease course and is not required for diagnosis. MRI of the spine may show nerve
root enhancement but is not a sensitive sign. Steroids have not been shown to have benefit
in acute inflammatory demyelinating polyneuropathy and are not recommended. Although
acute inflammatory demyelinating polyneuropathy is often preceded by an infection, this
patient does not have signs of active infection or meningitis/encephalitis requiring
antibiotics or antivirals.

20. The correct answer is: A. Nonsteroidal anti-inflammatory drugs (NSAIDs). First-line
agents for treating neuropathic pain include gabapentin, pregabalin, tricyclic
antidepressants (nortriptyline or amitriptyline), and serotonin and norepinephrine reuptake
inhibitors (duloxetine and venlafaxine). Second-line agents include tramadol and topical
agents, such as lidocaine and capsaicin. Third-line options include opiates and botulinum
toxin A. NSAIDs are not recommended for the treatment of neuropathic pain.

21. The correct answer is: A. 3,4-Diaminopyridine (3,4-DAP). The symptoms described in
the vignette reflect Lambert-Eaton myasthenic syndrome. This presynaptic neuromuscular
junction disorder is caused by autoantibodies against voltage-gated calcium channels,
most commonly associated with small-cell lung cancer. Lambert-Eaton myasthenic
syndrome is characterized by proximal limb weakness, preferentially involving the lower
extremities, and fatigability. Hyporeflexia or areflexia is also observed. Autonomic
nervous system abnormalities, in particular dry mouth but also pupillary abnormalities,
decreased sweating and lacrimation, and impotence are other important characteristics.
The weakness and hyporeflexia tend to improve temporarily with brief, repeated muscle
contractions. Diffusely reduced motor amplitudes on motor nerve conduction studies,
often <50% of the laboratory’s lower limits of normal, are commonly encountered at
baseline in Lambert-Eaton myasthenic syndrome. Electrophysiologic tests of particular
importance are repetitive nerve stimulation studies that when performed at a slow rate (3
Hz) show a decremental response similar to myasthenia gravis (MG). After rapid
repetitive nerve stimulation (30 to 50 Hz) or brief (10 sec), intense contractions, a marked
increase of the compound muscle action potential amplitude by >200% is seen.
The most effective symptomatic treatment in Lambert-Eaton myasthenic syndrome
is 3,4-DAP. Through blocking voltage-gated potassium channels on presynaptic motor
neurons, 3,4-DAP prolongs nerve terminal depolarization and increases acetylcholine
release. In theory, pyridostigmine should be synergistic with 3,4-DAP, but many patients
with Lambert-Eaton myasthenic syndrome have no benefit from pyridostigmine, either on
its own or in combination with 3,4-DAP.

22. The correct answer is: B. Perifascicular atrophy and perivascular inflammation. The
symptoms experienced in this case suggest a pattern of proximal muscle weakness.
Difficulty going up and down stairs and difficulty getting out of low chairs are symptoms
characteristic of proximal lower extremity weakness. On examination, proximal weakness
in both upper and lower extremities, as well as mild weakness in the neck flexors, was

Freemedicalbooks4download
found. In combination with the erythematous papules over the extensor surfaces (Gottron
papules) and the skin rash over the upper back (shawl sign), the clinical picture is
suggestive of dermatomyositis. The characteristic pathologic findings of dermatomyositis
on muscle biopsy are perifascicular atrophy, in which atrophic fibers are present at the
edges of fascicles that are otherwise composed of relatively normal-sized myofibers and
perivascular inflammation composed primarily of macrophages, B-cells, and plasma cells.

23. The correct answer is: A. Cluster headache. This patient’s history is classic for cluster
headache. Patients often describe an excruciating stabbing, boring, or burning pain that is
unilateral and periorbital. Ipsilateral autonomic symptoms, including lacrimation,
conjunctival injection, and rhinorrhea, are common. Patients often report a feeling of
restlessness or panic associated with the headaches. The headaches often come on at the
same time of day, with the evening being most common. Attacks can last 15 minutes to 3
hours, and patients can have up to eight attacks per day. The attacks often occur in clusters
lasting weeks to months, and patients can be headache-free between clusters. Like cluster
headache, short-lasting unilateral neuralgiform pain with conjunctival injection and
tearing syndrome is classified as a trigeminal autonomic cephalalgia. Pain quality is
similar, but pain is much briefer and more frequent, with attacks lasting 5 seconds to 4
minutes and happening up to 200 times per day. His symptoms are not typical for
trigeminal neuralgia (shooting pain in the face) or tension headache (bilateral, mild-to-
moderate pressure pain). Evaluation of new headaches should always include review of
red flag symptoms and signs that could indicate a secondary headache. These include
explosive onset (or “worst headache of life”), signs of increased intracranial pressure
(including worse with lying down), vision symptoms (including eye pain, diplopia, and
blurred vision), abnormal neurologic examination, age >50 years, and immunosuppressed
state.

24. The correct answer is: A. Indomethacin. This patient’s presentation is consistent with
hemicrania continua—a chronic headache syndrome of severe attacks with autonomic
features superimposed on a continuous headache of at least 3 months’ duration. Brain
imaging is usually indicated to exclude a structural lesion and dedicated venous imaging
should also be considered to exclude venous sinus thrombosis in patients with risk factors
for hypercoagulability. Hemicrania continua should resolve completely with
indomethacin; this treatment response is part of the diagnostic criteria. An indomethacin
trial is typically done over several days with incremental dose increases. After a
successful trial, the headache resolves and indomethacin is stopped. If the headache does
not completely resolve, an alternative diagnosis should be considered. Tricyclic
antidepressants and topiramate are treatments for many headache types, but not for
hemicrania continua. Verapamil is particularly effective in cluster headache. Triptans are
used as abortive medications for migraines.

25. The correct answer is: D. NOTCH3. This patient’s presentation is highly suggestive of
cerebral autosomal dominant arteriopathy with subcortical infarcts and
leukoencephalopathy (CADASIL). This is based on her history of subcortical infarcts
without the typical risk factors associated with vascular disease (eg, hypertension, diabetes
mellitus, dyslipidemia), her history of migraine with aura, and her family history of
migraines, stroke, and early cognitive impairment. Imaging in CADASIL shows diffuse
white matter hyperintensities on T2-weighted imaging, often including the anterior
temporal region and the external capsule (also known as the O’Sullivan sign). CADASIL
is an autosomal dominant disorder caused by a NOTCH3 gene mutation. It is the most
common cause of inherited stroke and vascular dementia in adults. The NOTCH3 gene
encodes a transmembrane receptor that contains 34 epidermal growth factor repeats—a
large transmembrane protein necessary for vascular smooth muscle differentiation and
development.

26. The correct answer is: E. Send thrombophilia workup and start therapeutic
anticoagulation with heparin. The patient has several red flags in her history that
suggest a secondary cause of headaches, such as worsening in frequency and persistence,
worsening with recumbent position, and awakening from sleep. Her examination is
notable for bilateral papilledema, suggesting that her intracranial pressure might be
elevated. The cranial imaging depicted in the figure is a magnetic resonance venogram
showing nonvisualization of the right transverse, sigmoid, and jugular venous system
consistent with venous sinus thrombosis. The correct management in this case would be to
send thrombophilia workup and start the patient on therapeutic anticoagulation as soon as
possible.

27. The correct answer is: B. L5 nerve root. This patient’s symptoms are consistent with an
L5 radiculopathy causing foot drop, likely due to herniation of the L4-L5 disc.
Radiculopathy can present with shooting pain down the affected extremity, sensory loss,
and motor weakness. The L5 nerve root innervates the tibialis anterior, which dorsiflexes
the foot; weakness of this muscle causes foot drop. Sensory loss is over the lateral calf and
great toe, with paresthesias shooting from the low back down the lateral thigh into the
same distribution. Reflexes are typically normal, as the patellar reflex is mediated by L4
and the Achilles reflex is mediated by S1. L4 radiculopathy presents with quadriceps
weakness (knee extension) and numbness over the anteromedial lower leg and inner foot.
S1 radiculopathy presents with gastrocnemius weakness (ankle plantar flexion) and
sensory loss over the lateral foot and sole of the foot. A thoracic spinal cord lesion would
be more likely to present with a sensory level and upper motor neuron signs and would
not cause radicular pain. Peroneal neuropathy is another common cause of foot drop, most
often due to compression at the fibular head, but can be distinguished from L5
radiculopathy by the sparing of foot inversion, which is in the L5 myotome but innerved
by the tibial nerve.

28. The correct answer is: B. Calf cramping. Both lumbar spinal stenosis and peripheral
vascular disease with limb ischemia can cause lower extremity pain with certain activities.
In lumbar spinal stenosis (neurogenic claudication), patients typically report radicular pain
that is worse with walking, standing, or lying prone and better with bending forward or
sitting. Sitting forward can help relieve mechanical compression of the nerve roots. There
may also be associated focal weakness, sensory changes, and decreased reflexes. In
peripheral artery disease (vascular claudication), the pain is a cramping that is
predominantly in the calves and can radiate up the legs. It is worse with activity and better
with rest, but there is no change between sitting and standing. On examination, there may
be pale, cool extremities with diminished pulses, but neurologic findings would not be
expected.

Freemedicalbooks4download
29. The correct answer is: A. Administer dexamethasone 10 mg IV, with urgent
evaluation for thoracic spine decompression. This patient presents with symptoms of
rapidly progressing weakness and numbness of her bilateral lower extremities. On
examination, she has evidence of a thoracic myelopathy based on her pattern of weakness
(involving the lower but sparing the upper extremities), hyperreflexia, upgoing toes, and
T4 sensory level to pinprick. Her imaging reveals an epidural mass lesion compressing the
spinal cord at the level of the T5 vertebra concerning for metastatic disease. The next step
in management would involve administration of steroids and evaluation for urgent spinal
cord decompression.
Abbreviations

ABG arterial blood gas


ACE angiotensin-converting enzyme
AF atrial fibrillation
AFB acid-fast bacilli
ALT alanine aminotransferase
ARB angiotensin receptor blocker
AST aspartate aminotransferase
BCG bacillus Calmette-Guérin
BMI body mass index
BP blood pressure
BUN blood urea nitrogen
CABG coronary artery bypass grafting
CAD coronary artery disease
CBC complete blood count
CK creatine kinase
CNS central nervous system
COPD chronic obstructive pulmonary disease
CPR cardiopulmonary resuscitation
CSF cerebrospinal fluid
CT computed tomography
CTA CT angiography
CVA cerebrovascular accident
CXR chest radiograph
DDAVP desmopressin
DM diabetes mellitus
ECG electrocardiogram
ED emergency department
EEG electroencephalogram
EF ejection fraction
EIA enzyme-linked immunoassay
ELISA enzyme-linked immunosorbent assay
EMG electromyography

Freemedicalbooks4download
ENT ears, nose, and throat
ER emergency room
ESR erythrocyte sedimentation rate
FSH follicle-stimulating hormone
FVC forced vital capacity
GERD gastroesophageal reflux disease
GFR glomerular filtration rate
GI gastrointestinal
HAV hepatitis A virus
HBV hepatitis B virus
HCV hepatitis C virus
HEV hepatitis E virus
HF heart failure
Hgb hemoglobin
HIV human immunodeficiency virus
HR heart rate
HSCT hematopoietic stem cell transplantation
IBD inflammatory bowel disease
ICU intensive care unit
INR international normalized ratio
IVC inferior vena cava
IVIG intravenous immunoglobulin
JVD jugular venous distention
JVP jugular venous pressure
LBBB left bundle branch block
LDH lactate dehydrogenase
LDL low-density lipoprotein
LDL-C lipoprotein cholesterol
LFT liver function test
LMWH low-molecular-weight heparin
LP lumbar puncture
MAP mean arterial pressure
MCV mean corpuscular volume
MELD model for end-stage disease
MI myocardial infarction
MR magnetic resonance
MRA MR angiography
MRI magnetic resonance imaging
MRSA methicillin-resistant Staphylococcus aureus
MRV MR venography
MSSA methicillin-sensitive Staphylococcus aureus
MTb Mycobacterium tuberculosis
MVA motor vehicle accident
NPH neutral protamine Hagedorn
NPO nil per os (nothing by mouth)
NSAID nonsteroidal anti-inflammatory drug
OCP oral contraceptive pill
PCP primary care physician
PCR polymerase chain reaction
PET positron emission tomography
PFT pulmonary function test
PMH past medical history
PMN polymorphonuclear leukocyte
PO oral intake
PSA prostate-specific antigen
PT prothrombin time
RBC red blood cell
ROS review of systems
RR respiratory rate
RUQ right upper quadrant
SC subcutaneous
SPEP serum protein electrophoresis
SPS sodium polystyrene sulfonate
STS Society of Thoracic Surgery
T1DM type 1 diabetes mellitus
T2DM type 2 diabetes mellitus
TB tuberculosis
TSH thyroid-stimulating hormone
U/A urinalysis
URI upper respiratory infection
USPSTF US Preventive Services Task Force
UTI urinary tract infection
VBG venous blood gas
WBC white blood cell
WHO World Health Organization

Freemedicalbooks4download

You might also like